You are on page 1of 602

The Exponential Function

Of primary importance in this course is the exponential function x (t) = e at , where a is a constant. We will assume you are completely familiar with the properties and graphs of this function.

Properties: 1. e0 = 1. 2. e at+c = ec e at . 3. e at is never 0. 4. If a > 0 then lim e at = and


t t

lim e at = 0.
t

5. If a < 0 then lim e at = 0 and


t

lim e at = .

6. For any positive a, e at grows much faster than any polynomial. Examples. lim et /t3 = ,
t t

lim tet = 0.

Graphs
y 6 5 4 3 2 1 2 1 .5 1 1.5 2 t
2 1 6 5 y

y=e

y=e

4 3 2 1 1 2 t

Fig. 1. Graphs of et and et .

Variables and Parameters 1. Independent and Dependent Variables


When we write a function such as f ( x ) = 3x2 + 2x + 1 we say that x is an independent variable: it can be freely set to any value (or any value within the given domain) and the value of the function is then computed. When we give a name to the value of the function, such as y = 3 x2 + 2 x + 1 or y = f (x)

we say that y is a dependent variable. That is, the value of y depends on the value we choose for x. We can have systems of equations with more than one dependent variable. For example, x = t2 1 y = 3e t . Here the dependent variables x and y depend on the independent variable t. We can have functions with more than one independent variable. For example, x = st2 t s Here the independent variables are t and s and the dependent variable is x. And, of course, we can have more than one of each: x = st2 t s y = 3e t + s . As a matter of notation (often referred to by mathematicians as abuse of notation) we can use the dependent variable to also denote the function. So, for example, we can write x = x (t) = t2 1. Most of what we do will involve ordinary differential equations. These have only one independent and one dependent variable. Differential equations arise from many sources, and the independent variable can signify

Variables and Parameters

OCW 18.03SC

many different things. Nonetheless, very often it represents time, and the dependent variable is some dynamical quantity which depends upon time. For this reason, in this course we will often use t for the independent variable.

2.

Parameters

Parameters are similar to variables that is, letters that stand for numbers but have a different meaning. We use parameters to describe a set of (usually) similar things. Parameters can take on different values, with each value of the parameter specifying a member of this set of similar objects. An example should make this clear. In calculus you learned to nd the antiderivative (integral) of t2 . There are many functions whose derivative is t2 . For example, t3 /3 + 2 or t3 /3 + . So, to give the full answer we write

t2 dt = t3 /3 + c.

where c is called the constant of integration. In this case, each value of c species a single antiderivative. We call c the parameter of the set of all the antiderivatives of t2 . Each value of the parameter c species a single antiderivative. Sets are written formally using curly braces, e.g., {t3 /3 + c : c any number}, but we will rarely do this. For example, we will write, x (t) = t3 /3 + c, where c is an arbitrary constant. (1)

This means a set of functions x = x (t) parametrized by c. Sets can depend on more than one parameter. For example, x ( t ) = c 1 e t + c 2 e 7t where c1 , c2 are arbitrary constants. (2)

Because each of the functions in (1) are similar they all have a family resemblance we say equation (1) gives a 1-parameter family of functions. Likewise, we say (2) gives a 2-parameter family of functions. You see the pattern!

Notations for Derivatives


dy , y and Dy dx to all mean the derivative of y with respect to x. Only the rst one species the independent variable x. In the other two you can only determine the independent variable from context. When the independent variable is time t we will usually adopt the . physicists notation x for the derivative. For second derivatives we have d2 y = y = D2 y dx2 all mean the second derivative of y with respect to x. If x = x (t) is a func.. tion of time we will also write x. For higher derivatives we will use the notations dn y = y(n) = D n y dx n to mean the nth derivative. We will write

Differential Equations 1. Denition of Differential Equations


A differential equation is an equation expressing a relation between a function and its derivatives. For example, we might know that x is a function of t and .. . x + 8x + 7x = 0. (1) or perhaps the relation is more complicated, like xx (5) + cos(t)etx + ( x x x )6 = sin(5t).

(2)

When the function in the differential equation has a single independent variable we call it an ordinary differential equation. That is, the derivatives are ordinary derivatives, not partial derivatives. This course is almost exclusively concerned with ordinary differential equations. The Order of a Differential Equation The order of a differential equation is the order of the largest derivative appearing in it. Equation (1) is a second order differential equation. Equation (2) is a fth order equation since the highest derivative is x (5) (in the rst term).

2. Solving a Differential Equation


Solving a differential equation means nding a function that satises the equation. For many equations it can be hard or impossible to nd a solution. One thing that is easy however is to check a proposed solution. We demonstrate with a few examples. Example 1. Checking a Solution By Substitution Verify the y(t) = e3t is a solution to the differential equation y = 3y.

(3)

Solution. To do this we simply substitute y = e3t into (3), and check that . the equation holds. On the left hand side of (3) we have y = 3e3t . On the right hand side we have 3y = 3e3t . Since both sides are equal, y = e3t is a solution. Example 2. Rejecting a Solution by Substitution Show that y(t) = t3 is not a solution to the differential equation y = y / t.

(4)

Differential Equations

OCW 18.03SC

Solution. Again, we substitute the expression for y into (4). . Left hand side: y = 3t2 . Right hand side: y/t = t2 . Since the two sides are not equal, y = t3 is not a solution.

3.

Parametrizing the Set of Solutions of a Differential Equation

Differential equations usually have more than one solution. We can describe them all at once using a parameter. Example. Find all the solutions to

..

x = 2t

(5)

This is a standard calculus problem. Integrating twice and remembering to include the constants of integration gives x (t) = t3 + c1 t + c2 , 3

where c1 and c2 are arbitrary constants. This expression gives a parametrization of the set of solutions to equation (5). The constants c1 and c2 are parameters. Every choice of c1 and c2 gives a different solution to (5). For example, x = t3 /3 + 2t + 1 and t3 /3 + t + 2.718 are both solutions.

4.

Initial Value Problems

Sometimes we have a differential equation and initial conditions. Together they make up an initial value problem. The meaning of the term initial conditions is best illustrated by example. Example. Solve the initial value problem x = 2t with the initial conditions . x (1) = 1, x (1) = 2. Solution. In the previous example we found the general solution of this differential equation t3 x ( t ) = + c1 t + c2 . 3 We use the initial conditions to nd the values of c1 and c2 . x (t) = t2 + c1 x (1) = 1 + c1 = 2. x (t) = t3 /3 + c1 t + c2 x (1) = 1/3 + c1 + c2 = 1. Solving for c1 and c2 we get c1 = 1, c2 = 1/3. Thus, the solution to the initial value problem is x (t) = t3 /3 + t 1/3. 2

..

Differential Equations

OCW 18.03SC

5. Acronyms
It will be convenient at times to allow ourselves to use acronyms. Some of the most common are 1. Differential equation (DE). 2. Ordinary differential equation (ODE). 3. Initial value problem (IVP). 4. Initial conditions (IC).

Solution to an ODE
Quiz: Solution to an ODE. Which of the following is a solution to the ODE dy/dx = 2y + 1? Choices: a) y = ce2x 1. b) y = x2 + x + c. c) y = e x/2 + c. d) y = ce2x 1/2. e) y = e2x + c. f) None of the above

Answer: (d) This is a little long because at this point our only strategy is to check each potential solution by substitution. (We will remedy that soon!) Briey: a) Left side: dy/dx = 2ce2x . Right side: 2y + 1 = 2ce2x 2 + 1 = 2ce2x 1. Not equal. b) Left side: dy/dx = 2x + 1. Right side: 2y + 1 = 2x2 + 2x + 2c + 1. Not equal.
x /2 . Right side: 2y + 1 = 2e x /2 + 2c + 1. Not c) Left side: dy/dx = 1 2e equal.

d) Left side: dy/dx = 2ce2x . Right side: 2y + 1 = 2ce2x 1 + 1 = 2ce2x . Equal! This is the answer. e) Left side: dy/dx = 2e2x . Right side: 2y + 1 = 2e2x + 2c + 1. Not equal.

Introduction: The Most Important DE 1. The Most Important DE


The most important differential equation we will study is y = ay. In words the equation says the rate of change of y is proportional to y. It is hardly an exaggeration to say that much of 18.03 is an elaboration on this fundamental equation. In any case, we need to understand this DE and its solutions thoroughly. Because of its importance we will write down some other ways you might see it y = ay;

(1)

dy = ay(t); dt

y = ay;

y ay = 0.

(2)

You should recognize all these as the same equation. The solution to this equation is y(t) = Ce at , where C is any constant. This is easily checked by substitution. Again, because this equation is so important we show the details. Left side of (1): y = aCe at . Right side of (1): ay = aCe at . Since, after substitution the left side equals the right side, y(t) = Ce at is indeed a solution of (1). Because of the exponential in the solution equation (1) is said to model expontial growth (when a > 0) or decay (when a < 0). The constant a is known as the growth or decay constant. In this course we will learn many techniques for solving differential equations. We will test almost all of them on equation (1). You should, of course, understand how to use these techniques to solve (1). However: whenever you see this equation you should remind yourself it models exponential growth or decay and know the solution without computation.

Other Basic Examples 1. Other Basic Examples

Here are some basic examples of DEs taken from math and science. Except for example 1 we will not give solutions. We will do that and more with these DEs as we go through the course. Example 1. (From Calculus) dy Solve for y satisfying = 2x dx Solution. This problem is just asking for the anti-derivative of 2x: y ( x ) = x 2 + c. Notice that there are many solutions, parametrized by c. An expression like this, which parametrizes all the solutions is called the general solution. Example 2. (Heat Diffusion) A body at temperature T sits in an environment of temperature TE . Newtons law of cooling models the rate of change in temperature by T ' = k ( T TE ), where k is a positive constant. Note, the minus sign guarantees that the temperature T is always heading towards the temperature of the environment TE . Example 3. (Newtons Law of Motion: Constant Gravity) Near the earth a body falls according to the law d2 y = g, dt2 where y is the height of the body above the Earth and g is the acceleration due to gravity, 9.8 m/sec2 . Example 4. (Newtons Law of Gravitation) Newtons law of gravity says that the acceleration due to gravity of a body at distance r from the center of the Earth is d2 r = GME /r2 , dt2

Other Basic Examples

OCW 18.03SC

where ME is the mass of the Earth and G is the universal gravitational constant. Example 5. (Simple Harmonic Oscillator: Hookes Law) Suppose a body of mass m is attached to a spring. Let x be the amount the spring is stretched from its unstretched equilibrium position. Hookes law combined with Newtons law of motion says mx = kx

..

mx + kx = 0,

..

where k is the spring constant. The minus sign indicates that the force always points back towards equilibrium, as it does in the real world. Example 6. (Damped Harmonic Oscillator) If we add a damping force proportional to velocity to the spring-mass system in example 5, we get mx = kx bx

..

mx + bx + kx = 0,

..

here bx is the damping force and b is called the damping constant. Example 7. (Damped Harmonic Oscillator with an External Force) If we add a time varying external force F (t) to the system in example 6, we get .. . .. . mx = kx bx + F (t) mx + bx + kx = F (t).

Separation of Variables 1. Separable Equations


We will now learn our rst technique for solving differential equation. An equation is called separable when you can use algebra to separate the two variables, so that each is completely on one side of the equation. We illustrate with some examples. Example 1. Solve y' = x (y 1) Solution. We rewrite the equation as variables dy = x (y 1). Then separate the dx

dy = x dx. y1 dy = y1 x2 + c1 . 2

Next we integrate both sides. x dx

ln |y 1| + c2 =

We can amalgamate the two constants of integration into one constant: ln |y 1| = x2 + c3 . 2

(We label the constant of integration c3 so well have c still available later.) Next we solve for y as a function of x.

| y 1| = e x

2 /2+ c

= e c3 e x

2 /2

The absolute value signs can be removed, but then the right hand side might be positive or negative. We write this as y 1 = e c3 e x
2 /2

1 + e c3 e x

2 /2

Finally we replace the constant ec3 by C to get the solution y(t) = 1 + Ce x Note.
2 /2

dy = x dx y1 might give pause. However, this formal method is justied by the chain rule, in the same way change of variable (u-substitution) is justied for integration. For the more rigorously minded an expression like

Separation of Variables

OCW 18.03SC

1.1.

Lost Solutions

Example 2. Find all the solutions to the DE y ' = 2 x (1 y )2 . (1)

Solution. First, note there is a constant solution: y( x ) = 1. It is easy to see this is a solution by substituting it into (1) both sides of the equation become 0. We need to note this because, as we will see, the separation of variables method will not nd this particular solution. Now lets solve the DE by separation of variables. 1. Separate variables: 2. Integrate: 3. Solve for y: dy = 2x dx. (1 y )2 1 = x2 + C. 1y 1 . y = 1 2 x +C

Notice that the constant solution y( x ) = 1 is not in the parametrized family found by separation variables. We call this a lost solution because it is lost by separation of variables. How did it get lost? The answer is in step (1) above, where the term dy is only valid if y = 1. (1 y )2 In general, for the separable DE y' = f ( x ) g(y), all the roots of g(y) give lost (constant) solutions. Example 3. Find all the lost solutions of y' = ( x + 1)e x (y2 8y + 7). Solution. The factor y2 8y + 7 has roots y = 1 and y = 7. Therefore the lost solutions are the constant functions y( x ) = 1 and y( x ) = 7. 1.2. The Most Important DE Even though we already know the solution, we should test our new technique on the DE for exponential growth/decay. Example 4. Solve y = ky. Solution. Separate variables: Integrate: ln |y| = kt + c1 . Exponentiate: |y| = ekt+C1 = ec1 ekt . 2 dy = k dt. y

Separation of Variables Remove absolute value: y = ec1 ekt . Let the constant ec1 = C: y = Cekt . All solutions to the DE are y(t) = Cekt .

OCW 18.03SC

If you look carefully youll see we did one rather sneaky thing. The solution y(t) = 0 is a lost solution, yet it appears to have been found by the separation of variables (set C = 0). What happened is that when we renamed ec1 as C we should have noticed that the exponential is never 0, so C = 0. Essentially, we included the lost solution by being a little sloppy and then getting lucky. We do not recommend this technique as a way to do mathematics!

Separation of Variables
Quiz: Separation of Variables. What is the general solution to the ODE dy/dx = 2y + 1? (Use separation of variables.) Choices: a) y = Ce2x 1. b) y = Ce x/2 2. c) x = y2 + y + C. d) y = e x/2 + C. e) y = Ce2x + 1. f) y = Ce2x 1/2. g) y = e2x + C. h) None of the above.

Answer: Separate variables: dy/(2y + 1) = dx Integrate both sides: (1/2)ln|2y + 1| + c1 = x + c2 . Amalgamate the constants: ln |2y + 1| = 2x + c3 . Exponentiate and solve (if possible) for y in terms of x:

|2y + 1| = ec3 e2x 2y + 1 = Ce2x y = Ce2x 1/2.


So the answer is: (f)

Separation of Variables
Quiz: Separation of Variables. What is the general solution to the ODE dy/dx = 2y + 1? (Use separation of variables.) Choices: a) y = Ce2x 1. b) y = Ce x/2 2. c) x = y2 + y + C. d) y = e x/2 + C. e) y = Ce2x + 1. f) y = Ce2x 1/2. g) y = e2x + C. h) None of the above. Pick what you think is the correct choice and then look at the answer.

Is it Separable?
Quiz: (Is it Separable?.) Is y + xy = x separable? Choices: a) Yes. b) No. Answer: Well, y = x xy = x (1 y) so dy/(1 y) = xdx: Yes, the equation is separable. We could go on to solve this, but you can do that on your own.

Is it Separable?
Quiz: (Is it Separable?.) Is y + xy = x separable? Choices: a) Yes. b) No. Pick what you think is the correct choice and then look at the answer.

Is it Separable?
Quiz: (Is it Separable?.) Is y + xy = x separable? Think about your answer and then look at the choices.

Solutions that Blow Up: The Domain of a Solution


Example 1. Solve the IVP y = y2 , y(0) = 1. Solution. We can solve this using separation of variables. Separate: dy = dx. y2

Integrate: 1/y = x + C.

Find C using the IC: y(0) = 1 = 1/C, therefore C = 1. Solution: y = 1/( x 1) = 1/(1 x ).
5 4 3 2 1 2 1 1 2 3 4 5 1 2 t y

Solve for y: y = 1/( x + C ).

The graph has a vertical asympote at x = 1.

Starting at x = 0 the graph goes to innity as x 1. Informally, we say y blows up at x = 1. The graph has two pieces. One is dened on (, 1) and the other is dened on (1, ). For technical reasons we prefer to say that we actually have two solutions to the DE. We indicate this by carefully specifying the domain of each. y( x ) = 1/(1 x ) y( x ) = 1/(1 x ) y in the interval (, 1) y in the interval (1, ). (1) (2)

Fig. 1. Graph of y = 1/(1 x ).

Thus, the solution to the IVP in this example is solution (1).

Solutions that Blow Up: The Domain of a Solution

OCW 18.03SC

The rule being followed here is that solutions to ODEs have domain consisting of a single interval. The example shows one reason for this: starting at (0, 1) on solution (1) there is no way to follow the solution continuosly to solution (2).

Modeling by First Order Linear ODEs 1. Introduction


If we have a DE which models a situation involving a physical quantity y(t) then solving the DE means nding the unknown function y. Knowing the possible solutions y allows to understand the physical system. Of course someone needs to build the DE doing the modeling. In this note we will see how to do this for some real systems.

2. The savings account model


Modeling a savings account gives a good way to understand the signicance of many of the features of a general rst order linear ordinary differential equation. Write x (t) for the number of dollars in the account at time t. It accrues interest at an interest rate r. The interest rate has units of percent/year. The more money in the account the more interest you earn. At the end of an interest period of t years (e.g. t = 1/12, or t = 1/365) the bank adds r x (t) t dollars to your account. This means the change x in your account is x = rx (t)t . r has units of (years)1 . Mathematicians and some bankers like to take things to the limit. Rewrite our equation as x = rx (t) , t and suppose that the interest period is made to get smaller and smaller. In the limit as t 0, we get the differential equation x = rx One of the beautiful facts about this type of modeling is that it covers more complicated situations. In our computation, there was no assumption that the interest rate was constant in time; it could well be a function of time, r (t). In fact it could have been a function of both time and the existing balance, r ( x, t). Banks often do make such a dependenceyou get a better interest rate if you have a bigger bank account. If x is involved, however, the equation is what is called nonlinear and we will not consider that case in this session.

Modeling by First Order Linear ODEs

OCW 18.03SC

Now suppose we make contributions to this savings account. Well record this by giving the rate of savings, q. This rate has units dollars per year, so if you contribute every month then the monthly payments will be q t with t = 1/12. This payment also adds to your account, so, when we divide by t and take the limit, we get x = rx + q. Once again, your rate of payment into the account may not be constant in time; we might have a function q(t). Also, we can allow q(t) to be negative, which corresponds to withdrawing money from the account. What we have, then, is the general rst order linear ODE: x r ( t ) x = q ( t ).

(1)

3. Linear insulation
Here is another example of a linear ODE. The linear model here is not as precise as in the bank account example. A cooler insulates my lunchtime root beer against the warmth of the day, but ultimately heat penetrates. Lets see how you might come up with a mathematical model for this process. You can jump right to equation (2) if you want, but we would like to spend a some time talking about how one might get there, so that you can carry out the analogous process to model other situations. The rst thing to do is to identify relevant parameters and give them names. Lets write t for the time variable, x (t) for the temperature inside the cooler, and y(t) for the temperature outside. Lets assume that the insulating properties of the cooler dont change over time. (Were not going to watch this process for so long that the aging of the cooler itself becomes important! ) However, the insulating properties probably do depend on the inside and outside temperatures. Insulation affects the rate of change of the temperature: the rate of change at time t of temperature inside depends upon the temperatures inside and outside at time t. This gives us a rst order differential equation of the form x = F ( x, y) Now its time for the next simplifying assumption, namely that this rate of change depends only on the difference y x between the temperatures, 2

Modeling by First Order Linear ODEs

OCW 18.03SC

and not on the temperatures themselves. This means that x = f (y x ) for some function f of one variable. If the temperature inside the cooler equals the temperature outside, we expect no change. This means that f (0) = 0. Now, any reasonable function has a tangent line approximation, and since f (0) = 0 we have f (z) kz . That is, when |z| is fairly small, f (z) is fairly close to kz. (From calculus you know that k = f (0), but we wont use that here.) When we replace f (y x ) by k (y x ) in the differential equation, we are linearizing the equation. We get the ODE . x = k ( y x ). The nal assumption we are making, in justifying this last simplication, is that we will only use the equation when z = y x is reasonably small small enough so that the tangent line approximation is reasonably good. For large temperature differences the linearized model will not generally give realistic results. We can write this equation as x + kx = ky. This is Newtons law of cooling. The constant k is called the coupling constant. It mediates between the two temperatures. It will be large if the insulation is poor, and small if its good. If the insulation is perfect, then k = 0. The factor of k on the right might seem odd, but it you can see that it is forced on us by checking units: the left hand side is measured in degrees per hour, so k must be measured in units of (hours)1 . We can see some general features of insulating behavior from this equation. For example, the times at which the inside and outside temperatures coincide are the times at which the inside temperature is at a critical point: x ( t1 ) = 0

(2)

exactly when

x ( t1 ) = y ( t1 ).

(3)

Geometric Methods: Introduction


In studying the rst order ODE y = f ( x, y) the main emphasis is on learning different ways of nding explicit solutions. However, you should realize that most rst order equations cannot be solved explicitly. For such equations, one tool we can resort to is graphical methods. Mostly we will use the computer to make the visualizations. But we will also learn to carry them out by hand to give rough qualitative information about how the graphs of solutions to the differential equation look geometrically. We rst introduce the basic protagonists of graphical methods: direction elds, isoclines and integral curves; the latter correspond to solutions. Look out for the intersection principle, a key theorem about them. The isoclines applet is a great tool for getting a feel for geometric methods; make sure to familiarize yourself with it. It will be particularly important for illustrating the power of graphical methods for understanding long-term behavior of solutions the nal topic in this session.

Direction Fields, Isoclines, Integral Curves


Graphical methods are based on the construction of what is called a direction eld for the equation y = f ( x, y). To get this, we imagine that through each point ( x, y) of the plane is drawn a little line segment whose slope is f ( x, y). Example. dy = 2 x. dx
y

Notice that the slope f ( x, y) does not depend on y here. It is invariant under vertical translation. In practice, the segments are drawn in at a representative set of points in the plane; if a computer draws them, the points are (usually) evenly spaced in both directions. If drawn by hand, however, they are not, because a different procedure is used, better adapted to human speed. To construct a direction eld by hand, draw in lightly (or in dashed lines) what are called the isoclines for the equation y = f ( x, y). These are the one-parameter family of curves given by the equations f ( x , y ) = m, m constant.

Along a given isocline, the line segments all have the same slope m; this makes it easy to draw in those line segments, and you can put in as many as you want. (Note: iso-cline = equal slope.) Example. The gure below shows a direction eld for the equation y = x y. The isoclines are the lines x y = m, two of which are shown in dashed lines, corresponding to the values m = 0, 1.

Direction Fields, Isoclines, Integral Curves

OCW 18.03SC

y' = x - y

y
4 3 2 1 0 -1 -1 0 1 2

m=-1 m=0

x
3 4

The m = 0 isocline is of special interest, as the direction eld is horizontal along it; it is called the nullcline. Once you have sketched the direction eld for the equation y = f ( x, y) by drawing some isoclines and drawing in little line segments along each of them, the next step is to draw in (with a solid line) curves which are at each point tangent to the line segment at that point. Such curves are called integral curves or solution curves for the direction eld. Their signicance is this: The integral curves are the graphs of the solutions to y = f ( x, y). By denition, this is the curve y = y(t) dened so that its slope at the point ( x, y) is f ( x, y). Two integral curves (in solid lines) have been drawn for the equation y = x y. Notice they have the same slope as the direction eld at every point they pass through. Remark. While it is not compulsory to use light or dashed lines for isoclines, and solid lines for integral curves, it is a very good habit, especially when working by hand.

Isoclines
Exercise. What are the isoclines for y = y? Make a large diagram, and draw the isoclines for m = -2, -1, 0, 1, 2; use these to sketch the direction eld. Draw some integral curves; how many different types of behaviors do there seem to be? Answer.
y m=2 m = 1.5 m=1 m = 0.5 xm = 0 m = 0.5 m = 1 m = 1.5 m = 2

The isoclines are horizontal lines y = m. We can see in the gure three types of behavior for the integral curves. We know by solving the DE that they are given by y( x ) = Ce x , and these types are classied by the sign of C: positive, zero, or negative. Remark. As the slope eld is invariant under horizontal translation, integral curves are horizontal translations of each other. This will be discussed in much greater detail in the session on autonomous equations.

Numerical Methods: Introduction


The study of differential equations has three main facets: Analytic methods (also known as exact or symbolic methods). Geometric methods. Numerical methods. In the rst two sessions we introduced some of the tools from the rst two categories; in this session, some methods from the third are presented. Before proceeding, one should stress that most differential equations cannot be solved exactly; the importance of geometric and numerical methods should not be underestimated. Most of the session is devoted to one of the most basic numerical technique, Eulers method. We will learn how to implement it, and work through several examples. However, the method has limitations: without sufcient caution, it can often return answers with high errors. We will learn how to try to control and estimate these errors, and about some pitfalls to avoid. In the nal part of this session, we will be introduced briey to some more sophisticated and accurate numerical techniques.

Motivation and Implementation of Eulers Method 1. What Would One Use Numerical Methods For?
The graphical methods described in the previous session give one a quick feel for how the solutions to a differential equation behave; they can also be very accurate at predicting long-term behaviour, e.g. in the presence of funnels. However, when the medium range solutions must be known accurately, and the equation cannot be solved exactly, numerical methods are usually the best option. Also, even when an equation can be solved with an exact formula, it still might not be straightforward to compute values of a solution. For example, the equation y = y with initial condition y(0) = 1 can be solved exactly: y( x ) = e x . The number e is the value y(1). But how do you nd out that in fact e = 2.718282828459045.... ? Here, too, you would use some kind of numerical methods. For ODEs the simplest numerical method is called Eulers method.

2. Review: The Tangent Line Approximation


Consider a function y( x ), and a point ( a, y( a)) on its graph. Call Ta ( x ) the function describing the tangent line to the graph at this point, The slope of the tangent line is y ( a) and it satises the equation Ta ( a + h) = y( a) + y ( a) h Suppose we are given a differential equation y = f ( x, y), with initial condition y( a) = c. One simple way to approximate y near a is to use the tangent line at a: For example, consider the differential equation y = y, with initial condition y(0) = 1. The tangent line approximation gives: y (1) y (0) + y (0) 1 = 1 + 1 = 2 Of course, we know y( x ) = e x , so y(1) = e 2.718. So, our estimate of 2 is rather crude. Suppose instead we were trying to approximate e (= 1.649...). We get e = y(1/2) y(0) + y (0) 0.5 = 1 + 0.5 = 1.5 y( a + h) y( a) + y ( a) h

Motivation and Implementation of Eulers Method

OCW 18.03SC
y

a better estimate. Graphically, we can see the tangent line separates from the curve. The bigger the step h in the x direction the farther away the tangent line is from the curve. The point is that the tangent line approximation works best for small h. The idea behind Eulers method is to use a sequence of successive tangent line approximations, each of them with a fairly small h.

e 2 1.5 1 1/2 1

y = ex

3. Eulers method
We want to estimate the solution (integral curve) to = f ( x, y) passing through ( x0 , y0 ). It is shown as a curve in the picture. We rst choose a step size, denoted h. Starting at ( x0 , y0 ) we approximate the integral curve over the interval [ x0 , x0 + h] by the tangent line, which has slope f ( x0 , y0 ). (This is the slope of the integral curve, since y = f ( x, y).) This takes us as far as the point ( x1 , y1 ), which is calculated by the equations (see the picture) y x1 = x0 + h, y1 = y0 + h f ( x0 , y0 ) .
y (x1 , y1 ) y1 y0 (x0 , y0 ) h h x0 x1 x

slope f (x0 , y0 )

Now we are at ( x1 , y1 ). We repeat the process, using as the new approximation to the integral curve the line segment having slope f ( x1 , y1 ). This takes us as far as the next point ( x2 , y2 ), where x2 = x1 + h, y2 = y1 + h f ( x1 , y1 ) .

We continue in the same way. The general formulas telling us how to get from the (n 1)-st point to the n-th point are x n = x n 1 + h , y n = y n 1 + h f ( x n 1 , y n 1 ) . (1)

In this way, we get an approximation to the integral curve consisting of line segments joining the points ( x0 , y0 ), ( x1 , y1 ), . . . as shown in the gure below.

Motivation and Implementation of Eulers Method


y Integral curve

OCW 18.03SC

Euler approx. x0 x1 x2 x3 x

We will call the line segments "Euler struts", and their union the Euler polygon. It is an approximation to the integral curve y = y( x ). In doing a few steps of Eulers method by hand, as you are asked to do in some of the exercises to get a feel for the method, its best to arrange the work systematically in a table. Example. For the IVP: y = x2 y2 , step size .1 to nd y(1.2). Solution. yn . n xn 0 1 1 1.1 2 1.2 We use f ( x, y) = x2 y2 , yn 0 .1 .22 f ( xn , yn ) 1 1.20 h f ( xn , yn ) .1 .12 y(1) = 0, use Eulers method with h = .1, and (1) above to nd xn and

Exercise: An Example of Eulers Method


Exercise. Consider the rst order ODE y = f ( x, y) = y2 x with initial condition y(0) = 1. Estimate y(1), using Eulers method with h = 0.5. Organize your answer in a table. Answer. n xn 0 0.0 1 0.5 2 1.0 yn -1.00 -0.50 -.63 f ( x n , y n ) = y2 n xn 1.00 -0.25 h f ( xn , yn ) 0.50 -0.13

Exercise: An Example of Eulers Method


Exercise. Consider the rst order ODE y = f ( x, y) = y2 x with initial condition y(0) = 1. Estimate y(1), using Eulers method with h = 0.5. Organize your answer in a table.

Eulers Method: Exercises and Exploration


Here are some problems for you to further explore the Eulers method applet. Take time to play around with all three of them. Some of the phenomena you encounter will be explained in the next note in this session: Errors in Eulers method. Start by opening the Eulers Method . Concave and convex functions: overshooting and undershooting 1. Call up the DE y = 0.5y + 1, use the initial point (2, 1), and construct the Euler solutions for increasing x using the stepsizes h = 0.5, 0.25, and 0.125. Then compare with the actual solution. At each step, which is the best approximation? Is it too high or too low? 2. Now do the same for y = y2 2y + 1, with initial point (1, 1). Do you think you can predict when the approximation is going to be too high or too low? A cautionary tale: separatrix crossing 3. Call up y = y2 x, use as the initial point (0.98, 0). Select Actual and Euler 0.50, and have the Applet draw the solutions. Then use the same initial condition with All Euler. What happened? Can you describe the problem in the language of graphical solutions?

Errors In Eulers Method


As we have seen with the applet, Eulers method is rarely exact. In this section we try to understand potential sources of error, and nd ways to estimate or bound it.

1. Common Error Sources


Let us stress this again: the Euler polygon is not an exact solution; the direction eld at its vertices usually differs more and more from the direction eld along the actual solution. At places where the direction eld is changing rapidly, this can quickly produce very bad approximations: the variation of the direction eld causes the integral curve to bend away from its approximating Euler strut. One trick ODE solvers use is to take smaller step sizes when the direction eld is steep. As a general rule Eulers method becomes more accurate the smaller the step-size h is taken. However, if h is too small, round-off errors can appear and will accumulate, particularly on a pocket calculator; whenever in doubt, try to do all computations on a computer, keeping a high number of signicant gures.

2. Estimating the Sign of the Error: Concave and Convex Functions


Can we predict whether the Euler approximation is too big or too small? In our rst example when exploring the Euler Method applet they were too small. Pictorially, this was because the solution was curving upwards, leaving the polygons below it. How would we know without a picture whether a solution is curving up or curving down? The mathematical concept corresponding to curving up is convexity: a function y( x ) is called convex on an interval if y ( x ) > 0 on that interval. Curving down is called concavity; the corresponding condition is y ( x ) < 0. Any tangent to a convex function at a point lies below the function at that point. Thus, intuitively, if y ( x0 ) > 0, Eulers estimate at x1 is likely to be too low. Similarly, if y ( x0 ) < 0, it is likely to be too high. Let us do a worked example. Example. Well use Eulers method to estimate the value at x = 1.5 of the solution to y = f ( x, y) = y2 x2 with y(0) = 1, using h = 0.5. We get the table:

Errors In Eulers Method n 0 1 2 3 xn 0 0.5 1.0 1.5 yn -1 -0.5 -0.5 -0.875 f ( xn , yn ) 1 0 -0.75 h f ( xn , yn ) 0.5 0 -0.375

OCW 18.03SC

We want to use the intuition developed in the paragraph above; how can we compute y ? Differentiate both sides of the equation y = y2 x2 dy2 with respect to x and use the chain rule for the term . to get: dx y = 2yy 2x Thus y (0) = 2(1)(1) 0 = 2. This means that the estimate is likely to be too large. First: y = x 1 with slope 1, so y = y2 x2 = ( x 1)2 x2 = 2x + 1. When x is in the interval [0, 0.5] we have 2x + 1 1. Second: y = 0.5 with slope zero, so y = y2 x2 = 0.25 x2 . When x is in [0.5, 1], this is nonpositive. Third: y = 0.75x + 0.25 with slope 0.75, so y = y2 x2 = 0.4375 x2 0.375x + 0.0625. We would like to compare this with -0.75 in the interval [1, 1.5]. At x = 1, we have equality, so it sufces to show that the rst derivative of y on the segment, or y = 0.875x 0.375, is nonpositive for x in [1, 1.5], and it is.

3. Two cautionary tales


3.1. Dramatic overshoot In the third example in our Euler Applet exploration we looked at the DE y = y2 x starting at (0.98, 0). The actual solution was drastically different from any of the Euler polygons. This phenomenon is best explained in the language graphical solutions. There is a separatrix for y = y2 x that passes just above (-0.98,0); you can test this by playing around with the applet a little. The integral curve through (-0.98,0) remains below the separatrix; however, all the Euler polygons cross it, and subsequently behave like integral curves from the other side of the separatrix. Depending on the initial point, this sort of phenomenon can be very difcult to avoid; one strategy can be to rst study the equation using geometric techniques.

Errors In Eulers Method

OCW 18.03SC

3.2.

Divergent estimates

Consider the IVP y = y2 , y(0) = 1. Let us try to estimate y(1) using Eulers method. For h = 0.2, we get n xn yn n xn yn 0 0 1 1 0.2 1.2 2 0.4 1.49 3 0.6 1.93 4 0.8 2.68 5 1.0 4.11 (We omit the columns with f ( xn , yn ) and f ( xn , yn )h.) For smaller step sizes, we get the following estimates: h Estimate for y(1) 0.1 37.6 0.05 91.25 0.02 238.21 What is going on? We can actually solve this equation explicitly, for instance with the separation of variables method of session one. The solution is: y( x ) = 1/(1 x ). This is not dened for x = 1: as x 1 , y +. The lesson is that in practice, one should never simply choose a step size and accept the answer. You should try smaller and smaller h until the answer settles down. If it does, you have one good bit of evidence to accept the approximation; if it doesnt, the method has failed. The computer does not eliminate the need to think!

Further Numerical Methods


Eulers method is a rst order method (no relation to rst order equations). It is possible to show theoretically that for small enough h, the error in Eulers method is at most C1 h, where C1 is a constant that depends on the IVP. It is very hard to know ahead of time what C1 will be. In the previous section, we saw that making h smaller was a way to decrease the error caused by the variability of the direction eld. However, there are some more sophisticated methods that turn out to be even better.

1. General Approach
Looked at broadly Eulers method is a way of stepping discretely from one point to the next to approximate the integral curve. The general formula for stepping from ( xn , yn ) to ( xn+1 , yn+1 ) is x n +1 = x n + h , yn+1 = yn + mh,

where h is the stepsize in the x direction and m is the slope of the line we step along. In Eulers method h is xed ahead of time and m = f ( xn , yn ). (It would be more precise to write mn instead of m. Well use the simpler looking notation, with the understanding that m changes with each step.) Other methods use other (and better) ways of choosing h and m. We start with some xed stepsize methods. As the name suggests, we x the stepsize h ahead of time and put all the work into nding m

2. The Improved Euler method


This is also called the Runge-Kutta 2 method or RK2, or the Heun method. We start with the same data as for Eulers method: an initial value problem y = f ( x, y), y( x0 ) = x0 , and a step size h. We construct an RK2 polygon, made out of segments called RK2 struts, with endpoints ( xn , yn ). As before, xn+1 = xn + h. The difference between RK2 and Eulers method is the rule for choosing the slope m for each strut. At each step we start by constructing the Euler strut. We let m be the average of the slope eld at the two ends of the strut. Example. Consider the differential equation y = f ( x, y) = y2 x with initial condition y(0) = 1. Let us compute one step for the RK2 polygon with h = 1/2. Because m, x and y are reserved well use the letters k, a and b for intermediate slopes and points.)

Further Numerical Methods

OCW 18.03SC

1. Compute the slope at ( x0 , y0 ): k1 = f (0, 1) = 1. 2. Take an Euler step from ( x0 , y0 ) to ( a, b): a = x0 + h = .5, b = y0 + k1 h = .5. 3. Compute the slope at ( a, b): k2 = f ( a, b) = f (.5, .5) = .25. 4. Average k1 and k2 to get m: m = (k1 + k2 )/2 = .375. 5. Use m and h to take step from ( x0 , y0 ) to ( x1 , y1 ): x1 = x0 + h = .5, y1 = y0 + mh = .8125. You can check, e.g. by using the applet, that this brings us down closer to the actual solution curve than Eulers method. RK2 is a second order method: for small enough h, the error is at most C2 h2 , where the constant C2 depends on the IVP. Each evaluation of the direction eld takes time, which usually costs money. Eulers method uses one evaluation per step, whereas RK2 uses two; therefore, if we want to compare efciencies, we should compare Eulers method with step size h to RK2 with 2h. In those cases, the error for Eulers method is around C1 h, whereas it is around C2 (2h)2 = 4C2 h2 for RK2. Even if C2 is larger than C1 , for small enough h, the RK2 error will be signicantly smaller than the Euler error. Besides, C2 is usually smaller than C1 , which gives a second advantage to using RK2 over Eulers method.

3. Runge-Kutta 4 method
This is usually shortened to RK4. It is a renement of RK2; we start with the same data, and also build a polygon, whose segments are called RK4 struts. Again, at each step, the difference is in choosing the slope of the segment. In RK4 you evaluate the direction eld slope four times for each step. We wont give the details, they are easy enough to look up. Remark 1. While its straightforward to compute by hand, most people leave the computations in RK4 to a computer. Remark 2. You might have noticed a pattern in the numbering of the Runge-Kutta techniques; Eulers method is sometimes referred to as RK1. RK4 is a fourth order method. For small enough h, its error is approximately C4 h4 . Again, the constant C4 depends on the IVP. It is fair to compare the errors for Eulers method with step size h, RK2 with step size 2h, and RK4 with 4h. Regardless of the values of C1 , C2 and C4 , for sufciently small h, the RK4 error of C4 (4h)4 will be signicantly less 2

Further Numerical Methods

OCW 18.03SC

than the RK2 error of C2 (2h)2 or the Euler error of C1 h. Besides, C4 itself is usually smaller than C2 and C1 . Example. Let us go back to our original problem: estimating e by viewing it as the value at 1 of the solution to the initial value problem y = y, y(0) = 1. We compare the errors of our three methods. In all cases, we use 1000 evaluations of the direction eld. Method Step size Error RK1 = Euler 0.001 1.3 103 RK2 = Heun 0.002 1.8 106 RK4 0.004 5.8 1012 We can also estimate the constants Ci for this particular IVP: C1 1.3; C2 0.45; C4 0.023. The (short) moral is that Eulers method often offers poor precision, and that RK4 is essentially always the most accurate. As you might have guessed, there are plenty of methods of higher order still; however, they also involve more overhead. Experience has shown that RK4 is a good compromise.
Remark. The x initial value problem y = f ( x ) , y( a) = y0 has solution y( x ) = y0 + a f (t)dt Our numerical methods for approximating y(x) correspond to integration approximation techniques:

Eulers method gives the the left end-point Riemann sum; RK2 gives the trapezoidal rule; RK4 gives Simpsons rule.

4. Variable Stepsize methods


We saw that it is not wise to pick a single stepsize and accept the results of the Euler method. Likewise RK2 and RK4 can be fooled. With the xed stepsize methods you need to choose a value of h; do your computation; then redo it using stepsize h/2. You keep cutting the stepsize in half this until the answers stop changing. The variable stepsize techniques carry this out at each step. There are an enormous number of such methods. What they have in common is estimating at each step whether the stepsize needs to be made smaller or can safely be made larger. In general, these provide the most accurate numerical methods at an acceptable cost in additional computation. 3

First Order Linear ODEs: Introduction


Linear equations are the most basic and probably the most important class of differential equations. They will be the main focus of this course. In this session we will introduce rst order linear ordinary differential equations. Thats a long name, so we will typically shorten it to rst order linear equations or rst order linear ODEs. In later sessions we will look at higher order linear equations. After dening rst order linear ODEs, we will spend some time introducing the terminology that is used to describe them. In particular, we will borrow the language of systems and signals from engineering. Along the way we will see some examples of physical systems modeled by rst order linear equations. Finally, we will state and prove the superposition principle. This principle is not difcult, but it is very important. We will show you several examples of superposition and remind you of its importance several times in this session.

First order Linear Differential Equations


To start we will dene rst order linear equations by their form. Soon, we will understand them by their properties. In particular, you should be on the lookout for the statement of the superposition principle and in later sessions a conceptual denition of linearity. Denition. The general rst order linear ODE in the unknown function x = x (t) has the form: A(t) dx + B ( t ) x ( t ) = C ( t ). dt (1)

As long as A(t) = 0 we can simplify the equation by dividing by A(t). dx + p(t) x (t) = q(t) dt Well call (2) the standard form for a rst order linear ODE. (2)

1. Terminology and Notation


The functions A(t), B(t) in (1), and p(t) in (2), are called the coefcients of the ODE. If A and B (or p) are constants (i.e. do not depend on the variable t) we say the equation is a constant coefcient DE. We use the familiar notations y or y for the derivative of y. With some . dy exceptions, well use y = dt to mean the derivative with respect to time dy and y for derivatives with respect to some other variable, e.g. y = dx . If there is any danger of confusion well revert to the unambiguous Liebnitz dy dy notatiation: dt , dx , etc.

2. Homogeneous/Inhomogeneous
If C (t) = 0 in (1) the resulting equation: A(t) x + B(t) x = 0 is called homogeneous1 . Likewise, in standard form, x + p(t) x = 0 is homogeneous. Otherwise the equation is inhomogeneous.
The syllable ge has a long e and is stressed in homogeneous, while the syllable mo is stressed in homogenous.
1 Homogeneous is not the same as homogenous (or homogenized).

First order Linear Differential Equations

OCW 18.03SC

3. Examples
We will give two examples where we construct models that give rst order linear ODEs. Example 1. In session 1 we modeled an oryx population x with natural growth rate k and harvest rate h: x = kx h, or x kx = h.

Fig. 1. Oryx. Image courtesy of Cape Town Craig on ickr. We repeat the argument leading to this model. We start with the population x (t) at time t. A natural growth rate k means that after a short time t we would expect there to be approximately kx (t)t more oryx. However, in that same time ht oryx are harvested. So we have the net change in the oryx population: x kx (t)t ht

x kx (t) h. t

dx dt

Now, letting the time interval t approach 0 we get the ODE = kx (t) h.

Note: if the rates k and h are not constant, but vary with time, the modeling process will lead to the same differential equation: dx = k(t) x (t) h(t) dt or dx k ( t ) x ( t ) = h ( t ). dt

Example 2. (Bank account) I have a bank account. It has x (t) dollars in it, i.e., x is a function of time. I can deposit money in the account and make withdrawals from it. The bank pays me interest for the money in my account. We will call the interest rate r, it has units of (year)1 . 2

First order Linear Differential Equations

OCW 18.03SC

In the old days a bank would pay interest at the end of the month on the balance at the beginning of the month. We can model this mathematically. With t = 1/12, the statement at the end of the month will read: x (t + t) = x (t) + rx (t)t + [deposits withdrawals between t and t + t]. These days r is typically very small, say 1%/year = 0.01/year. And, you dont get 1% each month! You get 1/12 of that. You can think of a withdrawal as a negative deposit, so I will call everything a deposit and allow the sign to positive or negative. Nowadays interest is usually computed daily. This is a step on the path to the enlightenment afforded by calculus, in which t 0 and the interest is computed continuously. In order to reach enlightenment, I want to record deposits minus withdrawals as a rate, in dollars per year. Suppose I contribute $100 sometime every month, and make no withdrawals. My total deposits up to time t, that is, my cumulative total deposit Q(t) has a graph like the following gure.
Q 400 300 200 100
1 12 2 12 3 12 4 12 5 12

Fig. 2. With periodic deposits Q(t) is a step function. In keeping with letting t 0, we should imagine that I am making this contribution continually at the constant rate of $1200/year. Then the graph of Q(t) is a straight line with slope 1200, shown in gure below. In this case, the derivative Q (t) = q(t) is constant.
Q 400 300 200 100
1 12 2 12 3 12 4 12 5 12

Fig. 3. With continuous deposits the graph of Q(t) is a straight line. 3

First order Linear Differential Equations

OCW 18.03SC

In general, say I deposit at the rate of q(t) dollars per year. The value of q(t) might vary over time, and might be negative from time to time, because, with our convention, withdrawals are merely negative deposits. So, (assuming q(t) is continuous), x (t + t) x (t) + rx (t)t + q(t)t. Now subtract x (t) and divide by t: x (t + t) x (t) rx + q t Next, let the interest period t tend to zero: x = rx + q. Note: q(t) can certainly vary in time. The interest rate can too. In fact the interest rate might depend upon x as well: a larger account will probably earn a better interest rate. Neither feature affects the derivation of this equation, but if r does depend upon x as well as t , then the equation we are looking at is no longer linear. So, for this example, lets say r = r (t) and q = q ( t ). We can put the linear ODE into standard form: x r ( t ) x = q ( t ).

Is is Linear?
Quiz: Is it Linear? We will develop a theory of linear equations, complete with an algorithm for solving them. Its important to recognize them when you see them. 1. 2. 3. Which of the following are linear ODEs? . x + x2 = t . x = (t2 + 1)( x 1) . x + x = t2

Choices: a) None b) (1) only c) (2) only d) (3) only e) All f) All but (1) g) All but (2) h) All but (3) Answer: Equations (2) and (3) are linear; (1) is not: the correct answer is (f).

Is is Linear?
Quiz: Is it Linear? We will develop a theory of linear equations, complete with an algorithm for solving them. Its important to recognize them when you see them. 1. 2. 3. Which of the following are linear ODEs? . x + x2 = t . x = (t2 + 1)( x 1) . x + x = t2

Choices: a) None b) (1) only c) (2) only d) (3) only e) All f) All but (1) g) All but (2) h) All but (3) Pick what you think is the correct choice and then look at the answer.

Is is Linear?
Quiz: Is it Linear? We will develop a theory of linear equations, complete with an algorithm for solving them. Its important to recognize them when you see them. 1. 2. 3. Which of the following are linear ODEs? . x + x2 = t . x = (t2 + 1)( x 1) . x + x = t2 Think about your answer and then look at the choices.

Interpret the Graph


Quiz: Interpret the Graph Referring to the bank account example, at the point A on the graph of Q(t), what sort of transaction am I making?
Q t

Choices: a) deposits b) withdrawals Answer: (a), deposits. When the slope is positive Q is increasing; i.e. I am making deposits.

Solutions by Integrating Factors: Introduction


Our goal in this session is to derive formulas for solving both homogeneous and inhomogeneous rst order linear ODEs. For the inhomogeneous equations we will use what are called integrating factors. The method of integrating factors is a beautiful technique for solving the general rst order linear equations as well as some other types of DEs. In later sessions we will learn other techniques that are easier and apply to higher order equations, but only apply in specic cases. The main example in this session will be about the diffusion of heat. This is an interesting physical application and we will return to it several times in the course. Finally we will connect the formula for the solution to the inhomogeneous equation with the superposition principle. The main point here will be to write the general solution to the inhomogeneous equation as the sum of a particular solution to that equation and the general solution to the corresponding homogeneous equation. In symbols, x = x p + C xh . A word of advice: pay special attention to what we mean by the phrase particular solution. The meaning differs slightly from the standard English sense and can be a little confusing at rst.

Solutions to Linear First Order ODEs 1. First Order Linear Equations


In the previous session we learned that a rst order linear inhomogeneous ODE for the unknown function x = x (t), has the standard form x + p ( t ) x = q ( t ). (To be precise we should require q(t) is not identically 0.) We saw a bank example where q(t), the rate money was deposited in the account, was called the input signal. We also saw an RC circuit example where the input signal was the voltage V (t) and q(t) = V (t). A rst order linear homogeneous ODE for x = x (t) has the standard form x + p(t) x = 0.

(1)

(2)

We will call this the associated homogeneous equation to the inhomogeneous equation (1) In (2) the input signal is identically 0. We will call this the null signal. It corresponds to letting the system evolve in isolation without any external disturbance. In the bank example: if there are no deposits and no withdrawals the input is 0. In the RC circuit example: if the power source is turned off and not providing any voltage increase then the input is 0.

2. Solutions to the Homogeneous Equations


The homogeneous linear equation (2) is separable. We can nd the solution as follows: Separate variables: Integrate: ln | x | = dx = p(t)dt. x

p(t)dt + c1 . (We use c1 to save C for later.)

p(t)dt .

Exponentiate:

| x | = e c1 e

Solutions to Linear First Order ODEs Rename ec1 as C:

OCW 18.03SC

p(t)dt ;

| x | = C e

C > 0.

Drop the absolute value and recover the lost solution x (t) = 0: This gives the general solution to (2) x (t) = C e

p(t)dt

where

C = any value.

(3)

A useful notation is to choose one specic solution to equation (2) and call it xh (t). Then the solution (3) shows the general solution to the equation is x (t) = Cxh (t). (4) There is a subtle point here: formula (4) requires us to choose one solution to name xh , but it doesnt matter which one we choose. We can say this somewhat awkwardly as choose an arbitrary specic solution. A typical choice is to set the parameter C = 1, but this is not necessary. Example. Solve x + 2tx = 0. Solution. Separate variables: Integrate: dx = 2tdt. x

ln | x | =

2tdt = t2 + c1 .

Exponentiate and substitute C for ec1 :

| x | = e c1 e t = C e t .
x ( t ) = C et .
2

Drop the absolute value and also recover the lost solution:
2

In this example an obvious choice for xh is xh (t) = et . It is clear the general solution to the example is x (t) = C xh (t) where C = any number.

3. Solution to Inhomogeneous DEs Using Integrating Factors


We start with the integrating factors formula: the general solution to the . inhomogeneous rst order linear ODE (1) (x + p(t) x = q(t)) is 1 x (t) = u(t)q(t)dt + C , where u(t) = e p(t) dt . (5) u(t) 2

Solutions to Linear First Order ODEs

OCW 18.03SC

The function u is called an integrating factor. This method, due to Euler, is easy to apply. We deduce it by the method of optimism, i.e., we introduce an integrating factor u and hope that it will help us. Proof: We start with the product rule for differentiation

. . d (ux ) = ux + ux. dt
and the equation (1): x + p ( t ) x = q ( t ). Multiply both sides of the equation by some function u(t), whose value we will determine later: . ux + upx = uq. (6) In order to be able to apply the product rule we want the sum on the left . . d hand side of the equation to have the form dt (ux ) = ux + ux. There may be many functions u for which the left hand side has this form; we only need to nd one of them. To do this, note that

. . . . . d (ux ) = ux + upx ux + ux = ux + upx u = up. dt The last equation is a separable DE for the unknown function u:
du = p(t) dt u and so: ln |u| =

p(t) dt
p dt

u=e

(7)

Remember, we are looking for just one u, so any choice of anti-derivative of p(t) in equation (7) will do. Now replace the left-hand side of (6) by ux + upx = uq d (ux ) = uq dt u(t) x (t) =
d dt ( ux )

and solve for x:

u(t)q(t)dt + c 1 x (t) = u(t)q(t)dt + c u(t) 3

Solutions to Linear First Order ODEs

OCW 18.03SC

This last equation is exactly the formula (5) we want to prove. Example. factors. Solve the ODE x + 2x = e3t using the method of integrating

Solution. Until you are sure you can rederive (5) in every case it is worthwhile practicing the method of integrating factors on the given differential equation. (At the end, we will model a solution that just plugs into (5).) Multiply both sides by u: ux + 2u(t) x (t) = u(t) e3t . Next, nd an integrating factor u so that the left-hand side is equal to . . (which equals ux + ux). ux + ux = ux + 2ux

(8)
d dt ( ux )

u = 2u u ( t ) = e 2t (we choose any one u that works).


d dt ( ux )

Now substitute u(t) = e2t into (8), then replace the left-hand side by and solve for x. d 2t ( e x ) = e 2t e 3t dt 1 e2t x = e5t + C (integrate the previous equation) 5 1 x (t) = e3t + Ce2t (solve for x (t)). 5 Here is a model of the same solution using (5) directly. Integrating factor: u(t) = e Solution:

2 dt

= e 2t

(choose any one possibility).

1 x (t) = u(t)

u(t)e3t dt e5t dt 1 5t e +C 5

=e =e

2t 2t

1 = e3t + Ce2t . 5 4

Solutions to Linear First Order ODEs

OCW 18.03SC

4. Comparing the Integrating Factor u and xh


Recall that in section 2 we xed one solution to the homogeneous equation (2) and called it xh . The formula for xh is xh (t) = e

p(t) dt

where we can pick any one choice for the antiderivative. Comparing this with the formula for the integrating factor u=e

p(t)dt

we get the following relationship between the two functions: xh (t) = 1 . u(t)

The solution to the homogeneous equation (or for short the homogeneous solution) xh will play an extremely prominent role in the rest of the course.

Example: Heat Diffusion


Example. Heat Diffusion Here we will model heat diffusion with a rst order linear ODE. We will solve the DE using the method of integrating factors. Every summer I put my root beer in a cooler, but after a while it still gets warm. Lets model its temperature by an ODE. First we need to name the function that measures the temperature: x (t) = root beer temperature at time t. The greater the temperature difference between inside and outside, the faster x (t) changes. The simplest (linear) model of this is: x (t) = k ( Text (t) x (t)), where k > 0 and Text (t) is the external temperature. One check that this makes sense (with k > 0) is to note that when the outside temperature Text is greater than the inside temperature x (t), then . x (t) > 0 (since k > 0), so the temperature is increasing. Likewise, Text < x (t) x (t) < 0 the temperature is decreasing. This is indeed how heat behaves! Rearranging the DE, we get the linear equation in standard form: x kx = kText (t).

(1)

This is Newtons law of cooling; k could depend upon t and we would still have a linear equation, but lets suppose that we are not watching the process for so long that the insulation of the cooler starts to break down! Systems and signals analysis: The system is the cooler. The input signal is the external temperature Text (t). The output signal or system response is x (t), the temperature inside the cooler.

Example: Heat Diffusion

OCW 18.03SC

Note that the right-hand side of equation (1) is k times the input signal, not the input signal itself. As usual, what constitutes the input and output signals is a matter of the interpretation of the equation, not of the equation itself. To take a specic example, let x (0) = 32 F, k= 1 3 and Text (t) = 60 + 6t in F,

where t denotes hours after 10AM. (That is, outside temperature is rising linearly.) We get the following differential equation and initial value:

. 1 x + x = 20 + 2t, 3

x (0) = 32.

(2)

Solution. Again, until you can do it every time you should practice rederiving the integrating factors formula. Here we will use it directly. Integrating factor: u(t) = e 3 dt = e 3 t (choose any one possibility). Solution: 1 x (t) = u(t) (20 + 2t) dt + C u(t) t/3 t/3 =e e (20 + 2t) dt + C

1 1

= et/3 (60e 3 t + 6te 3 t 18e 3 t + C )


x (t) = 60 + 6t 18 + Ce
1 3

(using integration by parts)

= 42 + 6t + Ce

1 3t

All thats left is to use the initial condition to nd C. We plug in t = 0, x (0) = 32 and solve for C. 32 = x (0) = 42 + C

C = 10.

The equation describing the temperature inside my cooler is: x (t) = 42 + 6t 10et/3 . We can use this to nd how long it will take for my root beer to reach 60 F. (I dont like it any warmer than that.) We need to solve 42 + 6t 10et/3 = 60. 2

Example: Heat Diffusion

OCW 18.03SC

It is probably easiest to graph this function and read the correct value off the graph.

60 45 30 15 t

3.52

Fig. 1. I have roughly 3.5 hours to enjoy my root beer. Remark: At this point the method of integrating factors is the only technique we have to solve this problem. For many problems it is the only technique, but as mentioned in the session introduction, we will eventually learn easier methods that work in this case.

The Meaning of k
Quiz: The meaning of k. In the root beer cooling example the DE was: x (t) = k ( Text (t) x (t)). What does it mean for k to be large? Choices: 1. good insulation 2. bad insulation 3. nothing to do with insulation Answer: When the insulation is good, k is small; when the insulation is bad k is large. When the insulation is perfect k is zero. k is a coupling constant; when it is zero, the temperature inside the cooler is decoupled from the temperature outside. In the construction industry a number like k is pasted on windows; its called the U-value of the window.

Units
Quiz: Units Let x (t) be the temperature of my house in degrees Celsius with t in hours. Suppose it satises the ODE: dx + kx = kTe (t). dt 1. What are the units on k? 2. What are the units on Te ? Choices: 1. Units on k: a)
degrees hour

b) degees Celsius

c)

1 hour

d) k is dimensionless

2. Units on Te : a)
degrees hour

b) degrees Celsius

c)

1 hour

d) Te is dimensionless

Answer:
1 1. The units on k are hour : Since x is in degrees Celsius and t has units degrees degrees dx in hours, dt has units hour . Thus, kx has units hour , which implies 1 k has units hour .

2. The units on Te are degrees Celsius: From the equation we see that Te has the same units as x.

Units
Quiz: Units Let x (t) be the temperature of my house in degrees Celsius with t in hours. Suppose it satises the ODE: dx + kx = kTe (t). dt 1. What are the units on k? 2. What are the units on Te ? Choices: 1. Units on k: a)
degrees hour

b) degees Celsius

c)

1 hour

d) k is dimensionless

2. Units on Te : a)
degrees hour

b) degrees Celsius

c)

1 hour

d) Te is dimensionless

Pick what you think is the correct choice and then look at the answer.

Units
Quiz: Units Let x (t) be the temperature of my house in degrees Celsius with t in hours. Suppose it satises the ODE: dx + kx = kTe (t). dt 1. What are the units on k? 2. What are the units on Te ? Think about your answer and then look at the choices.

Superposition and the Integrating Factors Solution 1. Another Proof of the Superposition Principle
The superposition principle is so important a concept that it is worth reviewing yet again. Here we will use the integrating factors formula for the solution to rst order linear ODEs to give another simple proof of this principle. Recall, the standard rst order linear ODE is x + p ( t ) x ( t ) = q ( t ). We derived the integrating factors solution 1 x (t) = u(t)q(t) dt + c , where u(t) = e u(t)

(1)

p(t) dt

(2)

and where the integral is any specic choice of the antiderivative and c is the constant of integration. The superposition principle says that if: . x1 is a solution to x + p(t) x (t) = q1 (t) and . x2 is a solution to x + p(t) x (t) = q2 (t) then for any constants a and b, ax1 + bx2 is a solution to . x + p(t) x (t) = aq1 (t) + bq2 (t). More briey, we can write q1 x1 and q2 x2 aq1 + bq2 ax1 + bx2 . (3)

To provide another way of thinking about this key principle, well rephrase it again in physical terms. If equation (1) models a physical situation and we consider q(t) to be the input then the principle shown in (3) says superposition of inputs leads to superposition of outputs. In fact, the proof takes only a few lines. Given the separate inputs q1 (t) and q2 (t), formula (2) gives the separate outputs 1 1 x1 ( t ) = u(t)q1 (t) dt + c1 and x2 (t) = u(t)q2 (t) dt + c2 u(t) u(t) Now we use (2) to nd the output for input q = aq1 + bq2 . We will be able to choose any constant of integration, so, ahead of time, we choose

Superposition and the Integrating Factors Solution

OCW 18.03SC

the constant of integration to be of the form c1 + c2 . Using the standard properties of integrals, the output is then 1 x (t) = u(t)( aq1 (t) + bq2 (t)) dt + c1 + c2 u(t) a b = u(t)q1 (t) dt + c1 + u(t)q2 (t) dt + c2 u(t) u(t) = ax1 (t) + ax2 (t) (which is what needed to be proved).

2. General = Particular + Homogeneous


In the general solution (2). we made a specic choice of the integral. By setting c = 0 this leads to a specic choice of the solution 1 xp = u(t)

u(t)q(t) dt.

We call x p a particular solution, but this is a very poor name because there is nothing particularly particular about it. It is simply one specic solution. We could have chosen any other. In the rst note of this session we saw that the solution to the homogeneous equation (i.e., when q(t) 0). is related to the integrating factor u by xh (t) = 1/u(t). Using x p and xh we can rewrite the general solution (2) as x (t) = x p (t) + cxh (t). This tells us something interesting: one way to fully solve the inhomogeneous equation (1) is to rst solve the homogeneous equation and then nd any one solution, i.e., a particular solution, to the inhomogeneous equation. We can use any method we want to nd x p . One method is the method of integrating factors, but for many equations we will have easier methods. Example 1. an x p .
2 Find the general solution to x + 1 t x = t by nding an x h and

Solution. First we nd xh . The associated homogeneous equation is x + 1 t x = 0. This is separable and we easily solve it as follows. Separate variables: Integrate: Set c = 0, drop absolute values and exponentiate: 2 ln | x | xh (t)
dx x

= 1 t dt = ln |t| + c =1 t.

Superposition and the Integrating Factors Solution

OCW 18.03SC

Next, we use an integrating factor to nd x p . Formula (2) says u(t) = e 1/t dt = eln(t) = t. (Of course, we knew this since u = 1/ xh .) Thus, (again arbitrarily choosing the constant of integration to be 0) x p (t) = 1 u(t)

u(t)t2 dt =

1 t

t3 dt =

t3 . 4

The general solution to the problem is therefore x (t) = x p (t) + cxh (t) = t3 c + . 4 t (4)

Notice, if we were a computer that didnt know any better, we might have 3 chosen a different x p (t), say x p (t) = t4 + 1 t . We know this is a solution to our DE and so it has every right to be called a particular solution. In this case we would write our general solution as 3 t 1 c x (t) = x p (t) + cxh (t) = + + . (5) 4 t t Equations (4) and (5) are both valid as general solutions. This is because both equations really represent a whole family of solutions (you get a different family member for each value of c) and each family contains the same set of solutions. For example, we get the same solution if we take c = 5 in equation (4) or if we take c = 4 in equation (5). Example 2. Find the general solution to x + 2x = 4.

Solution. The associated homogeneous equation is x + 2x = 0. This models exponential decay and has a solution xh (t) = e2t . Well use the method of optimism to nd a particular solution. Since the right-hand side is a constant we guess a constant solution. By inspection we see that x p (t) = 2 is one solution. Combining the homogeneous and particular solutions, we get that the general solution is x (t) = 2 + ce2t . Example 3. Use the superposition principle to explain why x (t) = x p (t) + cxh (t) is a solution to (1). Solution. Lets use the language of inputs/outputs and call the righthand side of (1) the input. The superposition principle says a superposition of inputs leads to a superposition of outputs. 3

Superposition and the Integrating Factors Solution

OCW 18.03SC

That is, since x p is a solution to the ODE with input q(t) and xh is a solution with input 0, we get the superposition x p + cxh is a solution with input q(t) + c 0 = q(t). This is exactly what we were asked to show.

Is it Particular?
Quiz: The rst order linear DE x + kx = t has general solution x (t) = t/k 1/k2 + cekt . Which of the following could be chosen as a particular solution to the DE? a. t/k 1/k2 b. t/k 1/k2 + 3ekt c. t/k 1/k2 + cekt d. ekt Choices: 1. (a) only 2. (b) only 3. (d) only 4. (a) and (b) only 5. (a), (b) and (c) only 6. All of them. Answer: (4): (a) and (b). (a) and (b) are both specic solutions so they can be particular solutions. (c) is the general solution, so it is not a particular solution. (We will accept the argument that c could be a specic constant and therefore this could be a particular solution.) (d) is a homogeneous solution not an inhomogeneous one.

Is it Particular?
Quiz: The rst order linear DE x + kx = t has general solution x (t) = t/k 1/k2 + cekt . Which of the following could be chosen as a particular solution to the DE? a. t/k 1/k2 b. t/k 1/k2 + 3ekt c. t/k 1/k2 + cekt d. ekt Choices: 1. (a) only 2. (b) only 3. (d) only 4. (a) and (b) only 5. (a), (b) and (c) only 6. All of them. Pick what you think is the correct choice and then look at the answer.

Is it Particular?
Quiz: The rst order linear DE x + kx = t has general solution x (t) = t/k 1/k2 + cekt . Which of the following could be chosen as a particular solution to the DE? a. t/k 1/k2 b. t/k 1/k2 + 3ekt c. t/k 1/k2 + cekt d. ekt Think about your answer and then look at the choices.

Complex Arithmetic and Exponentials: Introduction


Complex numbers will be a fundamental part of the toolkit for this course. Using the complex roots of polynomials and allowing exponentials with complex exponents will simplify and unify our study of constant coefcient linear ODEs. The key will be Eulers formula eit = cos(t) + i sin(t). This formula will allow us to replace trigonometric functions which have hard-to-remember, hard-to-manipulate identities by complex exponentials which have easy to remember and manipulate rules.

Complex Arithmetic 1. History


Most people think that complex numbers arose from attempts to solve quadratic equations, but actually they rst appeared in connection with cubic equations. Everyone knew that certain quadratic equations, like x2 + 1 = 0 or x2 + 2 x + 5 = 0

had no solutions. The problem was with certain cubic equations, for example x3 6x + 2 = 0. This equation was known to have three real roots, given by simple combinations of the expressions 3 3 A = 1 + 7 B = 1 7. (1) For instance, one of the roots is A + B; it may not look like a real number, but it turns out to be one. What was to be made of the expressions A and B? They were viewed as some sort of imaginary numbers which had no meaning in themselves, but which were useful as intermediate steps in calculations which would ultimately lead to the real numbers you were looking for (such as A + B). This point of view persisted for several hundred years. But as more and more applications for these imaginary numbers were found, they gradually began to be accepted as valid numbers in their own rights, even though they did not measure the length of any line segment. Nowadays we are fairly generous in our use of the word number: numbers of one sort or another dont have to measure anything, but to merit the name they must belong to a system in which some type of addition, subtraction, multiplication, and division is possible, and where these operations obey those laws of arithmetic one learns in elementary school and has usually forgotten by high school the commutative, associative, and distributive laws.

2. Denitions
ing To describe the complex numbers, we use a formal symbol i represent1 ; then a complex number is an expression of the form: a + ib a, b real numbers. (2)

Complex Arithmetic

OCW 18.03SC

If a = 0 or b = 0, they are omitted (unless both are 0); thus we write a + i 0 = a, 0 + ib = ib, 0 + i0 = 0

The denition of equality between two complex numbers is a + ib = c + id

a = c; b = d

( a, b, c, d real numbers).

(3)

This shows that the numbers a and b are uniquely determined once the complex number a + ib is given; we call them respectively the real and imaginary parts of a + ib. (It might seem logical to call ib the imaginary part, but this would be less convenient.) In symbols, a = Re( a + ib) b = Im( a + ib) (4)

Addition and multiplication of complex numbers are dened in the familiar way, making use of the fact that i2 = 1: Addition

( a + ib) + (c + id) = ( a + c) + i (b + d)
Multiplication

(5)

( a + ib)(c + id) = ( ac bd) + i ( ad + bc).

(6)

Division is a little more complicated; what is important is not so much the nal formula as the procedure that produces it; assuming c + id = 0, it is: Division a + ib a + ib c id ac + bd bc ad = = 2 +i 2 . (7) c + id c + id c id c + d2 c + d2 This division prodcedure made use of complex conjugation: if z = a + ib, we dene the complex conjugate of z to be the complex number z = a ib (note that zz = a2 + b2 ). (8)

The size of a complex number is measured by its absolute value, or modulus, dened by: |z| = | a + ib| = a2 + b2 ; (thus: zz = |z|2 ). (9)

Complex Arithmetic Examples


In the following we let z = 2 + 3i and w = 4 + 5i.

1. Real and Imaginary Parts


Re(z) = 2, Im(z) = 3, Re(w) = 4, Im(w) = 5.

Note: the imaginary part does not include i.

2. Addition and Subtraction

z + w = ( 2 + 3i ) + ( 4 + 5i ) = 6 + 8i z w = ( 2 + 3i ) ( 4 + 5i ) = 2 2i .

3. Multiplication
z w = (2 + 3i )(4 + 5i ) = 8 15 + i (10 + 12) = 7 + 22i.

4. Complex Conjugate and Magnitude

z = 2 + 3i = 2 3i |z| = 4 + 9 = 13 z + z = 2 + 3i + 2 3i = 4 = 2 Re(z) z z = (2 + 3i )(2 3i ) = 4 + 9 = 13 = |z|2

5. Division
Multiply numerator and denominator by the complex conjugate of the denominator: w 4 + 5i 4 + 5i 2 3i 8 + 15 + i (12 + 10) 23 2 = = = = i. 2 + 3i 2 + 3i 2 3i 13 13 13 z

Multiplication by i
Quiz: Multiplication by i. Multiplication by i has what effect on a complex number in the complex plane? Choices: a) It rotates the number around the origin by 90 degrees counterclockwise. b) It rotates the number around the origin by 90 degrees clockwise. c) It takes a number to the number pointing in the opposite direction with the same distance from the origin. d) It reects the number across the imaginary axis. e) It reects the number across the real axis. f) None of the above. g) I dont know.

Answer: Answer: (a). We compute: i ( a + bi ) = b + ai, which is rotated by 90 degrees counterclockwise.


y i b + ai a + bi i x i x y

For example, i 1 = i and i i = 1.

Multiplication by i
Quiz: Multiplication by i. Multiplication by i has what effect on a complex number in the complex plane? Choices: a) It rotates the number around the origin by 90 degrees counterclockwise. b) It rotates the number around the origin by 90 degrees clockwise. c) It takes a number to the number pointing in the opposite direction with the same distance from the origin. d) It reects the number across the imaginary axis. e) It reects the number across the real axis. f) None of the above. g) I dont know. Pick what you think is the correct choice and then look at the answer.

Multiplication by i
Quiz: Multiplication by i. Multiplication by i has what effect on a complex number in the complex plane? Think about your answer and then look at the choices.

Complex Conjugation
Quiz: Complex Conjugation. If z = z, what does that tell us about the value of z = a + bi? Choices: a) z is purely imaginary. b) z is real. c) z has length 1. d) z = 0. e) None of the above.

Answer: Answer: (a) a + bi = ( a bi ) implies a = a = 0.

Complex Conjugation
Quiz: Complex Conjugation. If z = z, what does that tell us about the value of z = a + bi? Choices: a) z is purely imaginary. b) z is real. c) z has length 1. d) z = 0. e) None of the above. Pick what you think is the correct choice and then look at the answer.

Complex Conjugation
Quiz: Complex Conjugation. If z = z, what does that tell us about the value of z = a + bi? Think about your answer and then look at the choices.

Eulers Formula, Polar Representation 1. The Complex Plane


Complex numbers are represented geometrically by points in the plane: the number a + ib is represented by the point ( a, b) in Cartesian coordinates. When the points of the plane are thought of as representing complex numbers in this way, the plane is called the complex plane. By switching to polar coordinates, we can write any non-zero complex number in an alternative form. Letting as usual x = r cos( ), y = r sin( )

we get the polar form for a non-zero complex number: assuming x + iy = 0, (1) x + iy = r (cos( ) + i sin( )). When the complex number is written in polar form, r = | x + iy| = x2 + y2 . (absolute value, modulus). We call the polar angle or the argument of x + iy. In symbols, one sometimes sees: = arg( x + iy). (polar angle, argument). The absolute value is uniquely determined by x + iy but the polar angle is not, since it can be increased by any integer multiple of 2 . (The complex number 0 has no polar angle.) To make unique, one can specify 0 < 2 . (principal value).

This so-called principal value of the angle is sometimes indicated by writing Arg( x + iy). For example, Arg(1) = , arg(1) = , 3 , 5 ,

Changing between Cartesian and polar representation of a complex number is essentially the same as changing between Cartesian and polar coordinates: the same equations are used and the same triangle appears in the plane. The gure below shows this. (You will learn what ei means in the next section.)

Eulers Formula, Polar Representation

OCW 18.03SC

y z = x + iy = rei r = |z | x y

z = x iy = rei

Solution.

Example 1. Give the polar form for: i, 1 + i, 1 i, 1 + i 3. i = i sin(3 /2) 1 + i = 2(cos( /4) + i sin( /4)) 1 + i 3 = 2(cos(2 /3) + i sin(2 /3)) 1 i = 2(cos( /4) + i sin( /4)).

Fig 1. The complex plane.

2. Eulers Formula
The abbreviation cis is sometimes used for cos( ) + i sin( ); for students of science and engineering, however, it is important to get used to the exponential form for this expression: ei = cos( ) + i sin( ) Eulers formula. (2)

Equation (2) should be regarded as the denition of the exponential of an imaginary power. A good justication for it is found in the innite series: et = 1 + t t2 t3 + + + 1! 2! 3!

If we substitute i for t in the series and collect the real and imaginary parts of the sum (remembering that i2 = 1, i3 = i, i4 = 1, i5 = i, and so on), we get: 2 4 3 5 i e = 1 + +i + 2! 4! 3! 5! = cos( ) + i sin( ) 2

Eulers Formula, Polar Representation

OCW 18.03SC

in view of the innite series representations for cos( ) and sin( ). Since we only know that the series expansion for et is valid when t is a real number, the above argument is only suggestive it is not a proof of (2). What it shows is that Eulers formula (2) is formally compatible with the series expansions for the exponential, sine, and cosine functions.

3. Polar Representation
Using the complex exponential, the polar representation (1) is written: x + iy = rei . (3)

The most important reason for polar representation is that multiplication of complex numbers is particularly simple when they are written in polar form. Indeed, by using Eulers formula (2) and the trigonometric addition formulas, it is not hard to show that e i 1 e i 2 = e i ( 1 + 2 ) . (4)

This gives another justication for the denition (2) the complex exponential follow the same exponential addition rules as the real exponential. The law (4) leads to the simple rules for multiplying and dividing complex numbers written in polar form: Multiplication Rule r 1 e i 1 r 2 e i 2 = r 1 r 2 e i ( 1 + 2 ) . (5)

To multiply two complex numbers, you multiply the absolute values and add the angles. Reciprocal Rule 1 1 = e i ; i r re Division Rule (6)

r 1 e i 1 r = 1 e i ( 1 2 ) . (7) i 2 r2 r2 e To divide by a complex number, divide by its absolute value and subtract its angle. The reciprocal rule (6) follows from (5), which shows that 1 i e rei = 1. r 3

Eulers Formula, Polar Representation

OCW 18.03SC

Using (5), we can raise x + iy to a positive integer power by rst using x + iy = rei : ( x + iy)n = r n ein ; (8) DeMoivres formula: The special case when r = 1 is called DeMoivres Formula: (cos( ) + i sin( ))n = cos(n ) + i sin(n ). (9) Example 2. Express: a) (1 + i )6 in Cartesian form; 1+i 3 b) in polar form. 3+i Solution. a) Change to polar form, use (8), then change back to Cartesian form: (1 + i )6 = ( 2ei /4 )6 = ( 2)6 ei6 /4 = 8ei3 /2 = 8i.

1+i 3 2ei /3 b) Changing to polar form, = i /6 = ei /6 , using the division 2e 3+i rule (7).
You can check the answer to (a) by applying the binomial theorem to (1 + i )6 and collecting the real and imaginary parts; to (b) by doing the division in the Cartesian form then converting the answer to polar form. 3.1. Combining pure oscillations of the same frequency. The equation which does this is widely used in physics and engineering; it can be expressed using complex numbers: where a + bi = Aei ; a cos(t) + b sin(t) = A cos(t ), in other words, A = a2 + b2 , = tan1 (b/ a). To prove (10), we have: a cos(t) + b sin(t) = Re (( a bi ) (cos(t) + i sin(t))) (10)

= Re( Aei eit )

= Re( Aei(t) ) = A cos(t ).


4

Complex Powers
Quiz: Complex Powers. Choices: a) 1 b) 4 c) 4 d) 2 e) 4i f) None of the above

(1 + i )4 = ?

Answer: In polar form 1+i = Thus,

2ei /4 .

(1 + i )4 = ( 2ei /4 )4 = 4ei = 4.

These powers all lie on a spiral emanating from the origin. The answer is (c). (Thus, (1 + i ) is a fourth root of 4.)

Complex Powers
Quiz: Complex Powers. Choices: a) 1 b) 4 c) 4 d) 2 e) 4i f) None of the above Pick what you think is the correct choice and then look at the answer.

(1 + i )4 = ?

Complex Powers
Quiz: Complex Powers.

(1 + i )4 = ?

Think about your answer and then look at the choices.

Applet Exploration: Complex Exponential


Start by opening the Complex Exponential applet . The Unit Circle 1. Set a = 0 and b = 1. As you change t notice that eibt always lies on the unit circle. What happens if you change the value of b leaving a = 0? Explain this in terms of sin and cos. 2. When you increase a from 0 what happens to the circle? Explain this by expanding e(a+bi)t .

Complex Exponentials
Because of the importance of complex exponentials in differential equations, and in science and engineering generally, we go a little further with them. Eulers formula denes the exponential to a pure imaginary power. The denition of an exponential to an arbitrary complex power is: e a+ib = e a eib = e a (cos(b) + i sin(b)). (1)

We stress that the equation (1) is a denition, not a self-evident truth, since up to now no meaning has been assigned to the left-hand side. From (1) we see that Re(e a+ib ) = e a cos(b), Im(e a+ib ) = e a sin(b). (2)

The complex exponential obeys the usual law of exponents: ez+z = ez ez ,


(3)

as is easily seen by combining (1) with the multiplication rule for complex numbers. The complex exponential is expressed in terms of the sine and cosine functions by Eulers formula. Conversely, the sine and cosine functions can be expressed in terms of complex exponentials. There are two important ways of doing this, both of which you should learn: cos( x ) = Re(eix ), 1 cos( x ) = (eix + eix ), 2 sin( x ) = Im(eix ); 1 sin( x ) = (eix eix ). 2i (4) (5)

The equations in (5) follow easily from Eulers formula; their derivation is left as an exercise. Here are some examples of their use. Example. Express cos3 ( x ) in terms of the functions cos(nx ), for suitable n. Solution. We use (5) and the binomial theorem, then (5) again: cos3 ( x ) = 1 ix (e + eix )3 8 1 = (e3ix + 3eix + 3eix + e3ix ) 8 1 3 = cos(3x ) + cos( x ). 4 4

Complex Exponentials

OCW 18.03SC

As a preliminary to the next example, we note that a function like eix = cos( x ) + i sin( x ) is a complex-valued function of the real variable x. Such a function may be written as u( x ) + iv( x ) u, v real-valued and its derivative and integral with respect to x are dened to be

a) D (u + iv) = Du + iDv From this it follows easily that

b)

(u + iv)dx =

udx + i

vdx.

(6)

D (e(a+ib)x ) = ( a + ib)e(a+ib)x , and therefore

e(a+ib)x dx =

1 e(a+ib)x . a + ib

(7)

Example. Calculate

e x cos 2x dx by using complex exponentials.

Solution. The usual method is a tricky use of two successive integration by parts. Using complex exponentials instead, the calculation is straightforward. We have e(1+2i)x = e x cos(2x ) + ie x sin(2x ), by (1) therefore by (6b) e x cos(2x ) dx = Re( e(1+2i)x dx ). Calculating the integral,

e(1+2i)x dx =

1 e ( 1 + 2i ) x 1 + 2 i 1 2 = i (e x cos(2x ) + ie x sin(2x )), 5 5

by (7)

using (1) and complex division. According to the second line above, we want the real part of this last expression. Multiply and take the real part; you get the answer

e x cos(2x ) dx =

1 x 2 e cos(2x ) + e x sin(2x ). 5 5 2

Complex Exponentials

OCW 18.03SC

In this differential equations course, we will make free use of complex exponentials in solving differential equations, and in doing formal calculations like the ones above. This is standard practice in science and engineering, and its well worth getting used to.

Complex Exponentials
Quiz: Complex Exponentials. The magnitude of e(a+bi)t is e at , and the argument of e(a+bi)t is bt. When a > 0 and b > 0, we can think of e(a+bi)t as a point in the complex plane which traces out a path as t varies. The curve in the complex plane traced out by e (1+2 i ) t most closely resembles which of the following? Choices: a) A straight ray along the positive real axis b) A circle with radius e and center at the origin c) A circle with radius 1 and center at the origin d) A spiral moving inwards and counterclockwise e) A spiral moving outwards and counterclockwise f) A spiral moving inwards and clockwise g) A spiral moving outwards and clockwise

Answer: The magnitude of e(1+2 i)t is et and the argument is 2 t, so the answer is (e).

Complex Exponentials
Quiz: Complex Exponentials. The magnitude of e(a+bi)t is e at , and the argument of e(a+bi)t is bt. When a > 0 and b > 0, we can think of e(a+bi)t as a point in the complex plane which traces out a path as t varies. The curve in the complex plane traced out by e (1+2 i ) t most closely resembles which of the following? Choices: a) A straight ray along the positive real axis b) A circle with radius e and center at the origin c) A circle with radius 1 and center at the origin d) A spiral moving inwards and counterclockwise e) A spiral moving outwards and counterclockwise f) A spiral moving inwards and clockwise g) A spiral moving outwards and clockwise Pick what you think is the correct choice and then look at the answer.

Complex Exponentials
Quiz: Complex Exponentials. The magnitude of e(a+bi)t is e at , and the argument of e(a+bi)t is bt. When a > 0 and b > 0, we can think of e(a+bi)t as a point in the complex plane which traces out a path as t varies. The curve in the complex plane traced out by e (1+2 i ) t most closely resembles which of the following? Think about your answer and then look at the choices.

Finding n-th Roots


To solve linear differential equations with constant coefcients, we need to be able to nd the real and complex roots of polynomial equations. Though a lot of this is done today with calculators and computers, one still has to know how to do an important special case by hand: nding the roots of z n = , where is a complex number, i.e., nding the n-th roots of . Polar representation will be a big help in this. Lets begin with a special case: the n-th roots of unity: the solutions to zn = 1. To solve this equation, we use polar representation for both sides, setting z = rei on the left, and using all possible polar angles on the right; using the exponential law to multiply, the above equation then becomes r n ein = 1 e(2k i) , k = 0, 1, 2, .

Equating the absolute values and the polar angles of the two sides gives r n = 1, n = 2k , k = 0, 1, 2, ,

from which we conclude that r = 1, = 2k , n k = 0, 1, , n 1. (1)

In the above, we get only the value r = 1, since r must be real and nonnegative. We dont need any integer values of k other than 0, , n 1, since they would not produce a complex number different from the above n numbers. That is, if we add an, an integer multiple of n, to k, we get the same complex number: = 2(k + an) = + 2a ; n and ei = ei ,

since e2a i = (e2 i ) a = 1.

We conclude from (1) therefore that the n-th roots of 1 are the numbers e2k i/n , k = 0, , n 1. (2)

Finding n-th Roots

OCW 18.03SC

This shows there are n complex n-th roots of unity. They all lie on the unit circle in the complex plane, since they have absolute value 1; they are evenly spaced around the unit circle, starting with the root z = 1; the angle between two consecutive roots is 2 /n. These facts are illustrated for the case n = 6 in the gure below
y

Fig. 1. The six solutions to the equation z6 = 1 lie on a unit circle in the complex plane. From (2), we get another notation for the roots of unity ( is the Greek letter zeta): the n-th roots of 1 are 1, , 2 , , n1 , where = e2 i/n . (3)

We now generalize the above to nd the n-th roots of an arbitrary complex number w. We begin by writing w in polar form: w = rei ; = Argw, 0 < 2 ,

i.e., is the principal value of the polar angle of w. Then the same reasoning as we used above shows that if z is an n-th root of w, then zn = w = rei so z = n rei( +2k )/n , k = 0, 1, , n 1. (4) Comparing this with (3), we see that these n roots can be written in the suggestive form n w = z 0 , z 0 , z 0 2 , , z 0 n 1 , where z0 = n rei /n . (5) As a check, we see that all of the n complex numbers in (5) satisfy zn = w :

Finding n-th Roots

OCW 18.03SC
n ni = zn 1i , z0 0 w,

( z0 i ) n

= =

since n = 1, by (3); by the denition (5) of z0 and (4).

Example. Find in Cartesian form all values of

a) 3 1

b)

Solution. a) According to (3), the cube roots of 1 are 1, , and 2 , where 1 3 2 i /3 =e = cos(2 /3) + i sin(2 /3) = + i 2 2 1 3 2 = e2 i/3 = cos(2 /3) + i sin(2 /3) = i . 2 2 The greek letter (omega) is traditionally used for this cube root. Note that for the polar angle of 2 we used 2 /3 rather than the equivalent angle 4 /3, in order to take advantage of the identities cos( x ) = cos( x ) sin( x ) = sin( x ).

Note that 2 = . Another way to do this problem would be to draw the position of 2 and on the unit circle and use geometry to gure out their coordinates. 4 b) To nd i, we can use (5). We know that 4 1 = 1, i, 1, i (either by drawing the unit circle picture or by using (3)). Therefore by (5), we get 4 i = z0 , z0 i , z0 , z0 i , where z0 = e i/8 = cos( /8) + i sin( /8);

= a + ib, b + ia, a ib, b ia

where z0 = a + ib = cos( /8) + i sin( /8).

Example. Solve the equation x6 2x3 + 2 = 0. Solution. Treating this as a quadratic equation in x3 , we solve the quadratic by using the quadratic formula; the two roots are 1 + i and 1 i (check this!), so the roots of the original equation satisfy either x3 = 1 + i or x3 = 1 i.

This reduces the problem to nding the cube roots of the two complex numbers 1 i. We begin by writing them in polar form: 1 + i = 2e i/4 , 1 i = 2e i/4 . (Once again, note the use of the negative polar angle for 1 i, which is more convenient for calculations.) The three cube roots of the rst of these are (by (4)), 3

Finding n-th Roots

OCW 18.03SC

6 6

2e i/12 = 2e3 i/4 =

6 6

2 (cos( /12) + i sin( /12)) 2 (cos(3 /4) + i sin(3 /4)) , since

2 3 + = ; 12 3 4 2 7 6 6 2e7 i/12 = 2 (cos(7 /12) i sin(7 /12)) , since = . 12 3 12 1 + i 1 + i 6 The second cube root can also be written as 2 = . 3 2 2 This gives three of the cube roots. The other three are the cube roots of 1 i, which may be found by replacing i by i everywhere (i.e., taking the complex conjugate). The cube roots can also be described according to (5) as 6 6 z1 , z1 , z1 2 and z2 , z2 , z2 2 where z1 = 2e i/12 , z2 = 2e i/12 .

Sinusoidal Functions: Introduction


In ways that will become clear as the course progresses sinusoidal functions play a key role. In particular, for constant coefcient equations they are the most important type of input. In this session we will use Eulers formula to gain a more detailed understanding and description of sinusoidal functions in terms of their amplitudes and phase angles. These notions will be used throughout the remainder of the course.

Sinusoidal Functions 1. Denitions


A sinusoidal function (or sinusoidal oscillation or sinusoidal signal) is one that can be wrtten in the form f (t) = A cos( t ). (1)

The function f (t) is a cosine function which has been amplied by A, shifted by / , and compressed by . A > 0 is its amplitude: how high the graph of f (t) rises above the t-axis at its maximum values; is its phase lag: the value of t for which the graph has its maximum (if = 0, the graph has the position of cos( t); if = /2, it has the position of sin( t)); = / is its time delay or time lag: how far along the t-axis the graph of cos( t) has been shifted to make the graph of (1); (to see this, write A cos( t ) = A cos( (t / ))) is its angular frequency: the number of complete oscillations f (t) makes in a time interval of length 2 ; that is, the number of radians per unit time; = /2 is the frequency of f (t): the number of complete oscillations the graph makes in a time interval of length 1; that is, the number of cycles per unit time; P = 2 / = 1/ is its period, the t-interval required for one complete oscillation. One can also write (1) using the time lag = / f (t) = A cos ( (t )) .

2. Discussion
Here are the instructions for building the graph of (1) from the graph of cos(t). First scale, or vertically stretch, cos(t) by a factor of A; then shift the

Sinusoidal Functions

OCW 18.03SC

result to the right by units (if < 0 the shift will actually be to the left); and nally scale it horizontally by a factor of 1/ . In the gure below the dotted curve is cos(t) and the solid curve is 2.5 cos( t /2). The solid curve has A = 2.5, = , = /2, = 1/2.

Vertically, the solid curve is 2.5 times the dotted one. Horizontally, the solid curve it 1/ times the dotted one. (The dotted curve takes 2 units of time to go through one cycle and the solid curve takes only 2 units of time.) The solid curve hits its rst maximum at t = 1/2, i.e. at the t = , the time lag.
amplitude = 2.5 2 1 t

-3

-2

-1 -1 -2

time lag = 1/2 one period = 2

one period = 2

Fig. 1. Features of the graph of a sinusoid.

Mystery Sinusoid
Quiz: Mystery Sinusoid
2.5 2 1.5 1 0.5 0 -0.5 -1 -1.5 -2 -2.5 -2 -1 0 1 2 3 4 5 6 7 8 t

Fig. 1. Mystery sinusoid. The graph of a sinusoidal function is displayed. The problem is to express it in the standard form f (t) = A cos( t ). Choices: a) 2 cos 4 t + 4 d) 2 cos 4t 4 b) 2 cos 4t+ 4 e) 2 cos (4t + 1) c) 2 cos 4 t 4 f) 2 cos (4t 1)

Answer: The answer is (b) The graph runs vertically between 2 and -2, so the amplitude is A = 2. There are consecutive peaks at -1 and 7, so the period P = 8. Therefore, the angular frequency = 2 / P = /4. The curve has a time lag of = 1 (see the peak at -1). Since = / , we have = = /4. Hence the equation of the sinusoid is: A cos( t ) = 2 cos t+ . 4 4

Mystery Sinusoid
Quiz: Mystery Sinusoid
2.5 2 1.5 1 0.5 0 -0.5 -1 -1.5 -2 -2.5 -2 -1 0 1 2 3 4 5 6 7 8 t

Fig. 1. Mystery sinusoid. The graph of a sinusoidal function is displayed. The problem is to express it in the standard form f (t) = A cos( t ). Choices: a) 2 cos 4 t + 4 d) 2 cos 4t 4

b) 2 cos 4t+ 4 e) 2 cos (4t + 1)

c) 2 cos 4 t 4 f) 2 cos (4t 1)

Pick what you think is the correct choice and then look at the answer.

Mystery Sinusoid
Quiz: Mystery Sinusoid
2.5 2 1.5 1 0.5 0 -0.5 -1 -1.5 -2 -2.5 -2 -1 0 1 2 3 4 5 6 7 8 t

Fig. 1. Mystery sinusoid. The graph of a sinusoidal function is displayed. The problem is to express it in the standard form f (t) = A cos( t ).

Think about your answer and then look at the choices.

Applet Exploration: Trigonometric Identity


Start by opening the Trigonometric Identity applet . This mathlet illustrates sinusoidal functions and the trigonometric identity a cos( t) + b sin( t) = A cos( t ), where a + ib = Aei .

That is, ( A, ) are the polar coordinates of ( a, b). The sinusoidal function A cos( t ) is drawn here in red. A and are the amplitude and phase lag of the sinusoid. They are both controlled by sliders. 1. The phase lag measures how many radians the sinuoid falls behind the standard sinusoid, which we take to be the cosine. So when = /2 you have the sine function. Verify this in the applet. 2. The nal parameter is , the angular frequency. High frequency means the waves come faster. Frequency zero means constant. Play with the slider and understand this statement. Return the angular frequency to 2. 3. The trigonometric identity shows the remarkable fact that the sum of any two sinuoidal functions of the same frequency is again a sinusoid of the same frequency. Use the a and b sliders to select coefcients for cos( t) and sin( t). The a slider modies the yellow cosine curve in the window at bottom and the b slider modies the blue sine curve. Notice that the sum of a cos(t) and b sin(t) is displayed in the top window in green (which is a combination of blue and yellow). There it is! - the linear combination is again sinusoidal, or at least appears to be. 4. The window at the right shows the two complex numbers a + ib and Aei . The sinusoidal identity says that the green and red sinusoids will coincide exactly when the complex numbers a + ib and Aei coincide. Verify this on the applet by pickong values of A and . and then adjusting a and b until the green and red sinusoids are the same.

The Sinusoidal Identity


The sum of two sinusoidal functions of the same frequency is another sinusoidal function with that frequency! For any real constants a and b, a cos( t) + b sin( t) = A cos( t ) where A and can be described in at least two ways: A= a2 + b2 , b = tan1 ; a (2) (3) (1)

a + bi = Aei . Conversely, we have a = A cos() and b = A sin(). Geometrically this is summarized by the triangle in the gure below.

(4)

A a

Fig. 1. a + bi = Aei . One proof of (1) is a simple application of the cosine addition formula cos( ) = cos() cos( ) + sin() sin( ). We will now give an equivalent proof using Eulers formula and complex arithmetic: The triangle in Figure 1 is the standard polar coordinates triangle. It shows a + ib = Aei or a ib = Aei . Thus A cos( t ) = Re( Aei( t) )

= Re(ei t Aei ) = Re((cos( t) + i sin( t)) ( a ib)) = Re( a cos( t) + b sin( t) + i ( a sin( t) b cos( t))) = a cos( t) + b sin( t).
We should stress the importance of the trigonometric identity (1). It shows that any linear combination of cos( t) and sin( t) is not only periodic of

The Sinusoidal Identity

OCW 18.03SC

period 2 , but is also sinusoidal. If you try to add cos( t) to sin( t) by hand, you will probably agree that this is not at all obvious.

We will call A cos( t ) amplitude-phase form and a cos( t) + b sin( t) rectangular or Cartesian form. You should be familiar with amplitudephase form; we usually prefer it because both amplitude and phase have geometric and physical meaning for us.

Amplitude-Phase Form of a Sinusoid


Quiz: Amplitude-Phase Form of a Sinusoid. Put cos ( t) + 3 sin ( t) into amplitude-phase form A cos ( t )? Choices: a) 2 cos t 4 b) 3 cos t 3 c) 2 cos t 3 d) 2 cos t 3 e) 3 cos t 3 f) 3 cos t 4 Answer: The answer is (d) because A =

12 +

3 = 2, and = tan1

3 1

= 3.

First Order Constant Coefcient Linear ODEs


In this session we start our transition to studying constant coefcient linear ODEs by looking at constant coefcient rst order equations. Here we will continue to solve them using integrating factors. Starting in the next session we will learn some simpler methods for solving constant coefcient equations. We will see that for many physical systems we can split the output (i.e. the solution to the DE) into pieces called the steady-state and the transient. This will correspond exactly to the way we have already written solutions as a particular solution plus the general homogeneous solution: x ( t ) = x p ( t ) + x h ( t ). With the steady-state solution equaling x p and the transient being xh . We have already encountered examples of constant coefcient equations when modeling exponential growth (a bank account), heat diffusion (root beer cooling) and an RC circuit. To nish the session we will add examples of radioactive decay and mixing tanks to this list.

Solution to the Constant Coefcient First Order Equation


In this section we will consider the constant coefcient equation y + ky = q(t). (Constant coefcient means k is a constant.) Solving is easy using the integrating factor u(t) = e session 5. We get the solution kt kt y=e e q(t) dt + c

k dt

(1)

= ekt from

(2) (3)

= ekt

ekt q(t) dt + cekt .

As usual, we have a particular solution and a homogeneous solution, respectively y p (t) = e


kt

ekt q(t) dt

and

yh (t) = ekt .

The general solution to (1) is then y ( t ) = y p ( t ) + c y h ( t ). The Case k > 0. If k > 0 the system in (1) models exponential decay. That is, when the input is 0 the system response is y(t) = cekt , which decays exponentially to 0 as t goes to . The term ekt
kt the transient because it goes to 0. In the general solution we call ce

ekt q(t) dt is called the steady-state or long-term solution.

That is, cyh is the transient and y p is the steady-state solution. The value of c in (2) is determined by the initial value y(0). The initial condition only affects the transient and not the long-term behavior of the solution. No matter what the initial condition, every solution goes asymptotically to the steady-state. That is, all solution curves approach the steady-state as t .

Solution to the Constant Coefcient First Order Equation

OCW 18.03SC

y' = -y - 1/(x + 1) + 1

y
Steady state Other solutions

Fig. 1. In the case k > 0 all solutions go asymptotically to the steady-state. Since all the solutions approach each other, there is no precise way to choose the one we call the steady-state. In fact, we can choose any one to be the steady-state solution. Generally, we just choose the simplest looking solution. The case k 0. When k 0 the homogeneous solution ekt does not go asymptotically to 0. In other words it is not transient. In this case it does not make sense to talk about the steady-state solution.

Examples of Constant Coefcient First order Equations


We have seen a number of physical systems that are modeled by rst order constant coefcient linear ODEs. Here we will recall some of them and introduce examples of radioactive decay and of mixing tanks. Example 1. Radioactive Decay. Suppose we have some radioactive matter with decay constant k1 . This means that the rate at which the radioactive material decays is proportional to the amount present at any given time. Let A(t) be the amount of matter at time t. The rate equation modeling this system is dA dA = k1 A(t) + k1 A = 0. dt dt The solution to this is easily seen to be A(t) = A0 ek1 t . Now, suppose that when A decays it becomes a different radioactive substance B, which has its own decay constant k2 . The rate equation for the amount B(t) of B is dB = k 1 A ( t ) k 2 B ( t ), dt i.e. dB dt = (rate B is created from A) - (rate B decays). Using the solution for A(t) = A0 ek1 t this becomes dB + k 2 B ( t ) = k 1 A0 e k 1 t . dt Example 2. Temperature-Concentration or Conduction-Diffusion. tons law of cooling gives the equation dT + kT = kTe (t). dt Here, T (t) is the temperature of a body and Te (t) is the temperature of the surrounding environment. (See the heat-diffusion example in session 5.) Example 3. Mixing Tanks. A tank containing a volume V of salt solution has solution entering the tank at rate r (in, say, liters/minute). The incoming solution has a concentration Ce (t) of salt. Solution leaves the tank at the same rate r. (This means the volume of solution in the tank is constant we say the inow and outow rates are balanced.) New-

Examples of Constant Coefcient First order Equations

OCW 18.03SC

V
r

Fig. 1. A mixing tank with balanced inow and outow. We will assume the concentration of salt stays uniform throughout the tank. If the solution is continuously stirred this is a reasonable assumption. Question. Let x (t) be the amount of salt in the tank. How do we write a differential equation modeling x (t)? Answer. The rate of change of salt in the tank is the rate salt ows in - the rate it ows out. rate in = inow rate inow concentration = r Ce (t). x rate out = outow rate ouow concentration = r V , since the outow concentration is the concentration in the tank which is x /V . Therefore, . dx x r = rCe (t) r x + x = rCe (t). (1) dt V V Notes: 1. In building the model it is best to let the dependent variable be the amount of salt and not the concentration. One good reason for this is that amounts add, but concentrations do not. For example, if I combine a solution with 2 grams of salt and one with 3 grams, I will have a solution with 5 grams of salt. But, if I combine a 2 g/liter solution with a 3 gm/liter solution, the new solution will have concentration somewhere between 2 and 3 g/liter, depending on how much of each solution is combined. 2. If we choose we can write the DE in terms of the concentration C (t) = x (t)/V . Simply divide the equation by V :

. . x r x r r r + = Ce (t) C + C (t) = Ce (t). V VV V V V


If we let k = r /V this DE becomes C + kC = kCe , which looks like the conduction-diffusion equation in example 2. 3. If the tanks are not balanced we can use the same logic to build the ODE 2

Examples of Constant Coefcient First order Equations

OCW 18.03SC

modeling the mixing tanks. It becomes the linear DE x+

r2 x = r1 Ce , V

where r1 , r2 are the inow and outow rates respectively. We can also write . r2 r1 this in terms of concentration C + C = Ce . V V Example 4. RC Circuits In session 4 we discussed a cicuit with a resistor, capacitor and input voltage. It satises the ODE R dI 1 + I = E . dt C (2)

Here, R is the resistance, C is the capacitance, E is the voltage source and I is the current through the resistor.
I R

capacitor

Fig. 2. RC circuit with input voltage E . Let q(t) be the charge on the capacitor, then I = be written in terms of q as R dq 1 + q = E ( t ). dt C
dq dt

and equation (2) can

(3)

Well take a moment to remind you that the right-hand side of the DE is not always exactly the same as the input. In both (2) and (3) we consider E to be the input to the system, but in (2) the right-hand side of the equation is E .

Response to Discontinuous Input


We will continue looking at the constant coefcient rst order linear DE y + ky = q(t). It has the integrating factors solution y = ekt ekt q(t)dt + c .

(1)

In this note we want to do an example where the input q(t) is discontinuous. The most basic discontinuous function is the unit-step function at a point a, dened by: 0 t<a u a (t) = (2) 1 t > a. (We leave its value at a undened, though some books give it the value 0 there, others the value 1 there.) Example 1. Well look again at Newtons law of cooling and my root beer cooler: . y + ky = k f (t), where, y(t) is the temperature inside the cooler and f (t) is the temperature of the air. Its a nice, cool morning with constant temperature. Suddenly the sun comes out and the air warms up to a higher constant temperature. Whats the response of my cooler to this signal? Well assume the sun comes out at time t = a, my cooler starts at t = 0 with temperature 0 and (somewhat idealized) the air temperature jumps instantly from 0 to 20 at time t = a. So f (t) = 20 u a (t) and our IVP is y + ky = k20u a (t),

y(0) = 0.

Solution. For t < a we have the input is 0. Since y(0) = 0, the response is y(t) = 0. For t a the DE becomes y + ky = 20k with y( a) = 0. The solution (which we have found before) is y(t) = 20 + cekt . Now we use the initial condition y( a) = 0 to the nd the value of c. We get c = 20eka , so y(t) = 20 20eka ekt for t a.

Response to Discontinuous Input

OCW 18.03SC

We can now assemble the results for t < a and t a into one expression; for the latter, we also put the exponent into a more suggestive form. 0 0 < t < a; input = 20u a (t) response = y(t) = k ( t a ) 20 20e t a. (3) Note that the response is just the translation a units to the right of the response to the unit-step input at 0. Our next example continues the temperature model with a different discontinuous input. In this case, the physical input is an external bath which is initially ice-water at 0 degrees, then replaced by water held at a xed temperature for a time interval, then replaced once more by ice-water. To model the input we need the unit box function on [ a, b]: 1 atb u ab = 0 a < b; (4) 0 otherwise Example 2. Find the response of the system y + ky = kq, to input q(t) = 20u ab (t). Solution. There are at least three ways to do this: a) Express u ab as a sum of unit step functions and use (3) together with superposition of inputs; b) Use the function u ab directly in a denite integral expression for the response; c) Find the response in two steps: rst use (3) to get the response y(t) for the input u a (t); this will be valid up till the point t = b. Then, to continue the response for values t > b, evaluate y(b) and nd the response for t > b to the input 0, with initial condition y(b). We will follow (c), leaving the rst two as exercises. By (3), the response to the input u a (t) is: 0 y(t) = 20 20ek(ta) 2

with IC y(0) = 0

0t<a t a.

Response to Discontinuous Input

OCW 18.03SC

This is valid up to t = b, since u ab (t) = u a (t) for t b. Evaluating at b, y(b) = 20 20ek(ba) . (5)

For t > b we have u ab = 0, so the DE is just y + ky = 0. This models exponential decay (our most important DE) and we know the solution: y(t) = cekt . (6)

We determine c from the initial value (5). Equating the initial values y(b) from (5) and (6), we get: cekb = 20 20ekb+ka from which: c = 20ekb 20eka . By (6): y(t) = 20(ekb eka )ekt , t b. (7)

After combining exponents in (7) to give an alternative form for the response we assemble the parts, getting: 0 y(t) = 20 20ek(ta) k(tb) 20ek(ta) 20e 0ta a<t<b t b.

(8)

Exponential Input; Gain and Phase Lag: Introduction


The case of sinusoidal input is of great importance in applications. A sinusoidal function is a pure oscillation like cos( t) or sin( t), or more generally, A cos( t ). (As you can see, the last form includes both of the previous two by letting A = 1 and = 0 or /2). In the temperature model, sinusoidal input could represent the diurnal (day and night) varying of outside temperature. In the concentration model it could represent the diurnal varying of the level of some hormone in the bloodstream, or the varying concentration in a sewage line of some waste product produced periodically by a manufacturing process. In this session we are going to restrict our attention to rst order constant coefcient ODEs. Before looking at sinusoidal input we will look at exponential input. For constant coefcient equations we will be able to use the method of optimism to nd a solution without having to compute integrals. For sinusoidal input, we will use Eulers formula to convert sinusoids to complex exponentials, and so our solutions for exponential input will apply to sinusoids as well.

First Order Response to Exponential Input


We start with an example of a linear constant coefcient ODE with exponential input signal. Example 1. Solve x + 2x = 4e3t . Solution. We could use our integrating factor, but instead lets use the method of optimism, i.e., the inspired guess. The inspiration here is based on the fact that differentiation reproduces exponentials: d rt e = rert . dt Since the right hand side is an exponential, maybe the output signal x (t) will be also. Lets try x p (t) = Ae3t . This is not going to be the general solution, so we use the subscript p to indicate it is just one particular solution. We dont know what A is yet, but we will be led to its value by substitution. Substituting x p into the DE we get . Left hand side: x p + 2x p = 3 Ae3t + 2 Ae3t = 5 Ae3t . Right hand side: 4e3t . Equating the two sides we get 5 Ae3t = 4e3t 5 A = 4 A = 4/5. So, we were led to the value of A and we have that one solution to the DE is 4 x p ( t ) = e 3t . 5 The associated homogeneous equation x + 2x = 0 has general solution xh (t) = Ce2t . By the superposition principle, we add x p and xh to get the general solution to our DE: x (t) = x p (t) + xh (t) = 4 3t e + Ce2t . 5

First Order Response to Sinusoidal Input 1. Introduction


We are going to solve a rst order constant coefcient DE with sinusoidal input: . x + kx = B cos( t). (1) Our strategy will be to use Eulers formula to replace cos( t) by the complex exponential ei t . We call this technique complex replacement. We illustrate it with an example and then rework the example in the general case.

2. Illustrative Example
Solve the ODE x + 2x = 2 cos(2t).

(2)

Solution. We will go through this example very carefully. After sufcient practice many of the steps can be done in your head. The key is to introduce a new variable y with its own related ODE y + 2y = 2 sin(2t).

(3)

Now we combine x and y to make a complex variable z = x + iy. Combining equations (1) and (3) in the same manner we get z + 2z = 2 cos(2t) + 2i sin(2t) = 2e2it .

(4)

We note that x = Re(z), so once weve found z(t) we can easily nd x (t). Equation (4) has exponential input and we know how to solve it: try a solution of the form z p (t) = Ae2it . Substituting this into the equation gives Left hand side: z p + 2z p = 2iAe2it + 2 Ae2it = (2 + 2i ) Ae2it . Right hand side: 2e2it . Equating the two sides we get

(2 + 2i ) Ae2it = 2e2it A = 1/(1 + i ).


Thus, z p (t) = e2it /(1 + i ). The problem asks for x which is the real part of z. We can nd x using polar or Cartesian coordinates. We will do it both ways. For most students using polar coordinates is less familiar. You should therefore learn it well

First Order Response to Sinusoidal Input

OCW 18.03SC

because polar coordinates are easier to interpret and we are generally prefered. The sinusoidal identity can be used to convert one form to the other. Polar Coordinates. In polar coordinates, 1 + i = 2 ei /4 . Using this in the formula for z p : z p (t) = e2it e2it ei(2t /4) 1 = = = (cos(2t /4) + i sin(2t /4)) . 1+i 2ei /4 2 2

Taking the real part we get 1 x p (t) = Re(z p (t)) = cos(2t /4). 2 Finally, as always, we add the homogeneous solution to this to get the general solution: 1 x (t) = x p (t) + Cekt = cos(2t /4) + Cekt . 2 Cartesian Coordinates. We use the complex conjugate to handle the denominator: z p (t) = e2it cos(2t) + i sin(2t) 1 i cos(2t) + sin(2t) + i (sin(2t) cos(2t)) = = . 1+i 1+i 1i 2

Taking the real part we get x p (t) = cos(2t) + sin(2t) . 2

Exercise. Use the sinusoidal identity to show that the two solutions given in the previous example are, in fact, identical.

3. General Case
Solve the ODE x + kx = B cos( t). (We assume k, and B are all positive.) Solution. This is really just a matter of replacing the numbers in our illustrative example by the letters k, B and . We will not write down as much as before. If something is unclear you can go to the corresponding part of the example above to understand it. 2

(5)

First Order Response to Sinusoidal Input

OCW 18.03SC

Do the complex replacement: z + kz = Bei t ,

where cos( t) = Re(ei t ) and x = Re(z).

(6)

Equation (5) has exponential input, so we try a solution of the form z p (t) = Ae2it . Substituting this into the equation gives . Left hand side: z p + kz p = i Aei t + kAei t = (k + i ) Aei t . Right hand side: Bei t . Equating the two sides we get

(k + i ) Aei t = Bei t A = B/(k + i ).


Thus, z p (t) = Bei t /(k + i ). In polar coordinates k + i = k2 + 2 ei , where = tan1 ( /k ) in the rst quadrant.

(Because tan1 is ambiguous, e.g tan( /4) = tan(5 /4) = 1, we x the value of tan1 by saying which quadrant the complex number is in. In this case, since k, > 0, k + i is in the rst quadrant. Another way to do this would be to write = Arg(k + i ).) Thus, z p (t) = Bei t Bei t Bei( t) = = . k + i k2 + 2 ei k2 + 2

Taking the real part we get x p (t) = B k2 + 2 cos( t ).

Finally, as always, we add the homogeneous solution to this to get the general solution: x (t) = x p (t) + Cekt = B k2 cos( t ) + Cekt .

+ 2

Amplitude, Phase, Gain and Bode Plots


We found that the ODE x + kx = kB cos( t) has a particular solution x (t) = kB k2 + 2 cos( t ) = gB cos( t ) (2)

(1)

where = tan1 ( /k). If we consider the input to be B cos( t) then the gain (= output amplitude/input amplitude) is g = k/ k2 + 2 . There is a lot more to learn from the formula (2) and its various pieces. The terminology applied below to solutions of the rst order equation (1) applies equally well to solutions of second and higher order equations. We will discuss this more when we study second order equations. See also the Mathlet Amplitude and Phase: First Order for a dynamic illustration. Lets gather all the terminology in one place. 1. B cos( t) is the input (or input signal). 2. B is the input amplitude and is the input circular frequency. 3. x (t) is the output or response. 4. g = k/ k2 + 2 is called the gain or amplitude response. The input amplitude is scaled by the gain to give the output amplitude. 5. is called the phase lag. Lets x the coupling constant k and think about how g and vary as we vary , the circular frequency of the signal. Thus we will regard them as functions of , and we may write g( ) and ( ) in order to emphasize this perspective. We are supposing that we always have the same system and are watching its response to a variety of input signals. Graphs of g( ) and ( ) for values of the coupling constant k = .25, .5, .75, 1, 1.25, 1.5 are given below.

Amplitude, Phase, Gain and Bode Plots

OCW 18.03SC

g = gain
1.0 0.9 0.8 0.7 0.6 0.5 0.4 0.3 0.2 0.1 0.0

.5

1 1.5 2 2.5 Circular frequency of signal

Fig. 1. First order amplitude response curves

/ = phase shift in multiples of


0 -0.05 -0.10 -0.15 -0.20 -0.25 -0.30 -0.35 -0.40 -0.45 -0.50

.5

1 1.5 2 2.5 Circular frequency of signal

Fig. 2. First order phase response curves These graphs are essentially Bode plots. (Technically, the Bode plots display log g( ) and ( ) against log .)

In this course we will focus more on the amplitude response curve (graph of gain vs. ) than the phase response curve. The phase response is important, we just wont have time to explore it. For equation (1) the amplitude response is rather simple: for any value of k the gain starts at 1 and decreases to 0 as goes to innity.

First Order Autonomous DEs: Introduction


In the last topic of this unit we will study autonomous rst order differential equations. These are (in general) nonlinear equations of the form x = f ( x ). (Compare this with the general rst order ODE x = f ( x, t).) The word autonomous means self governing and indicates that the rate of change of x is governed by x itself and is not dependent on time. Autonomous systems have the following properties: 1. They model conditions which are constant in time, though they may depend on the current value of x. 2. They are separable. 3. They can be hard to integrate. 4. We can say a lot about them without solving them. 5. They are time invariant: if y(t) is a solution then so is y(t t0 ) for any value of t0 . Our goal will be to learn to get qualitative information about the solutions without actually nding them. For example, in population models we might want to know if the population stabilizes, crashes or explodes. For physical models, we may want to know if the system is self correcting or if it can, all by itself, fall apart. To visualize the qualitative results we will use the phase line. This is a very simple one dimensional plot consisting of critical points and arrows.

Logistic Model: Qualitative Analysis


We will approach this topic through examples. As stated in the introduction, a rst order autonomous equation is one of the form y = g ( y ).

1. Simple Examples
Example 1. Natural growth or decay with constant growth-rate k: y = ky. Example 2. Bank account with interest rate not depending on time but possibly depending upon current balance and constant savings rate: y = I ( y ) y + q.

2. Logistic Population Model


Example 3. The logistic population model is a simple model that takes into account the limits the environment imposes on population growth. Suppose we have a model for a population y that has a variable growth rate k (y) which depends on the current population but not on time. That is, y = k ( y ) y.

(1)

Suppose that when y is small the growth rate is approximately k0 , but that there is a maximal sustainable population M. This means that as y gets near M the growth rate decreases to zero. And, if y > M , the growth rate becomes negative and the population declines back to the maximal sustainable population. In the simplest version of this, the graph of k (y) is a straight line with k = k0 when y = 0 and k = 0 when y = M.

Logistic Model: Qualitative Analysis

OCW 18.03SC

k k0 k = k (y ) = k0 (1 y/m)

Fig. 1. Line with vertical intercept k0 and horizontal intercept M. The equation of this line is k ( y ) = k 0 (1 y / M ). (You can check that k (0) = k0 , k ( M) = 0 and k(y) < 0 for y > M.) In this case equation (1) is known as the Logistic Population Model : y = k0 (1 (y/ M))y = f (y).

(2)

This is more realistic than natural growth when you want to account for limits to growth. It is nonlinear but it is autonomoous. Autonomous equations are always separable and, in this case, we could compute the resulting integral using partial fractions. But we are aiming for a qualitative grasp of the solutions, which we develop in the next example. Example 4. Give a qualitiative picture of the solutions without solving equation (2). Solution. We start by looking for constant solutions y(t) = y0 . Since a constant has derivative 0, plugging this into (2) gives 0 = f ( y0 ) We see that y0 = 0 and y0 = M are the two points where f (y0 ) = 0. Thus we have two constant solutions y(t) = 0 and y(t) = M. Because a system at equilibrium is unchanging, we will call these solutions equilibrium so. lutions. Since y = f (y) = 0 when y = 0 and y = M we call 0 and M the critical points of the DE. To summarize, the following all say the same thing: 1. f (y0 ) = 0. 2

Logistic Model: Qualitative Analysis 2. y(t) = y0 is an equilibrium solution. 3. y = y0 is a critial point.

OCW 18.03SC

To tie this to previous work, note the equation is separable and our constant solutions are none other than the lost solutions of the separable equation. To understand the non-constant solutions we will sketch and analyze the direction eld for equation (2). Clearly, each isocline, f (y) = c, is a horizontal straight line. For a xed slope c, the isocline will consist of a horizontal line y = y0 where f (y0 ) = c. As usual, rst we look at the nullcline f (y) = 0. We already know the zeros of f (y) are 0 and M. So the nullclines are the pair of lines y = 0 and y = M. These are exactly the constant solutions found above.
y y (t) = M

y (t) = 0

Fig. 2. The nullclines are also solution curves. To get a clear picture of the other isoclines we will draw a graph of f (y) as a function of y. Its a parabola opening downward, meeting the horizontal axis at y = 0 and y = M.
f (y ) O

< f (y ) < 0

> f (y ) > 0

/y < M f (y ) < 0

Fig. 3. The graph of f (y) tells us where y is positive and negative. The graph shows that . for y < 0 y = f (y) is negative, . for 0 < y < M y = f (y) is positive, . for M < y y = f (y) is negative. This is indicated on the graph by the arrows on the horizontal axis. The

Logistic Model: Qualitative Analysis

OCW 18.03SC

arrow points left (towards decreasing y) where y is negative and right (to. wards increasing y) where y is positive. To make things clear, we also label the intervals as having f (y) positive or negative. Now we can sketch the direction eld. First, we draw the nullclines and since these are horizontal lines, we dont need to sketch the direction eld elements (little line segments) along them. Then we choose a horizontal line above y = M and sketch the direction eld elements along it. (We . know they are negative because for y > M we know y < 0.) Similarly, we add an isocline between 0 and M and one below 0. Finally we can sketch some solution curves:
y

Fig. 4. Direction eld and solution curves for the logistic equation (2). 1. Since the slope eld is constant in the t direction any solution curve can be translated left or right and still be a solution. 2. Since the lines y = 0 and y = M are solutions the other curves cant cross them. 3. The solutions that start just above the equilibrium solution y = 0 must increase. Since they cant cross the solution y = M they must go asymptotically to towards it. These bounded solutions are called logistic curves or S-curves. They represent the population drifting from just above the equilibrium y = 0 towards the one at y = M. 4. If the population exceeds the M, it tends back towards it. This represents environmental pressure related to overpopulation. M is called the carrying capacity of the environment.

Logistic Model: Qualitative Analysis

OCW 18.03SC

5. In a population model we would never see y < 0. Mathematically, the solution curves that start below y = 0 decrease without bound.

3. Stable and Unstable Equilibria


Notice that in the logistic model all the solution curves that start near the equilibrium y = M go asymptotically towards it. (See gure 4.) We call such an equilibrium a stable equilibrium. Similarly, we call the equilibrium y = 0 an unstable equilibrium because all the curves that start near it move away.

4. Summary
The sketch of the solutions gives us our qualitive picture. We also dened a number of terms. 1. Autonomous equation: y = f (y). 2. Equilibrium solutions: Constant solutions y(t) = y0 where f (y0 ) = 0. 3. Critical points: The value of the equilibrium solutions, i.e., values y0 where f (y0 ) = 0. 4. Stable equilibrium: An equilibrium solution where all nearby solution curves tend towards it. 5. Unstable equilibrium: An equilibrium solution where all nearby solution curves tend away from it. 6. Logistic population model, logistic curves: see above. 7. Carrying capacity: The stable equilibrium in the logistic model that all (positive) populations approach asymptotically. In later examples we will learn how to systematically make a qualitative sketch of the solution curves.

Phase Lines 1. Denition


Since the direction eld for an autonomous DE, y = f (y), is constant on horizontal lines, its essential content can be conveyed more efciently using the following recipe: 1. Draw the y-axis as a vertical line and mark on it the equilibria, i.e. where f (y) = 0. 2. In each of the intervals delimited by the equilibria draw an upward pointing arrow if f (y) > 0 and a downward pointing arrow if f (y) < 0. This simple diagram tells you roughly how the system behaves. Its called the phase line. The phase line captures exactly the information we use to get the qualitative sketch of solution curves. We illustrate this with some examples.

2. Examples
Example 1. For the DE y = 3y: nd the critical points, draw the phase line, classify the critical points by stability and use the phase line to give a qualitative sketch of some solution curves. Solution. The steps to follow are: 1. Find the critical points. . 2. Plot the graph of f (y) and determine where y is positive and negative. 3. Draw the phase line and nd the stability of the critical points. 4. Sketch the solution curves. 1. We have f (y) = 3y. We easily see that the only critical point (root of f ) is y = 0. 2. The plot of f (y) is a straight line. We see y > 0 for y > 0 and y < 0 for y < 0.

Phase Lines

OCW 18.03SC

f (y ) = 3 y

y <0

y >0

3. We use the information from steps 1 and 2 to draw the phase line. We . put a large dot at the critical point. Since y > 0 in the interval y > 0 we add an arrow pointing upwards in that interval. Similarly the interval y < 0 gets a down arrow. Since the arrows point away from the critical point, the equilibrium is unstable. This is all shown in the gure below. 4. Once we have the phase line we can make a qualitative sketch of the solution curves. The equilibrium solution corresponds to the critical point. It is the horizontal line y(t) = 0. The solutions that start positive increase and those that start negative decrease. We present the solution curves next to the phase line so you can see that the phase line arrows represent the y-direction of the integral curve.
y y

unstable 0

3. The phase line. 4. Qualitative sketch of solution curves. Example 2. Repeat example 1 for the logistic equation y = k0 (1 y/ M)y. Solution. We did all of this earlier except draw the phase line. 1. Critical points: y = 0, y = M.

Phase Lines

OCW 18.03SC

y O y = f (y ) < f (y ) < 0 > f (y ) > 0 /y < M f (y ) < 0

2. Graph of f (y)

y O

unstable 0

stable M

/t

3. Phase line. 4. Sketch of solution curves.

3. Solutions Can Be Shifted in Time . In an autonomous equation y = f (y), the direction eld is constant in
the horizontal direction. Said differently, the conditions represented by the ODE are constant in time. Consequently, any horizontal (time) translate of a solution is another solution. A "time translate" of a function y(t) is a function y(t t0 ); the graph is shifted horizontally (to the right) by t0 units. Solutions of y = k0 y exhibit three different behaviors, illusy(t) = ek0 t , y(t) = 0 and y = ek0 t .

Example 3. trated by

Any solution is a horizontal translate of one of these three: y ( t ) = e k 0 ( t t0 ) , y(t) = 0 (whose only time-translate is itself), or y ( t ) = e k 0 ( t t0 ) . See the answer to example 1 for the graphical version of this. 3

Phase Lines

OCW 18.03SC

4. Semistable Equilibria
Some equilibria are half stable and half unstable. We call them semistable Example 4. Repeat examples 1 and 2 for the DE y = y2 . Solution. Once again the algebra is trivial. 1. The only critical point is y = 0. . 2. The graph of f (y) shows y is always positive (except at 0). 3. The phase line shows the critical point at 0 is semistable.
y y

y y = y2 y semistable 0 t

y >0

y >0

Graph of f (y).

Phase line.

Sketch of solution curves.

5. Conclusion
The phase line shows the qualitative behavior of a system at a glance: the critical points are shown and you can tell the stability of each critical point by looking at the arrows around it; the arrows also tell you what happens to the integral curves in the long-run, as t goes to innity.

Stability
Quiz: Stability. . In the autonomous equation y = f (y) , where f (y) has the graph shown, descibe the rightmost critical point.

y
y = f (y ) -1 1

Choices: a) stable b) unstable c) semistable c) cant tell, could be any of them Answer: Unstable. This is evident from the phase line.

y
unstable 1

stable 0

unstable -1

The phase line shows that the critical point at y = 1 is unstable.

Stability
Quiz: Stability. . In the autonomous equation y = f (y) , where f (y) has the graph shown, descibe the rightmost critical point.

y
y = f (y ) -1 1

Choices: a) stable b) unstable c) semistable c) cant tell, could be any of them Pick what you think is the correct choice and then look at the answer.

Stability
Quiz: Stability. . In the autonomous equation y = f (y) , where f (y) has the graph shown, descibe the rightmost critical point.

y
y = f (y ) -1 1

Think about your answer and then look at the choices.

Can Solutions have a Local Maximum?


Quiz: Can Solutions have a Local Maximum?. Can solutions of autonomous equations have a strict local maximum? (For a function f (t) a strict local maximum at time t = a means f ( a) is larger than any nearby values of f (t). Graphically, it is at the top of a hill.) Choices: a) Yes. b) No.

Answer: No.

Suppose y(t0 ) = y0 and y(t0 ) = 0 then there is an equilibrium solution y(t) = y0 . By the existence and uniqueness theorem this is the only solution with y(t0 ) = y0 . We have shown that non-constant solutions never have derivative equal to 0, i.e. they dont have any local maxima or minima. We had to be careful in phrasing the question because constant functions have local maxima, just not strict local maxima. That is, all values of the function are maximum values, but no value of the function is larger than nearby values.

Linear vs. Nonlinear


The general linear rst order ODE is r ( t ) x ( t ) + p ( t ) x ( t ) = q ( t ). The general rst order ODE is y(t) = F (t, y(t)). Representing the linear side of the debate is Linn E. R. and Chao S. representing the nonlinear side.

1.

The Debate

Linn: Id like to begin by making the point that there is a solution procedure for linear equations, which reduces the solution of any linear equation to integration. Multiply the equation through by a factor so that the two terms d(ux ) become the two terms in , then integrate. dt Sometimes you can just see this. For example, t2 x + 2tx =

d 2 ( t x ). dt

If we are in reduced standard form, i.e. when r = 1, then this can be done systematically with the following steps: 1. We seek u(t) such that u( x + px ) =

d(ux ) , dt

i.e. pu = u. This is separable, with solution u=e

p(t) dt

(Any constant of integration will do here.) 2. Then integrate both sides of d(ux ) = uq dt and solve for x. This gives x ( t ) = u 1 ( t )

u(t)q(t) dt.

Linear vs. Nonlinear

OCW 18.03SC

The constant of integration is in this integral, so the general solution has the form x (t) = x p (t) + cu1 (t). Another lovely feature of linear equations is that the constant of integration in the solution of a linear equation always appears right there. The associated homogeneous equation is x + px = 0. This is separable, with solution xh (t) = e cal of the integrating factor! Wonderful! cu1 (t)

p(t) dt

. Look! This is the recipro-

In most applications, u1 (t) falls off to zero as t gets large; the term is a transient.

Chao: Thats a lot of integration. Im more interested in the general behavior of solutions, rather than an incomprehensible expression of them as integrals or a boring expression of them in terms of sin, cos, and exponentials. I prefer arguments like this: take an equation like y = y2 x . This doesnt have a single solution which you, Linn, in your linear cave, have anything to say about. But I can look at the direction eld, recognize that there is a funnel along the curve y = x . This means all solutions near there are trapped and are asymptotic to x. I can even argue that they are all ultimately a bit larger than x. No integration involved and very good information. Or take an equation like the logistic equation . y y = k0 y 1 . p This is an auotomous equation, and remains so even if I allow a harvest rate, even one depending upon y : . y y = k0 y 1 a ( y ). p This equation gives genuine insight into real population dynamics. By looking at the phase line it is easy to analyze the behavior of solutions, in a way useful for policy makers. 2

Linear vs. Nonlinear

OCW 18.03SC

Linn: Well, now, most of the time a system is near equilibrium. Engineers get very anxious when their systems get too far from equilibrium. Lets look at your nice nonlinear logistic equation. As you said, theres a critical point at y = p, and so an equilbrium solution. Just how does the system relax to this equilibrium? Lets write y = p + u and change variables using 1 y u = . p p

Substituting this in the logistic equation gives u = k0

u u2 ( p u) = k0 u + k0 . p p

For small u the second term is very small, and can be ignored. This is called LINEARIZING!! the equation near equilibrium. Near equilbrium solutions to the nonlinear equation behave a lot like p + u where u is a solution to the linear equation . u = k 0 u. So we can say that the population relaxes to equilibrium exponentially, as ek0 t approaches 0. Chao: There you go again with your fancy exponentials. You think you know all about them, but your computer has to compute their values, after all, and the methods it uses are no different from the methods used to compute the values of linear equations. It has to use Eulers method, or its fancier variants. Linn: Speaking of fancy exponentials, Id like to point out that smart people almost never use integrating factors to integrate linear equations with constant coefcients: . x + kx = q(t). (LCC) Yeah, the integrating factor is ekt , but even I dont like the integrals that come out. But there are these great tricks, Chao! Suppose q(t) = Bert . Then, be optimistic! Maybe theres an exponential solution of the form Aert . When we substitute into the two sides of equation (LCC) we get: . Left side: x + kx = A(r + k)ert Right side: Bert . Setting them equal to each other we get: Bert = A(k + r )ert 3

Linear vs. Nonlinear

OCW 18.03SC

Thus, A=

B k+r

and

x p (t) =

Weve found a particular solution to x + kx = Bert . Because it is the response to exponential input we call this the Exponential Response Formula. (ERF) It works as long as k + r is not 0. Chao: Bravo. Linn: Thank you. And whats better, did I ever tell you about the complex exponential? In the ERF r can be a complex number! Euler told us that ei = cos( ) + i sin( ) its a point on the unit circle in the complex plane. So, trig functions are incorporated into the complex exponential!! To solve x x = 3 cos(t) I replace it by the different equation z z = 3eit of which my original DE is the real part. Then I can use the ERF (with k = 1, r = i, B = 3) to get zp = 3 eit ( 1 + i ).

B rt e k+r

(ERF)

Since x p = Re(z p ), all thats left is to nd the real part of z p . zp = 3 eit 1 i 3 cos(t) + 3 sin(t) + i ( cos(t) sin(t)) = . ( 1 + i ) 1 i 2

3 3 cos(t) + sin(t). 2 2 For the general solution we just add in the general solution to the homogeneous equation, which is cet . Its a little funny to call this a transient and I dont, but it does give you the general solution. x p = Re(z p ) =

Therefore,

Linear vs. Nonlinear

OCW 18.03SC

Chao: You know, your method results in these sums of sines and cosines, which is very nice but I want to know what they look like. The nonlinear view of sines and cosines writes a cos( t) + b sin( t) = A cos( t ), where A, and are the polar coordinates of the point ( a, b). In your example, A = 3 2/2 and = 3 /4. So, the solution is easy to draw and compare with the input signal. I want to point out another charming feature of solutions to many non. linear equations. Take a simple one, for example y = y2 . This is separable and can be solved in three short steps: y2 dy = dx

y 1 = x + c y = 1/(c x ).
So the IVP with y(0) = 1 has c = 1 and y = 1/(1 x ). Its graph is asymptotic to the vertical line at x = 1. In other words, it is able to go off to innity in nite time. It ends. The equation y = 1/(1 x ) actually represents two solutions: one for x < 1, and another for x > 1. If we are to say that a solution of a differential equation is determined by an initial value, we have to require that the graph be connected. Linn: You call that a feature? That never happens to solutions to my equations. If they are going to go south on me, I know it from the coefcients or the input signal. As long as p(t) and q(t) are nice and nite (and r (t) is nonzero) so are all solutions. They live as long as I do! Chao: Well, the world really is nonlinear. Newtons law of gravitation is highly nonlinear. This kind of explosion actually happens in the case of Newtons laws: Jeff Xia showed that in a certain 5-planet system two of sin(1/t) the planets behaves more or less like , oscillating with increasing t amplitude and increasing frequency as t 0 (from the negative side). Solutions to linear equations are not nearly as diverse and exciting!

2.

Conclusion

In both 18.03 classes in spring 2010, Linear won the debate, but Nonlinears supporters were more enthusiastic. 5

Introduction
Constant coefcient linear DEs lie at the heart of this course. In this session will see that they can be used to model many physical systems. Here we will focus on the damped harmonic oscillator. In particular we will discuss spring-mass-dashpot systems Because second order equations are algebraically tractable we will focus on them. Fortunately they are varied enough to give us a lot of insight into the behavior of higher order systems. In this session we will learn how to solve homogeneous equations, i.e. those where the input function is 0. The key step will be nding which exponential functions ert satisfy the DE. These will be our modal solutions. We will use superposition to build all the solutions out of the modal solutions. The algebra will demand that we allow r to be a complex number, so we will need to use Eulers formula to convert the complex exponential solution into solutions involving sines and cosines.

Second Order Physical Systems 1. Second Order Physical Systems


Second order equations are the basis of analysis of mechanical and electrical systems. Well build this important subject up slowly, starting with a simple mechanical system. A spring is attached to a wall and a cart:
Fext spring mass x x=0 x

We set up the coordinate system so that at x = 0 the spring is relaxed, which means that it is exerting no force. This is called the equilibrium position. In addition to the spring, suppose that there is another force acting on the cart an external force, maybe wind blowing on a sail attached to it, maybe gravity, or some other force. Then mx = Fspr + Fext The spring force is characterized by the fact that it depends only on position. In fact: if x > 0, if x = 0, if x < 0, Fspr ( x ) < 0 Fspr ( x ) = 0 Fspr ( x ) > 0.

..

The simplest way to model this behavior (and one which is valid in general for small x, by the tangent line approximation) is Fspr ( x ) = kx, where k > 0 This is called Hookes Law and k is called the spring constant. Replacing Fspr by kx we get mx + kx = Fext .

..

Second Order Physical Systems

OCW 18.03SC

Any real mechanical system also has friction. Friction takes many forms. It is characterized by the fact that it depends on the motion of the mass. We will suppose that it depends only on the velocity of the mass and not on its position. Often the damping is controlled by a device called a dashpot. This is a cylinder lled with oil, that a piston moves through. Door dampers and car shock absorbers often actually work this way. We write . Fdash ( x ) for the force exerted by the dashpot. It opposes the velocity: if x > 0, if x = 0, if x < 0,

. . .

Fdash ( x ) < 0 Fdash ( x ) = 0 Fdash ( x ) > 0

. . .

The simplest way to model this behavior (and one which is valid in general . for small x, by the tangent line approximation) is Fdash ( x ) = bx, where b > 0. This is therefore called linear damping and b is called the damping constant. Putting this together we get the differential equation for the displacement x of the mass from equilibrium is mx + bx + kx = Fext .

..

(1)

Equation (1) will be a rich source of examples in the remainder of the course. Diagramatically this looks like:
Fext spring mass x x=0 x dashpot

MIT OpenCourseWare http://ocw.mit.edu

Fall 2011

18.03SC Differential Equations

For information about citing these materials or our Terms of Use, visit: http://ocw.mit.edu/terms.

The Characteristic Polynomial 1. The General Second Order Case and the Characteristic Equation
For m, b, k constant, the homogeneous equation mx + bx + kx = 0.

..

(1)

is a lot like x + kx = 0, which has as solution x = ekt . Well be optimistic and try for exponential solutions, x (t) = ert , for some as yet undetermined constant r. To see which values of r might work, plug x (t) = ert into (1). Organize the calculation: the k ] , b] , m] are ags indicating that we should multiply the corresponding line by this number. k] b] m] x = ert . x = rert .. x = r2 ert

m x + bx + kx = (mr2 + br + k)ert = 0.
An exponential is never zero, so we can divide this equation by ert . We have found that ert is a solution to (1) exactly when r satises the characteristic equation mr2 + br + k = 0. The left hand side is a polynomial called, naturally enough, the characteristic polynomial and usually denoted p(r ). (You will often also see s used as the variable instead of r. With this notation the characteristic polynomial is p(s) = ms2 + bs + k.) Example. Find all the solutions to x + 7x + 8x = 0. Solution. The characteristic polynomial is r2 + 8r + 7 . We want the roots. One reason we wrote out the polynomial was to remind you that you can nd roots by factoring it. This one factors as (r + 1)(r + 7) so the roots are r = 1 and r = 7, with corresponding exponential solutions are x1 (t) = et and x2 (t) = e7t . By superposition, the linear combination of independent solutions gives the general solution: x ( t ) = c 1 e t + c 2 e 7t .

..

..

The Characteristic Polynomial

OCW 18.03SC

Suppose that we have initial conditions x (0) = 2 and x (0) = 8 then . we can solve for c1 and c2 . Use x (t) = c1 et 7e7t and substitute t = 0 to get x (0) = c1 + c2 = 2 . x (0) = c1 7c2 = 8 Adding these two equations yields 6c2 = 6, so c2 = 1 and c1 = 1. The solution to our DE with the given initial conditions is then x (0) = 2, . x (0) = 8 is x ( t ) = e t + e 7t .

2. The General nth Order Case


In the same way we can take the homogeneous constant coefcient linear equation of degree n a n x ( n ) + + a1 x + a0 x = 0 and get its characteristic polynomial, p (r ) = a n r n + + a1 r + a0 The exponential x (t) = ert is a solution of the homogeneous DE if and only if r is a root of p(r ), i.e. p(r ) = 0. By superposition, any linear combination of these exponentials is also a solution.

MIT OpenCourseWare http://ocw.mit.edu

Fall 2011

18.03SC Differential Equations

For information about citing these materials or our Terms of Use, visit: http://ocw.mit.edu/terms.

Linear Differential Equations 1. Linear Differential Equations


A linear differential equation is of the following form: a n x ( n ) + a n 1 x ( n 1) + + a 1 x + a 0 x = q ( t ).

(1)

The ak s are the coefcients. They may depend upon t (but not on x ). If an is not zero then the differential equation is said to be of order n. If this models a physical system then the left hand side represents the system and the right hand side represents the input signal. The coefcients represent parameters of the system. For example, the mass, damping and .. . spring constants m, b and k in mx + bx + kx are the parameters of the system. In general, they may depend on time, e.g. maybe the force is actually a rocket, and the fuel burns so m decreases. Or maybe the spring gets softer as it ages. Maybe the honey in the dashpot gets thicker with time. We will generally assume the coefcients are constant. In which case equation (1) is said to be a constant coefcient linear equation. It is, in fact, a good approximation of the non-constant coefcient equation as long as the coefcients vary on a time-scale that is much greater than the timescale of the dynamical variable x.

2. Second Order Homogeneous Constant Coefcient Linear Equations .. . We will study the spring system mx + bx + kx = Fext starting with the case Fext = 0. .. . mx + bx + kx = 0. (2)
To ensure that (2) is of second order (and a realistic physical system) we always assume m > 0, but we will allow the case b = 0 and occasionally k = 0. With no external force the system evolves on its own. Think of a door that can swing back and forth or a ball on the end of a rubber band. As we did in rst order equations we will call (2) a homogeneous linear differential equation.

3. The Undamped Case


The special case b = 0 is called undamped. This is called the simple .. harmonic oscillator. Its ODE is mx + kx = 0 or

..

x+

k x = 0. m

Linear Differential Equations

OCW 18.03SC

If we let =

k/m our equation becomes

..

x + 2 x = 0.

We have seen before (and you can easily check) that x1 (t) = cos( t) and x2 (t) = sin( t) are solutions to this equation. Since the input is 0 and the equation is linear, we can use superposition of solutions to get the general solution x (t) = a cos( t) + b sin( t) = A cos( t ) (3) This is another fundamental fact you should memorize! (The second equality comes from the sinusoidal identity, which gives a = A cos and b = A sin .) We know (3) gives every solution because x (0) = a and x (0) = b, so you can solve (uniquely) for a and b to give any desired intial condition.

MIT OpenCourseWare http://ocw.mit.edu

Fall 2011

18.03SC Differential Equations

For information about citing these materials or our Terms of Use, visit: http://ocw.mit.edu/terms.

Period of the Simple Harmonic Oscillator


Quiz: What is the period of a nonzero solution of x + 4x = 0? Choices: a) Depends upon the solution b) 2 c) d) 4 e) 2 f) /2 g) None of these.

..

Answer: (c) . We have the natural frequency 0 = k/m = 2, so the general solution is x (t) = c1 cos(2t) + c2 sin(2t) = A cos(2t ) in both rectangular and phase-amplitude form respectively. (As a check, think of what t has to do to take 2t from 0 to 2 ; or alternatively use P = 2 /0 , with 0 = 2.)

MIT OpenCourseWare http://ocw.mit.edu

Fall 2011

18.03SC Differential Equations

For information about citing these materials or our Terms of Use, visit: http://ocw.mit.edu/terms.

Period of the Simple Harmonic Oscillator


Quiz: What is the period of a nonzero solution of x + 4x = 0? Choices: a) Depends upon the solution b) 2 c) d) 4 e) 2 f) /2 g) None of these. Pick what you think is the correct choice and then look at the answer.

..

MIT OpenCourseWare http://ocw.mit.edu

Fall 2011

18.03SC Differential Equations

For information about citing these materials or our Terms of Use, visit: http://ocw.mit.edu/terms.

Period of the Simple Harmonic Oscillator


Quiz: What is the period of a nonzero solution of x + 4x = 0? Think about your answer and then look at the choices.

..

MIT OpenCourseWare http://ocw.mit.edu

Fall 2011

18.03SC Differential Equations

For information about citing these materials or our Terms of Use, visit: http://ocw.mit.edu/terms.

Modes and Roots


A solution of the form x (t) = cert to the homogeneous constant coefcient linear equation a n x ( n ) + a n 1 x ( n 1) + + a 1 x + a 0 x = 0

(1)

is called a modal solution and cert is called a mode of the system. We saw previously that ert is a solution exactly when r is a root of the characteristic polynomial p ( s ) = a n s n + a n 1 s n 1 + + a 1 s + a 0 . Warning: This only works for homogeneous constant coefcient linear equations. It does not work for non-constant coefcient or inhomogeneous or nonlinear equations. The roots of polynomials can be real or non-real complex numbers. (We need to be a little careful with our language because a real number is also a complex number with imaginary part 0.) Roots can also be repeated. Studying the second order equation will be enough to help us understand all of these possibilities. So, we study is (with a2 = m, a1 = b, a0 = k) mx + bx + kx = 0.

..

(2)

which models a spring-mass-dashpot system with no external force. The characteristic equation is ms2 + bs + k = 0.

1. Real Roots
We have already done this case earlier in this session. If the characteristic polynomial has real roots r1 and r2 then the modal solutions to (2) are x1 (t) = er1 t and x2 (t) = er1 t . The general solution if found by superposition x ( t ) = c 1 x 1 ( t ) + c 2 x 2 ( t ) = c 1 e r1 t + c 2 e r2 t . Example 1. (Real roots) Solve the x + 5x + 4x = 0. Solution. The characteristic equation is s2 + 5s + 4 = 0. This factors as (s + 1)(s + 4) = 0, so it has roots -1, -4. The modal solutions are x1 (t) = et and x2 (t) = e4t . Therefore, the general solution is x ( t ) = c 1 e t + c 2 e 4t .

..

Modes and Roots

OCW 18.03SC

2. Complex Roots
(Again, if we were being completely precise, this section would be called non-real complex roots to indicate a complex number with non-zero imaginary part.) Example 2. Solve the equation x + 4x + 5x = 0.

..

Solution. The characteristic polynomial is s2 + 4s + 5. Using the quadratic formula the roots are 4 16 20 s= = 2 1 = 2 i . 2 So our exponential solutions are z 1 ( t ) = e ( 2+ i ) t and z 2 ( t ) = e ( 2 i ) t .

We use the letter z here to indicate the functions are complex valued. The general solution is a linear combination of these two basic solutions. But, because the DE has real coefcients, we were expecting real valued solutions. We will nish this example and get our real solutions after stating and proving the following theorem. Theorem (Real Solution Theorem): .. . If z(t) is a complex-valued solution to mz + bz + kz = 0, where m, b, and k are real, then the real and imaginary parts of z are also solutions. Proof: Let u(t) be the real part of z and v(t) the imaginary part, so z(t) = u(t) + iv(t). Now, build the table. k] b] m] z = u + iv . . . z = u + iv .. .. .. z = u + iv

Summing with the coefcients (and remembering z is a solution to the homogeneous DE) gives

(mu + bu + ku) + i (mv + bv + kv) = 0.


Both expressions in parentheses are real, so the only way the sum can be zero is for both of them to be zero. That is, both u and v are solutions of (2) as claimed. Back to the example: Using Eulers formula z1 (t) = e(2+i)t = e2t cos t + ie2t sin t. 2

..

..

Modes and Roots

OCW 18.03SC

The real part of e2t cos t + ie2t sin t is e2t cos t and the imaginary part is e2t sin t. We now have two basic solutions and can use superposition to nd the general real valued solution x (t) = c1 e2t cos(t) + c2 e2t sin(t). Or we could have also written it as x (t) = e2t (c1 cos t + c2 sin t) = Ae2t cos(t ). This is a damped sinusoid with circular pseudo-frequency 1. If we had chosen the other exponential solution z2 (t) = e(2i)t = e2t (cos(t) + i sin(t)) then the basic real solutions would be e2t cos(t) = e2t cos(t) and e2t sin(t) = e2t sin(t). Up to a sign these are the same basic solutions as was obtained from z1 , so z2 (t) would have work just as well. Example 3. Solve x + x + x = 0.

..

2 Solution. Characteristic equation: s + s + 1 = 0. 1 1 4 1 3 = i . Roots: 2 2 2 Complex exponential solutions: z1 (t) = e(1+i 3)t/2 , z2 (t) = e(1i 3)t/2 Basic real solutions: x1 (t) = Re(z1 (t)) = et/2 cos( 3t/2), Im(z1 (t)) = t /2 e sin( 3t/2). t/2 ( c cos( 3t /2) + + c sin( 3 t /2)) = General real solution: x ( t ) = e 2 1 Aet/2 cos( 3 t/2 ).

Example 4. Suppose that the equation mx + bx + kx = 0 has characteristic roots a ib. Give the general real solution. Solution. In the previous examples we have established a pattern: Two basic real solutions are e at cos(bt) and the general real solution is x (t) = c1 e at cos(bt) + c2 e at sin(bt) = Ae at cos(bt ). 3 and e at sin(bt)

..

Modes and Roots

OCW 18.03SC

In words, the real part of the root is the coefcient of t in the exponential and the imaginary part is the angular pseudo-frequency in the trig functions. For completeness we will walk through the derivation of this. One exponential solution is z1 (t) = e(a+ib)t = e at (cos(bt) + i sin(bt)). The two basic solutions are the real and imaginary parts of z1 . That is, e at cos(bt) as claimed. and e at sin(bt),

Example 5. Use the characteristic equation to solve x + 4x = 0. Solution. You should have memorized the solution to this equation. We will check the characteristic equation technique against this known solution. Characteristic equation: s2 + 4 = 0. Roots: s2 = 4 s = 2i. Complex exponential solutions: z1 = e2it , z2 = e2it . Basic real solutions: Re(z1 ) = cos(2t), Im(z1 ) = sin(2t). General real solution: x = c1 cos(2t) + c2 sin(2t) = A cos(2t ) (as expected).

..

Example 6. Solve x + 4x + 4x = 0. Then p(s) = (s + 2)2 has r = -2 as a repeated root. The only exponential solution is e2t . Another solution, which is not a constant multiple of e2t , is given by tet . We will not check this for now, you know how to do it: plug in and use the product rule. So the general solution is x (t) = c1 e2t + c2 te2t or x (t) = e2t (c1 + c2 t). Example 7. (Its all about the roots) Suppose the roots with multiplicity of a certain homogeneous constant coefcient linear equation are 3, 4, 4, 4, 5 2i, 5 2i. 4

3. Repeated Roots ..

Modes and Roots

OCW 18.03SC

Give the general real solution to the equation. What is the order of the equation? Solution. The basic solutions are e3t , e4t , te4t , t2 e4t , e5t cos(2t), e5t sin(2t), te5t cos(2t), te5t sin(2t). (For each repeated root we added a multiple of t to the basic solution.) Using superposition, the general solution is x (t) = c1 e3t + c2 e4t + c3 te4t + c4 t2 e4t

+ c5 e5t cos(2t) + c6 e5t sin(2t) + c7 te5t cos(2t) + c8 te5t sin(2t).


There are 8 roots, so the order of the differential equation is 8.

MIT OpenCourseWare http://ocw.mit.edu

Fall 2011

18.03SC Differential Equations

For information about citing these materials or our Terms of Use, visit: http://ocw.mit.edu/terms.

Damped Harmonic Oscillators: Introduction


In this session we will look carefully at the equation mx + bx + kx = 0 as a model of the damped harmonic oscillator. When the damping constant b equals zero we know the solution to this equation is x (t) = c1 cos( t) + c2 sin( t) = A cos( t ), where = k/m. Since this has a pure sinusoidal solution we call the system a simple harmonic oscillator. When b = 0 we call the system a damped harmonic oscillator. Our goal is to understand the effect of b on the system. Well see that when b is small the system is underdamped and the output is a damped sinusoid or damped oscillation. When b is large the system is overdamped and it no longer oscillates. Right between under and overdamping is a value of b called critical damping. We will learn how to nd the critical damping value. Our main tool will be the method of characteristic roots discussed in the last session. We will use the mathlet Damped Vibrations to visualize what we have learned.

..

MIT OpenCourseWare http://ocw.mit.edu

Fall 2011

18.03SC Differential Equations

For information about citing these materials or our Terms of Use, visit: http://ocw.mit.edu/terms.

Damped Harmonic Oscillators


In the last session we modeled a spring-mass-dashpot system with the constant coefcient linear DE mx + bx + kx = Fext , where m is the mass, b is the damping constant, k is the spring constant and x (t) is the displacement of the mass from its equilibrium position.
Fext spring mass x x=0 x dashpot

..

We then assumed the external force Fext = 0 and used the characteristic equation technique to solve the homogeneous equation mx + bx + kx = 0.

..

(1)

Restrictions on the coefcients: The algebra does not require any restrictions on m, b and k (except m = 0 so that the equation is genuinely second order). But, since this is a physical model, we will now require m > 0, b 0 and k > 0. The Damped Harmonic Oscillator: The undamped (b = 0) system has equation .. mx + kx = 0. At this point you should have memorized the solution and also be able to solve this equation using the characteristic roots. The solution is x (t) = c1 cos( t) + c2 sin( t) = A cos( t ).

Here = k/m and the solution is given in both rectangular and amplitudephase form. The solution is always a sinusoid, which we consider a simple oscillation, and we call this system a simple harmonic oscillator.

Damped Harmonic Oscillators


x t

OCW 18.03SC

Figure 2: The output of a simple harmonic oscillator is a pure sinusoid. When we add damping we call the system in (1) a damped harmonic oscillator. This is a much fancier sounding name than the spring-massdashpot. It emphasizes an important fact about using differential equations for modeling physical systems. It doesnt matter whether x measures position or current or some other quantity. Any system modeled by equation (1) will respond just like the spring-mass-dashpot; that is, all damped harmonic oscillators exhibit similar behavior. We will see an important example of this principle whe we study the case of an RLC electrical circuit.

MIT OpenCourseWare http://ocw.mit.edu

Fall 2011

18.03SC Differential Equations

For information about citing these materials or our Terms of Use, visit: http://ocw.mit.edu/terms.

Under, Over and Critical Damping 1. Response to Damping


As we saw, the unforced damped harmonic oscillator has equation mx + bx + kx = 0, with m > 0, b 0 and k > 0. It has characteristic equation ms2 + bs + k = 0 with characteristic roots

..

(1)

b2 4mk 2m

(2)

There are three cases depending on the sign of the expression under the square root: i) b2 < 4mk (this will be underdamping, b is small relative to m and k). ii) b2 > 4mk (this will be overdamping, b is large relative to m and k). iii) b2 = 4mk (this will be critical damping, b is just between over and underdamping. Mathematically, the easiest case is overdamping because the roots are real. However, most people think of the oscillatory behavior of a damped oscillator. Since this is connected to underdamping we start with that case. Case (i) Underdamping (non-real complex roots) If b2 < 4mk then the term under the square root is negative and the characteristic roots are not real. In order for b2 < 4mk the damping constant b must be relatively small. First we use the roots (2) to solve equation (1). Let d = |b2 4mk|/2m. Then we have b characteristic roots: i d . leading to 2m complex exponential solutions: e(b/2m+id )t , e(b/2mid )t . The basic real solutions are ebt/2m cos(d t) and ebt/2m sin(d t). The general real solution is found by taking linear combinations of the two basic solutions, that is: x (t) = c1 ebt/2m cos(d t) + c2 ebt/2m sin(d t)

Under, Over and Critical Damping

OCW 18.03SC

or x (t) = ebt/2m (c1 cos(d t) + c2 sin(d t)) = Aebt/2m cos(d t ). (3)

Lets analyze this physically. When b = 0 the response is a sinusoid. Damping is a frictional force, so it generates heat and dissipates energy. When the damping constant b is small we would expect the system to still oscillate, but with decreasing amplitude as its energy is converted to heat. Over time it should come to rest at equilibrium. This is exactly what we see in (3). The factor cos(d t ) shows the oscillation. The exponential factor ebt/2m has a negative exponent and therefore gives the decaying amplitude. As t , the exponential goes asymptotically to 0, so x (t) also goes asympotically to its equilibrium position x = 0. We call d the damped angular (or circular) frequency of the system. This is sometimes called a pseudo-frequency of x (t). We need to be careful to call it a pseudo-frequency because x (t) is not periodic and only periodic functions have a frequency. Nonetheless, x (t) does oscillate, crossing x = 0 twice each pseudo-period. Example 1. Show that the system x + 1x + 3x = 0 is underdamped, nd its damped angular frequency and graph the solution with initial conditions . x (0) = 1, x (0) = 0. Solution. Characteristic equation: s2 + s + 3 = 0. Characteristic roots: 1/2 i 11/2. Basic real solutions: et/2 cos( 11 t/2), et/2 sin( 11 t/2). General solution: x (t) = et/2 (c1 cos( 11 t/2) + c2 sin( 11 t/2)) = Aet/2 cos( 11 t/2 ). Since the roots have nonzero imaginary part, the system is underdamped. The damped angular frequency is d = 11/2. The initial conditions are satised when c1 = 1 and c2 = 1/ 11. So, 1 t/2 x (t) = e cos( 11 t/2) + sin( 11 t/2) 11 12 = et/2 cos( 11 t/2 ), 11 where = tan1 (1/ 11).

..

Under, Over and Critical Damping

OCW 18.03SC

Figure 1: The damped oscillation for example 1. Case (ii) Overdamping (distinct real roots) If b2 > 4mk then the term under the square root is positive and the characteristic roots are real and distinct. In order for b2 > 4mk the damping constant b must be relatively large. One extremely important thing to notice is that in this case the roots are both negative. You can see this by looking at the formula (2). The term under the square root is positive by assumption, so the roots are real. Since b2 4mk < b2 the square root is less than b and therefore the root b + b2 4mk < 0. The other root is clearly negative. Now we use the roots to solve equation (1) in this case. b + b2 4mk b b2 4mk Characteristic roots: r1 = , r2 = . 2m 2m Exponential solutions: er1 t , er2 t . General solution: x ( t ) = c 1 e r1 t + c 2 e r2 t . Lets analyze this physically. When the damping is large the frictional force is so great that the system cant oscillate. It might sound odd, but an unforced overdamped harmonic oscillator does not oscillate. Since both exponents are negative every solution in this case goes asymptotically to the equilibrium x = 0. At the top of many doors is a spring to make them shut automatically. The spring is damped to control the rate at which the door closes. If the damper is strong enough, so that the spring is overdamped, then the door just settles back to the equilibrium position (i.e. the closed position) without oscillating which is usually what is wanted in this case. Example 2. Show that the system x + 4x + 3x = 0 is overdamped and . graph the solution with initial conditions x (0) = 1, x (0) = 0. Which root controls how fast the solution returns to equilibrium? 3

..

Under, Over and Critical Damping Solution. Characteristic equation: s2 + 4s + 3 = 0. Characteristic roots: (this factors) 1, 3. Exponential solutions: et , e3t . General solution: x ( t ) = c 1 e t + c 2 e 3t .

OCW 18.03SC

Because the roots are real and different, the system is overdamped. The intial conditions are satised when c1 = 3/2, c2 = 1/2. So, x (t) = 3et /2 e3t /2.

x 0.0 0 0.2 0.4

0.6

0.8

1.0

2 t

Figure 2: The overdamped graph for example 2. goes to 0 more slowly than e3t/2 it controls the rate at which x goes to 0. (Remember, it is the term that goes to zero slowest term that controls the rate.) Case (iii) Critical Damping (repeated real roots) If b2 = 4mk then the term under the square root is 0 and the characteristic polynomial has repeated roots, b/2m, b/2m. Now we use the roots to solve equation (1) in this case. We have only one exponential solution, so we need to multiply it by t to get the second solution. Basic solutions: ebt/2m , tebt/2m . General solution: x (t) = ebt/2m (c1 + c2 t). As in the overdamped case, this does not oscillate. It is worth noting that for a xed m and k, choosing b to be the critical damping value gives the fastest return of the system to its equilibrium position. In engineering design this is often a desirable property. This can be seen by considering the roots, but we will not go through the algebra that shows this. (See gure (4).) 4 Because et

Under, Over and Critical Damping

OCW 18.03SC

Example 3. Show that the system x + 4x + 4x = 0 is critically damped and . graph the solution with initial conditions x (0) = 1, x (0) = 0. Solution. Characteristic equation: s2 + 4s + 4 = 0. Characteristic roots: (this factors) 2, 2. Exponential solutions: (only one) e2t . General solution: x ( t ) = e 2t ( c 1 + c 2 t ). Because the roots are repeated, the system is critically damped. The intial conditions are satised when c1 = 1, c2 = 2. So, x ( t ) = e 2t ( 1 + 2 t ).

..

x 0.0 0 0.2 0.4

0.6

0.8

1.0

2 t

Figure 3: The critically damped graph for example 3. Notice that qualitatively the graphs for the overdamped and critically damped cases are similar. The following gure shows plots for solutions to x + bx + x = 0 with . initial conditions x (0) = 1, x (0) = 0. The three plots are b = 1 underdamped; b = 2 critically damped (dashed line); b = 3 overdamped. Notice that the critically damped curve has the fastest decay.

..

x1 -0.2 0 0.2

0.6

1.0

4 t

Figure 4: Plots of solutions to x + bx + x = 0.

..

MIT OpenCourseWare http://ocw.mit.edu

Fall 2011

18.03SC Differential Equations

For information about citing these materials or our Terms of Use, visit: http://ocw.mit.edu/terms.

Introduction
We will continue our study of the mass-spring-dashpot system, governed by the differential equation mx + kx + bx = Fext (t). Remember that m represents the mass of the dashpot, k the strength of the spring, and b the damping. Fext (t) represents some external driving force. Weve already seen how to solve this equation if there is no driving force, i.e., if we have mx + kx + bx = 0. We will now discuss how to handle certain kinds of external driving functions, namely exponential and sinusoidal driving. We will nd a general formula to handle these cases, and touch on the phenomenon of resonance, a very important concept which well discuss in more detail in a few lectures. The method of superposition, which we saw already, will be an important tool for us again.

MIT OpenCourseWare http://ocw.mit.edu

Fall 2011

18.03SC Differential Equations

For information about citing these materials or our Terms of Use, visit: http://ocw.mit.edu/terms.

Superposition 1. Superposition I
We saw the principle of superposition already, for rst order equations. For example, we saw that if y1 is a solution to y + 4y = sin(3t) and y2 a solution to y + 4y = 2, then y1 + y2 is a solution to y + 4y = sin(3t) + 2. Superposition will be useful for us again, though now we will use it in two slightly different ways. The rst version we already used in a previous session, but lets state it carefully and explicitly: Superposition I: If y1 and y2 are solutions of a homogeneous linear equation, then so is any linear combination; that is, for any constants c1 and c2 , the function y3 = c1 y1 + c2 y2 will also be a solution. Example. Consider the ODE t2 y + ty 4y = 0. This is homogeneous, since the constant term (the one not involving y or any of its derivatives) is zero. You can easily check by substitution that y1 (t) = t2 and y2 (t) = 1/t2 are both solutions. Thus y ( t ) = c1 t2 + c2 / t2 is a solution for any c1 and c2 . Notice that we didnt need the differential equation to have constant coefcients: linearity and homogeneity is enough. If the equation is of second order with two solutions y1 and y2 such that neither is a multiple of the other, then c1 y1 + c2 y2 will be the general solution. It has the right number of parameters. The restriction on the solutions is to make sure that they are really different solutions, for instance, in the above example, it would be incorrect to take y1 = t2 and y2 = 3t2 , and then claim that y ( t ) = c1 t2 + c2 3t2 = ( c1 + 3c2 ) t2 is the general solution.

Superposition

OCW 18.03SC

2. Superposition II
Now consider the linear second order equation mx + bx + kx = Fext (t), and its associated homogeneous equation mx + bx + kx = 0. (2) (1)

Superposition II: Suppose x p is any solution to (1). If xh is any solution to (2), then x = x p + xh is again a solution to (1). This is similar to the way we used superposition for rst order equations. To prove this, we just need to substitute x into (1) and check that it really is a solution: mx + bx + kx = m( xh + x p ) + b( xh + x p ) + k ( xh + x p )
+ mx p ) + (bxh + bx p ) + (kxh + kx p ) = (mxh + bxh + kxh ) = (mxh

+ +

(mx p + bx p + kx p )
Fext .

=
So indeed, it is a solution.

An important fact: if xh is the general solution to (2) (so it should have two parameters) then x p + xh is the general solution to (1). Well see an example of this shortly. This proof works for linear equations of any order. For example, we already saw it as a consequence of the method of integrating factors for rst order equations. Weve already seen how to nd the general solution to the associated homogeneous equation (2) using the characteristic equation. Thus to nd the general solution to (1), we simply need to do is nd a single solution to this particular equation. This is what well discuss next.

MIT OpenCourseWare http://ocw.mit.edu

Fall 2011

18.03SC Differential Equations

For information about citing these materials or our Terms of Use, visit: http://ocw.mit.edu/terms.

Exponential Input
Lets consider the case where the driving function is an exponential Ae at , where A and a are constants. We will allow A and a to be complex, so this will also be useful for dealing with sinusoidal driving functions, e.g., Fext (t) = 3 cos(2t). Lets try to solve a particular example. Example. Find the general solution to x + 8x + 7x = 9e2t . We have no method yet, but we can at least try to guess (the method of optimism). We hope that we can get a solution which is similar in form to the right hand side. So lets guess x (t) = Ae2t , where A is an unknown constant. Substituting we get x + 8x + 7x = 4 Ae2t + 16 Ae2t + 7 Ae2t = 27 Ae2t . 1 2t e . 3 We are not done yet, since we want the general solution. Now we only need to solve the homogeneous equation, and then we can apply Superposition II. The associated homogeneous equation is Success! Setting A = 1/3, we have a solution x p = x + 8x + 7x = 0. The characteristic polynomial is p(r ) = r2 + 8r + 7 = (r + 7)(r + 1). The roots are 7 and 1, so we deduce that x h = c 1 e 7t + c 2 e t is the general solution to the homogeneous equation. Thus the general solution to the original equation is 1 x = x h + x p = c 1 e 7t + c 2 e t + e 2t . 3

Exponential Input

OCW 18.03SC

Make sure you understand why the rst two terms have a parameter and why the third does not. We can try this same approach to the general form mx + bx + kx = Be at , where B and a are constants. Again, we use the method of optimism, and try a solution of the form x (t) = Ae at , A being an unknown constant. Substituting, we nd mx + bx + kx = m a2 Aeet + b aAe at + k Ae at

= (ma2 + ba + k) Aeet .
Thus to be a solution, we must set A= B . ma2 + ba + k

Notice that the denominator in this expression can be written succinctly as just p( a), where p is the characteristic polynomial we saw in the context of the homogeneous equation. We have Exponential Response Formula (ERF). Consider the second order equation mx + kx + bx = Be at , and let p(r ) = mr2 + kr + br be its characteristic polynomial. Then x (t) = B at e p( a)

is a particular solution, as long as p( a) = 0. (You might worry about the restriction p( a) = 0and you should. Well come back to that shortly.) This formula works essentially unchanged for higher order equations toowell see that in a future session. Dont forget that this only gives a single particular solution. For the general solution, you must still solve the associated homogeneous problem and then apply Superposition II.

MIT OpenCourseWare http://ocw.mit.edu

Fall 2011

18.03SC Differential Equations

For information about citing these materials or our Terms of Use, visit: http://ocw.mit.edu/terms.

Sinusoidal Input
The exponential response formula works perfectly even if the number a in the exponential is complex. Lets use this to solve problems with a sinusoidal driving. Example. Find the general solution to x + 8x + 7x = 9 cos(2t). We begin by using complex replacement and considering instead the equation (1) z + 8z + 7z = 9e2it . Now we can apply the exponential response formula to obtain as a particular solution, z p (t) = 9 2it e p ( 2i ) 9 = e2it 2 (2i ) + 16i + 7 9 = e2it . 3 + 16i

Be careful with signs when you do these calculations! Remember i2 = 1. To get a particular solution to (1), we must take the real part. We prefer the solution in amplitude-phase form, so we write 3 + 16i = 265ei where = tan1 (16/3). Thus x p (t) = (z p (t)) = 1 265

cos(2t ).

To get the general solution we must add the general solution of the homogeneous problem, which we already saw: x h ( t ) = c 1 e 7t + c 2 e t . Thus we obtain the general solution x = x p + x h = c 1 e 7t + c 2 e t + 9 265 cos(2t ).

Sinusoidal Input

OCW 18.03SC

Notice that from the exponential response formula, the amplitude of the particular solution is given by A= B . | p( a)|

The ratio between the amplitude B of the driving force and the resulting amplitude of the solution is called the gain. So the gain is given by the formula gain = 1/| p( a)|. Lets apply the above sequence of steps to the general case of a sinusoidal driving: mx + bx + kx = B cos( t). The complixied equation is mz + bz + kz = Bei t . From the exponential response formula with a = i , a particular solution is B i t zp = e . p (i ) Converting to polar form and then taking the real part, we get xp = B cos( t ), | p (i ) |

where = arg( p(i )). Notice that since a = i , the gain is given by 1/| p( a)| = 1/| p(i )|.

MIT OpenCourseWare http://ocw.mit.edu

Fall 2011

18.03SC Differential Equations

For information about citing these materials or our Terms of Use, visit: http://ocw.mit.edu/terms.

Generalized Exponential Response Formula


We can also solve an LTI DE p( D ) x = q(t) with exponential input q(t) = Be at even when p( a) = 0. The answer is given by the following generalized Exponential Response formula (the proof of which we postpone to the session on Linear Operators. Generalized Exponential Response Formula. Let p( D ) be a polynomial operator with constant coefcients, and p(s) its s-th derivative. Then p( D ) x = Be at , has the particular solution Be at p( a) Bte at p ( a) xp = Bt2 e at p ( a) ... Bts e at (s) p ( a) where a is real or complex

if p( a) = 0 if p( a) = 0 and p ( a) = 0 if p( a) = p ( a) = 0 and p ( a) = 0

if a is an s-fold zero

Note: Later when we cover resonance the case p( a) = 0, p ( a) = 0 will be called the Resonant Response Formula Example 1. Find a particular solution to the equation

..

x + 8x + 15 x = e5t

Solution. The characteristic polynomial is p(r ) = r2 + 8r + 15. Since p(5) = 0 we need to use the generalized ERF. Computing p (r ) = 2r + 8, which implies p (5) = 2. Therefore the generalized ERF gives te5t te5t xp = = . p ( 5) 2 Example 2. Find a particular solution to

..

x + 2x + 2x = et cos t.

Generalized Exponential Response Formula

OCW 18.03SC

Solution. First we complexify the equation

..

z + 2 z + 2 z = e ( 1+ i ) t ,

where

x = Re(z).

The characteristic polynomial is p(r ) = r2 + 2r + 2. Computing, p(1 + i ) = (1 + i )2 + 2(1 + i ) + 2 = 0, p (r ) = 2r + 2, p ( 1 + i ) = 2i .

Since p(1 + i ) = 0 we use the generalized ERF zp = te(1+i)t te(1+i)t tet (cos t + i sin t) = = p ( 1 + i ) 2i 2i

Finally we take the real part to get x p = Re(z p ) = tet sin t . 2

MIT OpenCourseWare http://ocw.mit.edu

Fall 2011

18.03SC Differential Equations

For information about citing these materials or our Terms of Use, visit: http://ocw.mit.edu/terms.

The Simple Harmonic Oscillator


Lets investigate the case of no damping (b = 0) a bit more carefully: mx + kx = Fext (t). Lets dene the parameter n =

k/m

and use this to rewrite the equation in terms of m and n :


2 m( x + n x ) = Fext (t).

We saw this form of the equation in the previous session. Recall that the subscript n stands for natural. To remind ourselves why, consider the solution in the case of no driving force, i.e. Fext (t) = 0. The characteristic 2 ), has roots i . Thus, the general solution is equation p(r ) = m(r2 + n n xh = c1 cos(n t) + c2 sin(n t). So even without a driver, if we give the system a nudge it will oscillate at its natural frequency n . Now lets add some sinusoidal input: Fext (t) = B cos( t). Using complex replacement, we must nd a particular solution to
2 z) = Bei t . m(z + n

Applying the exponential response formula with a = i , we get zp = Taking the real part, x p = (z p ) =
2 m ( n

B i t B ei t . e = 2 p (i ) m ( n 2 )

B cos( t) 2 )

is a particular solution. All in all, our general solution is x = c1 cos( t) + c2 sin( t) +


2 m ( n

B cos( t). 2 )

Again, the gain is given by 1/| p( a)| = 1/| p(i )|.

The Simple Harmonic Oscillator

OCW 18.03SC

Example. Lets take m = 1, B = 1 and n = 2, and investigate the resulting particular solution as we vary , the frequency of the driving. The following gure shows the situation for = 3, 2.5 and 2.1.

3 2 1 0 1 2 30 2 4 6 8

=3 =2.5 =2.1

10

Fig. 1. Solutions for different values of . What do you think happens as approaches the natural frequency? Its no surprise that the solution breaks down when = n . This situation is called pure resonance and we will investigate it in detail in an upcoming session. Notice that it corresponds to the case p( a) = 0 in the exponential response formula, since p(i n ) = 0. For now, well note that it can be checked that the following is a solution: x p (t) = B t sin( t). 2m

Notice that the extra factor of t before the sine term. So the amplitude grows with time, as shown in the following gure.

The Simple Harmonic Oscillator

OCW 18.03SC

3 2 1 0 1 2 30 2 4 6 8 10

Fig. 2. Solution at pure resonance. More generally, the following is the counterpart to the exponential response formula in the pure resonance case, when p( a) = 0. Resonant Response Formula (RRF). Consider the second order equation mx + kx + bx = Be at , with characteristic polynomial p. Then if p( a) = 0 and p ( a) = 0, then x (t) = is a particular solution. Once we develop a small amount of algebraic machinery we will be able to give a simple proof of this formula. B p ( a) te at

MIT OpenCourseWare http://ocw.mit.edu

Fall 2011

18.03SC Differential Equations

For information about citing these materials or our Terms of Use, visit: http://ocw.mit.edu/terms.

Resonant Response Formula


Exercise. Find the general solution to x + 8x + 7x = 2et . Answer. The characteristic polynomial is p(r ) = r2 + 8r + 7. This has roots 7 and 1. Thus p(1) = 0 and we cant use the exponential response formula. We must use the resonant response formula instead. So we get 2 t xp = tet = 1 3 te p ( 1) as a particular solution. The general solution to the associated homogeneous problem is x h = c 1 e 7t + c 2 e t , and the nal solution is
t x = x h + x p = c 1 e 7t + c 2 e t + 1 3 te .

MIT OpenCourseWare http://ocw.mit.edu

Fall 2011

18.03SC Differential Equations

For information about citing these materials or our Terms of Use, visit: http://ocw.mit.edu/terms.

Resonant Response Formula


Exercise. Find the general solution to x + 8x + 7x = 2et .

MIT OpenCourseWare http://ocw.mit.edu

Fall 2011

18.03SC Differential Equations

For information about citing these materials or our Terms of Use, visit: http://ocw.mit.edu/terms.

Introduction
In this session we will continue to develop the important case of linear constant coefcient DEs with sinusodial input. We will start by dening stability. In a stable system the response to a periodic input will be essentially periodic. The word essentially indicates that there will be some transient behavior depending on the initial conditions, but this will die away over time. For constant coefcient equations the important fact is that stability is equivalent to all the characteristic roots being negative, or if they are complex having negative real part. This will turn out to be a simple consequence of the fact the if a is negative then e at goes to 0 as t grows to innity. The other main goal of this session is to introduce the operator D and the notation p( D ). We will use this to rephrase constant coefcient differential equations and to write elegant formulas for the gain, phase lag and response to sinusoidal input.

MIT OpenCourseWare http://ocw.mit.edu

Fall 2011

18.03SC Differential Equations

For information about citing these materials or our Terms of Use, visit: http://ocw.mit.edu/terms.

Stability 1. The Notion of Stability


A system is called stable if its long-term behavior does not depend signicantly on the initial conditions. It is an important result of mechanics that any system of masses connected by springs (damped or undamped) is a stable system. In network theory, there is a similar result: any RLC-network gives a stable system. In these notes, we investigate for the simplest such systems why this is so. In terms of differential equations, the simplest spring-mass system or RLC-circuit is represented by an ODE of the form a0 y + a1 y + a2 y = r (t), ai constants, t = time. (1)

For the spring-mass system, y is the displacement from equilibrium position, and r (t) is the externally applied force. For the RLC-circuit, y represents the charge on the capacitor, and r (t) is the electromotive force E (t) applied to the circuit (or else y is the current and r (t) = E ). By the theory of inhomogeneous equations, the general solution to (1) has the form y = c1 y1 + c2 y2 + y p , c1 , c2 arbitrary constants, (2)

where y p is a particular solution to (1), and c1 y1 + c2 y2 is the complementary function, i.e., the general solution to the associated homogeneous equation (the one having r (t) = 0). The initial conditions determine the exact values of c1 and c2 . So from (2), the system modeled by (1) is stable

for every choice of c1 , c2 , c1 y1 + c2 y2 0 as t .

(3)

Often one applies the term stable to the ODE (1) itself, as well as to the system it models. We shall do this here. If the ODE (1) is stable, the two parts of the solution (2) are named: y p = steady-state solution c1 y1 + c2 y2 = transient; (4)

Stability

OCW 18.03SC

the whole solution y(t) and the right side r (t) of (1) are described by the terms y(t) = response r (t) = input. From this point of view, the driving force is viewed as the input to the spring-mass system, and the resulting motion of the mass is thought of as the response of the system to the input. So what (2) and (4) are saying is that this response is the sum of two terms: a transient term, which depends on the initial conditions, but whose effects disappear over time; and a steadystate term, which represents more and more closely the response of the system as time goes to , no matter what the initial conditions are.

2. Conditions for Stability: Second Order Equations


We now ask under what circumstances the ODE (1) will be stable. In view of the denition, together with (2) and (3), we see that stability concerns just the behavior of the solutions to the associated homogeneous equation a0 y + a1 y + a2 y = 0 ; (5) the forcing term r (t) plays no role in deciding whether or not (1) is stable. There are three cases to be considered in studying the stability of (5); they are summarized in the table below, and based on the roots of the characteristic equation a0 r 2 + a1 r + a2 = 0 . (6) roots r1 = r2 r1 = r2 a ib solution to ODE c1 e
at r1 t

condition for stability r1 < 0, r2 < 0 r1 < 0 a<0

+ c2 e

r2 t

e ( c1 + c2 t ) e (c1 cos bt + c2 sin bt)

r1 t

The rst two columns of the table should be familiar, from your work in solving the linear second-order equation (5) with constant coefcients. Let us consider the third column, therefore. In each case, we want to show that if the condition given in the third column holds, then the criterion (3) for stability will be satised. Consider the rst case. If r1 < 0 and r2 < 0, then it is immediate that the solution given tends to 0 as t . On the other hand, if say r1 0, then the solution er1 t tends to (or to 1 if r1 = 0). This shows the ODE (5) is not stable, since not all solutions tend to 0 as t . 2

Stability

OCW 18.03SC

In the second case, the reasoning is the same, except that here we are using the limit lim tert = 0 r < 0
t

For the third case, the relevant limits are (assuming b = 0 for the second limit):
t

lim e at cos bt = 0 a < 0,

lim e at sin bt = 0 a < 0 .

The three cases can be summarized conveniently by one statement: Stability criterion for second-order ODEs root form all roots of a0 r2 + a1 r + a2 = 0 have negative real part. (7) Alternatively, one can phrase the criterion in terms of the coefcients of the ODE; this is convenient, since it doesnt require you to calculate the roots of the characteristic equation. a0 y + a1 y + a2 y = r (t) is stable

Stability criterion for second order ODEs coefcient form. Assume a0 > 0. a0 y + a1 y + a2 y = r (t) is stable

a0 , a1 , a2 > 0 .

(8)

The proof is left as an exercise; it is based on the quadratic formula.

3. Stability of Higher Order ODEs


The stability criterion in the root form (7) also applies to higher-order ODEs with constant coefcients:

( a 0 D n + a 1 D n 1 + . . . + a n 1 D + a n ) y = f ( t ) .

(9)

These model more complicated spring-mass systems and multi-loop RLC circuits. The characteristic equation of the associated homogeneous equation is a 0 r n + a 1 r n 1 + . . . + a n 1 r + a n = 0 . (10) The real and complex roots of the characteristic equation give rise to solutions to the associated homogeneous equation just as they do for second order equations. (For a k-fold repeated root, one gets additional solutions by multiplying by 1, t, t2 , . . . tk1 .) 3

Stability

OCW 18.03SC

The reasoning which led to the above stability criterion for secondorder equations applies to higher-order equations just as well. The end result is the same: Stability criterion for higher-order ODEs root form ODE (9) is stable

all roots of (10) have negative real parts;

(11)

that is, all the real roots are negative, and all the complex roots have negative real part. There is a stability criterion for higher-order ODEs which uses just the coefcients of the equation, but it is not so simple as the one (8) for secondorder equations. We will not use this in the course, but it is worth seeing. The key point is that the stability of the system can be found without nding the roots of a higher order polynomial. Without loss of generality, we may assume that a0 > 0. Then it is not hard to prove that ODE (9) is stable

a0 , . . . , a n > 0 .

(12)

The converse is not true. For an implication , the coefcients must satisfy a more complicated set of inequalities, which we give without proof, known as the Routh-Hurwitz conditions for stability stable Assume a0 > 0; ODE (9) is

in the determinant below, all of the n principal minors (i.e., the subdeterminants in the upper left corner having sizes respectively 1, 2, . . . , n) are > 0 when evaluated. a1 a0 0 0 0 0 ... 0 a3 a2 a1 a0 0 0 ... 0 a5 a4 a3 a2 a1 a0 . . . 0 (13) . . . . . . . . . . . . . . . . . . . . . . . . a 2n 1 a 2n 2 a 2n 3 a 2n 4 . . . . . . . . . a n In the determinant, we dene ak = 0 if k > n; thus for example, the last row always has just one non-zero entry, an .

MIT OpenCourseWare http://ocw.mit.edu

Fall 2011

18.03SC Differential Equations

For information about citing these materials or our Terms of Use, visit: http://ocw.mit.edu/terms.

p(D) Notation
We start by recalling the basic results we have developed so far. We are studying solutions x = x (t) of the linear constant coefcient DE an x (n) + an1 x (n1) + ... + a1 x + a0 x = q(t) with characteristic polynomial p(s) = an sn + an1 sn1 + ... + a1 s + a0 and homogeneous case (q = 0) an x (n) + an1 x (n1) + ... + a1 x + a0 x = 0 (H) (P) (I)

Notice that the left-hand sides of (I) and (H) have the same form. For this reason it will be useful to have a more compact notation. This is in fact provided by an important mathematical tool called operators. We will study these in more detail in the session on linear operators. For now, we d just note that we can write D = dt for the operation of differentiation applied to functions of t, i.e. if x = x (t), then Dx = dx dt , the rst derivative of x . 2 d In the same way we can write D2 = dt2 for differentiation twice, i.e. D2 x =
d the second derivative of x = x (t); similarly D3 = dt 3 for differentian n tion three times, and so on. Then if p(s) = an s + an1 + s 1 + ... + a1 s + a0 is any polynomial, we can write d2 x , dt2
3

p( D ) = an D n + an1 D n1 + ... + a1 D + a0 . The DEs (I) and (H) then become the statements p( D ) x = q p( D ) x = 0 respectively an efcient way to write the DEs indeed! Now lets recall the basics, but with our new operator notation. For the homogeneous case we have the following key theorem. Transience theorem. All solutions x = x (t) to the linear homogeneous constant coefcient DE p( D ) x = 0 (H) (I) (H)

p(D) Notation

OCW 18.03SC

decay to zero as t exactly when all roots r of the characteristic polynomial p(s) have negative real part. In this case the solutions to (H) are called transients. By superposition, all solutions to (I) then converge to the same solution as t gets large, and we say that the DE is stable. If we have a system modeled by a stable equation, but we are only interested in what it looks like after the transients have died down, we can ignore the initial condition:
input signal System steady state output signal xp

So in this case we are looking for particular solutions x p . If the input signal is sinusoidal, then we know from the results we obtained in the last session that there will be a particular solution which is also sinusoidal. This is the unique steady state solution which is periodic and it is of particular importance in many applications. Lets review how it goes and then introduce some useful denitions and terminology that apply to these solutions. The starting point is the Exponential Response Formula (ERF), which in the operator notation reads p( D ) x = Be at and has a solution xp = provided p( a) = 0. As we saw in the last session, the ERF and complex replacement can be used to obtain the periodic solution x p to the DE with sinusoidal input, i.e. p( D ) x = B cos( t). This is done as follows. Since B cos( t) = B Re(ei t ) we look at the complex equation p( D )z = Bei t , so x = Re(z). The exponential response formula gives B i t zp = e x p = B Re p (i ) 2 ei t p (i ) Be at p( a)

p(D) Notation

OCW 18.03SC

Thus, xp =

B cos( t ), | p (i ) |

where = Arg( p(i )). The solution x p is the particular steady-state periodic solution. Letse examine the relation between the periodic input q(t) = B cos( t) B and its periodic output x p (t) = cos( t ). We see that the am| p (i ) | B plitude of the input B is scaled and becomes the amplitude of the | p (i ) | output. We also see that the output sinusoid x p (t) is shifted by an angle = Arg( p(i )) relative to the input sinusoid q(t). This motivates the following denitions: for a CC linear DE P( D ) x = q(t) with sinusoidal input q(t).

Denition: 1. The gain is dened to be the the ratio of the amplitude of the output sinusoid to the amplitude of the input sinusoid. 2.The phase lag is dened to be the angle by which the output sinusoid is shifted relative to the input sinusoid. In the special case q(t) = B cos t which we solved above, we have that the gain g and the phase lag are g = 1 , | p (i ) | = Arg( p(i )).

When solving using p( D ) x = B cos t by complex replacement and the ERF we have x p = Re(z p ) where z p (t) is the complex solution to p( D )z p = Beiwt . That is B i t zp = e . p (i ) For this reason, we dene the complex gain in this case as 1 . p (i )

Note that the gain and the phase lag depend only on the frequency of the of the input signal (as well as on the system p( D ) of course).

MIT OpenCourseWare http://ocw.mit.edu

Fall 2011

18.03SC Differential Equations

For information about citing these materials or our Terms of Use, visit: http://ocw.mit.edu/terms.

Example
Lets apply what we just learned to a specic example. First, recall the basics. For the real homogeneous constant coefcient linear DE with sinusoidal input p( D ) x = B cos( t) we have the unique real periodic solution x p = B Re ei t p (i )

B cos( t ) | p (i ) |
1 , p (i )

where = Arg( p(i )). In this case the complex gain is phase lag is = Arg( p(i )). Example. Find the periodic solution to x + x + 2x = cos t.

and the

Solution. p(s) = s2 + s + 2, = 1, B = 1. i p ( i ) = p ( i ) = i 2 + i + 2 = 1 + i + 2 = 1 + i |1 + i | e i = 2e 4 , since = Arg(1 + i ) = tan1 (1/1) = . 4 1 1 Thus, Complex gain = = . p (i ) 1+i 1 1 Gain = = . | p (i ) | 2 Phase lag = = Arg( p(i )) = . 4 1 Periodic solution = x p = cos(t ). 4 2 Looking at the output x p in relation to the input signal we see q(t) = cos t. 1 1 The amplitude of x p = amplitude of q so the gain is . We also see 2 2 that x p lags behind q by /4 radians, so the phase lag = 4 .

MIT OpenCourseWare http://ocw.mit.edu

Fall 2011

18.03SC Differential Equations

For information about citing these materials or our Terms of Use, visit: http://ocw.mit.edu/terms.

Damping and Phase Lag


Quiz: Consider the equation

..

x + bx + 2x = cos(t).

If the damping constant b starts at 1 and is increased, what happens to the phase lag? Choices: a) It increases. b) It decreases. c) It stays the same.

Answer: The phase lag increases: The phase lag is the argument of p(i ) = 1 + bi. As b increases the argument increases.

MIT OpenCourseWare http://ocw.mit.edu

Fall 2011

18.03SC Differential Equations

For information about citing these materials or our Terms of Use, visit: http://ocw.mit.edu/terms.

Damping and Phase Lag


Quiz: Consider the equation

..

x + bx + 2x = cos(t).

If the damping constant b starts at 1 and is increased, what happens to the phase lag? Choices: a) It increases. b) It decreases. c) It stays the same. Pick what you think is the correct choice and then look at the answer.

MIT OpenCourseWare http://ocw.mit.edu

Fall 2011

18.03SC Differential Equations

For information about citing these materials or our Terms of Use, visit: http://ocw.mit.edu/terms.

Damping and Phase Lag


Quiz: Consider the equation

..

x + bx + 2x = cos(t).

If the damping constant b starts at 1 and is increased, what happens to the phase lag? Think about your answer and then look at the choices.

MIT OpenCourseWare http://ocw.mit.edu

Fall 2011

18.03SC Differential Equations

For information about citing these materials or our Terms of Use, visit: http://ocw.mit.edu/terms.

Damping and Amplitude


Quiz: Consider the equation

..

x + bx + 2x = cos(t).

If the damping constant b starts at 1 and is increased, what happens to the amplitude of the solution? Choices: a) It increases. b) It decreases. c) It stays the same.

Answer: The amplitude decreases. 1 . Since | p(i )| = |1 + bi | increases as b The amplitude of the solution is | p( i )| increases, the amplitude
1 | p (i ) |

decreases.

MIT OpenCourseWare http://ocw.mit.edu

Fall 2011

18.03SC Differential Equations

For information about citing these materials or our Terms of Use, visit: http://ocw.mit.edu/terms.

Damping and Amplitude


Quiz: Consider the equation

..

x + bx + 2x = cos(t).

If the damping constant b starts at 1 and is increased, what happens to the amplitude of the solution? Choices: a) It increases. b) It decreases. c) It stays the same. Pick what you think is the correct choice and then look at the answer.

MIT OpenCourseWare http://ocw.mit.edu

Fall 2011

18.03SC Differential Equations

For information about citing these materials or our Terms of Use, visit: http://ocw.mit.edu/terms.

Damping and Amplitude


Quiz: Consider the equation

..

x + bx + 2x = cos(t).

If the damping constant b starts at 1 and is increased, what happens to the amplitude of the solution? Think about your answer and then look at the choices.

MIT OpenCourseWare http://ocw.mit.edu

Fall 2011

18.03SC Differential Equations

For information about citing these materials or our Terms of Use, visit: http://ocw.mit.edu/terms.

Introduction
In this section we show how to solve the constant coefcient linear ODE with polynomial input. That is, p ( D ) y = q ( x ), where q( x ) is polynomial.

Any function can be approximated in a suitable sense by polynomial functions, and this makes polynomials an important tool. In addition the technique we will learn, called the method of undetermined coefcents, is a good example of a general class of method widely used in mathematics, which go as follows: make an intelligent guess as to the form of the solution, leaving as letters any unknowns; plug this trial solution into the equation to be solved; and use it to determine the unknown values. Hence the slightly inaccurate name undetermined coefcents in this case no worries, they wont be undetermined for long!

MIT OpenCourseWare http://ocw.mit.edu

Fall 2011

18.03SC Differential Equations

For information about citing these materials or our Terms of Use, visit: http://ocw.mit.edu/terms.

Polynomial Input: The Method of Undetermined Coefcients 1. The Basic Result


A polynomial is a function of the form q ( x ) = a n x n + a n 1 x n 1 + + a 0 . The largest k for which ak = 0 is the degree of q( x ). (The zero function is a polynomial too, but it doesnt have a degree.) Note that q(0) = a0 and q (0) = a1 . Here is the basic fact about the response of an LTI system with characteristic polynomial p(s) to polynomial signals: Theorem. (Undetermined coefcients) If p(0) = 0, and q( x ) is a polynomial of degree n, then p( D )y = q( x ) has exactly one solution which is a polynomial, and it is of degree n. The best way to see this, and to see how to compute this polynomial particular solution, is by examples.

2. The Method of Undetermined Coefcients


Given the linear time invariant (LTI) DE p( D )y = q( x ) with q( x ) is a polynomial of degree n, the Undetermined Coefcient (UC) solution method, as we discussed in the previous note, is to assume a particular solution of the form y p = h( x ), where h( x ) is a polynomial of degree n with unknown (undetermined) coefcients, and then to nd the coefcients by substituting y p into the ODE. Its important to do the work systematically; we suggest following the format given in the following example. Example 1. Find a particular solution y p to y + 3y + 4y = 4x2 2x. Solution. Our trial solution is y p = Ax2 + Bx + C; we format the work as follows. The lines show the successive derivatives; multiply each line by the factor given in the ODE, and add the equations, collecting like powers of x as you go. The fourth line shows the result; the sum on the left takes into account that y p is supposed to be a particular solution to the given

Polynomial Input: The Method of Undetermined Coefcients OCW 18.03SC

ODE.

4 y p = Ax2 + Bx + C 3 yp = 2 Ax + B
yp =
2 2

2A

4 x 2 x = (4 A ) x + (4 B + 6 A ) x + (4C + 3 B + 2 A ). Equating like powers of x in the last line gives the three equations 4 A = 4, 4 B + 6 A = 2, 4C + 3 B + 2 A = 0;

solving them in order gives A = 1, B = 2, C = 1, so that y p = x2 2x + 1. Example 2. Solve y + 5y + 4y = 2x + 3. Solution. Guess a trial solution of the form y p = Ax + B (same degree as input). Substitute in DE: yp + 5yp + 4y p = 0 + 5( A) + 4( Ax + B) = 2x + 3. 4 Ax + (5 A + 4 B) = 2x + 3. Equate coefcients: 4 A = 2, 5 A + 4 B = 3. Triangular system is easy to solve: A = 1/2, B = 1/8. 1 yp = 1 2x + 8. Find solution of homogeneous DE: y + 5y + 4y = 0 Char. equation: r2 + 5r + 4 = 0 r = 1, 4 y h = c 1 e t + c 2 e 4t general solution to DE = y = y p + yh . Example 3. Solve y + 5y + 4y = x2 + 3x Solution. Guess a trial solution of the form y p = Ax2 + Bx + C (same degree as input). Substitute this into the DE: yp + 3yp + 4y p = 2 A + 5(2 Ax + B) + 4( Ax2 + Bx + C ) = x2 + 3x

4 Ax2 + (10 A + 4 B) x + (2 A + 5 B + 4C ) = x2 + 3x Equate coefcients: 4 A = 1, 10 A + 4 B = 3, 2 A + 5 B + 4C = 0 Triangular system is easy to solve: A = 1/4, B = 1/8, C = 9/32 1 9 2 yp = 1 4 x + 8 x 32 . Use homogeneous solution from previous example to get the general solution to DE: y = y p + yh .
2

MIT OpenCourseWare http://ocw.mit.edu

Fall 2011

18.03SC Differential Equations

For information about citing these materials or our Terms of Use, visit: http://ocw.mit.edu/terms.

What Can Go Wrong


If the homogeneous DE p( D )y = 0 has polynomial solutions, then the polynomial solution of the inhomogeneous DE p( D )y = q will be of higher degree than the degree of q( x ). We illustrate with an example. Example. Solve y + y = x + 1 Try y p = Ax + B 0 + A = x + 1 cant solve. Problem: the constant term in y + ay + b is 0. Fix: bump all degrees up by order of lowest derivative: try y p = Ax2 + Bx. Substitute: 2 A + (2 Ax + B) = x + 1 2 Equate coeff: 2 Ax + (2 A + B) = x + 1 A = 1/2, B = 0 y p = 1 2x . Example. y + 3y = x2 + x Lowest order derivative is 2 bump up all degrees by 2. Try y p = Ax4 + Bx3 + Cx2 (24 Ax + 6 B) + 3(12 Ax2 + 6 Bx + 2C ) = x2 + x. Equate coefcients: 36 A = 1, 24 A + 18 B = 1, 6 B + 6C = 0 (well skip the algebra).

MIT OpenCourseWare http://ocw.mit.edu

Fall 2011

18.03SC Differential Equations

For information about citing these materials or our Terms of Use, visit: http://ocw.mit.edu/terms.

Solutions to Polynomial Input


Quiz: Which of the following are true about the differential equation 3x (4) + 2x (3) + x x + 4x = 2t2 + 1? Choices: a) It has no polynomial solutions. b) It has exactly one polynomial solution. c) It has many polynomial solutions. d) All its solutions are polynomials. e) We cant say from the information given.

Answer: The answer is b. The method of undetermined coefcients says there will be a particular solution of the form x p = At2 + Bt + C. Therefore there is at least one polynomial solution. The general solution is of the form x = x p + xh , where xh is a homogeneous solution. Since 0 is not a root of the characteristic equation, every (nonzero) homogeneous solution is a combination of exponentials and/or sinusoidal functions. Therefore x is a polynomial only for the case xh = 0. That is, x p is the only polynomial solution.
1 2 By the way, x p = 1 2t + 4t + 5 16 .

MIT OpenCourseWare http://ocw.mit.edu

Fall 2011

18.03SC Differential Equations

For information about citing these materials or our Terms of Use, visit: http://ocw.mit.edu/terms.

Solutions to Polynomial Input


Quiz: Which of the following are true about the differential equation 3x (4) + 2x (3) + x x + 4x = 2t2 + 1? Choices: a) It has no polynomial solutions. b) It has exactly one polynomial solution. c) It has many polynomial solutions. d) All its solutions are polynomials. e) We cant say from the information given. Pick what you think is the correct choice and then look at the answer.

MIT OpenCourseWare http://ocw.mit.edu

Fall 2011

18.03SC Differential Equations

For information about citing these materials or our Terms of Use, visit: http://ocw.mit.edu/terms.

Solutions to Polynomial Input


Quiz: Which of the following are true about the differential equation 3x (4) + 2x (3) + x x + 4x = 2t2 + 1? Think about your answer and then look at the choices.

MIT OpenCourseWare http://ocw.mit.edu

Fall 2011

18.03SC Differential Equations

For information about citing these materials or our Terms of Use, visit: http://ocw.mit.edu/terms.

Solutions to Polynomial Input 2


Quiz: Which of the following are true about the differential equation 3x (4) + 2x (3) + x = 2t2 + 1? Choices: a) It has no polynomial solutions. b) It has exactly one polynomial solution. c) It has many polynomial solutions. d) All its solutions are polynomials. e) We cant say from the information given.

Answer: The answer is c. Because the smallest derivative in the differential operator is 2, the method of undetermined coefcients says we should look for a particular solution of the form x p = At4 + Bt3 + Ct2 . Therefore there is at least one polynomial solution. But, for any D, E the function Dt + E is a homogenous solution. (You can see this directly or because 0 is a double root of the characteristic equation.) Thus, there a lots of polynomial solutions. Since there are nonzero roots of the characteristic equation not every solution is a polynomial. By the way, x p = 1 4 2 3 3 2 t t + t . 12 3 2

MIT OpenCourseWare http://ocw.mit.edu

Fall 2011

18.03SC Differential Equations

For information about citing these materials or our Terms of Use, visit: http://ocw.mit.edu/terms.

Solutions to Polynomial Input 2


Quiz: Which of the following are true about the differential equation 3x (4) + 2x (3) + x = 2t2 + 1? Choices: a) It has no polynomial solutions. b) It has exactly one polynomial solution. c) It has many polynomial solutions. d) All its solutions are polynomials. e) We cant say from the information given. Pick what you think is the correct choice and then look at the answer.

MIT OpenCourseWare http://ocw.mit.edu

Fall 2011

18.03SC Differential Equations

For information about citing these materials or our Terms of Use, visit: http://ocw.mit.edu/terms.

Solutions to Polynomial Input 2


Quiz: Which of the following are true about the differential equation 3x (4) + 2x (3) + x = 2t2 + 1? Think about your answer and then look at the choices.

MIT OpenCourseWare http://ocw.mit.edu

Fall 2011

18.03SC Differential Equations

For information about citing these materials or our Terms of Use, visit: http://ocw.mit.edu/terms.

Introduction
In this session we introduce the concept of an operator and see how they work in general. Then we specialize to the case of differential operators and show how they can be used to simplify the notations (as we already previewed in the session on Gain & Phase Lag) and the calculations used in solving linear constant-coefcient DEs.

MIT OpenCourseWare http://ocw.mit.edu

Fall 2011

18.03SC Differential Equations

For information about citing these materials or our Terms of Use, visit: http://ocw.mit.edu/terms.

Operators
Operators are to functions as functions are to numbers. An operator takes a function, does something to it, and returns this modied function. There are lots of examples of operators around: The shift-by-a operator (where a is a number) takes as input a function f (t) and gives as output the function f (t a). This operator shifts graphs to the right by a units. The multiply-by-h(t) operator (where h(t) is a function) multiplies by h(t): it takes as input the function f (t) and gives as output the function h(t) f (t). You can go on to invent many other operators. In this course the most important operator is: The differentiation operator, which carries a function f (t) to its derivative f ( t ). The differentiation operator is usually denoted by the letter D; so D f (t) is the function f (t). D carries f to f . For example, Dt3 = 3t2 . This is usually read as D applied to t3 . The identity operator takes an input function f (t) and returns the same function, f (t); it does nothing, but it still gets a symbol, I : I f = f . Operators can be added and multiplied by numbers or more generally by functions. Thus tD + 4 I is the operator sending f (t) to t f (t) + 4 f (t). The single most important concept associated with operators is that they can be composed with each other. Composition of two operators in a given order means that the two operators are applied to a function one after the other. For example, D2 , the second-derivative operator, means differentiation twice, sending f (t) to f (t). It is in fact the composition of D with itself: D2 = D D, so that D2 f = D ( D f ) = D ( f ) = f .

MIT OpenCourseWare http://ocw.mit.edu

Fall 2011

18.03SC Differential Equations

For information about citing these materials or our Terms of Use, visit: http://ocw.mit.edu/terms.

Linear Differential Operators With Constant Coefcients


The general linear ODE of order n for a function y = y(t) can be written as y ( n ) + p 1 ( t ) y ( n 1) + . . . + p n ( t ) y = q ( t ). (1)

From now on we will consider only the case where (1) has constant coefcients. This type of ODE can be written as y ( n ) + a 1 y ( n 1) + . . . + a n y = q ( t ) (2)

or, as we have seen, much more compactly using the differentiation operad tor D = : dt p( D ) y = q(t) , where p ( D ) = D n + a 1 D n 1 + . . . + a n . (3)
y

We call p( D ) a polynomial differential operator with constant coefcients. We think of the formal polynomial p( D ) as operating on a function y(t), converting it into another function; it is like a black box, in which the function y(t) goes in, and p( D )y (i.e., the left side of (2)) comes out.

p(D) p(D)y

The reason for introducing the polynomial operator p( D ) is that this allows us to use polynomial algebra to simplify, streamline and extend our calculations for solving CC DEs. Throughout this session we use the notation of equation (4): p ( D ) = D n + a 1 D n 1 + . . . + a n , ai constants. (4)

MIT OpenCourseWare http://ocw.mit.edu

Fall 2011

18.03SC Differential Equations

For information about citing these materials or our Terms of Use, visit: http://ocw.mit.edu/terms.

Operator Rules
Our work with these differential operators will be based on several rules they satisfy. In stating these rules, we will always assume that the functions involved are sufciently differentiable, so that the operators can be applied to them. Sum rule. If p( D ) and q( D ) are polynomial operators, then for any (sufciently differentiable) function u,

[ p( D ) + q( D )]u = p( D )u + q( D )u .
Linearity rule. If f and g are functions and c1 and c2 are constants, p ( D ) ( c1 f + c2 g ) = c1 p ( D ) f + c2 p ( D ) g .

(1)

(2)

Proof of the linearity rule: This rule follows from the linearity of differentiation. That is, D (c1 f + c2 g) = (c1 f + c2 g) = c1 f + c2 g = c1 Du1 + c2 Du2 . Similarly taking the second or higher derivative also follows the linearity rule . That is, D n ( c1 f + c2 g ) = dn ( c1 f + c2 g ) = c1 f ( n ) + c2 g ( n ) = c1 D n f + c2 D n g. dt

Next, we can scale the linear operator D n by a and it stays linear. That is, aD n (c1 f + c2 g) = a dn (c1 f + c2 g) = c1 a f (n) + c2 ag(n) = c1 aD n f + c2 aD n g dt

(Notice that a does not actually have to be a constant, it can be a function of t (or of whatever independent variable were using). ) Finally we can combine these operators into a polynomial operator D n + a 1 D n 1 + . . . + a n 1 D + a n which clearly still obeys the linearity rule.

Operator Rules Multiplication rule. If p( D ) = g( D ) h( D ) as polynomials in D, then p( D ) u = g( D ) h( D ) u . (3) The picture illustrates the meaning of the right side of (3). The property is true when h( D ) is the simple operator a D k , essentially because D m ( a D k u ) = a D m + k u. It extends to general polynomial operators h( D ) by linearity. Note that here a must be a constant; its false otherwise.

OCW 18.03SC

u h(D) h(D)u g(D)

p(D)u

An important corollary of the multiplication property is that polynomial operators with constant coefcients commute; i.e., for every function u(t), g( D ) h( D ) u = h( D ) g( D ) u . (4) As polynomials, g( D )h( D ) = h( D ) g( D ) = p( D ) therefore by the multiplication rule, both sides of (4) are equal to p( D ) u and therefore equal to each other. The remaining two rules are of a different type and are more concrete: they tell us how polynomial operators behave when applied to exponential functions and products involving exponential functions. Substitution rule. p( D )e at = p( a)e at Proof. We have, by repeated differentiation, De at = ae at , D2 e at = a2 e at , . . . , D k e at = ak e at ; therefore, (5)

( D n + c1 D n1 + . . . + cn ) e at = ( an + c1 an1 + . . . + cn ) e at ,
which is the substitution rule (5).

The exponential-shift rule This handles expressions such as tk e at and tk sin at. Let u = u(t). Then p( D ) e at u = e at p( D + a) u . (6)

Proof. We prove it in successive stages. First, it is true when p( D ) = D, since by the product rule for differentiation, De at u(t) = e at Du(t) + ae at u(t) = e at ( D + a)u(t). 2 (7)

Operator Rules

OCW 18.03SC

To show the rule is true for D k , we apply (7) to D repeatedly: D2 e at u = D ( De at u)

= D (e at ( D + a)u) = e at ( D + a) ( D + a)u , = e at ( D + a)2 u ,

by (7); by (7); by (3).

In the same way, D3 e at u = D ( D2 e at u)


2 = D (e at ( D + a) u) 2 at = e ( D + a) ( D + a) u , = e at ( D + a)3 u ,

by the above; by (7); by (3),

and so on. This shows that (6) is true for an operator of the form D k . To show it is true for a general operator p ( D ) = D n + a 1 D n 1 + . . . + a n , we write (6) for each D k (e at u), multiply both sides by the coefcient ak , and add up the resulting equations for the different values of k.

MIT OpenCourseWare http://ocw.mit.edu

Fall 2011

18.03SC Differential Equations

For information about citing these materials or our Terms of Use, visit: http://ocw.mit.edu/terms.

Example
Remark on complex numbers. As we saw in the session on Complex Arithmetic and Exponentials in Unit I, the formula D (c e at ) = c a e at (*)

remains true even when c and a are complex numbers. Therefore the rules and arguments above remain valid even when the exponents and coefcients are complex. We illustrate this with the following example. Example. Find D3 et sin t . Solution using the exponential-shift rule. Using the exponential shift rule and the binomial theorem, D3 et sin t

= et ( D 1)3 sin t = et ( D3 3 D2 + 3 D 1) sin t = et (2 cos x + 2 sin t),

since D2 sin t = sin t and D3 sin t = cos t. Solution using the substitution rule. have D 3 e ( 1+ i ) t Write et sin t = e(1+i)t . We

= ( 1 + i )3 e ( 1+ i ) t , by the substitution rule and (*); t = (2 + 2i ) e (cos t + i sin t),

by the binomial theorem and Eulers formula. To get the answer we take the imaginary part: et (2 cos t + 2 sin t). The operator method combined with the Exponential Response formula gives an efcient way to write and solve inhomogeneous DEs with real or complex exponential input. The following example again illustrates the usefulness of complex exponentials.

MIT OpenCourseWare http://ocw.mit.edu

Fall 2011

18.03SC Differential Equations

For information about citing these materials or our Terms of Use, visit: http://ocw.mit.edu/terms.

Time Invariance
In the case of constant coefcient operators p( D ), there is an important and useful relationship between solutions to p( D ) x = q(t) for input signals q(t) which start at different times t. The following result shows why these operators are called Linear Time Invariant (or LTI). Translation invariance. If p( D ) is an constant-coefcient differential operator and p( D ) x = q(t), then p( D )y = q(t c), where y ( t ) = x ( t c ). This is the time invariance of p( D ). Here is an example of its use.

Example. Suppose that we know that x p (t) = 2 sin(t/2 /4) is a solution to the DE .. . 2x + x + x = sin(t/2) (1)
Find a solution y p to 2x + x + x = sin(t/2 /3) Solution. By translation-invariance, we have immediately that y p = 2 sin(t/2 /4 /3) = 2 sin(t/2 7 /12).

..

(2)

MIT OpenCourseWare http://ocw.mit.edu

Fall 2011

18.03SC Differential Equations

For information about citing these materials or our Terms of Use, visit: http://ocw.mit.edu/terms.

Proof of the Generalized Exponential Response Formula


Using the exponential shift rule, we can now give a proof of the general case of the ERF which we stated without proof in the session on Exponential Response. This is a slightly complicated proof and you can safely skip it if you are not interested. Generalized Exponential Response Formula. Let p( D ) be a polynomial operator with constant coefcients and p(s) its s-th derivative. Then p( D ) x = e at , has the particular solution e at i) p( a) te at ii ) p ( a) xp = t2 e at iii ) p ( a) ... ts e at iv) p(s) ( a ) where a is real or complex (1)

if p( a) = 0 if p( a) = 0 and p ( a) = 0 if p( a) = p ( a) = 0 and p ( a) = 0

if a is an s-fold zero

Proof. That (i) is a particular solution to (1) follows immediately by using the linearity and substitution rules given earlier. p( D ) x p = p( D ) et 1 p( a)e at = p( D )e at = = e at . p( a) p( a) p( a)

Since cases (ii) and (iii) are special cases of (iv) we skip right to that. For case (iv), we begin by noting that to say the polynomial p( D ) has the number a as an s-fold zero is the same as saying p( D ) has a factorization p( D ) = q( D )( D a)s , We will rst prove that (2) implies p(s) ( a ) = q ( a ) s ! . (3) q( a) = 0. (2)

Proof of the Generalized Exponential Response Formula

OCW 18.03SC

To prove this, let k be the degree of q( D ) and write it in powers of ( D a): q( D ) p( D ) p(s) ( D )

= q ( a ) + c1 ( D a ) + . . . + c k ( D a ) k ; = q( a) s! + positive powers of D a.

then (4)

= q( a)( D a)s + c1 ( D a)s+1 + . . . + ck ( D a)s+k ;

Substituting a for D on both sides proves (3).

Using (3), we can now prove (iv) easily using the exponential-shift rule. We have p( D ) e at x s p(s) ( a ) e at p( D + a) xs , by linearity and ERF case (i); p(s) ( a ) e at = (s) q( D + a) D s x s , by (2); p ( a) e at = q( D + a) s!, by (3); q( a)s! e at = q( a) s! = e at , q( a)s!

where the last line follows from (4), since s! is a constant: q ( D + a ) s ! = ( q ( a ) + c1 D + . . . + c k D k ) s ! = q ( a ) s ! . Note: By linearity we could have stated the formula with a factor of B in the input and a corresponding factor of B to the output. That is, the DE p( D ) x = Be at has a particular solution xp = Be at , p( a) if p( a) = 0 etc.

MIT OpenCourseWare http://ocw.mit.edu

Fall 2011

18.03SC Differential Equations

For information about citing these materials or our Terms of Use, visit: http://ocw.mit.edu/terms.

Introduction
In this session we examine the important topic of resonance. Pure resonance occurs when an undamped system is forced at the same frequency as (one of) its natural frequencies. In this case the amplitude of the response grows without bound. An undamped system is an idealized case which can be considered as the limit of very light damping. In the lightly damped case the amplitude of the response is nite but it can be large. In this case we refer to the biggest possible amplitude as practical resonance. One common example of practical resonance is a childrens swing. If you push it in time with its natural frequency the amplitude of the swing will increase. Another example is a pair of guitar strings tuned to the same note. If you pluck one of them then the vibrating air will push the other one at its natural frequency and it too will start to vibrate.

MIT OpenCourseWare http://ocw.mit.edu

Fall 2011

18.03SC Differential Equations

For information about citing these materials or our Terms of Use, visit: http://ocw.mit.edu/terms.

The Exponential Response Formula: Resonant Case


The starting point for understanding the mathematics of pure resonance is the generalized Exponential Response formula. First recall the simple case of the Exponential Response formula: A solution to p( D ) x = Be at (1) is given by xp = B e at p( a) provided that p( a) = 0. (2)

In the session on Exponential Response we also saw the generalization of this formula when p( a) = 0. Here we will need to use the special case when p ( a) = 0: A solution to equation (1) is given by xp = B t e at p ( a) if p( a) = 0 and p ( a) = 0 (3)

We will call this the Resonant Response Formula. Lets look at an example of the type we will be using here to study pure resonance. Example. Find a particular solution to the DE x + 4x = 2 cos 2t. As usual, we try complex replacement and the ERF: if z p is a solution to the complex DE z + 4z = 2e2it , then x p = Re(z p ) will be a solution to x + 4x = 2 cos 2t. The characteristic polynomial is p(s) = s2 + 4, and a = 2i, so that we have p( a) = 0. But since p (s) = 2s, we have p ( a) = p (2i ) = 4i = 0. The resonant case of the ERF thus gives zp = 2 t e2it . 4i

Then taking the real part of z p gives us our particular solution xp = 1 t sin 2t. 2

MIT OpenCourseWare http://ocw.mit.edu

Fall 2011

18.03SC Differential Equations

For information about citing these materials or our Terms of Use, visit: http://ocw.mit.edu/terms.

Undamped Forced Systems


We now look at the pure resonant case for a second-order LTI DE. We will use the language of spring-mass systems in order to interpret the results in physical terms, but in fact the mathematics is the same for any second-order LTI DE for which the coefcient of the rst derivative is equal to zero. The problem is thus to nd a particular solution the DE
2 x + 0 x = F0 cos t.

The steps, as in the example in the last note, are 2 z = F ei t , x = Re( z ). Complex replacement: z + 0 0 2 2 p (i ) = 2 2 . Characteristic polynonial: p(r ) = r + 0 0 i t F e F0 ei t 0 = p (i ) 2 2 0 Exponential Response formula z p = F0 tei t F0 tei t = p (i ) 2i F cos t 0 if = 0 2 2 0 xp = F0 t sin 0 t if = 0 (resonant case). 2 0

if w = 0 if = 0 .

Resonance and amplitude response of the undamped harmonic oscillator F0 In x p the amplitude = A = A( ) = | 2 | is a function of . 0 2 The right plot below shows A as a function of . Note, it is similar to the damped amplitude response except the peak is innitely high. As w gets closer to 0 the amplitude increases. F0 t sin 0 t When = 0 we have x p = . This is called pure resonance 2 0 (like a swing). The frequency 0 is called the resonant or natural frequency of the system. In the left plot below notice that the response is oscillatory but not periodic. The amplitude keeps growing in time (caused by the factor of t in x p ). Note carefully the different units and different meanings in the plots below.

Undamped Forced Systems

OCW 18.03SC

The left plot is output vs. time (for a xed input frequency) and the right plot is output amplitude vs. input frequency. x and A are in physical units dependent on the system; t is in time; is in radians. x A t 0 Resonant response ( = 0 )

Undamped amplitude response

MIT OpenCourseWare http://ocw.mit.edu

Fall 2011

18.03SC Differential Equations

For information about citing these materials or our Terms of Use, visit: http://ocw.mit.edu/terms.

Introduction
In this session we will examine the response of a second order linear time invariant (LTI) system to a sinusoidal input. We will pay special attention to the way the output changes as the frequency of the input changes. This is what we mean by the frequency response of the system. In particular, we will look at the amplitude response and the phase response; that is, the amplitude and phase lag of the systems output considered as functions of the input frequency. We did something similar in the rst unit of the course where, in the session on Exponential Input, we discussed the frequency response of a rst order LTI system as the frequency of the input sinusoid varies. In the recent sessions on Exponential Response and Gain & Phase Lag, we worked out in detail the formulas which give the response to a sinusoidal input signal for an LTI DE of any order. Here we will specialize to the second order case where we will focus on the interpretation of these mathematical results for mechanical systems. In particular, we will look at damped-spring-mass systems. We will study carefully two cases: rst, when the mass is driven by pushing on the spring and second, when the mass is driven by pushing on the dashpot. Both these systems have the same form p ( D ) x = q ( t ), but their amplitude responses are very different. This is because, as we will see, it can make physical sense to designate something other than q(t) as the input. For example, in the system mx + bx + kx = by we will consider y to be the input. (Of course, y is related to the expression on the right-hand-side of the equation, but it is not exactly the same.)

..

MIT OpenCourseWare http://ocw.mit.edu

Fall 2011

18.03SC Differential Equations

For information about citing these materials or our Terms of Use, visit: http://ocw.mit.edu/terms.

Sinusoidally Driven Systems: Second Order LTI DEs


We start with the second order linear constant coefcient (CC) DE, which as weve seen can be interpreted as modeling a damped forced harmonic oscillator. If we further specify the oscillator to be a mechanical system with mass m, damping coefcient b, spring constant k, and with a sinusoidal driving force B cos t (with B constant), then the DE is mx + bx + kx = B cos t. (1)

For many applications it is of interest to be able to predict the periodic response of the system to various values of . From this point of view we can picture having a knob you can turn to set the input frequency , and a screen where we can see how the shape of the system response changes as we turn the -knob. In the sessions on Exponential Response and Gain & Phase Lag we worked out the general case of a sinusoidally driven LTI DE. Specializing these results to the second order case we have: Characteristic polynomial: p(s) = ms2 + bs + k. Complex replacement: mz + bz + kz = Bei t , x = Re(z). Exponential Response Formula: zp = Bei t Bei t = p (i ) k m 2 + ib B

cos( t ), ( k m 2 )2 + b2 2 b 1 where = Arg( p(i )) = tan . (In this case must be bek m 2 tween 0 and . We say is in the rst or second quadrants.) B , we can write the periodic response x p Letting A = ( k m 2 )2 + b2 2 as x p = A cos( t ). The complex gain, which is dened as the ratio of the amplitude of the output to the amplitude of the input in the complexied equation, is ( ) = g 1 1 = . p (i ) k m 2 + ib

x p = Re(z p ) =

Sinusoidally Driven Systems: Second Order LTI DEs

OCW 18.03SC

The gain, which is dened as the ratio of the amplitude of the output to the amplitude of the input in the real equation, is g = g( ) = The phase lag is = ( ) = Arg( p(i ) = tan1 ( and we also have the time lag = / . Terminology of Frequency Response We call the gain g( ) the amplitude response of the system. The phase lag ( ) is called the phase response of the system. We refer to them collectively as the frequency response of the system. Notes: 1. Observe that the whole DE scales by the input amplitude B. 2. All that is needed about the input for these formulas to be valid is that it is of the form (constant) (a sinusoidal function). Here we have used the notation B cos t but the amplitude factor in front of the cosine function can take any form, including having the constants depend on the system parameters and/or on . (And of course one could equally-well use sin t, or any other shift of cosine, for the sinusoid.) This point is very important in the physical applications of this DE and we will return to it again in a later session. 3. Along the same lines as the preceding: we always dene the gain as the the amplitude of the periodic output divided by the amplitude of the periodic input. Later in this session we will see examples where the gain is not just equal 1 1 to p(i (for complex gain) or | p(i (for real gain) stay tuned! ) )| b ) k m 2 (3) 1 1 = . | p (i ) | ( k m 2 )2 + b2 2 (2)

MIT OpenCourseWare http://ocw.mit.edu

Fall 2011

18.03SC Differential Equations

For information about citing these materials or our Terms of Use, visit: http://ocw.mit.edu/terms.

Frequency Response and Practical Resonance


In the previous note in this session we found the periodic solution to the equation mx + bx + kx = B cos( t). (1) The solution was x p = gB cos( t ), where g is the gain g = g( ) = and is the phase lag = ( ) = Arg( p(i )) = tan1 (b /(k m 2 )). (3) 1 (2)

( k m 2 )2 + b2 2

The gain or amplitude response is a function of . It tells us the size of the systems response to the given input frequency. If the amplitude has a peak at r we call this the practical resonance frequency. If the damping b gets too large then, for the system in equation (1), there is no peak and, hence, no practical resonance. The following gure shows two graphs of g( ), one for small b and one for large b. g g

r Fig 1a. Small b (has resonance).

Fig 1b. Large b (no resonance)

In gure (1a) the damping constant b is small and there is practical resonance at the frequency r . In gure (1b) b is large and there is no practical resonant frequency. Finding the Practical Resonant Frequency. We now turn our attention to nding a formula for the practical resonant frequency -if it exists- of the system in (1). Practical resonance occurs at the frequency r where g(w) has a maximum. For the system (1) with gain (2) it is clear that the maximum gain occurs when the expression under the radical has a minimum. Accordingly we look for the minimum of f ( ) = ( k m 2 )2 + b2 2 .

Frequency Response and Practical Resonance Setting f ( ) = 0 and solving gives f ( ) = 4m ( k m 2 ) + 2b2 = 0

OCW 18.03SC

= 0 or m2 2 = mk b2 /2.
We see that if mk b2 /2 > 0 then there is a practical resonant frequency k b2 r = . m 2m2 Phase Lag: In the picture below the dotted line is the input and the solid line is the response. The damping causes a lag between when the input reaches its maximum and when the output does. In radians, the angle is called the phase lag and in units of time / is the time lag. The lag is important, but in this class we will be more interested in the amplitude response. . . . ......

... . . . .....
/

. ...

. . . .. .....

.......

time lag

..... ...

.... ..... . . . . . . . ..

. ....

... ..... . . . . .

........ . . . . ...........

. . ...

.....

MIT OpenCourseWare http://ocw.mit.edu

Fall 2011

18.03SC Differential Equations

For information about citing these materials or our Terms of Use, visit: http://ocw.mit.edu/terms.

Mechanical Vibration System: Driving Through the Spring


The gure below shows a spring-mass-dashpot system that is driven through the spring.
y

Spring Mass

Dashpot

Figure 1. Spring-driven system Suppose that y denotes the displacement of the plunger at the top of the spring and x (t) denotes the position of the mass, arranged so that x = y when the spring is unstretched and uncompressed. There are two forces acting on the mass: the spring exerts a force force given by k(y x ) (where k . is the spring constant) and the dashpot exerts a force given by bx (against the motion of the mass, with damping coefcient b). Newtons law gives mx = k (y x ) bx or, putting the system on the left and the driving term on the right, mx + bx + kx = ky .

..

..

(1)

In this example it is natural to regard y, rather than the right-hand side q = ky, as the input signal and the mass position x as the system response. Suppose that y is sinusoidal, that is, y = B1 cos( t). Then we expect a sinusoidal solution of the form x p = A cos( t ).

Mechanical Vibration System: Driving Through the Spring OCW 18.03SC

By denition the gain is the ratio of the amplitude of the system response to that of the input signal. Since B1 is the amplitude of the input we have g = A/ B1 . In the previous note in this session, we worked out the formulas for g and , and so we can now use them with the following small change. The k on the right-hand-side of equation (1) needs to be included in the gain (since we dont include it as part of the input). We get A k k = = B1 | p (i ) | ( k m 2 )2 + b2 2 b ( ) = tan1 . k m 2 g( ) = Note that the gain is a function of , i.e. g = g( ). Similarly, the phase lag = ( ) is a function of . The entire story of the steady state system response x p = A cos( t ) to sinusoidal input signals is encoded in these two functions of , the gain and the phase lag. We see that choosing the input to be y instead of ky scales the gain by k and does not affect the phase lag. The factor of k in the gain does not affect the frequency where the gain is greatest, i.e. the practical resonant frequency. From the previous note in this session we know this is k b2 r = . m 2m2 Note: Another system leading to the same equation is a series RLC circuit. We will favor the mechanical system notation, but it is interesting to note the mathematics is exactly the same for both systems.

MIT OpenCourseWare http://ocw.mit.edu

Fall 2011

18.03SC Differential Equations

For information about citing these materials or our Terms of Use, visit: http://ocw.mit.edu/terms.

Mechanical Vibration System: Driving Through the Dashpot


Now suppose instead that we x the top of the spring and drive the system by moving the bottom of the dashpot instead. Suppose that the position of the bottom of the dashpot is given by y(t) and the position of the mass is given by x (t), arranged so that x = 0 when the spring is relaxed. Then the force on the mass is given by mx = kx + b

..

d (y x ) dt

since the force exerted by a dashpot is supposed to be proportional to the speed of the piston moving through it. This can be rewritten as mx + bx + kx = by .

..

(1)

Spring Mass x

Dashpot

Figure 2. Dashpot-driven system We will consider x as the system response, and again on physical grounds we specify as the input signal the position y of the back end of the dashpot. Note that the derivative of the input signal (multiplied by b) occurs on the right hand side of the equation. Again we suppose that the input signal is of sinusoidal form y = B1 cos( t). We will now work out the frequency response analysis of this problem. First, y = B1 cos( t) y = B1 sin( t), so our equation is mx + bx + kx = b B1 sin( t) .

..

(2)

Mechanical Vibration System: Driving Through the DashpotOCW 18.03SC

We know that the periodic system response will be sinusoidal, and as usual we choose the amplitude-phase form with the cosine function x p = A cos( t ) . Since y = B1 cos( t) was chosen as the input, the gain g is given by g =
A B1 .

As usual, we compute the gain and phase lag by making a complex replacement. One natural choice would be to regard q(t) = b B1 sin( t) as the imaginary part of a complex equation. This would work, but we must keep in mind that the input signal is B1 cos( t) and also that we want to express the solution x p as x p = A cos( t ). Instead we will go back to equation (1) and complexify before taking the derivative of the right-hand-side. Our input y = B1 cos( t) becomes = B1 ei t and the DE becomes y = i bB1 ei t . mz + bz + kz = by

..

(3)

) we have x = Re(z); that is, the sinusoidal system response Since y = Re(y x p of (2) is the real part of the exponential system response z p of (3). The Exponential Response Formula gives zp = where p(s) = ms2 + bs + k is the characteristic polynomial. The complex gain (scale factor that multiplies the input signal to get the output signal) is i b ( ) = g . p (i ) ( ) ei t . Thus, z p = B1 g = |g |ei , where = Arg( g ). (We use the minus sign We can write g so will come out as the phase lag.) Substitute this expression into the formula for z p to get | ei ( t) . z p = B1 | g Taking the real part we have | cos( t ). x p = B1 | g 2 i bB1 i t e p (i )

Mechanical Vibration System: Driving Through the DashpotOCW 18.03SC | and the phase lag = Arg( g ). All thats left is to compute the gain g = | g We have p(i ) = m(i )2 + bi + k = (k m 2 ) + bi , so, = g This gives | = g( ) = | g b b = . | p (i ) | ( k m 2 )2 + b2 2 i b i b = . p (i ) (k m 2 ) + bi (4)

In computing the phase we have to be careful not to forget the factor of i . After a little algebra we get in the numerator of g ) = tan1 ((k m 2 )/(b )). ( ) = Arg( g As with the system driven through the spring, we try to nd the input frequency = r which gives the largest system response. In this case we can nd r without any calculus by using the following shortcut: divide the numerator and denominator in (4) by bi and rearrange to get = g 1 1 = . 1 + ( k m 2 ) / (i b ) 1 i (k m 2 )/( b)

| can be written as Now the gain g = | g g = 1 1 + ( k m 2 )2 / ( b )2 .

Because squares are always positive, this is clearly largest when the term k m 2 = 0. At this point g = 1 and r = k/m = 0 , i.e. the resonant frequency is the natural frequency. (0 ) = 1, we also see that the phase lag = Arg( g ) is 0 at r Since g Thus the input and output sinusoids are in phase at resonance. We have found interesting and rather surprising results for this dashpotdriven mechanical system, namely, that the resonant frequency occurs at the systems natural undamped frequency 0 ; that this resonance is independent of the damping coefcient b; and that the maximum gain which can be obtained is g = 1. We can contrast this with the spring-side driven

Mechanical Vibration System: Driving Through the DashpotOCW 18.03SC

system worked out in the previous note, where the resonant frequency certainly did depend on the damping coefcient. In fact, there was no resonance at all if the system is too heavily damped. In addition, the gain could, in principle, be arbitarily large. Comparing these two mechanical systems side-by-side, we can see the importance of the choice of the specication for the input in terms of understanding the resulting behavior of the physical system. In both cases the right-hand side of the DE is a sinusoidal function of the form B cos t or B sin t, and the resulting mathematical formulas are essentially the same. The key difference lies in the dependence of the constant B on either the system parameters m, b, k and/or the input frequency . It is in fact the dependence of B on and b in the dashpot-driven case that results in the radically different result for the resonant input frequency r . Note: As with the mechanical system driven through the spring, the mechanical system driven through the dashpot has an exact mathematical analog in a series RLC circuit. We will discuss this in the next session.

MIT OpenCourseWare http://ocw.mit.edu

Fall 2011

18.03SC Differential Equations

For information about citing these materials or our Terms of Use, visit: http://ocw.mit.edu/terms.

Introduction
In this session we study one widely-used application of the linear time invariant DE analysis we have developed in this unit, namely, RLC circuits. Remarkably, these circuits can be modeled with the exact same differential equations as the mechanical systems studied in the previous sessions. The symbols used and their interpretation will change, but the fact that the DEs are identical means that, in some sense the behavior of the systems is the same. We will also use complex techniques to dene and understand impedance. Impedance generalizes the notion of resistance and like resistance it follows Ohms law. These techniques will also allow us to understand phasors and the phase angles between the different voltages in the circuit.

MIT OpenCourseWare http://ocw.mit.edu

Fall 2011

18.03SC Differential Equations

For information about citing these materials or our Terms of Use, visit: http://ocw.mit.edu/terms.

RLC Circuits 1. Simple circuit physics


The picture at right shows an inductor, capacitor and resistor in series with a driving voltage source. VL I (t) is the current in the circuit in amps. L is the inductance in henries. R is the resistance in ohms. C is the capacitance in farads. Vin is the input voltage to the circuit.

Vin

Q(t) is the charge on the capacitor, so I (t) = Q(t).

R I

VC


VR

From physics we get that the voltage drops across each of the circuit elements. .. . . Q VL = LI = LQ, VR = RI = RQ, VC = . C The amazing thing is that this and Kirchhoffs voltage law (KVL) is all the physics we need to understand this circuit. The rest is linear CC DEs and complex arithmetic. (KVL says that the net voltage drop around any closed loop is 0.)

2. Summary of the this Session.


We start with a summary of the physics and DEs covered in this session. Explanations will be given below and in the next note. Compatible Units: The units given with the circuit diagram above are compatible and we will assume throughout that we are using them. Voltage Drops: L I , RI ,

1 Q. (We will just accept these from the physicists.) C

DEs: Using the KVL and the voltage drops descibed above we get all of the

RLC Circuits

OCW 18.03SC

following physically equivalent DEs. LQ + RQ + L I + RI +

..

. 1 I = V in C .. . 1 1 LV C + RV C + VC = Vin C C .. . . 1 LV R + RV R + VR = RV in C

..

1 Q = Vin C

(1) (2) (3) (4)

Complex Replacement: If x is a real number or function, we will use the fol will be a complex replacement for x, in lowing notational convention here: x is complex, the same sense that we have use this term before, namely, x depending on whether the input and x is the real or imaginary part of x was cosine or sine. in = ei t ) Complex Impedance: (valid when V 1 L = iL , Z R = R, Z C = Z . iC =Z R + Z L + Z C = R + i ( L 1/( C )). Total impedance = Z in = Z L = Z L R = Z R C = Z C Complex Ohms Law: V I, V I, V I, V I. Phasors: All the output voltages are plotted in the complex plane as a rigid set of vectors that rotate at frequency . VR and I point in the same direc in by tion, VL leads I by /2, VC lags I by /2. I either leads or lags V 1 = tan (( L 1/(C ))/ R). Reactance and real impedance: = R + iS. Reactance = S = L 1/( C ). Z 2 2 Real impedance = | Z | = R + S = R2 + ( L 1/( C ))2 . E0 in = Z If Vin = E0 sin t then from V I we get I = sin( t ), | |Z with the phase angle = tan1 (S/ R). Practical resonance: In equations (2) and (4) the practical resonance is always at the natural frequency 0 = 1/ LC. 2

RLC Circuits

OCW 18.03SC

3. The Differential Equations


First, lets justify the differential equations 1-4. KVL implies the total voltage drop around the circuit has to be 0. If we follow the current I clockwise around the circuit adding up the voltage drops, we get the basic equation . 1 LI + RI + Q Vin = 0, (5) C here weve assume that the input provides a voltage gain. We can replace I by Q in (5) to get equation (1). If we differentiate (5) and replace Q by I we get (2). Now multiply (1) by 1/C to get (3) and multiply (2) by R to get (4).

4. The Electro-Mechanical Analogy


Notice that equation (3) has the same form as the DE for the spring-mass system driven through the spring. That is, if you substitute m, b, k and x for L, R, 1/C, VC in (3) and call Vin the input then you have the equation for the spring-mass system driven through the spring. Likewise equation (4) has the same form as the spring-mass system driven through the dashpot. Thus, ignoring their interpretations as voltage instead of postion, the outputs VC and VR behave exacly like the position x of the mass driven through the spring and dashpot respectively.

5. Resistance
Ohms law is most often given for the voltage VR = IR across a resistor. Recall: Two resistances R1 and R2 combine to give an equivalent resistance 1 1 1 R. For R1 , R2 in series R = R1 + R2 , and in parallel = + . R R1 R2 We are going to use the Exponential Response formula and complex arithmetic to understand the notions of complex impedance and phasor diagrams.

MIT OpenCourseWare http://ocw.mit.edu

Fall 2011

18.03SC Differential Equations

For information about citing these materials or our Terms of Use, visit: http://ocw.mit.edu/terms.

Impedance 1. Simple Complex Arithmetic Fact


Im iz
z

You should be clear that in the complex plane multiplication by i is the same as rotation by /2. Likewise division by i is the same as rotation by /2. The phase difference between two complex numbers a and b is simply the difference of their arguments, Arg( a) Arg(b). The simple arithmetic fact implies z and iz have a phase difference of /2. z and z/i have a phase difference of /2. We will need this when we discuss phasors. (1)

Re
z/i = iz

2.

Complex Impedance
We repeat for reference some of the DEs given in the previous note. LQ + RQ + L I + RI +

..

..

. 1 I = V in C

1 Q = Vin C

(2) (3)

Using complex arithmetic and the Exponential Response formula we can understand all the statements about impedance and phasors. First, note that if we remove the inductor and capacitor then (2) is just Ohms law, i.e. R Q = RI = Vin . Now we make the crucial assumption of sinusoidal input (alternating current): Vin (t) = V0 sin( t). With this input we will solve equation (3). First, complexify (3): (Because of the tildes ( I ) we use prime instead of dot to indicate derivatives.) L I + R I + 1 I = Vin = i V0 ei t , C I ). I = Im(

The Exponential Response formula gives the periodic solution: i V0 i t I= e . P (i ) (4)

Impedance A little algebra shows that the coefcient of V0 ei t in (4) is

OCW 18.03SC

i i 1 = = . 2 P (i ) L + 1/C + Ri iL + 1/(iCw) + R Accordingly we dene the complex impedance as = iL + 1 + R. Z iCw depends on the input frequency .) (Notice Z We can now write the complex version of Ohms law (always assuming Vin = V0 ei t ): 1 in = Z or V I. (6) I = V in Z We can associate a separate impedance to each circuit element: L = iL , Z R = R, Z C = Z 1 . iC (7) (5)

Comparing (5) and (7) we see that for a set of elements wired in series the total complex impedance is just the sum of the individual impedances. That is, impedance behaves just like resistance in series. Whats more, using the voltage drops across each element we see they individually satisfy a complex Ohms Law. L = L L V I = Li I=Z I, R = R V I, = 1 C = 1 Q V C C

I=

1 I = ZC I . iC

Note: the formulas involving depend crucially on the assumption that the complex input is V0 ei t .

3. Impedance in Parallel
It is also true and easy to show that for circuit elements in parallel the complex impedances combine like resistors in parallel. That is, if impedances 1 and Z 2 are in parallel then the total impedance of the pair, call it Z , satZ 1 1 1 ises = + . Z Z1 Z2 To see this we use Ohms law for a single circuit, KVL and Kirchoffs current law (KCL). They imply

Impedance

=Z 1 = 2 . I = I1 + I2 , V I1 , V I2 Z V V 1 1 I= + =V + 1 2 1 2 Z Z Z Z 1 = V I . QED 2 1/ Z1 + 1/ Z

OCW 18.03SC I I1 1 Z

I2


2 Z

4. Amplitude-Phase Form and Real Impedance


First we put the expression (5) for complex impedance in the form we need = iL + 1 + R = i ( L 1 ) + R = iS + R. Z iC C We call S = L 1/(C ) the reactance; note that S = 0 when 2 = 1/( LC ). = |Z |ei , where | Z | = S2 + R2 and = Arg( Z ) = tan1 (S/ R). In amplitude phase form Z Notice the sign of depends on the sign of S = L 1/C and also that is between /2 and /2. Thus, V0 V0 I = ei ( t) = ei ( t) . (8) 2 2 2 2 ( L 1/C ) + R S +R | = ( L 1/C )2 + R2 is called the real impedance. The term S2 + R2 = | Z Taking imaginary parts in (8) gives | = V0 sin( t ), I |Z which is like Ohms Law, except with a phase shift.

5. Phasors
(The term phasor just means ei t ). We have seen that each element of an LRC circuit obeys a complex Ohms law: L = Z L V I = Li I, R = R V I, C = Z C V I= 1 I. iC (9)

Each of the complex voltages is some constant factor I , which is, in turn, a multiple of ei t . If we plot the voltages in the complex plane then as t increases the entire picture will rotate at frequency . We call each of these voltages a phasor. 3

Impedance

OCW 18.03SC

We want to look at the phase difference between the various voltages. L and V C imply By our simple arithmetic fact (1), the factors i and 1/i in V L and V C are respectively /2 ahead and /2 behind V R . 1. The phasors V Equation (8) implies R is behind V in (if is negative then V R is ahead of V in . 2. The phasor V Later we will look at the excellent Series LRC Circuit applet which illustrates this. V Im in V L
I

R t V

Re

V C

6. Amplitude Response and Practical Resonance The natural frequency of the circuit is 0 = 1/ LC. This is the frequency of oscillation when the damping term R is zero. The practical resonance of the system (3) is independent of the value of R and always at the natural frequency 0 = 1/ LC (This is easy to see in (8), since | I | is clearly maximized when the term ( L 1/C )2 = 0.) That is, practical resonance occurs when V0 i t L + Z C = 0 iL i /C = 0 Z = R, Z I= e . R in , R all line up, i.e., In the phasor picture, at practical resonance V I and V lag is 0 and VR = Vin . This is one case where the corresponding sinusoidal graphs of the real voltages are neat enough to give a nice picture: the graph of VR is exactly in phase with Vin ; VL and VC have the same magnitude and are 180 out of phase; increasing R doesnt change VR , but decreases the amplitude of VL and VC . The applet Series LRC Circuit shows all this beautifully. 4

MIT OpenCourseWare http://ocw.mit.edu

Fall 2011

18.03SC Differential Equations

For information about citing these materials or our Terms of Use, visit: http://ocw.mit.edu/terms.

Series RLC Circuit Applet


Open the Series RLC Circuit applet. 1. Check the phasor diagram checkbox. 2. Check all the voltage checkboxes: VR , VL , VC , V , I . Note: if you click the checkboxes twice the graphs will be in color. 3. Animate the applet by clicking on the double arrow below the t axis. 4. Play with the applet. Suggested Applet Exercise Set the applet to show you all four voltages and the current I . Set L = 500 mH, C = 100 F, R = 250 ohms. Compute the resonant frequency of the system. Move to the resonant frequency, watch the phasors and the sinusoidal plots as you do this. With set at 0 watch the amplitudes of the three output voltages and the output current as R increases. Explain everything you see in terms of the i. complex Ohms laws and the Exponential Response formula solution for In the phasor diagram, can you see that the voltages across R and L are 90 out of phase? What about R and C? Are the voltages across C and L 180 out of phase? Why does the the angle between the input voltage and the output voltages vary as you vary ?

MIT OpenCourseWare http://ocw.mit.edu

Fall 2011

18.03SC Differential Equations

For information about citing these materials or our Terms of Use, visit: http://ocw.mit.edu/terms.

Fourier Series Basics: Introduction


In this session we will introduce the Fourier series of a periodic function and show how to compute it. Fourier analysis is a large subject with wideranging applications. The main use we will make of these powerful tools will be to solve the inhomogeneous linear time invariant DE p( D ) x = f (t), where f (t) is a periodic function. The Fourier series expansion of the periodic solution x p (t) will, for example, show clearly when resonances occur, and at what frequencies. Nature abounds with examples of these types of phenomena. To give just one example: the inner ear is equiped with a cellular array which acts as a Fourier analyzer" and allows us to detect the different frequencies of the in-coming sound by resonanting in tune with just those frequencies.

Periodic Functions
Periodic functions are functions which repeat: f (t + P) = f (t) for all t. For example, if f (t) is the amount of time between sunrise and sunset at a certain lattitude, as a function of time t, and P is the length of the year, then f (t + P) = f (t) for all t, since the Earth and Sun are in the same position after one full revolution of the Earth around the Sun. We state this explicitly as the following dention: a function f (t) is periodic with period P > 0 if f (t + P) = f (t) for all t.

Example. f (t) = sin(2t) is periodic with period P = . This is true because, for all t, f (t + ) = sin(2(t + ) = sin(2t + 2 ) = sin(2t) = f (t). Notice, though, that in the example above f (t) = sin(2t) also has period P = 2 and period P = 3 . In fact, it has period P = n for any integer n = 1, 2, 3 . . . . Graphically, a function with period P is one whose graph stays the same if it is shifted P to the left or right. Base Period Most periodic functions have a minimal period, which is often called either the period or the base period. For example, sin t has minimal period is 2 . It follows from this that the minimal period for sin(2t) is . The only exception is the constant function. Every value of P > 0 is a period and so it has no minimal period. (We dont allow P = 0 to be a period because then every function would be periodic with period P = 0.) Windows To fully describe a periodic function you only need to specify the period and the value of the function over one full period. We call an interval containing one full period a window. Typical choices for windows are [ P/2, P/2) and [0, P), but any interval of length P will work. Frequency Terminology Angular frequency, also called circular frequency has units of radians/unit time. Frequency has units of cycles/unit time. Since one cycle is 2 radians the relationship is

Periodic Functions angular frequency = 2 frequency.

OCW 18.03SC

The above is the ofcial terminology, but in actual practice many people say frequency when they mean angular frequency. In fact, that has been the general usage earlier in this course where we have called the frequency of cos( t). You will have to use the context to decide exactly which frequency is being used. For a function with period P the base angular frequency (also called the fundamental angular frequency) means the angular frequency corresponding to the base (or minimal) period P that is = 2 . P

Fourier Series We will see that a periodic function with base frequency can be written as a sum of sines and cosines whose frequencies are integer multiples of . This is called the Fourier series for the function. That is, sines and cosines, the simplest periodic functions, are the building blocks" for more general periodic functions. Later in this session we will see exactly how to compute the Fourier series for a periodic function.

Quiz: Cosines with Common Periods


Quiz: Which collection of cosine functions all have period 2 ? Choices: a) cos(t), cos(t/2), cos(t/3), . . . b) cos( t), cos(2 t), . . . c) cos(t), cos(2t), cos(3t), . . .

Answer: (c). For n = 1, 2, 3, . . . , cos(nt) has period 2 (and base period 2 /n) .

Quiz: Cosines with Common Periods


Quiz: Which collection of cosine functions all have period 2 ? Choices: a) cos(t), cos(t/2), cos(t/3), . . . b) cos( t), cos(2 t), . . . c) cos(t), cos(2t), cos(3t), . . . Pick what you think is the correct choice and then look at the answer.

Quiz: Cosines with Common Periods


Quiz: Which collection of cosine functions all have period 2 ? Think about your answer and then look at the choices.

Quiz: Cosines with Common Frequecies


Quiz: What is the base (fundamental) frequency of the function f (t) = cos(t) + cos(2t) + cos(3t)?

Choices: a) 1 b) 2 c) 3 d) 6 e) there is no base frequency.

Answer: (a): Base frequency = 1. The smallest common period of cos(t), cos(2t) and cos(2t) is 2 . Thus, f (t) = cos(t) + cos(2t) + cos(3t) has minimal period P = 2 , and therefore its base frequency is 2P = 1.

Quiz: Cosines with Common Frequecies


Quiz: What is the base (fundamental) frequency of the function f (t) = cos(t) + cos(2t) + cos(3t)?

Choices: a) 1 b) 2 c) 3 d) 6 e) there is no base frequency. Pick what you think is the correct choice and then look at the answer.

Quiz: Cosines with Common Frequecies


Quiz: What is the base (fundamental) frequency of the function f (t) = cos(t) + cos(2t) + cos(3t)?

Think about your answer and then look at the choices.

Fourier Series: Denitions and Coefcients


We will rst state Fouriers theorem for periodic functions with period P = 2 . In words, the theorem says that a function with period 2 can be written as a sum of cosines and sines which all have period 2 . Theorem (Fourier) Suppose f (t) has period 2 then we have f (t) a0 + a1 cos(t) + a2 cos(2t) + a3 cos(3t) + . . . 2 +b1 sin(t) + b2 sin(2t) + b3 sin(3t) + . . .
a0 + an cos(nt) + bn sin(nt), 2 n =1

(1)

where the coefcients a0 , a1 , . . . and b1 , b2 . . . are computed by 1 a0 = f (t) dt 1 an = f (t) cos(nt) dt 1 bn = f (t) sin(nt) dt

(2)

Some comments are in order. 1. As we saw in the quiz above, each of the functions cos(t), cos(2t), cos(3t), . . . all have 2 as a period. The same is clearly true for sin(t), sin(2t), sin(3t), . . . . 2. The series on the right-hand side (1) is called a Fourier series; and the coefcients , a0 , a1 , . . . b1 , b2 , . . . in (2) are called the Fourier coefcients of f (t). 3. The letter a is used in a0 /2 because we can think of it as the coefcient of cos(0 t) = 1. We dont need a b0 term because sin(0 t) = 0. The term constant term a20 is written in this way to make the formula for a0 look just like those of the other cosine coefcients an . (We will see why we need the factor of 1 2 in a later note when we prove that these formulas really do give the coefcients.) 4. In (1) we used the symbol instead of an equal sign because the two sides of (1) might differ at those values of t where f (t) is discontinuous. For us, this is a minor point and we will allow ourselves to use an equal sign from now on.

Fourier Series: Denitions and Coefcients

OCW 18.03SC

5. There is some terminology coming from acoustics and music: the n = 1 frequency is called the fundamental, and the frequencies n 2 are called the higher harmonics (or overtones). We will explore the connection between Fourier series and sound in a later session. Fourier series are a wonderful tool for breaking a periodic function, however complicated, into simple pieces. The superposition principle will then allow us to solve DEs with arbitrary periodic input in Fourier series form. In later notes we will extend Fouriers theorem to functions of other periods. The extension is straighforward, but requires more notation, so we will wait until you have gained some experience with Fourier series.

Examples
Example 1. Compute the Fourier series of f (t), where f (t) is the square wave with period 2 . which is dened over one period by 1 for t < 0 . f (t) = 1 for 0 t < The graph over several periods is shown below.

Solution. Computing a Fourier series means computing its Fourier coefcients. We do this using the integral formulas for the coefcients given with Fouriers theorem in the previous note. For convenience we repeat the theorem here. f (t) = where a0 = 1

a0 + ( an cos(nt) + bn sin(nt)), 2 n =1

f (t) dt,

an =

f (t) cos nt dt,

bn =

f (t) sin nt dt

In applying these formulas to the given square wave function, we have to split the integrals into two pieces corresponding to where f (t) is +1 and where it is 1. We nd 1 an =

f (t) cos(nt) dt =

(1) cos(nt) dt +

(1) cos(nt) dt.

Thus, for n = 0: 0 sin(nt) sin ( nt ) + =0 an = n n 0 and for n = 0: 1 a0 =


f (t) dt = 0.

(Note: its advisable to do a0 separately.)

Examples

OCW 18.03SC

Likewise 1 bn = 1 0 1 f (t) sin(nt) dt = sin(nt) dt + sin(nt) dt 0 0 cos(nt) cos(nt) = 1 cos(n ) cos(n ) 1 = n n 0 n n 4 for n odd 2 2 n (1 ( 1) n ) = = (1 cos(n )) = . n n 0 for n even

We have used the simplication cos n = (1)n to get a nice formula for the coefcients bn . (Note: when you get cos n in these calculations its always useful to make this substitution.) This then gives the Fourier series for f (t): 4 1 1 f (t) = bn sin(nt) = sin t + sin(3t) + sin(5t) + . 3 5 n =1 Example 2. seeing the convergence of a Fourier series 4 1 1 The claim is that f (t) = sin t + sin 3t + sin 5t + . However, it 3 5 is not easy to see that the sum on the right-hand side is in fact converging to the square wave f (t). So lets use a computer to plot the sums of the rst N terms of the series. for N = 1, 3, 9, 33. We get the following four graphs:

Notice that since a nite sum of sine functions is continuous (in fact smooth), the partial sums cannot jump when t is an integer multiple of , the way the square way f (t) does. But they are certainly trying" to become the square wave f (t)! And the more terms you add in, the better the t, with the theoretical limit as N being exactly equal to f (t) (except actually at the jumps t = n , as well see). Note: In this case we dont have any cosine terms, just sine. This turns out to be not an accident: it follows from the fact that f (t) here is an odd 2

Examples

OCW 18.03SC

function, i.e. f (t) = f (t), and such functions have only sines (which are also odd functions) in their Fourier series. Similarly for even functions and cosine series: if f (t) is even ( f (t) = f (t)) then all the bn s vanish and a0 the Fourier series is simply f (t) = + an cos(nt); while if f (t) is odd 2 n =1 then all the an s vanish and the Fourier series is f (t) =
n =1

bn sin(nt).

Fourier Series for Functions with Period 2L


Suppose that we have a periodic function f (t) with arbitrary period P = 2 L, generalizing the special case P = 2 which we have already seen. Then a simple re-scaling of the interval ( , ) to ( L, L) allows us to write down the general Fourier series and Fourier coefcent formulas: f (t) = a0 + an cos n t + bn sin n t 2 L L n =1 (1)

with Fourier coefcients given by the general Fourier coefcent formulas 1 L a0 = f (t) dt, L L 1 L an = f (t) cos(n t) dt, L L L 1 L bn = f (t) sin(n t) dt. L L L

(2)

Note: The number L =

P 2

is called the half-period.

Example. Let f (t) be the period 2 function, which is dened on the window [1, 1) by f (t) = |t|. Compute the Fourier series of f (t). The graph of f (t) below shows why this function is called either a triangle wave or a continuous sawtooth function.

2 2 1 1 Figure 1: The period 2 triangle wave.


Solution. In this case the period is P = 2, so the half-period L = 1. This means n L = n and we compute the coefcients from the formulas in (2), using integration by parts, as follows. For n = 0: an = 1 1
1
1

|t| cos(n t) dt = 2

1
0 1

t cos(n t) dt

= 2

t sin(n t) cos(n t) 2 + = 2 2 ((1)n 1) = n n2 2 0 n

n24 2 0

for n odd for n even

Fourier Series for Functions with Period 2L

OCW 18.03SC

and for n = 0: 1 1 1 a0 = |t| dt = 2 t dt = 1 1 1 0 Since f (t) is an even function and sin(n t) is odd, the sine coefcients bn = 0. (We will justify this carefully in the next session. For now you can compute the integrals for bn as an exercise and verify it in this case.) Thus, the Fourier series for f (t) is 1 4 cos 3 t cos 5 t 1 4 f (t) = 2 cos t + + + = 2 2 2 2 3 5 2

cos(n t) . n2 n odd

Orthogonality Relations
We now explain the basic reason why the remarkable Fourier coefcent formulas work. We begin by repeating them from the last note: 1 L f (t) dt, L L 1 L f (t) cos(n t) dt, an = L L L 1 L f (t) sin(n t) dt. bn = L L L a0 =

(1)

The key fact is the following collection of integral formulas for sines and cosines, which go by the name of orthogonality relations: 1 n = m = 0 1 L L L cos( n L t ) cos( m L t ) dt = 0 n = m 2 n=m=0 1 L L L cos( n L t ) sin( m L t ) dt = 0 1 n = m = 0 1 L L L sin( n L t ) sin( m L t ) dt = 0 n = m Proof of the orthogonality relations: This is just a straightforward calculation using the periodicity of sine and cosine and either (or both) of these two methods: iat eiat iat eiat Method 1: use cos at = e + , and sin at = e . 2 2i
1 Method 2: use the trig identity cos() cos( ) = 2 (cos( + ) + cos( )), and the similar trig identies for cos() sin( ) and sin() sin( ).

Using the orthogonality relations to prove the Fourier coefcient formula Suppose we know that a periodic function f (t) has a Fourier series expansion a0 f (t) = + an cos n t + bn sin n t (2) 2 L L n =1 How can we nd the values of the coefcients? Lets choose one coefcient, say a2 , and compute it; you will easily how to generalize this to any other coefcient. The claim is that the right-hand side of the Fourier coefcient formula (1), namely the integral 1 L f (t) cos 2 t dt. L L L

Orthogonality Relations

OCW 18.03SC

is in fact the coefcent a2 in the series (2). We can replace f (t) in this integral t by the series in (2) and multiply through by cos 2 L , to get 1 L
L a0
L

cos 2 t + an cos n t cos 2 t + bn sin n t cos 2 t dt L 2 L L L L n =1

Now the orthogonality relations tell us that almost every term in this sum will integrate to 0. In fact, the only non-zero term is the n = 2 cosine term 1 L
L

a2 cos 2 t cos 2 t dt L L L

and the orthogonality relations for the case n = m = 2 show this integral is equal to a2 as claimed. a0 Why the denominator of 2 in ? 2 Answer: it is in fact just a convention, but the one which allows us to have the same Fourier coefcent formula for an when n = 0 and n 1. (Notice that in the n = m case for cosine, there is a factor of 2 only for n = m = 0.) a0 Interpretation of the constant term . 2 We can also interpret the constant term a20 in the Fourier series of f (t) as the L average of the function f (t) over one full period: a20 = 21L L f (t) dt.

Operations on Fourier Series


In this session we learn a number of useful techniques for computing and manipulating Fourier series. We will also see some ways to get a new Fourier series from a given one, including the use of shifts, differentiation and integration. These can provide calculation shortcuts: if we already have worked out a Fourier series, we can nd the coefcients of the series for a related function without having to use the integral formulas again.

Compute a Fourier Series


Exercise. We warm up with a reminder of how one computes the Fourier series of a given periodic function using the integral Fourier coefcient formulas. Compute the Fourier series for the period 2 continuous sawtooth function f (t) = |t| for t . Answer.

2 2 Figure 1. Graph of the period 2 continuous sawtooth function. The period is 2 , so the half-period L = . Since f (t) = |t| for t , it is an even function we know the Fourier sine coefcents bn must be zero. Computing the cosine coefcients we get: For n = 0: an = 2 t cos(nt) dt 0 4 n2 2 t sin(nt) cos(nt) 2 n = = + (( 1 ) 1 ) = n n2 0 n2 0

|t| cos(nt) dt =

for n odd for even

For n = 0: 2 1 a0 = |t| dt = t dt = . 0 Thus, f (t) has Fourier series 4 cos(3t) cos(5t) f (t) = cos t + + + 2 32 52

4 2

cos(nt) n2 n odd

Compute a Fourier Series


Exercise. We warm up with a reminder of how one computes the Fourier series of a given periodic function using the integral Fourier coefcient formulas. Compute the Fourier series for the period 2 continuous sawtooth function f (t) = |t| for t .

Even and Odd Functions


If a periodic function f (t) is an even function we have already used the fact that its Fourier series will involve only cosines. Likewise the Fourier series of an odd function will contain only sines. Here we will give short proofs of these statements. Even and odd functions. Denition. A function f (t) is called even if f (t) = f (t) for all t.

The graph of an even function is symmetric about the y-axis. Here are some examples of even functions:

1. t2 , t4 , t6 , . . . , any even power of t. 2. cos( at) (recall the power series for cos( at) has only even powers of t). 3. A constant function is even. We will need the following fact about the integral of an even function over a balanced interval [ L, L]. If f (t) is even then
L
L

f (t) dt = 2

L
0

f (t) dt.

This fact becomes clear if we think of the integral as an area (see g. 1).

t L L L L

Fig. 1: Even functions: ( total area = twice area of right half)

Fig. 2: Odd functions: (total (signed) area is 0)

The graph of an odd function is symmetric about the the origin. Here are some examples of odd functions: 1. t, t3 , t5 , . . . , any odd power of t. 2. sin( at) (recall the power series for sin( at) has only odd powers of t).

Denition. A function f (t) is called odd if f (t) = f (t) for all t.

Even and Odd Functions

OCW 18.03SC

We will need the following fact about the integral of an odd function over a balanced interval [ L, L]. If f (t) is odd then
L
L

f (t) dt = 0.

This fact becomes clear if we think of the integral as an area (see Fig. 2). Multiplying Even and Odd Functions When multiplying even and odd functions it is helpful to think in terms of multiply even and odd powers of t. This gives the following rules. 1. even even = even 2. odd odd = even 3. odd even = odd

All this leads to the even and odd Fourier coefcient rules: Assume f (t) is periodic then: 2 L 1. If f (t) is even then we have bn = 0, and an = f (t) cos n t dt. L 0 L L 2 2. If f (t) is odd then we have an = 0, and bn = f (t) sin n t dt. L 0 L Reason: Assume f (t) is even. The rule for multiplying even functions tells us that f (t) cos at is even and the rule for integrating an even function over a symmetric interval tell us that an = 1 L
L
L

2 L f (t) cos n t dt = f (t) cos n t dt. L L 0 L

Likewise, the rule even odd = odd tell us that f (t) sin at is odd, and so the integral for bn is 0. If f (t) is odd everything works much the same. The rule for multiplying odd functions tells us that f (t) sin at is even and therefore 1 bn = L
L
L

2 L f (t) sin n t dt = f (t) sin n t dt. L L 0 L

Likewise the rule odd even = odd tells us that f (t) cos( at) is odd, and so the integral for an is 0. Examples: In previous sessions we saw the odd square wave had only sine coefcients and the even triangle wave had only cosine coefcients. 2

Scaling and Shifting


There is a very useful class of shortcuts which allows us to use the known Fourier series of a function f (t) to get the series for a function related to f (t) by shifts and scale changes. We illustrate this technique with a collection of examples of related functions. We let sq(t) be the standard odd, period 2 square wave. 1 for t < 0 sq(t) = 1 for 0 t < 1
t

(1)

1 Figure 0: The graph of sq(t), the odd, period 2 square wave.


We already know the Fourier series for sq(t). It is 4 1 1 4 sin(t) + sin(3t) + sin(5t) + = sq(t) = 3 5

sin(nt) (2) n n odd

1.

Shifting and Scaling in the Vertical Direction

Example 1. (Shifting) Find the Fourier series of the function f 1 (t) whose graph is shown. 2
t Figure 1: f 1 (t) = sq(t) shifted up by 1 unit.

Solution. The graph in Figure 1 is simply the graph in Figure 0 shifted upwards one unit. That is, f 1 (t) = 1 + sq(t). Therefore f 1 (t) = 1 + 4 sin(nt) . n n odd

Example 2. (Scaling) Let f 2 (t) = 2 sq(t). Sketch its graph and nd its Fourier series.

Scaling and Shifting

OCW 18.03SC

Solution. 2
t

2 Figure 2: Graph of f 2 (t) = 2 sq(t).


The Fourier series of f 2 (t) comes from that of sq(t) by multiplying by 2. f 2 (t) = 8 sin(nt) . n n odd

Example 3. We can combine shifting and scaling along the vertical axis. Let f 3 (t) be the function shown in Figure 3. Write it in terms of sq(t) and nd its Fourier series. 1
t

1 Figure 3: f 3 (t) = sq(t) shifted by 1 and then scaled by 1/2. 1 1 2 sin nt . Solution. f 3 (t) = (1 + sq(t)) = + 2 2 n odd n

2.

Scaling and Shifting in t

Example 4. (Scaling in time) Find the Fourier series of the function f 4 (t) whose graph is shown. 1
t

1 Figure 4: sq(t) scaled in time.


InFigure 4 the point marked 1 on the t-axis corresponds with the point marked in Figure 0. This shows that f 4 (t) = sq( t) and therefore we replace t by t in the Fourier series of sq(t). f 4 (t) = 4 sin(n t) . n n odd

Scaling and Shifting

OCW 18.03SC

Example 5. (Shifting in time) Let f 5 (t) = sq(t + /2). Graph this function and nd its Fourier series. Solution. We have f 5 (t) is sq(t) shifted to the left by /2. Therefore 4 cos 3t 4 sin(3t + 3 /2) cos t sin(t + /2) + +... = +... f 5 (t) = 3 3 (To simplify the series we used the trig identities sin( + /2) = cos( ) and sin( + 3 /2) = cos( ) etc.) 1 /2 Figure 5: sq(t) shifted in time. Notice that f 5 (t) is even, and so must have only cosine terms in its series, which is in fact conrmed by the simplied form above.
t

Integration and Differentiation


We can integrate a Fourier series term-by-term: Example 1. Let f (t) = 1 + cos t + then, h(t) =
t
0

cos 2t cos 3t + +... 2 3 sin 2t sin 3t + 2 +... 22 3

f (u) du = t + sin t +

Note: The integrated function h(t) is not periodic (because of the t term), so the result is a series, but not a Fourier series. We can also differentiate a Fourier series term-by-term to get the Fourier series of the derivative function. Example 2. Let f (t) be the period 2 triangle wave (continuous sawtooth) given on the interval [ , ) by f (t) = |t|. Its Fourier series is 4 cos 3t cos 5t f (t) = cos t + + +... 2 32 52 In the previous session we computed the Fourier series of a period 2 triangle wave. This series can then be obtained from that one by scaling by in both time and the vertical dimension, using the methods we learned in the previous note. The derivative of f (t) is the square wave. (You should verify this). Differentiating the Fourier series of f (t) term-by-term gives 4 sin 3t sin 5t f (t) = sin t + + +... , 3 5 which is, indeed, the Fourier series of the period 2 square wave we found in the previous session.

2 2 Figure 1: The period 2 triangle wave.

Integration and Differentiation

OCW 18.03SC

Example 3. What happens if you try to differentiate the square wave 4 sin 3t sin 5t sq(t) = sin t + + +... ? 3 5 Solution. Differentiation term-by-term gives sq (t) = 4 (cos t + cos 3t + cos 5t + . . .) .

But, what is meant by sq (t)? Since sq(t) consists of horizontal segments its derivative at most places is 0. However we cant ignore the vertical segments where the function has a jump discontinuity. For now, the best we can say is that the slope is innite at these jumps and sq (t) doesnt exist. Later in this unit we will learn about delta functions and generalized derivatives, which will allow us to make better sense of sq (t).

Convergence of Fourier Series


The period 2 L function f (t) is called piecewise smooth if there are a only nite number of points 0 t1 < t2 < . . . < tn 2 L where f (t) is not differentiable, and if at each of these points the left and right-hand limits lim f (t) and lim f (t) exist (although they might not be equal).
+ t ti t ti

Recall that when we rst introduced Fourier series we wrote a0 f (t) + a1 cos(t) + a2 cos(2t) + a3 cos(3t) + . . . 2 +b1 sin(t) + b2 sin(2t) + b3 sin(3t) + . . .

a0 + an cos(nt) + bn sin(nt), 2 n =1

where we used instead of an equal sign. The following theorem shows that our subsequent use of an equal sign, while not technically correct, is close enough to be warranted. Theorem: If f (t) is piecewise smooth and periodic then the Fourier series for f 1. converges to f (t) at values of t where f is continuous 2. converges to the average of f (t ) and f (t+ ) where it has a jump discontinuity. Example. Square wave. No matter what the endpoint behavior of f (t) the Fourier series converges to:

Original f (t)

Fourier series for f (t)

Example. Continuous sawtooth: Fourier series converges to f (t). Example.

Original f (t)

Fourier series

An Interpretation of Fourier Series


Fourier Analysis is a name sometimes used to denote the decomposition of a function f (t) into the sum of its sinusoidal harmonics, which are also called its Fourier components. Fourier Synthesis is a name used to denote the building up of a function f (t) by adding up its successive Fourier components; that is, the reconstruction of a function f (t) from its Fourier components. The are electronic devices which can perform both types of operations, called Fourier analyzers and Fourier synthesizers. The ear and the brain also function as, respectively, a Fourier analyzer and a Fourier synthesizer! We describe this briey (in very general terms). The input to the ear is a time-varying pressure wave-form f (t). The inner ear has an array of about twenty thousand hair-like cells, each of which resonates at a different frequency. Each of the individual Fourier components of f (t) stimulates a different one of these hair" cells. Thus this array of cells acts all together as a Fourier analyzer. Each hair cell which gets selected by a component of f (t) to be driven into motion then stimulates an attached nerve which sends a signal to the brain. The brain (being the smart and capable device it is) then somehow combines or synthesizes these individual received Fourier components and produces a reconstructed approximation of the function f (t) as their sum. This reconstructed f (t)-pattern in the brain is then what we experience as the sound corresponding to the incoming pressure-wave signal f (t).

Gibbs Phenomenon
In practice it may be impossible to use all the terms of a Fourier series. For example, suppose we have a device that manipulates a periodic signal by rst nding the Fourier series of the signal, then manipulating the sinusoidal components, and, nally, reconstructing the signal by adding up the modied Fourier series. Such a device will only be able to use a nite number of terms of the series. Gibbs phenomenon occurs near a jump discontinuity in the signal. It says that no matter how many terms you include in your Fourier series there will always be an error in the form of an overshoot near the discontinuity. The overshoot always be about 9% of the size of the jump. We illustrate with the example. of the square wave sq(t). The Fourier series of sq(t) ts it well at points of continuity. But there is always an overshoot of about .18 (9% of the jump of 2) near the points of discontinuity.
1.18 1 1.18 1

t
1 1 1 1

1 1.18

1 1.18

Gibbs: max n = 1
1.18 1

Gibbs: max n = 3
1.18 1

t
1 1 1 1

1 1.18

1 1.18

Gibbs: max n = 9

Gibbs: max n = 33

In these gures, for example, max n=9 means we we included the terms for n = 1, 3, 5, 7 and 9 in the Fourier sum 4 sin 3t sin 5t sin 7t sin 9t sin t + + + + . 3 5 7 9

Application to Innite Series


There is a famous formula found by Euler:

n =1

1 2 = . 6 n2

(1)

Well show how you can use a Fourier series to get this result. t on [0, 2 ]. Consider the period 2 function given by f (t) = t 2

t
2

Figure 1: Graph of f (t). First, we compute the Fourier series of f (t). Since f is even, the sine terms are all 0. For the cosine terms it is slightly easier to integrate over a full period from 0 to 2 rather than doubling the integral over the halfperiod. We give the results, but leave the details of the integration by parts to the reader. For n = 0 we have 1 2 2 2 a0 = t( t/2) dt = 0 3 and for n = 0 we have 1 an =
2
0

t( t/2) cos(nt) dt

2 1 t sin(nt) cos(nt) t2 sin(nt) t cos(nt) sin(nt) 2 = + + = 2. n n2 2n n2 n3 0 n


2 cos(nt) . 2 3 n2 n =1 Since the function f (t) is continuous, the series converges to f (t) for all t. Plugging in t = 0, we then get

Thus the Fourier series is f (t) =

f (0) = 0 =

2 2 2. 3 n n =1

A little bit of algebra then gives Eulers result (1).

ODEs with Periodic Input, Resonance


Fourier series can be used to solve an inhomogeneous linear time invariant (LTI) DE p( D ) x = f (t) (1) in the case where f (t) is a periodic function. The main idea is to use Fourier series and the superposition principle to reduce the problem to solving (1) when the input f (t) is a sinusoid. Very briey: if f (t) is periodic then its Fourier series gives f as a sum of sines and cosines. The superposition principle allows us to nd the response of (1) to each of the sine or cosine terms separately and then superposition these individual responses to nd the response of the system to the periodic signal f . The particular solution found in this way will also be a sum of sines and cosines with the same period as the input. We will call it the steady-periodic solution, xsp (t). One way in which the steady-periodic solution is useful is that it reveals when resonances will occur, and at what frequencies. In this session we will also show how Fourier series can be used to analyze sound waves.

Example: Simple Harmonic Oscillator


Example. Let f (t) = the odd square wave of period 2 with f (t) = 1 for 0 < t < 1. Use Fourier series to solve the DE

..

x + 9.1x = f (t).

(1)

Solution. From previous examples we know the Fourier series for f (t), 4 sin 3t sin 5t 4 sin nt sin t + + +... = f (t) = 3 5 n odd n So the DE (1) becomes

..

4 x + 9.1x =

sin 3t sin 5t sin t + + +... . 3 5

(2)

Step 1: Solve the DE with a single sine function as input. That is, solve

..

x n + 9.1xn =

sin nt . n

(3)

Notice, we use the index n so we can tell our solutions apart. Also notice 4 that equation (3) does not include the factor ; we will bring that back in the superposition step. We have a lot of experience solving equation (3). using complex replacement and the Exponential Response formula. We get particular solutions x n, p ( t ) = sin nt . n(9.1 n2 )

Step 2: Use superposition to get a particular solution x p to (2). Here we line up the DE and the solution so you can see superposition in action:

..

x + 9.1x xsp (t)

= = =

4 (sin t 4 ( x1, p ( t ) 4 sin t 9.11 .

3t + sin 3

5t + sin 5

+ . . .) + . . .) +...

= = =

4 4 4

sin nt n n odd

+ x3, p (t)
3t + 3(sin 9.19)

+ x5, p (t)
sin 5t + 5(9.1 25)

x n, p ( t )

n odd

sin nt . n(9.1 n2 ) n odd

(4)

This is called the steady periodic solution.

Example: Simple Harmonic Oscillator

OCW 18.03SC

Near resonance: The amplitudes of each of the terms in (4) are:

1 9.1 1

0.157,

1 3(9.1 9)

4.244,

1 5(9.1 25)

0.016

for n = 1, 3, 5 respectively. Then for n > 5 the amplitudes are much smaller. We see the n = 3 term in the steady periodic response xsp (t) has by far the biggest amplitude. We can explain this by noticing that the natural frequency of this system is 9.1 3 and so, the system has a resonant-type response to the 3t embedded third harmonic sin 3 in the input signal. Notice that the input signal has base (fundamental) frequency 1, so the presence of this third harmonic is not apparent to the eye, and yet the driven oscillator picked it out in its response, which has a dominant frequency three times the fundamental frequency of the input. There is a simple way to visualize this type of phenomenon: you can push a pendulum swing into resonance even if you give it a push only every third time it comes momentarily to rest at its maximum height, instead of pushing it every time.

Harmonic Frequency Response Applet


As usual, start the applet and play with it a little bit. In this applet the input has a xed angular frequency of 1 and the n slider adjusts the resonant frequency of the system. Choose f (t) to be the sine wave. Look at what happens as you change n . Why does the amplitude of the response go to innity when n = 1. Now choose f (t) to be the square wave. Notice that the amplitude of the response becomes innite at n = 1, 3 or 5. Question: As n gets close to 1, 3 or 5 what is the dominant frequency in the ouput? Answer: You should have seen that with n near 1 the output resembles a frequency 1 sine wave. For n near 3 the dominant frequency in the output is 3, i.e. there are three peaks in the oscillation over one cycle of the square wave. Likewise for wn near 5 the dominant frequency is 5. We can explain this using Fourier series. The square wave has Fourier series 4 sin nt f (t) = . n odd n Each term in the series affects the system. If the system has natural frequency 3 then the sin 3t term causes it to resonate with a large amplitude at that frequency. Thus, the response to that term is far larger than the response to any other term. We will explore this further later in this session.

Example: Damped Harmonic Oscillator


Example. Let f (t) be the triangle wave shown in gure 1. Solve the differential equation .. . x + 2 x + 9 x = f ( t ). 1

Solution. Using a previous example, or computing directly, we have the Fourier series for f (t) is 1 4 cos 3t cos 5t f (t) = 2 cos t + + +... . 2 32 52 We follow the same steps as in the example in the previous note. Step 1: Solving for the individual components: Solve: .. . x n + 2x n + 9xn = cos nt If n = 0 we get xn, p = For n 1 we have .. . Complex replacement: z n + 2zn + 9zn = eint , xn = Re(zn ) eint Exponential Response formula: zn, p = . 9 n2 + 2in Polar coords: 9 n2 + 2in = Rn ein , where Rn = (9 n2 )2 + 4n2 and n = Arg(9 n2 + 2in) = tan1
1 9.

(1)

2n 9 n2 (since the complex number is in the rst or second we must take the arctangent between 0 and ). 1 i(ntn ) 1 Thus, zn, p = e , which implies xn, p = cos(nt n ) Rn Rn Step 2: Superposition. To make things easier in step one we did not include the Fourier coefcients of the input in the DE (1). To use superposition we need to include them here. 1 4 cos(t 1 ) cos(3t 3 ) cos(5t 5 ) xsp (t) = + + +... , 18 2 R1 32 R 3 52 R5 with the formulas for Rn and n as above.

General Case
It is actually just as easy to write out the formula for the Fourier series expansion of the steady-periodic solution xsp (t) to the general secondorder LTI DE p( D ) x = f (t) with f (t) periodic as it was to work out the previous example - the only difference is that now we use letters instead of numbers. We will choose the letters used for the spring-mass-dashpot system, but clearly the derivation and formulas will work with any three parameters. For simplicity we will take the case of f (t) even (i.e. cosine series). Problem: Solve mx + bx + kx = f (t), for the steady-periodic response a0 xsp (t), where f (t) = + an cos n t dt 2 L n =1 Solution Characteristic polynomial: p(s) = ms2 + bs + k. Solving for the component pieces: .. . mx n + bx n + kxn = cos n Lt 1 For n = 0 we get x0, p = . k For n 1: .. . Complex replacement: m z n + bzn + kzn = ein L t , xn = Re(zn ) ein L t Exponential Response formula: zn, p (t) = . p(in L) 2 i n Polar coords: p(in + ibn L ) = k m(n L ) L = | p (in L ) | e , 2 2 2 where | p(in L )| = k m(n ) + b2 ( n and L L) bn 1 L n = Arg( p(in (phase lag). 2 L )) = tan
km(n L )

..

Thus, zn, p (t) = gn

ei(n L tn ) ,

with gn =

1 | p(in L )|

(gain).

Taking the real part of xn, p we get xn, p (t) = gn cos(n L t n ). Now using superposition and putting back in the coefcients an we get: xsp (t) =
a0 a0 x0, p + an xn, p (t) = + 2 2 k n =1

n =1

gn an cos(n t n ) L

This is the general formula for the steady periodic response of a secondorder LTI DE to an even periodic driver f (t)

Fourier Coefcients: Complex with Sound


Caution: If you listen to the sound through headphones, start by setting the volume low and increase it slowly. Only let the volume get high enough to hear the sounds. This is especially important when playing pure sine waves because brain is used to auditory input of many frequencies at once. To get the same apparent loudness from a single frequency requires a large amplitude. Prolonged exposure to a high amplitude sound of a given frequency can damage your hearing at that frequency. As usual, rst open the applet and play with it. The applet makes a Fourier series of a function out of complex exponentials instead of sines and cosines. It then graphs the real part of the function plays the periodic sound whose pressure wave corresponds to the graph. Using the applet. 1. The fundamental (base) frequency is given in kilohertz (kHz). Therefore, the fundamental angular frequency is = 2000 . There is a slider for setting the value of . 2. The function f (t) is given by f (t) = cn ein t , where the cn are complex coefcients. Therefore f (t) is a complex-valued function. 3. There is a parameter with a slider. Leave it at 0 for now. 4. The graph shows Re(ei f (t)). The sound played corresponds to this graph. 5. To select one of coefcients cn , click on the yellow dot for that coefcient in either of the windows on the lower right. There are two ways to adjust the value of cn . First, you can use the mouse to drag the white dot representing the value of cn around the complex plane displayed on the lower left. The second method is to use the sliders on the lower right. The top slider represents the magnitude |cn | and the bottom slider the polar angle Arg(cn ). Notice that as you move the sliders the point in the complex plane also moves. Exploring the Applet Now click the reset button to set all the coefcients to 0. Set to 0.

Fourier Coefcients: Complex with Sound

OCW 18.03SC

Make sure the f (t) real checkbox is not selected. Set the frequency to the lower A value. (This is the musical note A440 (Hz).) Select the rst coefcient (c1 ) and set the magnitude to 2. The graph should be a sinusoid. Now start the sound, you should hear a steady pure tone. While the sound is playing adjust the Arg(c1 ). What happened to the graph? What happened to the sound? The graph should slide left or right as you change the phase angle. The sound shouldnt change: you ear does not detect phase. Play with setting the higher harmonics, i.e. setting the other coefcients. How does the sound change? Does the pitch that youre hearing change? If the higher harmonics have much lower amplitudes than the fundamental frequency, then the fundamental pitch will stay the same but the quality of the sound will change. If the amplitude of a higher harmonic approaches that of the fundamental you may begin to hear it as a separate note. Make sure the f (t) real checkbox is not selected and = 0. Try to adjust the coefcients to get a square wave. Hint: Which coefcients of the square wave are nonzero? How do you get a sine function out of complex exponentials. As we found in an earlier session, the Fourier series of the square wave with fundamental angular frequency = 2 and amplitude 1 is 4 (sin( t) + sin(3 t)/3 + sin(5 t)/5 + . . .) . Since sin( x ) is the real part of ieix and we selected amplitude 2, we should set c1 = 8i 8i 8i 8i 8i , c3 = , c5 = , c7 = , c9 = . 3 5 7 9

(All the even coefcients are 0.) Could you get the square wave using the coefcients c1 , c3 etc.? Could you get the square wave using real coefcients and adjusting the value of ?

Step and Delta Functions: Introduction


This session will make two additions to our mathematical modeling toolkit: step functions and delta functions. These are simple functions modeling idealized signals. A step function represents an idealized signal that switches from off to on at a specic time. That is, its value jumps from 0 to 1. In the real world the signal would go through a transition phase, which would take a small amount of time to get from 0 to 1. We idealize it as making the transition instantaneously. The most basic step function is the unit or Heaviside step function, u(t). It is 0 for t < 0 and 1 for t > 0. Its graph looks like
u(t) 1 t

The graph of the unit step function. A delta function represents an idealized input that acts all at once. If a nite force pushes on a mass it changes the momentum of the mass over time. We can achieve the same change in momentum with a small force acting over a long time or a large force acting over a short time. If the force acts over a very short time we call it an impulse. The unit delta function (t) (also called the unit impulse function) models an idealized impulse, which can be thought of as an innite force acting over an innitesimal amount of time and causes a unit change in the momentum of the mass. In the above, the delta function represented an idealized impulsive force acting on a second order mechanical system. First order system, and indeed systems of any order, also have the notion of an impulse, which can also be modeled by a delta function. Step functions and delta functions are not differentiable in the usual sense, but they do have what we call generalized derivatives. In fact, as a generalized derivative we have u (t) = (t). Since step and delta functions can also be integrated they can used in DEs. Step and delta functions are of fundamental importance in our study of LTI systems. For example, if we know the response of such a system to either the unit delta or unit step function then we can compute its response to any input whatsoever.

Step and Box Functions 1. Heaviside Unit Step Function


The unit step function is dened by 0 for t < 0 u(t) = 1 for t > 0 The reason for the name unit step can be seen in the graph.
u(t) 1 t 1 t u(t a)

The rst graph shows the function u(t). The second graph shows u(t a), which is simply u(t) shifted to the right. 0 for t < a u(t a) = 1 for t > a A few details need to be highlighted. 1. u(t) is also called the Heaviside function. 2. u(t) is not dened when t = 0. Looking at the graph we see that u(t) has a jump discontinuity at t = 0. 3. The graph shows that u(0 ) = 0 and u(0+ ) = 1. Here, u(0 ) means the limit of u(t) as t approaches 0 from the left called the left-hand limit. Likewise, u(0+ ) means the limit of u(t) as t approaches 0 from the right. 4. In the graphs we used dashed lines at the jump discontinuity. These lines are not part of the graph and we could have left them out. It is also common to use solid lines. Strictly speaking this is incorrect, but it gives nicer looking gures (for more on this see the next section).

2. Models
We can use u(t) to model an on/off process. Suppose a light turns on; rst it is dark, then it is light. The basic model is the unit step function

Step and Box Functions

OCW 18.03SC

Of course a light doesnt reach its steady state instantaneously; it takes a small amount of time. If we use a ner time scale, you can see what happens. It might move up smoothly; it might overshoot; it might move up in ts and starts as different elements come on line. If we zoomed in near t = 0 the graph might actually look like

1 t

.01

At the longer time scale, we dont care about these details. Modeling the process by u(t) lets us ignore them.

3. Box Functions
When we modeled the light with u(t) we assumed the light went on and stayed on forever. Eventually the light will be turned off or burn out. To be general, lets assume the light goes on at time a and off at time b. We can model this with the function 0 for t < a 1 for a < t < b u ab (t) = 0 for b < t. The graph of this is
uab (t) = u(t a) u(t b) 1

The graph shows why this is often called a box function. If you plot u(t a) and u(t b) on the same axes you will nd that u ab (t) = u(t a) u(t b). 2

Step and Box Functions

OCW 18.03SC

We will usually dispense with the notation u ab (t) and the formuala u(t a) u(t b) for the box function.

4. Switches
By multiplying by a function f (t) we can use step and box functions as switches to turn f (t) on or off.

The graphs show step and box functions acting as switches.

The rst plot shows f (t). The second shows u(t a) being used to switch f (t) on at t = a. That is, u(t a) f (t) is 0 for t < a and agrees with f (t) for t > a. The third shows u(t a) u(t b) being used to turn f (t) on in the window a < t < b and off outside it. The fourth graph shows u(t a) f (t a). That is, rst f (t) is translated to the right a units and the result is switched on at time a.

5.

u-format and Cases Format

We now have two ways to express functions that change formulas for different intervals of t. Example. Suppose f (t) is 0 for t < 0, t for 0 < t < 1, t2 for 1 < t < 2 and 2t for t > 2. Express f (t) in both u and cases formats. Solution. Cases format expresses f (t) by specifying the formula for each case: 0 for t < 0 t for 0 < t < 1 f (t) = 2 t for 1 < t < 2 2t for 2 < t. u-format uses step and box functions to turn on and off expressions: f (t) = (u(t) u(t 1)) t + (u(t 1) u(t 2)) t2 + u(t 2) 2t. 3

Step and Box Functions

OCW 18.03SC

Notice how each case tells us which step or box functions to use as switches and how each u function tells us where the cases change. Example. format. Solution. Write f (t) = u(t) 4t + u(t 2) t2 + u(t 4) t3 /4 in cases 0 4t f (t) = 4t + t2 4t + t2 + t3 /4 for t < 0 for 0 < t < 2 for 2 < t < 4 for 4 < t.

Notice how there are no off switches in the expression for f (t), so in cases format the number of terms in each successive case grows as the u-switches turn on.

Using Step Functions as Switches


Quiz: What is the equation for the function which agrees with f (t) between a and b (assume a < b) and is zero outside this window? Choices: a) (u(t b) u(t a)) f (t) b) (u(t a) u(t b)) f (t a) c) (u(t a) u(t b)) f (t) d) u(t a) f (t a) u(t b) f (t b) e) none of these

Answer: (c): (u(t a) u(t b)) f (t). The box function u(t a) u(t b) is 1 between a and b and 0 outside it.

Using Step Functions as Switches


Quiz: What is the equation for the function which agrees with f (t) between a and b (assume a < b) and is zero outside this window? Choices: a) (u(t b) u(t a)) f (t) b) (u(t a) u(t b)) f (t a) c) (u(t a) u(t b)) f (t) d) u(t a) f (t a) u(t b) f (t b) e) none of these Pick what you think is the correct choice and then look at the answer.

Using Step Functions as Switches


Quiz: What is the equation for the function which agrees with f (t) between a and b (assume a < b) and is zero outside this window? Think about your answer and then look at the choices.

Delta Functions: Unit Impulse 1. Introduction


In our discussion of the unit step function u(t) we saw that it was an idealized model of a quantity that goes from 0 to 1 very quickly. In the idealization we assumed it jumped directly from 0 to 1 in no time. In this note we will have an idealized model of a large input that acts over a short time. We will call this model the delta function or Dirac delta function or unit impulse. After constructing the delta function we will look at its properties. The rst is that it is not really a function. This wont bother us, we will simply call it a generalized function. The reason it wont bother us is that the delta function is useful and easy to work with. Inside integrals or as input to differential equations we will see that it is much simpler than almost any other function.

2. Delta Function as Idealized Input


Suppose that radioactive material is dumped in a container. The equation governing the amount of material in the tank is x + kx = q(t), where, x (t) is the amount of radioactive material (in kg), k is the decay rate of the material (in 1/year), and q(t) is the rate at which material is being added to the dump (in kg/year). The input q(t) is in units of mass/time, say kg/year. So, the total amount dumped into the container from time 0 to time t is Q(t) = Equivalently
t
0

q(u) du.

Q ( t ) = q ( t ). To keep things simple we will assume that q(t) is only nonzero for a short amount of time and that the total amount of radioactive material dumped over that period is 1 kg. Here are the graphs of two possibilities for q(t) and Q(t).

Delta Functions: Unit Impulse

OCW 18.03SC

q (t) 2

= 1 1/2 t

Q(t)

t 1/2

8 q (t) Q(t)

= 1

1/8

t 1/8

Figure 1: two possible graphs of q(t) and Q(t), both with total input = 1. It is easy to see that each of the boxes on the left side of Figure 1 has total area equal to 1. Thus, the graphs for Q(t) rise linearly to 1 and then stay equal to 1 thereafter. In other words, the total amount dumped in each case is 1. Now let qh (t) be a box of width h and height 1/h. As h 0, the width of the box becomes 0, the graph looks more and more like a spike, yet it still has area 1 (see Figure 2).
2 1 t
1 2

16

1 h=1

h = 1/2

h = 1/16

1 16

t h 0, q (t) = (t)

Figure 2: Box functions qh (t) becoming the delta function as h 0. We dene the delta function to be the formal limit (t) = lim qh (t).
h 0

Graphically (t) is represented as a spike or harpoon at t = 0. It is an innitely tall spike of innitesimal width enclosing a total area of 1 (see gure 2, rightmost graph). 2

Delta Functions: Unit Impulse

OCW 18.03SC

As an input function (t) represents the ideal case where 1 unit of material is dumped in all at once at time t = 0.

3. Properties of (t)
We list the properties of (t) below. 1. From the previous section we have 0 (t) =

if t = 0, if t = 0.

The graph is represented as a spike at t = 0. (See gure 2 2. Because (t) is the limit of graphs of area 1, the area under its graph is 1. More precisely: d 1 if c < 0 < d (t) dt = 0 otherwise c

3. For any continuous function f (t) we have f ( t ) ( t ) = f (0) ( t ) and


d
c

f (t)(t) dt =

f (0) 0

if c < 0 < d otherwise

The rst statement follows because (t) is 0 everywhere except at t = 0. The second follows from the rst and property (2). 4. We can place the delta function over any value of t: (t a) is 0 everywhere but at t = a. Its total area remains 1. Its graph is now a spike shifted to be over t = a; and we have f ( t ) ( t a ) = f ( a ) ( t a ). d f ( a) if c < a < d f (t)(t a) dt = 0 otherwise c 5. (t) = u (t), where u(t) is the unit step function. Because u(t) has a jump at 0, (t) is not a derivative in the usual sense, but is called a generalized derivative. This is explained below.

(t a)

Delta Functions: Unit Impulse

OCW 18.03SC

6. We dened (t) as a limit of a sequence of box functions, all with unit area and which, in the limit, become a innite spike over t = 0. Box functions are simple, but not special. Any sequence of functions with these properties has (t) as its limit. 7. In practical terms, you should think of (t) as any function of unit area, concentrated very near t = 0. 8. (t) is not really a function. We call it a generalized function. 9. In arriving at these properties we have skipped over some important technical details in the analysis. Generally property (3) is taken to be the formal denition of (t), from which the other properties follow.

4. Examples of integration
Properties (3) and (2) show that (t) is very easy to integrate, as the following examples show:
5

Example 1.

integrand at t = 0.
5

7et cos(t)(t) dt = 7. All we had to do was evaluate the

Example 2.

evaluate the integrand at t = 2.


1

7et cos(t)(t 2) dt = 7e4 cos(2). All we had to do was


2

Example 3.

of integration the integrand is 0 on the entire interval.

7et cos(t)(t 2) dt = 0. Since t = 2 is not in the interval

The value t = 0 represents the left-side of 0 and t = 0+ is the rightside. So, 0 is in the interval [0 , ) and not in [0+ , ). Thus

0

(t) dt = 1

and


0+

(t) dt = 0.

In fact, since all the area under the graph is concentrated at 0, we can even write +
0 0

(t) dt = 1.

5. Generalized Derivatives
Our goal in this section is to explain property (5). A look at the graph of the unit step function u(t) shows that it has slope 0 everywhere except 4

Delta Functions: Unit Impulse at t = 0 and that its slope is at t = 0.


1 u(t)

OCW 18.03SC

That is, its derivative is u (t) =

if t = 0 if t = 0.

Since u(t) has a jump of 1 at t = 0 this derivative matches properties (1) and (2) of (t) and we conclude that u (t) = (t). Now this derivative does not exist in the calculus sense. The function u(t) is not even dened at 0. So we call this derivative a generalized derivative. We can also explain property (5) by looking at the anti-derivative of (t). Let f (t) =
t

( ) d .

The fundamental theorem of calculus leads us to say that f (t) = (t). (Again, this is only in a generalized sense since technically the fundamental theorem of calculus requires the integrand to be continuous.) Property (3) makes it easy to compute 0 if t < 0 f (t) = 1 if t > 0. That is, f (t) = u(t), so u(t) is the antiderivative of (t). In general, a jump discontinuity contributes a delta function to the generalized derivative. Example 4. Suppose f (t) has the following graph.
2 f (t) = t2 f (t) = 3t 7 t f (t) = 2

2 -1

Delta Functions: Unit Impulse

OCW 18.03SC

The formula for each piece of the graph is indicated. For the smooth parts of the graph the derivative is just the usual one. Each jump discontinuity adds a delta function scaled by the size of the jump to f (t). 2t if t < 0 0 if 0 < t < 2 f ( t ) = 2 ( t ) 3 ( t 2) + 3 if 2 < t In the graph for f (t) we represent the delta functions as spikes with the magnitude written next to the spike. The sign is indicated by the direction of the spike. The rest of the f (t) is plotted normally.
2

We say f (t) is a generalized function. In 18.03 a generalized function will mean a sum of a regular function and a linear combination of delta functions. (In the wider world of mathematics there are other generalized functions.) If we want to refer to the different parts of a generalized function we will call the delta function pieces the singular part and the remainder will be called the regular part. If the singular part contains a multiple of (t a) we will say the function contains (t a).

Example. Consider f (t) = u(t) + (t) + et + 3(t 2). The regular part of f is u(t) + et . The singular part is (t) + 3(t 2). The function contains (t) and (t 2). It does not contain (t 1). Important: In this unit, whenever a discontinuous function is differentiated we will mean the generalized derivative.

Integration with Delta Functions


Quiz: Compute
10
0

5(t + 1) + 3(t) + t2 (t 5) + t(t 20) dt.

Choices: a) 0 b) 25 c) 28 d) 33 e) 48 f) 53 g) none of these

Answer: (c) 28. The interval of integration contains 0 and 5, but not -1 or 20. So, only the (t) and (t 5) terms contribute to the integral. Their contributions are 3 and 25 (t2 evaluated at 5).

Integration with Delta Functions


Quiz: Compute
10
0

5(t + 1) + 3(t) + t2 (t 5) + t(t 20) dt.

Choices: a) 0 b) 25 c) 28 d) 33 e) 48 f) 53 g) none of these Pick what you think is the correct choice and then look at the answer.

Integration with Delta Functions


Quiz: Compute
10
0

5(t + 1) + 3(t) + t2 (t 5) + t(t 20) dt.

Think about your answer and then look at the choices.

Generalized Derivatives.
Quiz: When you re a gun, you exert a very large force on the bullet over a very short period of time. If we integrate F = ma = mx we see that a large force over a short time creates a sudden change in the momentum, mx . This is called an "impulse." If the gun is red straight up, the graph of the elevation of the bullet, plotted against t, starts at zero, then rises in an inverted parabola, and then when it hits the ground it stops again. The velocity (derivative of the position function) is zero for t < 0; then it rises to v0 (the initial velocity of the bullet); then it falls at constant rate (the acceleration of gravity) until the instant when it hits the ground, when it returns abruptly to zero. The graph of v(t) looks like this:
v0 v (t) t

v0

What does the graph of the generalized derivative of v(t) look like? Choices:
a) v0 v0 b) v0 c) v0 d) t t

t v0

e) None of these. Answer: (a).

Generalized Derivatives.
Quiz: When you re a gun, you exert a very large force on the bullet over a very short period of time. If we integrate F = ma = mx we see that a large force over a short time creates a sudden change in the momentum, mx . This is called an "impulse." If the gun is red straight up, the graph of the elevation of the bullet, plotted against t, starts at zero, then rises in an inverted parabola, and then when it hits the ground it stops again. The velocity (derivative of the position function) is zero for t < 0; then it rises to v0 (the initial velocity of the bullet); then it falls at constant rate (the acceleration of gravity) until the instant when it hits the ground, when it returns abruptly to zero. The graph of v(t) looks like this:
v0 v (t) t

v0

What does the graph of the generalized derivative of v(t) look like? Choices:
a) v0 v0 b) v0 c) v0 d) t t

t v0

e) None of these. Pick what you think is the correct choice and then look at the answer.

Generalized Derivatives.
Quiz: When you re a gun, you exert a very large force on the bullet over a very short period of time. If we integrate F = ma = mx we see that a large force over a short time creates a sudden change in the momentum, mx . This is called an "impulse." If the gun is red straight up, the graph of the elevation of the bullet, plotted against t, starts at zero, then rises in an inverted parabola, and then when it hits the ground it stops again. The velocity (derivative of the position function) is zero for t < 0; then it rises to v0 (the initial velocity of the bullet); then it falls at constant rate (the acceleration of gravity) until the instant when it hits the ground, when it returns abruptly to zero. The graph of v(t) looks like this:
v0 v (t) t

v0

What does the graph of the generalized derivative of v(t) look like? Think about your answer and then look at the choices.

Unit Step and Unit Impulse Response: Introduction


In real life, we often do not know the parameters of a system (e.g. the spring constant, the mass, and the damping constant, in a spring- massdashpot system). We may not even know the order of the system. For example, there may be many interconnected springs or diodes. Instead, we often learn about a system by watching how it responds to various input signals. In this session we will study the response of a linear time invariant (LTI) system from rest initial conditions to two standard and very simple signals: the unit impulse (t) and the unit step function u(t). Reasonably enough we will call these responses the unit impulse response and the unit step response. The theory of the convolution integral studied in the next session will give us a method of dertemining the response of a system to any input once we know its unit impulse response. Because both (t) and u(t) are discontinuous at t = 0 we will have to be careful with our denition of initial conditions. The most sensible mathematical and physical way to do this is to dene our initial conditions at 0 . As input an impulse causes a jump when it is applied. This means that the conditions at 0+ will be different than those at 0 . To distinguish these two cases we will use the terms pre-initial conditions (at 0 ) and postinitial conditions (at 0+ ). We will be able to state precisely the effect of a unit impulse on these conditions.

Initial Conditions 1. Introduction


Before we try to solve higher order equations with discontinuous or impulsive input we need to think carefully about what happens to the solution at the point of discontinuity. Recall that we have the left and right limits of a function as t 0: x (0 ) = lim x (t)
t 0

and

x (0+ ) = lim x (t).


t 0

(Note that we can dene these limits as t goes to any value a.) For a continuous function these two limits are the same, and they are both equal to x (0). For the unit step function we have u(0 ) = 0, u(0+ ) = 1, u(0) is undened.

For the unit impulse function (t) we have (0 ) = 0, (0+ ) = 0, (0) = .

In this unit our differential equations will always have initial conditions at t = 0. The above examples show that when there is a discontinuity we might need to distinguish between 0 and 0+ . Assuming x is the output, . we will do this by calling x (0 ), x (0 ), . . . the pre-initial conditions and . x (0+ ), x (0+ ), . . . the post-initial condition. Important: Hereafter when we just say initial conditions we will mean the pre-initial conditions. In cases where x (t) is smooth the pre and post-initial conditions are the same and their is no need to distinguish between them.

2. Simple Examples
Example 1. Consider the initial value problem x = u ( t ),

x (0 ) = 0.

This is a simple calculus problem and has solution x (t) = 0 t for t < 0 for t > 0.
x(t) t

Initial Conditions

OCW 18.03SC

It is easy to see that x (0+ ) = 0, so the post-initial condition is the same as the pre-initial condition. This should not surprise us. Although the rate of input jumps from 0 to 1, it is still only inputting an innitesimal amount at a time. So, the response x (t) should be continuous. But, note . . that x (0 ) = 0 = x (0+ ) = 1. Example 2. Consider the initial value problem x = ( t ),

x (0 ) = 0.

We know how to integrate (t) to get x (t) = u(t).


1 x = u(t) t

Here the pre-initial condition x (0 ) = 0 does not match the post-initial condition x (0+ ) = 1. The impulse causes a jump in the value of x. Example 3. Consider a second order IVP

..

x = u ( t ),

x (0 ) = 0, x (0 ) = 0. 0 t2 /2 for t < 0 for t > 0.


1 x(t) t

Integrating twice we get x (t) =

Again, its easy to check that x (0 ) = x (0+ ) and x (0 ) = x (0+ ). That .. is, the pre and post initial conditions are the same. (But, x (0 ) = 0 = .. + x (0 ) = 1.) Example 4. Consider the initial value problem

..

x = ( t ),

x (0 ) = 0, x (0 ) = 0.

Integrating once gives x (t) = u(t). Integrating a second time gives x (t) = 0 t for t < 0 for t > 0.
x(t) t

Checking the pre and post initial conditions gives x (0 ) = 0 = x (0+ ) . . x (0 ) = 0 = x (0+ ) = 1 2

Initial Conditions

OCW 18.03SC

In other words, x (t) itself is continuous, but for the second order equation the input (t) caused a jump in the rst derivative. If we continued these examples wed nd that for an nth -order equation an input of (t) causes a jump in the derivative of order n 1.

3. Rest Initial Conditions


The case where x (t) = 0 for t < 0 is called rest initial conditions. If we have a DE of order n this translates into pre-initial conditions x (0 ) = 0, x (0 ) = 0, . . . , x (n1) (0 ) = 0.

4. Conclusion
A unit step input u(t) causes a smooth response with matching pre and post-initial conditions. For a unit impulse input (t) the pre and post initial conditions match except for the derivative one less than the order of the equation.

First order Unit Step Response 1. Unit Step Response


Consider the initial value problem x + kx = ru(t),

x (0 ) = 0,

k, r constants.

This would model, for example, the amount of uranium in a nuclear reactor where we add uranium at the constant rate of r kg/year starting at time t = 0 and where k is the decay rate of the uranium. As in the previous note, adding an innitesimal amount (r dt) at a time leads to a continuous response. We have x (t) = 0 for t < 0; and for t > 0 we must solve . x + kx = r, x (0) = 0. The general solution is x (t) = (r /k) + cekt . To nd c, we use x (0) = 0: 0 = x (0) = r r +c c = . k k r (1 ekt )u(t). k

Thus, in both cases and u-format 0 for t < 0 x (t) = r kt ) ( 1 e for t > 0 k

(1)

With r = 1, this is the unit step response, sometimes written v(t). To be more precise, we could write v(t) = u(t)(1/k)(1 ekt ). The claim that we get a continuous response is true, but may feel a bit unjustied. Lets redo the above example very carefully without making this assumption. Naturally, we will get the same answer. The equation is x + kx =

0 r

for t < 0 for t > 0,

x (0 ) = 0.

(2)

Solving the two pieces we get c1 ekt x (t) = r kt k + c2 e

for t < 0 for t > 0.

This gives x (0 ) = c1 and x (0+ ) = r /k + c2 . If these two are different there is a jump at t = 0 of magnitude x (0+ ) x (0 ) = r / k + c 2 c 1 .

First order Unit Step Response

OCW 18.03SC

The initial condition x (0 ) = 0 implies c1 = 0, so our solution looks like 0 for t < 0 x (t) = r kt + c e for t > 0. 2 k To nd c2 we substitute this into our differential equation (2). (We must use the generalized derivative if there is a jump at t = 0.) After substitution the left side of (2) becomes . 0 for t < 0 x + kx = (r /k + c2 )(t) + kc2 ekt + r + kc2 ekt for t > 0 0 for t < 0 = (r / k + c2 ) ( t ) + r for t > 0. Comparing this with the right side of (2) we see that r /k + c2 = 0, or c2 = r /k. This gives exactly the same solution (1) we had before. Figure 1 shows the graph of the unit step response (r = 1). Notice that it starts at 0 and goes asymptotically up to 1/k.
1/k v (t) t

Figure 1. Unit step is the response of the system x + kx = f (t) when f (t) = u(t). The Meaning of the Phrase Unit Step Response In this note looked at the system with equation x + kx = f (t) and we considered f (t) to be the input. As we have noted previously, it sometimes makes more sense to consider something else to be the input. For example, in Newtons law of cooling T + kT = kTe it makes physical sense to call Te , the temperature of the environment, the input. In this case the unit step response of the system means the response to the input Te (t) = u(t), i.e. the solution to T + kT = ku(t).

Unit Step Response: Post-initial Conditions


Quiz: Consider the equation v + kv = u(t) with rest initial conditions, v(0 ) = 0. For the solution v(t) what is v(0+ )? Choices: a) v(0+ ) = 0 b) v(0+ ) = 1/k c) v(0+ ) = 1 d) v(0+ ) = k e) None of these.

. . . .

Answer: (c) v(t) is continuous so v(0 ) = v(0+ ) = v(0) = 0 Therefore the DE shows . v(0+ ) = u(0+ ) = 1.

Unit Step Response: Post-initial Conditions


Quiz: Consider the equation v + kv = u(t) with rest initial conditions, v(0 ) = 0. For the solution v(t) what is v(0+ )? Choices: a) v(0+ ) = 0 b) v(0+ ) = 1/k c) v(0+ ) = 1 d) v(0+ ) = k e) None of these. Pick what you think is the correct choice and then look at the answer.

. . . .

Unit Step Response: Post-initial Conditions


Quiz: Consider the equation v + kv = u(t) with rest initial conditions, v(0 ) = 0. For the solution v(t) what is v(0+ )? Think about your answer and then look at the choices.

First order Unit Impulse Response 1. Unit Impulse Response


Consider the initial value problem x + kx = (t),

x (0 ) = 0,

k, r constants.

This would model, for example, the amount of uranium in a nuclear reactor where at time t = 0 we add 1 kilogram of uranium all at once and k is the decay rate of the uranium. Because of the rest initial conditions we have x (t) = 0 for t < 0. The effect of the input is to cause the amount x (t) to jump from 0 to 1 at t = 0. That is, x (0+ ) = 1. For t > 0 the input (t) = 0 and, therefore, for t > 0 we should solve . x + kx = 0, x (0) = 1. The general solution is x (t) = cekt . To nd c, we use x (0) = 1, which gives c = 1. Thus, in both cases and u-format 0 for t < 0 x (t) = = ekt u(t). (1) ekt for t > 0 This is called the unit impulse response, which we denote w(t). In some sense it is the simplest nontrivial solution; you just give the system a unit kick at t = 0, stand back, and watch the result. For t > 0 it is just the homogeneous solution with initial condition x (0) = 1.

2. Graph of the Unit Impulse Response w(t)


Figure 1 shows the graph of the unit impulse response. Notice that at t = 0 it jumps to x = 1 and then decays exponentially to 0.
1 w(t) t

Figure 1. The unit impulse response of the system x + kx.

3.

(t) as a limit of box functions

Originally we found (t) as a limit of box functions of area 1. In this section we will compute the unit impulse response as the limit of the responses

First order Unit Impulse Response

OCW 18.03SC

to these box functions. The main two points in doing this are: rst, to gain more comfort and facility with this circle of ideas and second, to convince you that the delta function is much nicer to work with than box functions. We invite you to compare the amount of work required for solving the unit impulse with the amount of work needed in the unit step case. for t < 0 0 A quick review: Dene the box function as uh (t) = 1/h for 0 < t < h 0 for h < t. It has total area 1 for all h > 0 and the graph of uh (t) becomes a spike as h 0, i.e. lim uh (t) = (t).
h 0

For the equation

x + kx = uh (t),

x (0 ) = 0

the three pieces of the solution are easily found to be kt for t < 0 c1 e 1 kt x (t) = hk + c2 e for 0 < t < h kt c3 e for h < t. Using the initial condition x (0 ) = 0 and matching the value of x at the endpoints of each piece we nd c1 = 0, c2 = 1/hk, c3 = (ekh 1)/hk. This gives the solution for t < 0 0 1 kt x (t) = hk (1 e ) for 0 < t < h 1 kh kt for h < t. kh ( e 1) e Letting h 0 this becomes (since lim x (t) = 0 ekt ekh 1 = 1) hk h 0 for t < 0 for 0 < t.

This limit is exactly the unit impulse response w(t) we found in a previous note. Figure 2 shows this graphically by plotting the input and output for several values of h. 2

First order Unit Impulse Response

OCW 18.03SC

3 2 1 1/1 t 1 1/2 t 1 1/3 t 1 t

Figure 2. Responses for h = 1, h = .5, h = .333, and h 0. The input is plotted in black and the output in red. Notice how the output rises faster and gets closer to 1 as h 0. Finally, in the limit of small h, it jumps directly to 1. The Meaning of the Phrase Unit Impulse Response Exactly as in the case of the unit step response, the unit impulse response means the response of the system when the input is a unit impulse. In this note we looked at the the system x + kx = f (t) and we considered f (t) to be the input. Suppose, instead, we have the system . T + kT = kTe , where we consider Te to be the input. Then the unit impulse response is the response of the system to input Te (t) = (t), i.e. the solution to T + kT = k(t).

Unit Impulse Response: Post-initial Conditions


Quiz: Consider the equation w + kw = (t) with rest initial conditions, w(0 ) = 0. For the solution w(t) what is w(0+ )? Choices: a) w(0+ ) = 0 b) w(0+ ) = 1/k c) w(0+ ) = 1 d) w(0+ ) = k e) None of these.

. . . .

Answer: (d). . Using the DE we get w(0+ ) + kw(0+ ) = (0+ ). We know w(0+ ) = 1 and . (0+ ) = 0. Therefore w(0+ ) = k. We could also look at the solution w(t) = ekt for t > 0. Thus w(t) = . kekt for t > 0. This implies w(0+ ) = k. Using the solution to the DE probably seems easier than the rst method, but it is important to be able to draw conclusions without knowing the solution.

Unit Impulse Response: Post-initial Conditions


Quiz: Consider the equation w + kw = (t) with rest initial conditions, w(0 ) = 0. For the solution w(t) what is w(0+ )? Choices: a) w(0+ ) = 0 b) w(0+ ) = 1/k c) w(0+ ) = 1 d) w(0+ ) = k e) None of these. Pick what you think is the correct choice and then look at the answer.

. . . .

Unit Impulse Response: Post-initial Conditions


Quiz: Consider the equation w + kw = (t) with rest initial conditions, w(0 ) = 0. For the solution w(t) what is w(0+ )? Think about your answer and then look at the choices.

Second order Unit Step Response 1. Unit Step Response


We will use the example of an undamped harmonic oscillator with input f (t) modeled by .. mx + kx = f (t). The unit step response is the solution to this equation with input u(t) and rest initial conditions x (t) = 0 for t < 0. That is, it is the solution to the initial value problem (IVP) mx + kx = u(t),

..

x (0 ) = 0, x (0 ) = 0.

This could be an undamped spring-mass system with mass m and spring constant k. The mass is at rest at equilibrium until time t = 0 when a steady force starts to act on it. Force represents a change in momentum over time. A nite force F (t) can only cause an ininitesimal change in momentum (i.e. F (t) dt) at a time. Therefore, the mass does not change position abruptly, nor does it change velocity instantaneously. Because of this we should expect a solution which is continuous with continuous derivative. Only the acceleration experiences a discontinuity. For t < 0 we already know that x (t) = 0. For t > 0 the DE is mx + kx = 1. This has a constant particular solution x (t) = 1/k, and a general homogeneous solution xh (t) = c1 cos(n t) + c2 sin(n t), where n = k/m. Putting the two together gives the general solution x (t) = 1/k + c1 cos(n t) + c2 sin(n t) for t > 0.

..

The continuity of x and x implies x (0) = x (0 ) = 0 and x (0) = x (0 ) = 0. This allows us to nd c1 and c2 . 0 = x (0) = 1/k + c1 . 0 = x (0) = c2 n

c1 = 1/k c2 = 0.

Second order Unit Step Response

OCW 18.03SC

The unit step response for this system is (in both cases and u-format) 0 for t < 0 1 x (t) = 1 = (1 cos(n t))u(t). k for t > 0. k (1 cos( n t )) As in the rst order case, we will sometimes denote this v(t). The claim that we get a continuous response is true, but may feel a bit unjustied. Lets redo the above example very carefully without making this assumption. It will take more work, but we will get the same answer. In cases format the equation for the IVP is .. . 0 for t < 0 mx + kx = x (0 ) = 0, x (0 ) = 0. 1 for t > 0, Solving the two pieces we get c1 cos(n t) + c2 sin(n t) x (t) = 1/k + c3 cos(n t) + c4 sin(n t)

(1)

for t < 0 for t > 0.

The pre-initial conditions x (0 ) = x (0 ) = 0 easily imply c1 = c2 = 0. So our solution looks like 0 for t < 0 x (t) = 1/k + c3 cos(n t) + c4 sin(n t) for t > 0. To nd c3 and c4 we substitute x (t) into equation (1).

To measure the jumps we compute x (0+ ) = 1/k + c3 and x (0+ ) = c4 n . We use this as we compute derivatives of x. 0 for t < 0 . x (t) = (1/k + c3 )(t) + c3 n sin( n t) + c4 n cos(n t) for t > 0.

..

x (t) = (1/k + c3 ) (t) + c4 n (t) +

0 2 cos( t ) c 2 sin( t ) c3 n n n 4 n

for t < 0 for t > 0.

Since the right-hand side of equation (1) does not have any delta functions .. or any (t) the coefcients in front of these terms in the formula for x must be 0: 1/k + c3 = 0 c4 n = 0

c3 = 1/k c4 = 0.
2

Second order Unit Step Response

OCW 18.03SC

In the end, we have exactly the same solution as above for the unit step response. To summarize: the continuity assumptions follow because any jumps . in x (t) or x (t) would result in delta functions when x is substituted into equation (1). The generalized derivative (t) is not something weve seen before. It is often called a doublet. There is an entire theory of these and other generalized functions, but we will only use (t) in this course. Figure 1 shows the graph of the unit step response (with k = 1 and m = 0.5.
2/k

Figure 1. The unit step response for the system mx + kx = u(t). If we added some damping the homogeneous part of the solution would go to 0 and the unit step response would go asymptotically to 1/k. The Meaning of the Phrase Unit Step Response As we noted in the rst order case, the unit step response is the response of the system to a unit step input. For example, if our system is mx + bx + kx = by and we consider y to be the input, then the unit step response is the solution to .. . . .. . mx + bx + kx = bu(t) equivalently mx + bx + kx = b(t).

..

..

Second order Unit Impulse Response 1. Effect of a Unit Impulse on a Second order System
We consider a second order system mx + bx + kx = f (t).

..

(1)

Our rst task is to derive the following. If the input f (t) is an impulse c(t a), then the systems response to f (t) has the following properties. . 1. The momentum mx (t) jumps by c units at t = a. That is, mx ( a+ ) mx ( a ) = c. 2. The position x (t) is unchanged at t = a. That is, x ( a + ) = x ( a ). Recall the argument that we used before: If x (t) had a jump at a then .. x (t) would contain a multiple of (t a). So, mx (t) would contain a multiple of the doublet (t a). This is impossible since the input (t a) does not contain a doublet. This shows point (2) above.

To show point (1), we note that if mx (t) has a jump of c units at t a .. then mx (t) contains the term c(t a). This is needed to make the left-hand side of equation (1) match the right hand side when f (t) = c(t a). Another way to show points (1) and (2) is a physical argument. A force acting on the mass over time changes its momentum. In fact, the best way to state Newtons second law is that dp = f ( t ), dt where p(t) is the momentum of a system and f (t) is an external force acting on the system. If a force f (t) acts over the time interval [t1 , t2 ] the total change of momentum due to the force is
t2
t1

f (t) dt.

Physicists call this the impulse of the force f (t) over the interval [t1 , t2 ]. If a very large force is applied over a very short time interval and has total impulse of 1 the result will be a sudden unit jump in the momentum of the system.

Second order Unit Impulse Response

OCW 18.03SC

For a second order system the unit impulse function can be thought of as an idealization of this force. It is a force with total impulse 1 applied all at once. A third argument that we will skip would be to solve equation (1) with a box function for input and take the limit as the box gets narrower and taller always with area 1.

2. Unit Impulse Response


We consider once again the damped harmonic oscillator equation mx + bx + kx = f (t). The unit impulse response is the solution to this equation with input f (t) = (t) and rest initial conditions: x (t) = 0 for t < 0. That is, it is the solution to the initial value problem (IVP) mx + bx + kx = (t),

..

..

x (0 ) = 0, x (0 ) = 0.

This could be an damped spring-mass system with mass m, damping constant b and spring constant k. The mass is at rest at equilibrium until time t = 0 when it is hit by a sudden very brief very intense force, rather like getting hit on the head by a hammer. The effect is to increase the momentum instantaneously, without changing the position of the mass. Let w(t) denote the solution we seek. The rest initial conditions tell us that w(t) = 0 for t < 0. We know from section 1 that the effect of the input is to cause a unit jump in the momentum at t = 0 and no change in position. We also know that, for t > 0, the input (t) = 0. Putting this together, for t > 0 the w(t) satises the equation mw + bw + kw = 0,

..

w(0) = 1/m, w(0) = 0.

This is a homogeneous constant coefcient linear differential equation which we have lots of practice in solving. Example 1. Find the unit impulse response for the system 2x + 8x + 26x = f (t).

..

(2)

Solution. We will use the standard notation w(t) for the unit impulse response. We are looking for the response from rest to f (t) = (t). We know

Second order Unit Impulse Response

OCW 18.03SC

w(t) = 0 for t < 0. At t = 0 the input causes a unit jump in momentum, . i.e., 2w(0+ ) = 1. So, for t > 0 we have to solve 2w + 8w + 26w = 0,

..

w(0+ ) = 1/2, w(0+ ) = 0.

The roots of the characteristic polynomial are 2 3i. Which implies w(t) = c1 e2t cos(3t) + c2 e2t sin(3t), The initial conditions give 0 = w (0+ ) = c 1 , . 1/2 = w(0+ ) = 2c1 + 3c2 c2 = 1/6. Thus, the unit impulse response (in both cases and u-format) is 1 2t 0 for t < 0 w(t) = = e sin(3t)u(t). 1 2t sin(3t) for t > 0 6 6e for t > 0.

(3)

Figure 1 the graph of the unit impulse response. Notice that at t = 0 the . graph has a corner. This corresponds to the slope w jumping from 0 to 1/2. For t > 0 the graph decays to 0 while oscillating.
t

Figure 1. The unit impulse response for the system 2x + 8x + 26 x.

..

3. Checking Example 1 by Substitution


With any differential equation you can verify a solution by plugging it into the equation. We will do that with example 1 to gain some more insight into why we get the solution. First, we take the derivatives of the solution in equation (3) for t > 0
2t ( 2 sin(3t ) + 3 cos(3t )) for t > 0 w(t) = 1 6e .. 1 2t w(t) = 6 e (5 sin(3t) 12 cos(3t)) for t > 0

Next we look at the jumps at t = 0 w(0 ) = 0, . w(0 ) = 0, 3 w (0+ ) = 0 . w(0+ ) = 1/2

Second order Unit Impulse Response

OCW 18.03SC

Now we can compute the full generalized derivatives


2t ( 2 sin(3t ) + 3 cos(3t )) u ( t ) w(t) = 1 6e .. 1 2t ( 5 sin(3t ) 12 cos(3t )) u ( t ) w(t) = 2 (t) + 1 6e

Finally we substitute w for x in equation (2) 2w ( t ) 8w ( t ) 26w(t)

.. .

= (t) = =

5 2t sin(3t) 4e2t cos(3t) 3 e 8 2t sin(3t) + 4e2t cos(3t) 3 e


13 2t 3 e

sin(3t)

2w + 8w + 26w = (t). The Meaning of the Phrase Unit Impulse Response As weve noted several times already, the response to a given input depends on what we in our equation we consider to be the input. For example, if our system is .. . . mx + bx + kx = by and we consider y to be the input, then the unit impulse response is the solution to mx + bx + kx = b(t)

..

..

equivalently

mx + bx + kx = b(t).

..

(Here, is what weve called a doublet.)

Unit Impulse Response: Post-initial Conditions


Quiz: Let w(t) be the solution to mx + kx = (t) with rest initial condi. tions. What is w(0+ )? Choices: a) w(0+ ) = 0 b) w(0+ ) = m c) w(0+ ) = k d) w(0+ ) = k/m e) w(0+ ) = 1/m f) None of these.

..

. . . . .

Answer: (e). The unit impulse input causes a unit jump in momentum. Starting from . . rest this means mw(0+ ) = 1 or w(0+ ) = 1/m.

Unit Impulse Response: Post-initial Conditions


Quiz: Let w(t) be the solution to mx + kx = (t) with rest initial condi. tions. What is w(0+ )? Choices: a) w(0+ ) = 0 b) w(0+ ) = m c) w(0+ ) = k d) w(0+ ) = k/m e) w(0+ ) = 1/m f) None of these. Pick what you think is the correct choice and then look at the answer.

..

. . . . .

Unit Impulse Response: Post-initial Conditions


Quiz: Let w(t) be the solution to mx + kx = (t) with rest initial condi. tions. What is w(0+ )? Think about your answer and then look at the choices.

..

Higher Order Unit Impulse Response


We can extend our reasoning in the rst and second order cases to any order. Consider an nth order system with DE a n x ( n ) + a n 1 x ( n 1) + . . . + a 1 x + a 0 x = f ( t ) , (1)

where we take f (t) to be the input. The equation for the unit impulse response of this system is a n x ( n ) + a n 1 x ( n 1) + . . . + a 1 x + a 0 x = ( t ), with rest IC. (2)

The effect of the function input is to cause a jump in the n 1st derivative at time t = 0, while the lower order derivatives do not jump. That is, the system is put in the state x (0+ ) = 0, x (0+ ) = 0, . . . , x (n2) (0+ ) = 0, x (n1) (0+ ) = 1/ an . To show this we use the same reasoning as in the second order case. Suppose there was a jump in a lower derivative. For example, suppose x (n3) (0+ ) = b = 0. Then the expression for x (n2) (t) contains b(t), which implies that x (n1) (t) contains b (t) and x (n) (t) contains b (t). This is impossible because the right-hand side of (2) does not have any derivatives of the delta function. an x (n) ( t ) Since x n1 (t) has a jump of x (n1) (0+ ) = 1/ an at t = 0, its derivative has a unit impulse, (t), at t = 0.

We conclude that the solution to (2) is 0 for t < 0 and for t > 0 it is exactly the same as the solution to a n x ( n ) + a n 1 x ( n 1) + . . . + a 1 x + a 0 x = 0 with initial conditions x (0) = 0, x (0) = 0, . . . , x n2 (0) = 0, x n1 (0) = 1/ an .

Convolution: Introduction
The convolution product of two functions is a peculiar looking integral which produces another function. It is found in a wide range of applications, so it has a special name and a special symbol. The convolution of f and g is denoted f g and dened by

( f g)(t) =

t+
0

f (s) g(t s) ds.

We will start by stating this formula without any motivation. Its main properties are relatively easy to deduce from its denition. The motivation will come in the form of Greens formula. This important tool tells us how to solve a linear time invariant (LTI) system with any input (and rest IC) once we know its unit impulse response. The rest of the session is concerned with the proof of the Greens formula and examples of convolution and Greens formula. Technical Detail: Because we want convolution to work with delta functions we needed to be careful with the limits of integration. This explains the plus and minus on the limits.. If both functions are continuous or have at most jump discontinuities then the limits can be 0 and t.

Denition and Properties 1. Denition


The convolution of two functions f and g is a third function which we denote f g. It is dened as the following integral

( f g)(t) =

t+
0

f ( ) g(t ) d

for t > 0.

(1)

We will leave this unmotivated until the next note, and for now just learn how to work with it. There are a few things to point out about the formula. The variable of integration is . We cant use t because that is already used in the limits and in the integrand. We can choose any symbol we want for the variable of integration it is just a dummy variable. The limits of integration are 0 and t+ . This is important, particularly when we work with delta functions. If f and g are continuous or have at worst jump discontinuities then we can use 0 and t for the limits. You will often see convolution written like this: f g(t) =
t
0

f ( ) g(t ) d .

We are considering one-sided convolution. There is also a two-sided convolution where the limits of integration are . (Important.) One-sided convolution is only concerned with functions on the interval (0 , ). When using convolution we never look at t < 0.

2. Examples
Example 1 below calculates two useful convolutions from the denition (1). As you can see, the form of f g is not very predictable from the form of f and g. Example 1. Show that e at ebt = e at ebt , ab a = b; e at e at = t e at

Denition and Properties

OCW 18.03SC

Solution. We show the rst; the second calculation is similar. If a = b, e e =


at bt

t
0

e e

a b(t )

d = e

bt

t
0

( ab)

d = e

bt

e( ab) ab

= ebt
0

e( ab)t 1 e at ebt = . ab ab

Note that because the functions are continuous we could safely integrate just from 0 to t instead of having to specify precisely 0 to t+ . The convolution gives us a formula for a particular solution y p to an inhomogeneous linear ODE. The next example illustrates this for a rst order equation. Example 2. Express as a convolution the solution to the rst order constantcoefcient linear IVP. y + ky = q(t);

y(0) = 0.

(2)

Solution. The integrating factor is ekt ; multiplying both sides by it gives

(y ekt ) = q(t)ekt .
Integrate both sides from 0 to t, and apply the Fundamental Theorem of Calculus to the left side; since we have y(0) = 0, the solution we seek satises y p ekt =
t
0

q( )ek d ;

( is the dummy variable of integration.)

Moving the ekt to the right side and placing it under the integral sign gives yp =
t
0

q ( ) ek(t ) d

y p = q(t) ekt . Now we observe that the solution is the convolution of the input q(t) . with ekt , which is the solution to the corresponding homogeneous DE y + ky =, but with IC y(0) = 1. This is the simplest case of Greens formula, which is the analogous result for higher order linear ODEs, as we will see shortly.

Denition and Properties

OCW 18.03SC

3. Properties
1. Linearity: Convolution is linear. That is, for functions f 1 , f 2 , g and constants c1 , c2 we have

( c1 f 1 + c2 f 2 ) g = c1 ( f 1 g ) + c2 ( f 2 g ).
This follows from the exact same property for integration. This might also be called the distributive law. 2. Commutivity: f g = g f . Proof: This follows from the change of variable v = t . Limits: = 0 t = t+ and = t+ t = 0 t+ t+ Integral: ( f g)(t) = 0 f ( ) g(t ) d = 0 f (t v) g(v) dv = ( g f )(t) 3. Associativity: f ( g h) = ( f g) h. The proof just amounts to changing the order of integration in a double integral (left as an exercise).

4. Delta Functions
We have

( f )(t) = f (t)

and

((t a) f )(t) = f (t a).

(3)

The notation for the second equation is ugly, but its meaning is clear. We prove these formulas by direct computation. First, remember the rules of integration with delta functions: for b > 0
b
0

( ) f ( ) d = f (0).

The formulas follow easily for t 0

( f )(t) = ((t a) f )(t)

( ) f ( t ) d = f ( t 0) = f ( t ) t+ = 0 ( a ) f ( t ) d = f ( t a ) .
0

t+

5. Convolution is a Type of Multiplication


You should think of convolution as a type of multiplication of functions. In fact, it is often referred to as the convolution product. In fact, it has the properties we associate with multiplication: It is commutative. 3

Denition and Properties It is associative. It is distributive over addition.

OCW 18.03SC

It has a multiplicative identity. For ordinary multiplication, 1 is the multiplicative identity. For convolution, formula (3) shows that (t) is the multiplicative identity for the convolution product.

Greens Formula
In this note we state Greens formula and look at some examples. We will prove it in the next note.

1. Greens Formula
Suppose that we have a linear time invariant system with rest IC. P ( D ) y = f ( t ), y(t) = 0 for t < 0 (1)

As in previous sessions, we will consider f (t) to be the input to this system. Everything we say will also hold for systems like . .. . . T + kT = kTe with input Te and mx + bx + kx = by with input y. In this context, where we dont consider functions for t < 0, the initial conditions mean that y(t) and all its derivatives are 0 at t = 0 . P( D ) is a polynomial differential operator. Although it can be of any order, recall that we developed the second order case extensively in the last unit, where it was often written as P( D )y = my + by + ky. Suppose further that w(t) is the unit impulse response for (1). That is, w(t) satises P( D )w = (t), with rest IC. Then, for any input f (t) the solution to equation (1) is given by Greens formula y(t) = ( f w)(t) =
t+
0

..

f ( )w(t ) d .

(2)

This is a wonderful formula! It tells us the response to any input once we know the unit impulse response. Furthermore, it gives us that response as an integral which can be computed numerically if necessary. For many physical systems the impulse response can be measured directly or deduced from measurements. So, Greens formula gives us a method for predicting the systems response to any input.

2. Unit Impulse Response = Weight Function


The unit impulse response is also called the weight function. We will use the terms interchangeably. If we think of an integral as a sum then Greens formula shows the solution y(t) to (1) is given as a weighted sum

Greens Formula

OCW 18.03SC

of the small bits of input, f ( ) d from before time t. Each piece is weighted by w(t ). Before proceeding, let us recall the denition of the unit impulse response. The weight w(t) is the unique solution to the IVP P( D )y = (t) with rest IC (3)

In the previous session we learned how to rewrite (3) as a homegeneous equation. We will only restate this for second order equations. The weight function for the system mx + bx + kx = f (t) is 0 for t < 0 and the solution to mx + bx + kx = 0, for t > 0.

..

..

x (0) = 0, x (0) = 1/m

3. Examples
We now out Greens formula (2) in a couple of cases where we can check it by nding the particular solution y p by another method. Example 1. Find the particular solution given by (2) to

..

y + y = A,

y(0) = 0, y(0) = 0,

where A is a constant.

Solution. The unit impulse response is w(t) = sin t. Therefore for t 0, we have t t y p (t) = A sin(t ) d = A cos(t ) = A(1 cos t).
0 0

We check this by another method: The exponential response formula or the method of undetermined coefcients produces the particular solution y p = A. Adding in the homogeneous solution we get the general solution to the DE is y = A + c1 cos t + c2 sin t. You can easily compute that the rest initial conditions are matched by y = A A cos t, as found by Greens formula.

Greens Formula

OCW 18.03SC

Example 2. Find the particular solution for t 0 given by (2) to 1 for 0 t y + y = f (t) = 0 elsewhere Solution. Here the method of Example 1 leads to two cases: 0 t and t : t t t 0 sin(t ) d = cos(t ) = 1 cos t, for 0 t ; 0 yp = f ( ) sin(t ) d = 0 sin ( t ) d = cos ( t ) = 2 cos t, for t . 0
0

We leave it to ther reader to check this by our earlier methods.

Proof of Greens Formula


Greens Formula: For the equation P ( D ) y = f ( t ), the solution for t > 0 is given by y(t) = ( f w)(t) =
t+
0

y(t) = 0 for t < 0

(1)

f ( )w(t ) d ,

(2)

where w(t) is the weight function (unit impulse response) for the system. Proof: The proof of Greens formula is surpisingly direct. We will use the linear time invariance of the system combined with superposition and the denition of the integral as a limit of Riemann sums. To avoid worrying about 0 and t+ we will assume that f (t) is continuous. With appropriate care, the proof will work for an f (t) that has jump discontinuities or contains delta functions. As we saw in the session on Linear Operators in the last unit, linear time invariance means that y(t) solves P( D )y = f (t) y(t a) solves P( D )y = f (t a). (3)

Or, in the language of input-response, if y(t) is the response to input f (t) then y(t a) is the response to input f (t a). First we will partition time into intervals of width t. So, t0 = 0, t1 = t, t2 = 2t, etc.
t t ... 0 = t0 t1 t2 t ... tk tk+1

Figure 1: Division of the t-axis into small intervals. Next we decompose the input signal f (t) into packets over each interval. The kth signal packet, f k (t) coincides with f (t) between tk and tk+1 and is 0 elsewhere f (t) for tk < t < tk+1 f k (t) = 0 elsewhere.

Proof of Greens Formula

OCW 18.03SC

f (t)

fk (t)

t t ... 0 = t0 t1 t2

t ... tk tk+1

t t t ... 0 = t0 t1 t2

t ... tk tk+1

Figure 2: The signal packet f k (t). It is clear that for t > 0 we have f (t) is the sum of the packets f (t) = f 0 (t) + f 1 (t) + . . . + f k (t) + . . . A single packet f k (t) is concentrated entirely in a small neighborhood of tk so it is approximately an impulse with the same size as the area under f k (t). The area under f k (t) f (tk ) t. Hence, f k ( t ) ( f ( t k ) t ) ( t t k ). The weight function w(t) is response to (t). So, by linear time invariance the response to f k (t) is y k ( t ) ( f ( t k ) t ) w ( t t k ). We want to nd the response at a xed time. Since t is already in use, we will let T be our xed time and nd y( T ). Since f is the sum of f k , superposition gives y is the sum of yk . That is, at time T y ( T ) = y0 ( T ) + y1 ( T ) + . . . f ( t0 ) w ( T t0 ) + f ( t1 ) w ( T t1 ) + . . . t

(4)

We can ignore all the terms where tk > T . (Because then w( T tk ) = 0, since T tk < 0.) If n is the last index where tk < T we have y ( T ) f ( t0 ) w ( T t0 ) + f ( t1 ) w ( T t1 ) + . . . + f ( t n ) w ( T t n ) t 2

Proof of Greens Formula This is a Riemann sum and as t 0 it goes to an integral y( T ) =


T
0

OCW 18.03SC

f (t)w( T t) dt

Except for the change in notation this is Greens formula (2). Note on Causality: Causality is the principle that the future does not affect the past. Greens theorem shows that the system (1) is causal. That is, y(t) only depends on the input up to time t. Real physical systems are causal. There are non-causal systems. For example, an audio compressor that gathers information after time t before deciding how to compress the signal at time t is non-causal. Another example is the system with input f (t) and . output y(t) where y is the solution to y = f (t + 1).

Examples
We will give several examples of Greens formula. The rst we will build from scratch so you get a sense of how this formula arises naturally. The last example shows how Greens formula works for a system driven at its resonant frequency. Example 1. The build up of a pollutant in a lake Every good formula deserves a particularly illuminating example, and perhaps the following will serve for the convolution integral. It is also illustrated by the Mathlet Convolution: Accumulation. Problem: We have a lake, and a pollutant is being dumped into it, at a certain variable rate f (t). This pollutant degrades exponentially over time. If the lake begins at time zero with no pollutant, how much is in the lake at time t > 0? Solution. Let x (t) be the amount of pollutant in the lake at time t and a be the decay constant. For exponential decay we know that if a quantity p of pollutant is dropped in the lake at time k then at a later time t it will have been reduced to the amount pea( k ) . (1)

Here t k is the time elapsed between when the pollutant is added and when we check how much of it is left. In our system pollutant is not being added all at once. Rather, it is dripping continuously into the lake. We break the interval [0, t] into n small pieces of width as shown.
... 0 = 0 1 2 ... k k+1

n = t

Let pk be the amount of pollutant added in the interval [k , k+1 ]. Since is small we get the approximation pk f (k ) . (Remember f ( ) is a rate; to get a quantity you must multiple by time.) According to equation (1) the amount of this left at time t is approximately pk ea(tk ) f (k ) ea(tk ) .

Examples

OCW 18.03SC

This is approximately the contribution to x (t) from the interval [k , k+1 ]. To determine the x (t) we simply sum up the contributions of all the intervals. x (t) p0 ea(t0 ) + p1 ea(t1 ) + . . . + pn1 ea(tn1 ) a ( t ) a ( t ) a ( t ) 0 + f ( )e 1 + . . . + f ( n 1 f (0 )e . 1 n 1 ) e This is a Riemann sum. Taking the limit as 0 we get the convolution integral x (t) =
t
0

f ( ) e a(t ) d .

(2)

Example 2. In example 1 we constructed our formula by slicing an interval into pieces. You should know how to do this. But, we prove theorems and formulas to avoid always going back to rst principles. In this example we will solve the problem in example 1 using the differential for exponential decay and nding its weight function. (Of course, this DE was found by slicing an interval into pieces . . . .) The DE with rest IC is x + ax = f (t),

x (0 ) = 0

Its weight function w(t) is 0 for t < 0, and for t > 0 it is the solution to the IVP . w + aw = 0, w(0) = 1. We get w(t) = eat for t > 0. Using Greens formula we again get the convolution integral (2) Example 3. Resonance Use Greens formula to solve the DE with rest inital conditions 2x + 8x = cos(2t),

..

x (0 ) = 0, x (0 ) = 0

For t > 0, the weight function is the solution to 2w + 8w = 0, The solution is w(t) =

..

w(0) = 0, w(0) = 1/2.

1 sin(2t). For t > 0 Greens formula gives 4


t
0

x (t) =

1 sin(2 ) cos(2(t )) d . 4 2

Examples

OCW 18.03SC

This is an easy integral, we sketch the algebra to compute it. It uses the sin( A + B) + sin( A B) trigonometric identity: sin( A) cos( B) = . 2 t x (t) = 0 sin(2 ) 1 4 cos(2( t )) d . t = 1 8 0 (sin(2t ) + sin(4 2t )) d t cos(4 2t) 1 = 8 sin(2t) 4

t sin(2t) . 8

This is the answer we expected from our earlier work with the exponential response formula.

Laplace Transform Basics: Introduction


An operator takes a function as input and outputs another function. A transform does the same thing with the added twist that the output function has a different independent variable. The Laplace transform takes a function f (t) and produces a function F (s). We will allow the variable s to be complex. We will see that it can be thought of as complex circular frequency It is best to think of f (t) and F (s) as two views of the same underlying phenomenon. If we have a signal then f (t) is the familiar view of that signal in time and F (s) is the less familiar view in frequency. Everything about the signal is present in both views, but some things are easier to see in one view or the other. Using them together gives us a powerful tool for understanding systems and signals. In practice the Laplace transform has the following benets: It makes explicit the long-term behavior of f (t). It converts differential equations into algebraic equations. It allows you to nd the operator P( D ) if you know its weight function w(t). It converts Greens formula, which is a complicated convolution integral in the time view, into as simple algebraic statement in the frequency view. Most importantly, the frequency view can be summarized in something called the pole diagram. This diagram can show at a glance the stability and frequency response of a system. It is an important engineering design tool. We will see and explore each of these virtues in this and the next few sessions.

Denition of Laplace Transform 1. Denition of Laplace Transform


The Laplace transform of a function f (t) of a real variable t is another function depending on a new variable s, which is in general complex. We will denote the Laplace transform of f by L f . It is dened by the integral

(L f )(s) =

f (t)est dt,

(1)

for all values of s for which the integral converges. There are a few things to note. L f is only dened for those values of s for which the improper integral on the right-hand side of (1) converges. We will allow s to be complex. As with convolution the use of 0 , in the denition (1) is necessary to accomodate generalized functions containing (t). Many textbooks do not do this carefully, and hence their denition of the Laplace transform is not consistent with the properties they assert. In those cases where 0 isnt needed we will use the less precise form

(L f )(s) =

f (t)est dt.

(1)

Also, as with convolution, the limits of integration mean that the Laplace transform is only concerned with functions on (0 , ).

2. Notation, F (s)
We will adopt the following conventions: 1. Writing (L f )(s) can be cumbersome so we will often use an uppercase letter to indicate the Laplace transform of the corresponding lowercase function: (L f )(s) = F (s), (L g)(s) = G (s), etc. For example, in the formula

L( f ) = sF (s) f (0 )
it is understood that we mean F (s) = L( f ).

Denition of Laplace Transform

OCW 18.03SC

2. If our function doesnt have a name we will use the formula instead. For example, the Laplace transform of the function t2 is written L(t2 )(s) or more simply L(t2 ). 3. If in some context we need to modify f (t), e.g. by applying a translation by a number a, we can write L( f (t a)) for the Laplace transform of this translation of f . 4. Youve already seen several different ways to use parentheses. Sometimes we will even drop them altogether. So, if f (t) = t2 then the following all mean the same thing

(L f )(s) = F (s) = L f (s) = L( f (t))(s) = L(t2 )(s);

L f = F = L ( t2 ).

3. First Examples
For the rst few examples we will explicitly use a limit for the improper integral. Soon we will do this implicitly without comment. Example 1. Let f (t) = 1, nd F (s) = L f (s). Solution. Using the denition (1) we have

L (1) = F ( s ) =

st

est dt = lim T s

T
0

esT 1 = lim T s

T .
0

The limit depends on whether s is positive or negative. 0 if s > 0 sT lim e = T if s < 0. Therefore,
1 s

L (1) = F ( s ) =

diverges

if s > 0 if s 0.

(We didnt actually compute the case s = 0, but it is easy to see it diverges.) Example 2. Compute L(e at ). Solution. Using the denition (1) we have

L(e ) =

at

e e

at st

e( as)t dt = lim T a s

T
0

e( as) T 1 = lim T as

T .
0

Denition of Laplace Transform

OCW 18.03SC

The limit depends on whether s > a or s < a. 0 if s > a ( as) T lim e = T if s < a. Therefore,
1 s a

L(e at ) =

diverges

if s > a if s a.

(We didnt actually compute the case s = a, but it is easy to see it diverges.) We have the rst two entries in our table of Laplace transforms: f (t) = 1 f (t) = e at

F (s) = 1/s,

s>0

F (s) = 1/(s a), s > a.

4. Linearity
You will not be surprised to learn that the Laplace transform is linear. For functions f , g and constants c1 , c2

L ( c1 f + c2 g ) = c1 L ( f ) + c2 L ( g )
This is clear from the denition (1) of L and the linearity of integration.

Domain of F (s) 1. Complex s and region of convergence


We will allow s to be complex, using as needed the properties of the complex exponential we learned in unit 1. Example 1. In the previous note we saw that L(1) = 1/s, valid for all s > 0. Lets recompute L(1)(s) for complex s. Let s = + i . L(1)(s) = 0 est dt T est = limT s 0 T et (cos( t)+i sin( t)) = limT s
0

This converges if > and diverges if < 0. Since = Re(s) we have

L(1) = 1/s,

for Re(s) > 0.

The region Re(s) > 0 is called the region of convergence of the transform. It is a right half-plane.
Imag. axis Re(s) > 0 Real axis

Region of convergence: right half-plane Re(s) > 0. Frequency: The Laplace transform variable s can be thought of as complex frequency. It will take us a while to understand this, but we can begin here. Eulers formula says ei t = cos( t) + i sin( t) and we call the angular frequency. By analogy for any complex number exponent we call s the complex frequency in est . If s = a + i then s is the complex frequency and its imaginary part is an actual frequency of a sinusoidal oscillation.

2. Piecewise continuous functions and functions of exponential order


If the integral fails to converge for any s then the function does not have a Laplace transform.

Domain of F (s)
2

OCW 18.03SC

Example. It is easy to see that f (t) = et has no Laplace transform. The problem is the et grows to fast as t gets large. Fortunately, all of the functions we are interested in do have Laplace transforms valid for Re(s) > a for some value a. Functions of Exponential Order The class of functions that do have Laplace transforms are those of exponential order. Fortunately for us, all the functions we use in 18.03 are of this type. A function is said to be of exponential order if there are numbers a and M such that | f (t)| < Me at . In this case, we say that f has exponential order a. Examples. 1, cos( t), sin( t), tn all have exponential order 0. e at has exponential order a. A function f (t) is piecewise continuous if it is continuous everywhere except at a nite number of points in any nite interval and if at thesepoints it has a jump discontinuity (i.e. a jump of nite height). Example. The square wave is piecewise continuous. Theorem: If f (t) is piecewise continuous and of exponential order a then the Laplace transform L f (s) converges for all s with Re(s) > a. Proof: Suppose Re(s) > a and | f (t)| < Me at . Then we can write s = ( a + ) + ib, where > 0. Then, since |eibt | = 1,
2

| f (t)est | = | f (t)e(a+)t eibt | = | f (t)e(a+)t) | < Met ,


Since Met dt converges for > 0, the Laplace transform integral also 0 converges. Domain of F (s): For f (t) we have F (s) = 1/s with region of convergence Re(s) > 0. But, the function 1/s is well dened for all s = 0. The process of extending the domain of F (s) from the region of convergence is called analytic continuation. In this class analytic continuation will always consist of extending F (s) to the complex plane minus the zeros of the denominator.

More Entries for the Laplace Table


In this note we will add some new entries to the table of Laplace transforms. s 1. L(cos( t)) = 2 , with region of convergence Re(s) > 0. s + 2 2. L(sin( t)) = 2 , with region of convergence Re(s) > 0. s + 2 Proof: We already know that L(e at ) = 1/(s a). Using this and Eulers formula for the complex exponential, we obtain

L(cos( t) + i sin( t)) = L(ei t ) =

1 1 s + i s + i = = 2 . s i s i s + i s + 2

Taking the real and imaginary parts gives us the formulas. L(cos( t)) = Re L(ei t ) = s/(s2 + 2 ) L(sin( t)) = Im L(ei t ) = /(s2 + 2 ) The region of convergence follow from the fact that cos( t) and sin( t) both have exponential order 0. Another approach would have been to use integration by parts to compute the transforms directly from the Laplace integral. 3. For a positive integer n, L(tn ) = n!/sn+1 . The region of convergence is Re(s) > 0. Proof: We start with n = 1.

0

L(t) =
Using integration by parts: u=t du = 1 dv = est v = est /(s)

test dt

test L(t) = s

+
0

1 s

est dt.

2 For Re(s) > 0 the rst term is 0 and the second term is 1 s L (1) = 1/ s . Thus, L(t) = 1/s2 .

Next lets do n = 2:

L ( t2 ) =

t2 est dt

More Entries for the Laplace Table

OCW 18.03SC

Again using integration by parts: u = t2 du = 2t dv = est v = est /(s) t2 est L(t ) = s


2

1 + s

2test dt.

3 For Re(s) > 0 the rst term is 0 and the second term is 1 s L (2t ) = 2/ s . 2 3 Thus, L(t ) = 2/s .

We can see the pattern: there is a reduction formula for

L(tn ) =
Integration by parts: u = tn du = ntn1 dv = est v = est /(s)

tn est dt.

L(tn ) =

tn est s

+
0

1 s

ntn1 est dt.

n 1 ). For Re(s) > 0 the rst term is 0 and the second term is 1 s L ( nt n n n 1 Thus, L(t ) = s L(t ).

Thus we have

L ( t3 ) L ( t4 )
...

= =
n! . s n +1

3 2 s L(t ) 4 3 s L(t )

= =

32 3! =s 4 s4 43! 4! = s5 s5

L(tn ) =

4. (s-shift formula) If z is any complex number and f (t) is any function then L(ezt f (t)) = F (s z). As usual we write F (s) = L( f )(s). If the region of convergence for L( f ) is Re(s) > a then the region of convergence for L(ezt f (t)) is Re(s) > Re(z) + a. Proof: We simply calculate

L(ezt f (t))

= =

0 0

ezt f (t)est dt f (t)e(sz)t dt

= F ( s z ).

More Entries for the Laplace Table Example. Find the Laplace transform of et cos(3t).

OCW 18.03SC

Solution. We could do this by using Eulers formula to write et cos(3t) = (1/2) e(1+3i)t + e(13i)t but its even easier to use the s-shift formula with z = 1, which gives

L(et f (t)) = F (s + 1),


where here f (t) = cos(3t), so that F (s) = s/(s2 + 9). Shifting s by -1 according to the s-shift formula gives

L(et cos(3t)) = F (s + 1) =

s+1 . ( s + 1)2 + 9

We record two important cases of the s-shift formula: sz 4a) L(ezt cos( t)) = ( s z )2 + 2 4b) L(ezt sin( t)) = . ( s z )2 + 2 Consistency. It is always useful to check for consistency among our various formulas: 1. We have L(1) = 1/s, so the s-shift formula gives L(ezt 1) = 1/(s z). This matches our formula for L(ezt ). 2. We have L(tn ) = n!/sn+1 . If n = 1 we have L(t0 ) = 0!/s = 1/s. This matches our formula for L(1).

Computing the Laplace Transform


Quiz: What is L((1 + t)2 )? Choices: a. (1/s + 1/s2 )2 = 1/s2 + 2/s3 + 1/s4 . b. 1/s + 2/s2 + 2/s3 . c. (1 + t)(1/s + 1/s2 ). d. None of the above. 1 2 2! + 2 + 3. s s s

Answer: (b): L((1 + t)2 ) = L(1 + 2t + t2 ) =

Computing the Laplace Transform


Quiz: What is L((1 + t)2 )? Choices: a. (1/s + 1/s2 )2 = 1/s2 + 2/s3 + 1/s4 . b. 1/s + 2/s2 + 2/s3 . c. (1 + t)(1/s + 1/s2 ). d. None of the above. Pick what you think is the correct choice and then look at the answer.

Computing the Laplace Transform


Quiz: What is L((1 + t)2 )? Think about your answer and then look at the choices.

The Laplace Transform of the Delta Function


Since the Laplace transform is given by an integral, it should be easy to compute it for the delta function. The answer is 1. L((t)) = 1. 2. L((t a)) = eas for a > 0. As expected, proving these formulas is straightforward as long as we use the precise form of the Laplace integral. For (1) we have:

L((t)) =

(t)est dt = 1.

As we saw in a previous session, integrating est against (t) amounts to evaluating est at t = 0, and e0 = 1. Similarly for the shifted version (2), integrating est against (t a) amounts to evaluating est at t = a:

L((t a)) =

(t a)est dt = esa .

Notice that the two formulas are consistent: if we set a = 0 in formula (2) then we recover formula (1).

Partial Fractions and Inverse Laplace Transform


In order to use the Laplace transform we need to be able to invert it and nd f (t) when were given F (s). Often this can be done by using the Laplace transform table. So for example, if F (s) = 1/(s 5) then f (t) = e5t . More often we have to do some algebra to get F (s) into a form suitable for the direct use of the table. Our main technique for doing this is the partial fractions decomposition. You probably saw this before in calculus as a method for computing integrals. First we will learn how to do partial fractions in a straightforward algebraic way using the method of undetermined coefcients. Next we will learn the Heaviside coverup method which makes some of the algebra easier.

Laplace Inverse by Table Lookup


The rst thing we need to be able to do is to use the Laplace table to nd the inverse Laplace transform. We will illustrate this entirely by examples. Notation: The inverse Laplace transform will be denoted L1 . Example 1. Find L1 (1/(s 2)). Solution. Use the table entry L(e at ) = 1/(s a):

L1 (1/(s 2)) = e2t .


Example 2. Find L1 (1/(s2 + 9)). Solution. Use the table entry L(sin( t)) = /(s2 + 2 ) and linearity: 1 1 1 3 1 1 L = L = sin(3t). 2 2 2 s +9 3 s +3 3 Example 3. Find L1 (4/s2 ). Solution. Use the table entry L(t) = 1/s2 : L1 (4/s2 ) = 4t. Example 4. Find L1 (4/(s 2)2 ). Solution. Use the s-shift formula L(ezt f (t)) = F (s 2), where, in this case, F (s) = 4/s2 f (t) = 4t by example (3). Therefore, L1 (4/(s 2)2 ) = L1 ( F (s 2)) = e2t f (t) = e2t 4t. Example 5. Find L
1

1 . s2 + 4s + 13

Solution. We rst need to complete the square s2 + 4s + 13 = s2 + 4s + 4 + 9 = (s + 2)2 + 9. We have a shifted function F (s + 2), where F (s) = 1/(s2 + 9). Using example (2), we know that f (t) = sin(3t)/3, so using the s-shift rule we get 1 sin(3t) 1 L = L1 ( F (s + 2)) = e2t . s2 + 4s + 13 3

Laplace Inverse by Table Lookup s . ( s2 + 2 )2

OCW 18.03SC

Example 6. Find L

Solution. We havent seen this formula yet, but there is a table entry, which t gives: sin( t). 2 Example 7. Find L1
1 ( s2 + 2 )2

. 1 (sin( t) t cos( t)). 2 3

Solution. This is also a table entry, answer:

Partial Fractions: Undetermined Coefcients 1. Introduction


Not every F (s) we encounter is in the Laplace table. Partial fractions is a method for re-writing F (s) in a form suitable for the use of the table. In this note we will run through the various cases encountered when we apply the method of partial fractions decomposition to a rational function. In the next note we will learn the Heaviside cover-up method, which simplies some of the algebra. Note: We use the term undetermined coefcients in the same way it was use when solving an ODE with polynomial input. It involves setting a polynomial with unknown coefcients equal to a known polynomial and solving for the unknown coefcients by equating them with the known ones.

2. Rational Functions
A rational function is one that is the ratio of two polynomials. For example s+1 s2 + 7s + 9 and s2 + 7s + 9 s+1 are both rational functions. A rational function is called proper if the degree of the numerator is strictly smaller than the degree of the denominator; in the examples above, the rst is proper while the second is not. Long-division: Using long-division we can always write an improper rational function as a polynomial plus a proper rational function. The partial fraction decomposition only applies to proper functions. Example 1. Use long-division to write nomial and a proper rational function. Solution. s2 s1 s3 + s3 + s2 s2 s2 s3 + 2s + 1 as a the sum of a polys2 + s 2

+s2

2s 2s +4s s 5s

+1 +1 +2 1

Partial Fractions: Undetermined Coefcients

OCW 18.03SC

Therefore,

s3 + 2s + 1 5s 1 = s1+ 2 . s2 + s 2 s +s2

3. Linear Factors
Here we assume the denominator factors in distinct linear factors. We start with a simple example. We will explain the general principle immediately afterwords. s3 Example 2. Decompose R(s) = using partial fractions. Use (s 2)(s 1) this to nd L1 ( R(s)). Solution. s3 A B = + . (s 2)(s 1) s2 s1 Multiplying both sides by the denominator on the left gives s 3 = A ( s 1) + B ( s 2) (1)

The sure algebraic way is to expand out the right hand side and equate the coefcients with those of the polynomial on the left. coeff. of s: 1 = A+B s 3 = ( A + B)s + ( A 2 B) coeff. of 1: 3 = A 2 B We solve this system of equations to nd the undetermined coefcients A and B: A = 1, B = 2. Answer: R(s) = 1/(s 2) + 2/(s 1). Table lookup then gives L1 ( R(s)) = e2t + 2et . Note: In this example it would have been easier to plug the roots of each factor into equation (1). When you do this every term except one becomes 0. Plug in s = 1 2 = B(1) B = 2 Plug in s = 2 1 = A(1) A = 1. In general, if P(s)/ Q(s) is a proper rational function and Q(s) factors into distinct linear factors Q(s) = (s a1 )(s a2 ) (s an ) then P(s) A1 A2 An = + ++ . Q(s) s a1 s a2 s an The proof of this is not hard, but we will not give it. Remember you must have a proper rational function and each of the factors must be distinct. Repeated factors are discussed below. 2

Partial Fractions: Undetermined Coefcients

OCW 18.03SC 3 . s3 3s2 s + 3

Example 3. Use partial fractions to nd L

Solution. The hardest part of this problem is to factor the denominator. For higher order polynomials this might be impossible. In this case you can check s3 3s2 s + 3 = (s 1)(s + 1)(s 3). The partial fractions decomposition is 3 A B C = + + . (s 1)(s + 1)(s 3) s1 s+1 s3 Multiplying through by the denominator gives 3 = A(s + 1)(s 3) + B(s 1)(s 3) + C (s 1)(s + 1). Plugging in s = 1 gives A = 3/4, likewise s = 1 gives B = 3/2 and s = 3 gives C = 3/4. Our answer is 3 2 3 3 1 L = Aet + Bet + Ce3t = et + et e3t . 3 2 s 3s s + 3 4 2 4

4. Quadratic Factors
If the denominator has quadratic factors then, the numerator in the partial fraction decomposition will be a linear term instead of a constant. s1 1 Example 4. Find L . (s + 1)(s2 + 4) Solution. This is a proper rational function so s1 A Bs + C = + 2 . (s + 1)(s2 + 4) s+1 s +4 (2)

Notice the quadratic factor gets a linear term in the numerator. Notice also that the number of unknown coefcients is the same as the degree of the denominator in the original fraction. From (2) we can write s1 C 1 L = Aet + B cos(2t) + sin(2t). 2 (s + 1)(s + 4) 2

Partial Fractions: Undetermined Coefcients

OCW 18.03SC

All thats left is to do some algebra to nd the coefcients Muliplying (2) through by the denominator gives s 1 = A(s2 + 4) + ( Bs + C )(s + 1) = ( A + B)s2 + ( B + C )s + (4 A + C ). Equate the coefcients on both sides: s2 : s: s2 : 0 = A+B 1 = B+C 1 = 4 A + C

Solving, we get A = 2/5, B = 2/5, C = 3/5. Example 5. Dont be fooled by quadratic terms that factor into linear ones. 1 1 A B C = = + + . 2 (s + 1)(s 4) (s + 1)(s + 2)(s 2) s+1 s+2 s2 Dont forget that the rational function must be proper. For s3 + 2s + 1 example, decompose 2 using partial fractions. s +s2 Solution. First, we must use long-division to make this proper. From example (1) we have Example 6. s3 + 2s + 1 5s 1 5s 1 A B = s1+ 2 = s1+ = s1+ + . 2 s +s2 s +s2 (s + 2)(s 1) s+2 s1 Solving for the undetermined coefcients gives A = 11/3, B = 4/3.

5. Repeated Linear Factors


For repeated linear factors we need one partial fraction term for each power of the factor as illustrated by the following example. 2s 1 Example 7. Find L using partial fractions. s3 ( s + 1)2 ( s + 2) Solution. 2s A B C D E F = + 2+ 3+ + + s3 ( s + 1)2 ( s + 2) s s s s + 1 ( s + 1)2 s+2 Here the denominator has a linear factor s repeated three times (term s3 ), and a linear factor (s + 1) repeated twice (term (s + 1)2 ); hence three partial 4

Partial Fractions: Undetermined Coefcients

OCW 18.03SC

fractions are associated with the rst, while two are associated with the latter. The term (s+2) which is not repeated leads to one partial fraction as previously seen. You can check that the coefcients are A = 5/2, B = 1, C = 0, D = 2, E = 2, F = 1/2. Using the s-shift rule we have L1 (1/(s + 1)2 ) = tet . Thus, 2s C 1 L = A + Bt + t2 + Det + Etet + Fe2t 3 2 s ( s + 1) ( s + 2) 2 5 1 = + t + 2et + 2tet + e2t . 2 2

6. Repeated Quadratic Factors


Just like repeated linear factors, quadratic factors have one term for each power of the factor as illustrated in the following example. 2s Example 8. Find L1 using partial fractions. s ( s2 + 1)2 ( s2 + 4s + 2) Solution. The partial fractions decomposition is 2s A Bs + C Ds + E Fs + G = + 2 + 2 + s ( s2 + 1)2 ( s2 + 4s + 6) s s +1 ( s + 1)2 s2 + 4s + 6 Note the repeated factor (s2 + 1)2 lead to two partial fraction terms. We wont compute the coefcients you can do this by going through the algebra. Instead, well focus on nding the Laplace inverse. Our table contains the terms with repeated quadratic factors. Ds t 1 L = D sin(t) 2 2 ( s + 1) 2 E 1 L 1 = E (sin(t) t cos(t)). ( s2 + 1)2 2 For the terms with denominator s2 + s + 2 we need to complete the square. Notice that we make sure to also shift the s-term in the numerator. Fs + G 1 1 F ( s + 2 ) + G 2 F L = L s2 + 4s + 6 ( s + 2)2 + 2 1 = Fe2t cos( 2 t) + ( G 2 F )e2t sin( 2 t). 2 5

Partial Fractions: Undetermined Coefcients

OCW 18.03SC

Putting this together, the answer is 2s L 1 = s ( s2 + 1)2 ( s2 + 4s + 6)

A + B cos(t) + C sin(t) t 1 + D sin(t) + E (sin(t) t cos(t)) 2 2 1 2t + Fe cos( 2 t) + ( G 2 F )e2t sin( 2 t). 2

7. Complex Factors
We can allow complex roots. In this case all quadratic terms factor into linear terms. Example 9. Decompose s/(s2 + 2 ) using complex partial fractions and use it to show L1 (s/(s2 + 2 )) = cos( t). Solution. s s A B = = + . s2 + 2 (s i )(s + i ) s i s + i Multiplying through by the denominator gives s = A(s + i ) + B(s i ). Plug in s = i A = 1/2. Plug in s = i B = 1/2. From the table: s A B 1 1 1 L = L + = Aei + Bei = (ei + ei ) = cos( t). 2 2 s + s i s + i 2

Heaviside Cover-up Method 1. Introduction


The cover-up method was introduced by Oliver Heaviside as a fast way to do a decomposition into partial fractions. This is an essential step in using the Laplace transform to solve differential equations, and this was more or less Heavisides original motivation. The cover-up method can be used to make a partial fractions decomp(s) position of a proper rational function whenever the denominator can be q(s) factored into distinct linear factors.

2. Linear Factors
We rst show how the method works on a simple example, and then show why it works. s7 Example 1. Decompose into partial fractions. (s 1)(s + 2) Solution. We know the answer will have the form s7 A B = + . (s 1)(s + 2) s1 s+2 (1)

To determine A by the cover-up method, on the left-hand side we mentally remove (or cover up with a nger) the factor s 1 associated with A, and substitute s = 1 into whats left; this gives A: s7 17 = = 2 = A. (2) ( s + 2 ) s =1 1+2 Similarly, B is found by covering up the factor s + 2 on the left, and substituting s = 2 into whats left. This gives s7 2 7 = = 3 = B. ( s 1) 2 1 s = 2 Thus, our answer is s7 2 3 . = + (s 1)(s + 2) s1 s+2 (3)

Heaviside Cover-up Method

OCW 18.03SC

Why does the method work? The reason is simple. The right way to determine A from equation (1) would be to multiply both sides by (s 1); this would give s7 B = A+ ( s 1). ( s + 2) s+2 (4)

Now if we substitute s = 1, what we get is exactly equation (2), since the term on the right disappears. The cover-up method therefore is just an easy and efcient way of doing the calculations. In general, if the denominator of the proper rational function factors into the product of distinct linear factors: p(s) A1 Ar = + ...+ , (s a1 )(s a2 ) (s ar ) s a1 s ar ai = a j ,

then Ai is found by covering up the factor s ai on the left, and setting s = ai in the rest of the expression. 1 into partial fractions. s3 s Solution. Factoring, s3 s = s(s2 1) = s(s 1)(s + 1). By the cover-up method, 1 1/2 1/2 1 = + + . s(s 1)(s + 1) s s1 s+1 Example 2. Decompose To be honest, the real difculty in all of the partial fractions methods (the cover-up method being no exception) is in factoring the denominator. Even the programs which do symbolic integration, like Macsyma, or Maple, can only factor polynomials whose factors have integer coefcients, or easy coefcients like 2. and therefore they can only integrate rational functions with easily-factored denominators.

3. Quadratic Factors
Heavisides cover-up method can be used even when the denominator doesnt factor into distinct linear factors. This only gives partial results, but these can often be a big help, as the following example illustrates. 5s + 6 . Example 3. Decompose 2 (s + 4)(s 2) Solution. We write 5s + 6 As + B C = 2 + . (s2 + 4)(s 2) s +4 s2 2 (5)

Heaviside Cover-up Method

OCW 18.03SC

We rst determine C by the cover-up method, getting C = 2 . Then A and B can be found by the method of undetermined coefcients; the work is greatly reduced since we need to solve only two simultaneous equations to nd A and B, not three. Following this plan, using C = 2, we combine terms on the right of (5) so that both sides have the same denominator. The numerators must then also be equal, which gives us 5s + 6 = ( As + B)(s 2) + 2(s2 + 4). (6) Comparing the coefcients of s2 and of the constant terms on both sides of (6) gives the two equations 0 = A+2 and 6 = 2 B + 8,

from which A = 2 and B = 1 . In using (6), one could have instead compared the coefcients of s, getting 5 = 2 A + B, leading to the same result, but providing a valuable check on the correctness of the computed values for A and B. In Example 3, an alternative to undetermined coefcients would be to substitute two numerical values for s into the original equation (5), say s = 0 and s = 1 (any values other than s = 2 are usable). Again one gets two simultaneous equations for A and B. This method requires addition of fractions, and is usually better when only one coefcient remains to be determined (as in Example 4 below). Still another method would be to factor the denominator completely into linear factors, using complex coefcients, and then use the cover-up method, but with complex numbers. At the end, conjugate complex terms have to be combined in pairs to produce real summands, and the calculations can sometimes be longer.

4. Repeated Linear Factors


The cover-up method can also be used if a linear factor is repeated, but there too it gives just partial results. It applies only to the highest power of the linear factor. 1 Example 4. Decompose . ( s 1)2 ( s + 2) Solution. We write 1 A B C = + + . ( s 1)2 ( s + 2) ( s 1)2 s 1 s + 2 3 (7)

Heaviside Cover-up Method

OCW 18.03SC

To nd A cover up (s 1)2 and set s = 1; you get A = 1/3. To nd C, cover up s + 2, and set s = 2; you get C = 1/9. This leaves B which cannot be found by the cover-up method. But since A and C are already known in (7), B can be found by substituting any numerical value (other than 1 or 2) for s in (7). For instance, if we put s = 0 and remember that A = 1/3 and C = 1/9, we get 1 1/3 B 1/9 = + + , 2 1 1 2 giving B = 1/9. B could also be found by applying the method of undetermined coefcients to the (7); note that since A and C are known, it is enough to get a single linear equation in order to determine B simultaneous equations are no longer needed. The fact that the cover-up method works for just the highest power of the repeated linear factor can be seen just as before. In the above example for instance, the cover-up method for nding A is just a short way of multiplying (5) through by (s 1)2 and then substituting s = 1 into the resulting equation.

Table Entries: Repeated Quadratic Factors 1. Repeated Quadratic Factors


We will add three entries to our Laplace Table. 1 1 L (sin( t) t cos( t)) = 3 2 2 ( s + 2 )2 t s L sin( t) = 2 ( s2 + 2 )2 1 s2 L (sin( t) + t cos( t)) = 2 2 ( s + 2 )2

(1) (2) (3)

There are several ways to prove these formulas. We will give one using partial fractions by factoring the denominators on the frequency side into complex linear factors. Proof of 1. First some algebra: 1 A B C D + + + 2 2 (s a) s a (s + a) s+a

(s

a )2 ( s

a )2

Cover-up gives us A and C. Undetermined coefcients then gives B and D: 1 1 = C, D = 3 = B 2 4a 4a This gives the inverse Laplace transform A =

L 1 (

(s

a )2 ( s

a )2

)=

1 1 (te at + teat ) 3 (e at eat ). 2 4a 4a ei t ei t = 2i sin( t)

(4)

We will use this on the right hand side of (1), but rst recall ei t + ei t = 2 cos( t) and (5)

Let a = i , then (4) and (5) combine to prove formula (1). 1 1 1 1 L = L ( s2 + 2 )2 ( s i )2 ( s + i )2 1 1 (tei t + tei t ) + ( e i t e i t ). 2 4 4 3 i 1 1 = 2 (tei t + tei t ) + ( ei t e i t ) 4 4 3 i 1 1 = 2 t cos( t) + sin( t). 2 2 3

Table Entries: Repeated Quadratic Factors

OCW 18.03SC

The proofs of (2) and (3) are similar, and we will omit them.

2. Note on the Relation to Resonance:


Each of the formulas (1), (2), and (3) has a term with a factor of t. This is exactly what we saw with the response x in the resonance equation

..

x + 2 x = cos( t),

which has solution x (t) = t sin( t)/(2 ). Notice that L1 (1/s2 ) = t and the s-shift rule shows L1 (1/(s a)2 ) = te at . So repeated factors on the frequency side always lead to multiplication by t on the time side. If the repeated factor has a higher power then we get multiplication by a higher power of t.

Laplace Transform: Solving IVPs: Introduction


In this session we apply the Laplace transform techniques we have learned to solving intitial value problems for LTI DEs p( D ) x = f (t). For signals f (t) which are discontinuous or impulsive, using the Laplace transform is often the most efcient solution method. We start by deriving the simple relations between the Laplace transform of a derivative of a function and the Laplace transform of the function itself. Our goal is to use these formulas to solve IVPs of the form p( D ) x = f (t) (with initial conditions).

We do this by Laplace transforming both sides of the DE and solving for the function X (s) = L( x (t)). It turns out that the resulting equation for X (s) is a simple algebraic equation which can be solved immediately. Then one recovers the solution x (t) by computing the inverse Laplace transform x (t) = L1 ( X (s)).

Table Entries: Derivative Rules 1. t-derivative rule


This is a course on differential equations. We should try to compute

L( f ). (We use the notation f instead of f simply because we think the dot does not sit nicely over the tall letter f .)
As usual, let L( f )(s) = F (s). Let f be the generalized derivative of f . (Recall, this means jumps in f produce delta functions in f .) The tderivative rule is

L( f ) = sF (s) f (0 ) L( f ) = s2 F (s) s f (0 ) f (0 ) L( f
(n)

(1) (2) f (0 ) + . . . + f
( n 1)

) = s F (s) s

n 1

f (0 ) s

n 2

(0 ). (3)

Proof: Rule (1) is a simple consequence of the denition of Laplace transform and integration by parts.

L( f ) = =

f (t)est dt
st

u = est u = sest

0

v = f (t) v = f (t)

f (t)e

+s
0

f (t)est dt

= f (0 ) + sF (s).
The last equality follows from: 1. We assume f (t) has exponential order, so if Re(s) is large enough f (t)est is 0 at t = . 2. The integral in the second term is none other than the Laplace transform of f (t). Rule (2) follows by applying rule (1) twice.

L( f ) = sL( f ) f (0 ) = s(L( f ) f (0 )) f (0 ) = sF (s) s f (0 ) f (0 ).


Rule (3) Follows by applying rule (1) n times. Notes: 1. We will call the terms f (0 ), f (0 ) the annoying terms. We will be happiest when our signal f (t) has rest initial conditions, so all of

Table Entries: Derivative Rules

OCW 18.03SC

the annoying terms are 0. 2. A good way to think of the t-derivative rules is

L( f ) = F (s) L( f ) = sF (s) + annoying terms at 0 . L( f ) = s2 F (s) + annoying terms at 0 .


Roughly speaking, Laplace transforms differentiation in t to multiplication by s. 3. The proof of rule (1) uses integration by parts. This is clearly valid if f (t) is continuous at t = 0. It is also true (although we wont show this) if f (t) is a generalized function. See example 2 below. Example 1. Let f (t) = e at . We can compute L( f ) directly and by using rule (1). Directly: f (t) = ae at L( f ) = a/(s a). Rule (1): L( f ) = F (s) = 1/(s a) L( f ) = sF (s) f (0 ) = s/(s a ) 1 = a / ( s a ). Both methods give the same answer. Example 2. Let u(t) be the unit step function, so u(t) = (t). . Directly: L(u) = L() = 1. . Rule (1): L(u) = sL(u) u(0 ) = s(1/s) 0 = 1. Both methods give the same answer. Example 3. Let f (t) = t2 + 2t + 1. Compute L( f ) two ways. Solution. Directly: f (t) = 2 L( f ) = 2/s. Using rule (3): L( f ) = s2 F (s) s f (0 ) f (0 ) = s2 (2/s3 + 2/s2 + 1/s) s 1 2 = 2/s. Both methods give the same answer.

2.

s-derivative rule

There is a certain symmetry in our formulas. If derivatives in time lead to multiplication by s then multiplication by t should lead to derivatives in s. This is true, but, as usual, there are small differences in the details of the formulas. The s-derivative rule is

L(t f )(s) = F (s) L(tn f )(s) = (1)n F (n) (s)

(4) (5) (6)

Table Entries: Derivative Rules

OCW 18.03SC

Proof: Rule (4) is a simple consequence of the denition of Laplace transform. F (s) = L( f ) =

0

f (t)est dt

F (s) = =

d ds

f (t)est dt

t f (t)est

= L(t f (t)).
Rule (5) is just rule (4) applied n times. Example 4. Use the s-derivative rule to nd L(t). Solution. Start with f (t) = 1, then F (s) = 1/s. The s-derivative rule now says L(t) = F (s) = 1/s2 which we know to be the answer. Example 5. Use the s-derivative rule to nd L(te at and L(tn e at ). Solution. Start with f (t) = e at , then F (s) = 1/(s a). The s-derivative rule now says L(te at ) = F (s) = 1/(s a)2 . Continuing: L(t2 e at ) = F (s) = 2/(s a)3 , L(t3 e at ) = F (s) = 3 2/(s a)4 , L(t4 e at ) = F (4) (s) = 4 3 2/(s a)5 , L(tn e at ) = (1)n F (n) (s) = n!/(s a)n+1 . With Laplace, there is often more than one way to compute. We know L(tn ) = n!/sn+1 . Therefore the s-shift rule also gives the above formula for L(tn e at ).

3. Repeated Quadratic Factors


Recall the table entries for repeated quadratic factors 1 1 L (sin( t) t cos( t)) = 2 3 ( s2 + 2 )2 t s L sin( t) = 2 2 ( s + 2 )2 1 s2 L (sin( t) + t cos( t)) = 2 ( s2 + 2 )2

(7) (8) (9)

Previously we proved thesse formulas using partial fractions and factoring the denominators on the frequency side into complex linear factors. Lets prove them again using the s-derivative rule. 3

Table Entries: Derivative Rules

OCW 18.03SC

Proof of (8) using the s-derivative rule. Let f (t) = sin( t). We know F (s) = plies s2 . The s-derivative rule im+ 2

L(t sin t) = F (s) =

( s2

2 s . + 2 )2

This formula is (8) with the factor of 2 moved from one side to the other. The other two formulas can be proved in a similar fashion. We wont give the proofs here.

Precise Denition of Laplace Inverse 1. Domain of L1 ( F ).


We have been a bit vague on one key technical point which we aim to clear up in this note. We start with an example. Example. Let u(t) be the unit step function. Since u(t) = 1 for t > 0 we dont need the 0 in the Laplace limits of integration.

L(u) =

u(t)e

st

dt =

est dt = 1/s.

This is exactly the same as L(1). So, when we look for f (t) = L1 (1/s) is it f (t) = 1 or f (t) = u(t)? The answer is, it doesnt matter. Since we are only concerned with the interval (0 , ), you can choose either one. To be precise, for a function F (s) we allow any function f (t) with the following properties to be called its Laplace inverse L1 ( F ): 1. f (t) is a (possibly) generalized function. 2. f (t) is dened on (0, ), except possibly at a discrete set of points where there are jump discontinuities or f (t) is singular, e.g. has delta functions. 3. f (t) may also have a singular part at t = 0. 4. L( f ) = F. 5. In particular, the Laplace inverse is not dened at t = 0 and has nothing to say about f (0 ), f (0 ), f (0 ), . . .. Indeed, it has nothing to say about f (t) for all t < 0. When nding the inverse Laplace transform these values are either irrelevant or must be determined by other means. The functions in Figure 1 below all all have Laplace transform 1/s. Notice, they are all different for t < 0 and the last two are not dened at 0. Since they agree on t > 0 (and have no delta function at t = 0) they have the same Laplace transform.
f (t) = 1 1 t 1 t u(t) 1 t

Figure 1. Three functions with the same Laplace transform.

Precise Denition of Laplace Inverse

OCW 18.03SC

2. Removable Discontinuities
Were still were not done with our discussion og the possible differences between two functions with the same Laplace transform. Consider the two functions whose graphs are shown in Figure 2.
f (t) 1 a t 1 t f1 (t) = 1

Figure 2. Two functions that differ at one point. They differ only at the point t = a. It is easy to see that they have the same Laplace transform: L( f ) = L( f 1 ) = 1/s. (This is because the integral is an area and the areas under two curves that differ like these are the same.) According to our denition either f (t) and f 1 (t) can be chosen as L1 (1/s). But, here the continuous function f 1 (t) is usually the better choice. For example, if we are nding a physical quantity that varies over time then the continuous function is usually the better model. The discontinuity in f (t) looks physically spurious. Discontinuities like the one in f (t) are called removable discontinuities. That is, by changing the value of f ( a) the function can be made continuous. (Technical denition below). Course Convention. In this course we will follow the physically and mathematically reasonable convention that our signals do not have removable discontinuities. They can, however, have jump discontinuities and contain delta functions, which are idealizations of real physical signals. Technical Denition of Removable Discontinuity. Suppose a function is discontinuous at t = a. If it can be made continuous by changing just the value of at a then we call t = a a removable discontinuity. Graphically: if the curve is a continuous curve with a gap where one point was moved then the point is a removable discontinuity. In symbols: If f ( a ) = f ( a+ ) = b then we can make a new function, continuous at a by redening the value at t = a: f (t) for t = a f 1 (t) = b for t = a. Again, we say the discontinuity at t = a is removable. 2

Laplace: Solving Initial Value Problems 1. Introduction


We now have everything we need to solve IVPs using Laplace transform. We will show how to do this through a series of examples. To be honest we should admit that some IVPs are more easily solved by other techniques. However, we will also see some examples where the Laplace machinery weve developed is a big help.

2. Examples of Solving IVPs . Example 1. Solve x + 3x = et with rest initial conditions (rest IC). . Solution. Rest IC mean that x (t) = 0 for t < 0, so x (0 ), x (0 ), . . . are all 0. As usual, we let X = L( x ).
Using the t-derivative rule we can take the Laplace transform of (both sides) of the DE.

(sX (s) x (0 )) + 3X (s) = 1/(s + 1).


Next we substitute the known value x (0 ) = 0 and solve for X (s)

( s + 3) X ( s ) =

1 1 X (s) = . s+1 (s + 1)(s + 3)

(1)

Finally, we nd x (t) = L1 ( X ) by using cover-up to do the partial fractions decomposition. 1 1/2 1/2 1 1 = x ( t ) = e t e 3t (s + 1)(s + 3) s+1 s+3 2 2 for t > 0.

Notes: 1. The term et /2 is what the exponential response formula would give us. The term e3t /2 is the homogenous part of the solution, needed to match the IC. 2. This technique found x (t) for t > 0. The rest IC tell us x (t) = 0 for t < 0. 3. x (0+ ) = 0: Since the input does not contain (t). There is no jump in x (t) at 0. 4. The factor of (s + 3) in front of X (s) in (1) is none other than the characteristic polynomial of this system.

Laplace: Solving Initial Value Problems Example 2. Solve x + 3x = et , x (0 ) = 4. Solution. Laplace:

OCW 18.03SC

sX ( x ) x (0 ) + 3X (s) = 1/(s + 1) (s + 3) X (s) = 4 + 1/(s + 1). Solve for X (s): X (s) = 4 1 + s + 3 (s + 1)(s + 3) (2)

We can use the partial fractions work from example (1). 1 x ( t ) = 4 e 3t + e t 2 1 t 7 3t = e + e 2 2 Notes: (Same remarks as in example 1.) 1 3t e 2 for t > 0

for t > 0.

Example 3. Find the unit impulse response for the operator D + 3 I . Give your answer in both u and cases format. Solution. The unit impulse response is the solution to

( D + 3 I ) w = w + 3w = ( t ),
Taking the Laplace transform we get

with rest IC.

(sW (s) w(0 )) + 3W (s) = 1 (s + 3)W = 1 W =


Laplace inverse now implies w(t) = e3t for t > 0. Thus, 0 for t < 0 w(t) = = u ( t ) e 3t . 3 t e for t > 0

1 . s+3

Notes: 1. The post-initial condition is w(0+ ) = 1. This came out of the calculation, we didnt have to think about the effect of the input (t) at t = 0. 2. The Laplace transform method did not help us nd w(t) for t < 0. For this we used the rest IC that are part of the denition of the unit impulse function. 3. Since w(0 ) = 0 the output jumps by 1 unit at t = 0. 4. Once again you saw the characteristic polynomial appearing. 2

Laplace: Solving Initial Value Problems

OCW 18.03SC

Example 4. Find the unit impulse response for the system p( D ) x = f , where p( D ) = D2 + 2 D + 2 I and we consider f to be the input Give your answer in both u and cases format. Solution. (We outline the solution.) IVP: w + 2w + 2w = (t), with rest IC. Laplace: s2 W + 2sW + 2W = 1 W = 1/(s2 + 2s + 2). (Here we left out all the annoying terms because they are all 0 due to the rest IC.) Complete the square: s2 + 2s + 2 = (s + 1)2 + 1. Inverse Laplace: (using the s-shift rule) W = 1/((s + 1)2 + 1) w(t) = et L1 (1/(s2 + 1)) = et sin(t) for t > 0. Thus w(t) = 0 et sin(t) for t < 0 for t > 0 u(t)et sin(t).

..

Notes: . 1. The post-initial conditions emerge naturally from the solution and are w(0+ ) = 0, w(0+ ) = 1. . 2. Since w(0 ) = 0 the rst derivative jumps by 1 unit at t = 0. 3. Once again you saw the characteristic polynomial appearing. Example 5. Solve x + 2x = 4t, with initial condition x (0) = 1. Remark. Because the input contains no delta functions it is okay to specify the initial condition at t = 0 instead of t = 0 . There will be no jump in the output, i.e., x (0) = x (0 ) = x (0+ ). Solution. Laplace: sX x (0 ) + 2X = 4/s2 . Algebra and partial fractions: X (s) = s2 ( s 4 1 A B C 1 + = + 2+ + . + 2) s + 2 s s s+2 s+2

Cover-up gives B = 2, C = 1. Undetermined coefcients gives A = 1. Inverse Laplace: x (t) = 1 + 2t + 2e2t , for t > 0. Example 6. Solve x + 4x = cos(2t), with rest IC. Solution. Laplace: (s2 + 4) X (s) = s/(s2 + 4) X (s) = s/(s2 + 4)2 . This is a repeated quadratic factor and it is in our table: x (t) = t sin(2t)/4. Notes: 1. This is a response of pure resonance. 2. We could have turned the logic around and used our previous knowledge of the solution to this equation to give yet another proof for the table entry L(t sin( t)/2 ) = s/(s2 + 2 )2 . 3

..

IVPs and t-translation 1. Introductory Example . Consider the system x + 3x = f (t). In the previous note we found its
unit impulse response: w(t) = 0 e 3t for t < 0 for t > 0

u ( t ) e 3t .

This is the response from rest IC to the input f (t) = (t). What if we shifted the impulse to another time, say, f (t) = (t 5)? Linear time invariance tells us the response will also be shifted. That is, the solution to x + 3 x = ( t 2), is x ( t ) = w ( t 2) = 0 e 3t

with rest IC

(1)

for t < 2 for t > 2

u ( t 2 ) e 3( t 2) .

In words, this is a system of exponential decay. The decay starts as soon as there is an input into the system. Graphs are shown in Figure 1 below.

t 2

Figure 1. Graphs of w(t) and x (t) = w(t 2). We know that L((t a)) = eas . So, we can nd X = L( x ) by taking the Laplace transform of (1).

( s + 3 ) X ( s ) = e 2s X ( s ) =

e 2s = e 2s W ( s ), s+3

where W = Lw. This is an example of the t-translation rule.

2.

t-translation Rule
We give the rule in two forms.

L(u(t a) f (t a)) = eas F (s) L(u(t a) f (t)) = eas L( f (t + a)).

(2) (3)

IVPs and t-translation

OCW 18.03SC

For completeness we include the formulas for

L(u(t a)) = eas /s L((t a)) = eas .

(4) (5)

Remarks: 1. Formula (3) is ungainly. The notation will become clearer in the examples below. 2. Formula (2) is most often used for computing the inverse Laplace transform, i.e., as u(t a) f (t a) = L1 eas F (s) . 3. These formulas parallel the s-shift rule. In that rule, multiplying by an exponential on the time (t) side led to a shift on the frequency (s) side. Here, a shift on the time side leads to multiplication by an exponential on the frequency side. Proof: The proof of (2) is a very simple change of variables on the Laplace integral.

L(u(t a) f (t a)) = = =


0 a 0

u(t a) f (t a)est dt f (t a)est dt f ( ) es( + a) d



0

(u(t a) = 0 for t < a) (change of variables: = t a)

= eas

f ( ) es d

= eas F (s).
Formula (3) follows easily from (2). The easiest way to proceed is by introducing a new function. Let g(t) = f (t + a), so f (t) = g(t a) We get and G (s) = L( g) = L( f (t + a)).

L(u(t a) f (t)) = L(u(t a) g(t a)) = eas G (s) = eas L( f (t + a)).


The second equality follows by applying (2) to g(t). as ke 1 Example. Find L . s2 + k 2 2

IVPs and t-translation k 2 s + k2

OCW 18.03SC

Solution. f (t) = L

= sin(kt).

keas s2 + k 2

= u(t a) f (t a) = u(t a) sin k(t a).


1 3 + . s2 s

Example. L (u(t 3)t) = e3s L(t + 3) = e3s

Example. L(u(t 3) 1) = e3s L(1) = e3s /s. 0 for t < 2 Example. Find L( f ) for f (t) = 2 t for t > 2. Solution. f (t) = u(t 2)t2 F (s) = e2s L((t + 2)2 ) = e2s ( Example. Find L( f ) for f (t) = cos(t) 0 for 0 < t < 2 for t > 2 . 2 4 4 + 2 + ). s3 s s

Solution. f (t) = cos(t)(u(t) u(t 2 )) = u(t) cos(t) u(t 2 ) cos(t). s s F (s) = 2 e2 s L(cos(t + 2 )) = (1 e2 s ) 2 . s +1 s +1 We will look at more involved examples in the next note.

IVPs: Longer Examples


The sh population in a lake is not reproducing fast enough and the population is decaying exponentially with decay rate k. A program is started to stock the lake with sh. Three different scenarios are discussed below. Example 1. A program is started to stock the lake with sh at a constant rate of r units of sh/year. Unfortunately, after 1/2 year the funding is cut and the program ends. Model this situation and solve the resulting DE for the sh population as a function of time. Solution. Let x (t) be the sh population and let A = x (0 ) be the initial population. Exponential decay means the population is modeled by x + kx = f (t),

x (0 ) = A

(1)

where f (t) is the rate sh are being added to the lake. In this case r for 0 < t < 1/2 f (t) = 0 for 1/2 < t. First, write f in u-format: f (t) = r (1 u(t 1/2)). Next, take the Laplace transform and solve for X (s). r r F (s) = L( f )(s) = es/2 . s s r sX x (0 ) + kX = F (s) (s + k) X A = (1 es/2 ) s A r X (s) = + (1 es/2 ). s + k s(s + k) To nd x (t) we temporarily ignore the factor of es/2 and take Laplace inverse of whats left. (using partial fractions). A r r 1 kt 1 L = Ae , L = (1 ekt ). s+k s(s + k) k The t-translation formula says

res/2 s(s + k)

r = u(t 1/2) (1 ek(t1/2) ). k

IVPs: Longer Examples

OCW 18.03SC

Putting it all together we get (in u and cases format). r r x (t) = Aekt + (1 ekt ) u(t 1/2) (1 ek(t1/2) ) k k r Aekt + k (1 ekt ) for 0 < t < 1/2 = r kt kt k ( t 1/2 ) Ae k (e + e ) for 1/2 < t. Example 2. (Periodic on/off) The program is refunded and the have enough money to stock at a constant rate of r for the rst half of each year. Find x (t) in this case. Solution. All thats changed from example 1 is the input function f (t). We write it in cases-format and translate that to u-format so we can take the Laplace transform. r 0 f (t) = r 0 for 0 < t < 1/2 for 1/2 < t < 1 for 0 < t < 3/2 for 3/2 < t < 2

1 3 = r (1 u ( t ) + u ( t 1) u ( t ) + . . . ) 2 2 The computations from here are essentially the same as in the previous example. s/2 + es e3s/2 + . . . ) L( f ) = r s (1 e

X=

A s+k

r (1 es/2 s(s+k) r k

+ es . . . )

1 2

x (t) = Aekt +

(1 ekt ) u(t 1/2)(1 ek(t1/2) ) + . . .

x (t) =

r r kt kt Ae + k k e r kt (e ek(t1/2) ) Aekt k
r r kt Aekt + k k (e ek(t1/2) + . . . + ek(tn) ) r kt Aekt k (e ek(t1/2) + . . . ek(tn1/2) )

for 0 < t < for


1 2

<t<1
1 2

for n < t < n + for n +


1 2

< t < n+1

IVPs: Longer Examples Factoring out ekt gives: r r kt Aekt + k k e (1 ek/2 + ek e3k/2 + . . . + enk ) x (t) = r kt Aekt k e (1 ek/2 + ek . . . ek(n+1/2) )

OCW 18.03SC

for n < t < n + 1/2 for n + 1/2 < t < n + 1.

Note that the constant term r /k is only present during periods of stocking. Example 3. (Impulse train) The answer to the previous example is a little hard to read. We know from experience that impulsive input usually leads to simpler output. In this scenario suppose that once a year r /2 units of sh are dumped all at once into the lake. Find x (t) in this case. Solution. Once again, all thats changed from example 1 is the input function f (t). The IVP is still given by equation (1). r f ( t ) = ( ( t ) + ( t 1) + ( t 2) + ( t 3) + . . . ). 2 This is called an impulse train. Its Laplace transform is easy to nd. r F ( s ) = (1 + e s + e 2s + e 3s + . . . ). 2 One nice thing about delta functions is that they dont introduce any new terms into the partial fractions part of the problem. r sX (s) x (0 ) + kX (s) = (1 + e s + e 2s + e 3s + . . . ). 2 A r X (s) = + (1 + e s + e 2s + e 3s + . . . ). s + k 2( s + k ) Laplace inverse is easy: 1 1 L = ekt s+k Thus, r r r r x (t) = Aekt + ekt + u(t 1)ek(t1) + u(t 2)ek(t2) + u(t 3)ek(t3) + . . . 2 2 2 2 Here are graphs of the solutions to examples 2 and 3 (with A = 0, k = 1, r = 2). Notice how they settle down to periodic behavior.
1 t 1 2 3 4 1 2 3 4 1 t

ens s+k

= u ( t n ) ek(tn) .

Fig. 1. Graphs from example 2 (left) and example 3 (right). 3

Transfer and Weight Functions, Greens Formula


In this session we will introduce the transfer function (often called the system function). For a system p( D ) x = f (t), the transfer function is simply 1/ p(s). It is also the Laplace transform of the unit impulse response. The operator p( D ) contains complete information about the system, but it can be hard to see at a glance. Things like stability, oscillatory behavior and resonanant frequencies are not always obvious just by looking at the polynomial operator. The transfer function W (s) = 1/ p(s) also contains complete information. After all, you can easily nd p( D ) if you know W (s). The transfer function has several big advantages. 1. Not all linear time invariant systems come from differential equations. So they dont necessarily have a p( D ). However, they all have transfer functions which are used in exactly the same way as we will learn for our systems. 2. When we studied convolution we learned Greens formula. This allows us to solve an ODE with rest initial conditions and any input once we know the unit impulse response. The Laplace transform will make Greens formula very simple to understand and use. Since the transfer function is the Laplace transform of the unit impulse response it will play a prominent role in this. 3. We can understand a combination of systems if we know the system functions of its constituent parts. 4. In the next session we will use the transfer functions pole diagram to to make many of the systems properties obvious at a glance. In this session we will start by dening the transfer function and use it to solve IVPs with rest initial conditions. Then we will see that Laplace transform changes convolution of functions in time into multiplication of functions in frequency. This will make Greens formula particularly simple to state. Finally, we will look at block diagrams and use them to combine systems in much the same way that simple circuits can be combined to make complicated ones.

The Transfer Function 1. Denition


We start with the denition (see equation (1). In subsequent sections of this note we will learn other ways of describing the transfer function. (See equations (2) and (3).) For any linear time invariant system the transfer function is W (s) = L(w(t)), where w(t) is the unit impulse response. (1)

Example 1. Find the transfer function for the system x + 3x = f (t). Solution. The unit impulse response is the solution to w + 3w = ( t ),

with rest IC.

The Laplace transform method nds W (s) on the way to nding w(t). Since we only want W (s) we can stop when we get there. Taking the Laplace transform of the DE we get sW (s) w(0 ) + 3W = 1

W=

1 . s+3

The annoying term w(0 ) = 0 because we have rest initial conditions. (Subsequent to this we will not bother writing the annoying terms when we have rest IC.)

2. Other Standard Terminology


The unit impulse response is also called the weight function and the transfer function is also called the system function. All of these terms are widely used and we will use them all to help you become familiar with them.

3. Formula for W (s)


Example 2. Find the transfer function for mx + bx + kx = f (t). Solution. The unit impulse response is the solution to mw + bw + kw = (t),

..

..

with rest IC.

By denition, the transfer function W (s) = L(w). So, we take the Laplace transform of the DE. There are no annoying terms because with rest initial

The Transfer Function conditions L(w) = s2 W (s) and L(w) = sW (s). We get

OCW 18.03SC

..

(ms2 + bs + k)W (s) = 1

W (s) =

ms2

1 . + bs + k

In example 2, the differential operator is p( D ) = mD2 + bD + kI . That is, the characteristic polynomial is p(s) = ms2 + bs + k and the transfer function is W (s) = 1/ p(s). Exactly the same reasoning holds for operators of higher order. Formula: For any polynomial operator p( D ) the transfer function for the system p( D ) x = f (t) is given by W (s) = 1 . p(s) (2)

Example 3. Suppose W (s) = 1/(s2 + 4) is the transfer function for a system p( D ) x = f (t). What is p( D )? Solution. Since W (s) = 1/ p(s) we have p(s) = s2 + 4, which implies p( D ) = D2 + 4 I .

4. Another Characterization of the Transfer Function


The best way to think of the transfer function is as a ratio of output to input. By this we mean the following. Suppose we have an equation p ( D ) x = f ( t ), with rest IC. 1 F ( s ) = W ( s ) F ( s ). p(s)

Taking Laplace transform of both sides gives p(s) X (s) = F (s) Solving for W (s) shows W (s) = output X (s) = . F (s) input (3)

X (s) =

5. Conclusion
We have characterized the transfer functions in three different ways. Equations (1) and (3) are perfectly general and apply to any LTI system. Equation (2) is specic to constant coefcient linear differential equations. 2

I. Finding p( D )
1 is the transfer function for the sys(s + 2)(s + 3) tem p( D ) x = f (t). What is p( D )? Quiz: Suppose W (s) = Choices: a) e2t e3t b) D2 + 5 D + 6 I c) 1/( D + 3)( D + 2) d) It doesnt exist e) Cant be found with the data given

Answer: (b) W (s) = 1/ p(s) = 1/(s2 + 5s + 6) p( D ) = D2 + 5 D + 6 I .

MIT OpenCourseWare http://ocw.mit.edu

Fall 2011

18.03SC Differential Equations

For information about citing these materials or our Terms of Use, visit: http://ocw.mit.edu/terms.

I. Finding p( D )
1 is the transfer function for the sys(s + 2)(s + 3) tem p( D ) x = f (t). What is p( D )? Quiz: Suppose W (s) = Choices: a) e2t e3t b) D2 + 5 D + 6 I c) 1/( D + 3)( D + 2) d) It doesnt exist e) Cant be found with the data given Pick what you think is the correct choice and then look at the answer.

I. Finding p( D )
1 is the transfer function for the sys(s + 2)(s + 3) tem p( D ) x = f (t). What is p( D )? Quiz: Suppose W (s) = Think about your answer and then look at the choices.

II. Finding p( D )
s . Find p( D ) so that W (s) is the transfer func+1 tion for the system p( D ) x = f (t). Quiz: Suppose W (s) = s2 Choices: a) cos(t) b) D2 + I c) D + 1/ D d) It doesnt exist e) Cant be found with the data given

Answer: (d) The system p( D ) x = f (t) has transfer function 1/ p(s). Since W (s) is not one over a polynomial there is no such polynomial. Note that W (s) is the transfer function for the system x + x = y, where we consider y to be the input.

..

II. Finding p( D )
s . Find p( D ) so that W (s) is the transfer func+1 tion for the system p( D ) x = f (t). Quiz: Suppose W (s) = s2 Choices: a) cos(t) b) D2 + I c) D + 1/ D d) It doesnt exist e) Cant be found with the data given Pick what you think is the correct choice and then look at the answer.

II. Finding p( D )
s . Find p( D ) so that W (s) is the transfer func+1 tion for the system p( D ) x = f (t). Quiz: Suppose W (s) = s2 Think about your answer and then look at the choices.

Modied Input
Way back when we introduced the language of system, input and response we decided that the right hand side of our equations wasnt always the input. Sometimes it was a modied version of the input. Example 1. Recall the heat diffusion equation x + kx = kTe (t), where Te (t) is the temperature of the environment. Consider Te (t) to be the input and nd the system function. Solution. Look at the equation for the unit impulse response w + kw = k(t),

rest IC.

Notice that since Te (t) is the input, the unit impulse response comes by letting Te (t) = (t). The Laplace transform now gives

( s + k )W ( s ) = k

W (s) =

k . s+k

Note well, that with modied input on the right hand side of the DE, the system function does not automatically have a 1 in the numerator. You might have noticed that in the previous example we could have 1 written W (s) = , which has our usual form. The next example s/k + 1 shows that this is not always the case. Example 2. Consider an LC circuit with input voltage v(t). Well assume L and C are set so the differential equation for the current i (t) is i (t) + 4i = v (t). We consider the input to be v(t) and the output to be i (t). (We use primes instead of dots for the derivative because the i already has a dot.) Finding the unit impulse response is tricky, because if we set v(t) = (t) then we will have (t) on the right hand side of the DE. Lets avoid this by using the characterization of the transfer function as the ratio output/input. In this case, well have W (s) = I (s)/V (s).

Modied Input

OCW 18.03SC

Assuming rest IC, we have L(v ) = sV (s), where, as usual, we have let the uppercase letter be the Laplace transform of the lowercase one. Applying the Laplace transform gives

(s2 + 4) I (s) = sV (s)

W (s) =

I (s) s = 2 . V (s) s +4

The s in the numerator guarantees this cannot be written in the form 1/ p(s) for any polynomial p(s). As a concluding note, well say that we were too pessimistic about our ability to handle (t). We might not know what it is, but we do know how to nd its Laplace transform.

L1 ( (t)) = sL((t)) (0 ) = s.

Greens Formula, Laplace Transform of Convolution 1. Greens Formula in Time and Frequency
When we studied convolution we learned Greens formula. This says, the IVP p( D ) x = f (t), with rest IC (1) has solution x (t) = (w f )(t), where w(t) is the weight function. (2)

(Remember, the weight function is the same as the unit impulse response.) The Laplace transform changes these equations to ones in the frequency variable s. (3) p(s) X (s) = F (s) X (s) = 1 F ( s ) = W ( s ) F ( s ), p(s) (4)

where W (s) is the transfer function. Equation (2) is Greens formula in time and (4) is Greens formula in frequency. In words, viewed from the t side, the solution to (1) is the convolution of the weight function and the input. Viewed from the s side, the solution is the product of the transfer function and the input.

2. Convolution
Comparing equations (2) and (4) we see that

L ( w f ) = W ( s ) F ( s ).

(5)

It appears that Laplace transforms convolution into multiplication. Technically, equation (5) only applies when one of the functions is the weight function, but the formula holds in general. Theorem: For any two functions f (t) and g(t) with Laplace transforms F (s) and G (s) we have

L ( f g ) = F ( s ) G ( s ).

(6)

Remarks: 1. This theorem gives us another way to prove convolution is commutative. It is just the commutivity of regular multiplication on the s-side.

L ( f g ) = F G = G F = L ( g f ).

Greens Formula, Laplace Transform of Convolution

OCW 18.03SC

2. In fact, the theorem helps solidify our claim that convolution is a type of multiplication, because viewed from the frequency side it is multiplication. Proof: The proof is a nice exercise in switching the order of integration. We wont use 0 and t+ in the integrals, since they would just clutter the exposition. It is an amusing exercise to put them in and see that they transform correctly as we manipulate the integrals. We start by writing L( f g) as the convolution integral followed by the Laplace integral.

L( f g) = =

( f g)(t)est dt
f (t u) g(u)est du dt.

t
0 0

Next, we change the order of integration (see the gure below).


0 u

f (t u) g(u)est dt du.

Finally, change variables in the inner integral: substitute v = t u, dv = dt, (u a constant)

= =


0 0

f (v) g(u)es(v+u) dv du
sv

f (v)e

dv

g(u)esu du

= F ( s ) G ( s ).
tO t=u tO t=u

/u

/u

Fig. 1. Changing the order from du dt to dt du.

3. Integration Rule
If differentiation on the time side leads to multiplication by s on the frequency side then we should expect integration in time to lead to division by s. If f (t) is a function with Laplace transform F (s) then the integration 2

Greens Formula, Laplace Transform of Convolution

OCW 18.03SC

rule states:

t+ 0

f ( ) d

F (s) . s

Proof: One way to prove this is using the t-derivative rule. Lets be clever and use convolution instead. The integral is exactly f (t) 1. Thus,

t+ 0

f ( ) d

= L ( f 1) = F ( s ) L (1) =

F (s) . s

This is what we needed to show.

Block Diagrams 1. Introduction


We discussed some simple block diagrams when we introduced the notions of system, input, and output back in unit 1. Here, we will include the transfer function in the diagram and show how to use them to compute the transfer function of more complicated systems. As we do this, it will be useful to keep in mind the desciption of the transfer function as output/input.

2. Simple Examples
Example 1. Suppose we have the system mx + bx + kx = f (t), with input f (t) and output x (t). The Laplace transform converts this all to functions and equations in the frequency variable s. The transfer function for this system is W (s) = 1/(ms2 + bs + k ). We can write the relation between input and output as input F (s) output X (s) = W (s) F (s) As a block diagram we can represent the system by
F (s) W (s) X (s)

..

Fig. 1. Block diagram for a system with transfer function W (s). Sometimes we write the formula for the transfer function in the box representing the system. For the above example this would look like
F (s) 1 ms2 + bs + k X (s)

Fig. 2. Block diagram giving the formula for the transfer function. Example 2. (Cascading systems) Consider the cascaded system p1 ( D ) x = f , p2 ( D ) y = x , rest IC.

The input to the cascade is f and the output is y. The rst equation takes the input f and outputs x. This is the input to the second equation, which ouputs y.

Block Diagrams

OCW 18.03SC

This is easy to solve on the frequency side. Let W1 (s) = 1/ p1 (s) and W2 (s) = 1/ p2 (s) be the transfer functions for the two differential equations. Considering the two equations separately we have X (s) = W1 (s) F (s) and Y (s) = W2 (s) X (s).

It follows immediately that Y (s) = W2 (s) W1 (s) F (s). Therefore the transfer function for the cascade is output/input = Y (s)/ F (s) = W2 (s) W1 (s). In other words, for cascaded systems the transfer functions multiply. Representing this as block diagrams we have two equivalent diagrams
F (s) W1 (s) F (s) X (s) W2 (s) Y (s) Y (s)

W1 (s)W2 (s)

Fig. 3. Equivalent block diagrams for a cascaded system. Example 3. (Parallel systems) Suppose that we have a system consisting of two systems in parallel as shown in the block diagram.
F (s) W1 (s) + W2 (s) Y (s)

Fig. 4. Systems in parallel. Find the transfer function for the entire system. Solution. The plus sign in the circle indicates the two signals coming into the junction should be added. The split near the start indicates the input F (s) go into each system. The way to gure out the transfer function is to name the outputs of each individual system.
F (s) F F W1 (s) W2 (s) X1 + X2 Y (s)

Fig. 5. System with intermediate outputs labeled. 2

Block Diagrams

OCW 18.03SC

For each system we know output = transfer function input. Thus, X1 = W1 F, X2 = W2 F, Y = X1 + X2 . So, we easily compute Y = X1 + X2 = W1 F + W2 F = (W1 + W2 ) F. Therefore the transfer function is W1 + W2 .

3. Feedback Loops
This is a bonus section. You will not see it in the problems or tests. Many systems use feedback loops. That is, the output of the system is monitored and used to modify the input. It is very hard to control a system without a feedback loop. Suppose we start with a system with transfer function W (s).
F (s) W (s) X (s)

and modify it to have the feedback loop shown


F + V gY W g Y

The original system is known as the open loop system and the corresponding system with feedback is known as the closed loop system. Weve labeled the outputs from each system element. The symbol g means the input is scaled by g, that is apply a gain of g to the input. The symbol means the two inputs are combined; the plus and minus signs indicate to add or subtract the corresponding input. The method of nding the transfer function is the same as in the previous examples. A bit of algebra gives V = F gY, Y = WV

Y = W ( F gY )

Y=

W F. 1 + gW

As usual, the transfer function is output/input = Y / F = W /(1 + gW ). This formula is one case of what is often called Blacks formula Example 4. Suppose we have an open loop system, say a circuit, with transfer function W (s) = s/( as2 + bs + c). If we add a feedback loop with 3

Block Diagrams

OCW 18.03SC

gain g then using Blacks formula the closed loop transfer function is s/( as2 + bs + c) s = 2 . 2 as + (b + g)s + c 1 + gs/( as + bs + c)

Poles, Amplitude Response, Connection to ERF


For our standard LTI system p( D ) x = f , the transfer function is W (s) = 1/ p(s). In this case the poles of W (s) are simply the zeros of the characteristic polynomial p(s) (also known as the characteristic roots). We have had lots of experience using these roots in this course and know they give important information about the system. The reason we talk about the poles of the transfer function instead of just sticking with the characteristic roots is that all LTI systems have a transfer function, while the characteristic polynomial is dened only for systems described by constant coefcient linear ODEs. Much of units 1,2 and the current unit have been devoted to the study of DEs for LTI systems: p ( D ) x = f ( t ). (1) In units 1 and 2 we saw that the stability of the system is determined by the roots of the characteristic polynomial. We saw as well that the amplitude response of the system to a sinusoidal input of frequency is also determined by the characteristic polynomial, namely as the gain g( ) = 1 | p (i ) |

In this unit weve learned about the Laplace transform, which gives us another view of a signal by transforming it from a function of t, say f (t), to a function F (s) of the complex frequency s. A key object from this point of view is the transfer function. For the system (1), if we consider f (t) to be the input and x (t) to be the output, then the transfer function is W (s) = 1/ p(s), which is again determined by the characteristic polynomial. In this session we will learn about poles and the pole diagram of an LTI system. This ties together the notions of stability, amplitude response and transfer function, all in one diagram in the complex s-plane. The pole diagram gives us a way to visualize systems which makes many of their important properties clear at a glance; in particular, and remarkably, the pole diagram 1. shows whether the system stable; 2. shows whether the unforced system is oscillatory;

Poles, Amplitude Response, Connection to ERF

OCW 18.03SC

3. shows the exponential rate at which the unforced system returns to equilibrium (for stable systems); and 4. gives a rough picture of the amplitude response and practical resonances of the system. For these reasons the pole diagram is a standard tool used by engineers in understanding and designing systems. We conclude by reminding you that every LTI system has a transfer function. Everything we learn in this session will apply to such systems, including those not modeled by DEs of the form (1)

Denition of Poles 1. Rational Functions


A rational function is a ratio of polynomials q(s)/ p(s). Examples. The following are all rational functions. (s2 + 1)/(s3 + 3s + 1), 1/(ms2 + bs + k), s2 + 1 + (s2 + 1)/1. If the numerator q(s) and the denominator p(s) have no roots in common, then the rational function q(s)/ p(s) is in reduced form Example. The three functions in the example above are all in reduced form. Example. (s 2)/(s2 4) is not in reduced form, because s = 2 is a root of both numerator and denominator. We can rewrite this in reduced form as s2 s2 1 = = . (s 2)(s + 2) s+2 s2 4

2. Poles
For a rational function in reduced form the poles are the values of s where the denominator is equal to zero; or, in other words, the points where the rational function is not dened. We allow the poles to be complex numbers here. Examples. a) The function 1/(s2 + 8s + 7) has poles at s = 1 and s = 7. b) The function (s 2)/(s2 4) = 1/(s + 2) has only one pole, s = 2. c) The function 1/(s2 + 4) has poles at s = 2i. d) The function s2 + 1 has no poles. e) The function 1/(s2 + 8s + 7)(s2 + 4) has poles at -1, -7, 2i. (Notice that this function is the product of the functions in (a) and (c) and that its poles are the union of poles from (a) and (c).) Remark. For ODEs with system function of the form 1/ p(s), the poles are just the roots of p(s). These are the familiar characteristic roots, which are important as we have seen.

3. Graphs Near Poles


We start by considering the function F1 (s) = 1 s . This is well dened for every complex s except s = 0. To visualize F1 (s) we might try to graph it. However it will be simpler, and yet still show everything we need, if we graph | F1 (s)| instead.

Denition of Poles To start really simply, lets just graph | F1 (s)| = complex).
|1/s|
1 |s|

OCW 18.03SC

for s real (rather than

2 1 3 2 1 1
1 |s|

Figure 1: Graph of

for s real.

Now lets do the same thing for F2 (s) = 1/(s2 4). The roots of the denominator are s = 2, so the graph of | F2 (s)| = |s2 1 has vertical asymp4| totes at s = 2.
|1/(s2 4)|

3 2 1

1
1 | s2 4|

Figure 2: Graph of

for s real.

As noted, the vertical asymptotes occur at values of s where the denominator of our function is 0. These are what we dened as the poles. F1 (s) = F2 (s) =
1 s

has a single pole at s = 0. has two poles, one each at s = 2.

1 s2 4

Looking at Figures 1 and 2 you might be reminded of a tent. The poles of the tent are exactly the vertical asympotes which sit at the poles of the function. 2

Denition of Poles

OCW 18.03SC

Lets now try to graph | F1 (s)| and | F2 (s)| when we allow s to be complex. If s = a + ib then F1 (s) depends on two variables a and b, so the graph requires three dimensions: two for a and b, and one more (the vertical axis) for the value of | F1 (s)|. The graphs are shown in Figure 3 below. They are 3D versions of the graphs above in Figures 1 and 2. At each pole there is a conical shape rising to innity, and far from the poles the function fall off to 0.

Figure 3: The graphs of |1/s| and 1/|s2 4|. Roughly speaking, the poles tell you the shape of the graph of a function | F (s)|: it is large near the poles. In the typical pole diagams seen in practice, the | F (s)| is also small far away from the poles.

4.

Poles and Exponential Growth Rate

If a > 0, the exponential function f 1 (t) = e at grows rapidly to innity as t . Likewise the function f 2 (t) = e at sin bt is oscillatory with the amplitude of the oscillations growing exponentially to innity as t . In both cases we call a the exponential growth rate of the function. The formal denition is the following Denition: The exponential growth rate of a function f (t) is the smallest value a such that f (t) lim bt = 0 for all b > a. (1) t e In words, this says f (t) grows slower than any exponential with growth rate larger than a. Examples. 1. e2t has exponential growth rate 2. 2. e2t has exponential growth rate -2. A negative growth rate means that the function is decaying exponentially to zero as t . 3. f (t) = 1 has exponential growth rate 0.

Denition of Poles

OCW 18.03SC cos t =0 ebt

4. cos t has exponential growth rate 0. This follows because lim for all positive b.

5. f (t) = t has exponential growth rate 0. This may be surprising because f (t) grows to innity. But it grows linearly, which is slower than any positive exponential growth rate. 6. f (t) = et does not have an exponential growth rate since it grows faster than any exponential. Poles and Exponential Growth Rate We have the following theorem connecting poles and exponential growth rate. Theorem: The exponential growth rate of the function f (t) is the largest real part of all the poles of its Laplace transform F (s). Examples. Well check the theorem in a few cases. 1. f (t) = e3t clearly has exponential growth rate equal to 3. Its Laplace transform is 1/(s 3) which has a single pole at s = 3,and this agrees with the exponential growth rate of f (t). 2. Let f (t) = t, then F (s) = 1/s2 . F (s) has one pole at s = 0. This matches the exponential growth rate zero found in (5) from the previous set of examples. 3. Consider the function f (t) = 3e2t + 5et + 7e8t . The Laplace transform is F (s) = 3/(s 2) + 5/(s 1) + 7/(s + 8), which has poles at s = 2, 1, 8. The largest of these is 2. (Dont be fooled by the absolute value of -8, since 2 > 8, the largest pole is 2.) Thus, the exponential growth rate is 2. We can also see this directly from the formula for the function. It is clear that the 3e2t term determines the growth rate since it is the dominant term as t . 4. Consider the function f (t) = et cos 2t + 3e2t The Laplace transform is 3 F (s) = (s+1s)2 +4 + s+ 2 . This has poles s = 1 2i, -2. The largest real part among these is -1, so the exponential growth rate is -1. Note that in item (4) in this set of examples the growth rate is negative because f (t) actually decays to 0 as t . We have the following Rule: 1. If f (t) has a negative exponential growth rate then f (t) 0 as t . 2. If f (t) has a positive exponential growth rate then f (t) as t . 4
2

Denition of Poles

OCW 18.03SC

5.

An Example of What the Poles Dont Tell Us

Consider an arbitrary function f (t) with Laplace transform F (s) and a > 0. Shift f (t) to produce g(t) = u(t a) f (t a), which has Laplace transform G (s) = eas F (s). Since eas does not have any poles, G (s) and F (s) have exactly the same poles. That is, the poles cant detect this type of shift in time.

Pole Diagrams 1. Denition of the Pole Diagram


The pole diagram of a function F (s) is simply the complex s-plane with an X marking the location of each pole of F (s). Example 1. Draw the pole diagrams for each of the following functions. a) F1 (s) = d) F4 (s) = Solution.
1 s +2 s s2 +6s+10

b) F2 (s) = e) F5 (s) =

1 s 2 1 ((s2 +3)2 +1)(s+2)(s+4)

c) F3 (s) = f) F6 (s) =

1 s2 +4 1 ((s+3)2 +1)(s2)

( a)
3i i

(b)
3i i 3

(c)
3i

X
i

1i

1i

1i

X
3i 3i 3i

(d)
3i

(e)
3i i

(f)
3i i

X
3

X
3

X
3

1i

1i

1i

3i

3i

3i

For (d) we found the poles by rst completing the square: s2 + 6s + 10 = (s + 3)2 + 1, so the poles are at s = 3 i. Example 2. Use the pole diagram to determine the exponential growth rate of the inverse Laplace transform of each of the functions in example 1. Solution. a) The largest pole is at -2, so the exponential growth rate is -2. b) The largest pole is at 2, so the exponential growth rate is 2. c) The poles are 2i, so the largest real part of a pole is 0. The exponential growth rate is 0. d) The largest real part of a pole is -3. The exponential growth rate is -3. e) The largest real part of a pole is -2. The exponential growth rate is -2.

Pole Diagrams

OCW 18.03SC

f) The largest real part of a pole is 2. The exponential growth rate is 2. Example 3. Each of the pole diagrams below is for a function F (s) which is the Laplace transform of a function f (t). Say whether (i) f (t) 0 as t (ii) f (t) as t (iii) You dont know the behavior of f (t) as t 0,

( a)
X

3i i 3

(b)
X X X

3i i 3

(c)
X

3i i

(d)
X
3

3i

3 1 1i 3i

3 1 1i 3i

3 1 1i

3i

3 1 1i 3i

i 3

(e)
X X X

3i i 3

(f)
X X X

3i i

( g)
X X
3

3i

Xi
3 1X 1i 3i
3

3 1 1i 3i

3 1 1i

3i

Solution. a) Exponential growth rate is -2, so f (t) 0. b) Exponential growth rate is -2, so f (t) 0. c) Exponential growth rate is 2, so f (t) . d) Exponential growth rate is 0, so we cant tell how f (t) behaves. Two examples of this: (i) if F (s) = 1/s then f (t) = 1, which stays bounded; (ii) if F (s) = 1/s2 then f (t) = t, which does go to innity, but more slowly than any positive exponential. e) Exponential growth rate is 0, so dont know the behavior of f (t). f) Exponential growth rate is 3, so f (t) . g) Exponential growth rate is 0, so dont know the behavior of f (t). (e.g. both cos t and t cos t have poles at i.

2. The Pole Diagram for an LTI System


Denition: The pole diagram for an LTI system is dened to be the pole diagram of its transfer function. Example 4. Give the pole diagram for the system

..

x + 8 x + 7 x = f ( t ),

Pole Diagrams

OCW 18.03SC

where we take f (t) to be the input and x (t) the output. 1 1 = . s2 + 8s + 1 (s + 1)(s + 7) Therefore, the poles are s = 1, 7 and the pole diagram is Solution. The transfer function for this system is W (s) =
i

1 1i

Example 5. Give the pole diagram for the system

..

x + 4 x + 6 x = y,

where we consider y(t) to be the input and x (t) to be the output. Solution. Assuming rest ICs, Laplace transforming this equation gives us s (s2 + 4s + 6) X = sY. This implies X (s) = 2 Y (s) and the transfer s + 4s + 6 s function is W (s) = 2 . This has poles at s = 2 2 i. s + 4s + 6 X
2 1
2i i 1 i 2

2i

Figure: Pole diagram for the system in example 5.

Poles and Stability


Recall that the LTI system p( D ) x = f has an associated homogeneous equation p( D ) x = 0 In unit 2 we saw the following stability criteria. 1. The system is stable if every solution to (2) goes to 0 as t . In words, the unforced system always returns to equilibrium. 2. Equivalently, the system is stable if all the roots of the characteristic equation have negative real part. 1 the poles p(s) of the system are just the characteristic roots. Comparing this with the stability criterion 2, gives us another way of expressing the stability criteria. Now, since the transfer function for the system in (1) is 3. The system is stable if all its poles have negative real part. 4. Equivalently, the system is stable if all its poles lie strictly in the left half of the complex plane Re(s) < 0. Criterion 4 tells us how to see at a glance if the system is stable, as illustrated in the following example. Example. Each of the following six graphs is the pole diagram of an LTI system. Say which of the systems are stable.
(a)
3i

(1)

(2)

(b)
3i

(c)
3i i

X
i 3 i 3 3

1 i 3i

1 i 3i

X X X X
3

1 i 3i

Poles and Stability

OCW 18.03SC

(d)
3i i 3

(e) X X X
3

(f )
3i 3i

X
i i 3 3 1 i 3i

1 i

X X

1 i 3i

3i

Solution. (a), (c) and (e) have all their poles in the left half-plane, so they are stable. The others do not, so they are not stable.

Poles and Amplitude Response


We started the session by considering the poles of functions F (s), and saw that, by denition, the graph of | F (s)| went off to innity at the poles. Since it tells us where | F (s)| is innite, the pole diagram provides a crude graph of | F (s)|: roughly speaking, | F (s)| will be large for values of s near the poles. In this note we show how this basic fact provides a useful graphical tool for spotting resonant or near-resonant frequencies for LTI systems. Example 1. Figure 1 shows the pole diagram of a function F (s). At which of the points A, B, C on the diagram would you guess | F (s)| is largest?

C
X
2i

X A B
2 1

X X

i 2i

Figure 2: Pole diagram for example 1. Solution. Point A is close to a pole and B and C are both far from poles so we would guess point | F (s)| is largest at point A. Example 2. The pole diagram of a function F (s) is shown in Figure 2. At what point s on the positive imaginary axis would you guess that | F (s)| is largest? X X X X X
3i 2i i

2 1

1 i

2i 3i

Figure 2: Pole diagram for example 2.

Poles and Amplitude Response

OCW 18.03SC

Solution. We would guess that s should be close to 3 i, which is near a pole. There is not enough information in the pole diagram to determine the exact location of the maximum, but it is most likely to be near the pole.

1. Amplitude Response and the System Function


Consider the system p ( D ) x = f ( t ). (1) where we take f (t) to be the input and x (t) to be the output. The transfer function of this system is 1 W (s) = . (2) p(s) When f (t) = B cos( t) the Exponential Response Formula from unit 2 gives the following periodic solution to (1) x p (t) = B cos( t ) , | p (i ) | where = Arg( p(i )). (3)

If the system is stable, then all solutions are asymptotic to the periodic solution in (3). In this case, we saw in the session on Frequency Response in unit 2 that the amplitude response of the system as a function of is g( ) = 1 | p ( i ) |. (4)

Comparing (2) and (4), we see that for a stable system the amplitude response is related to the transfer function by g ( ) = |W ( i ) | . Note: The relation (5) holds for all stable LTI systems. Using equation (5) and the language of amplitude response we will now re-do example 2 to illustrate how to use the pole diagram to estimate the practical resonant frequencies of a stable system. Example 3. Figure 3 shows the pole diagram of a stable LTI system. At approximately what frequency will the system have the biggest response? (5)

Poles and Amplitude Response

OCW 18.03SC

X X X X X

3i 2i i

2 1

1 i

2i 3i

Figure 3: Pole diagram for example 3 (same as Figure 2). Solution. Let the transfer function be W (s). Equation (5) says the amplitude response g( ) = |W (i )|. Since i is on the positive imaginary axis, the amplitude response g( ) will be largest at the point i on the imaginary axis where |W (i )| is largest. This is exactly the point found in example 2. Thus, we choose i 3i, i.e. the practical resonant frequency is approximately = 3. Note: Rephrasing this in graphical terms: we can graph the magnitude of the system function |W (s)| as a surface over the s-plane. The amplitude response of the system g( ) = |W (i )| is given by the part of the system function graph that lies above the imaginary axis. This is all illustrated beautifully by the applet Amplitude: Pole Diagram explored in the next note in this session.

MIT OpenCourseWare http://ocw.mit.edu

Fall 2011

18.03SC Differential Equations

For information about citing these materials or our Terms of Use, visit: http://ocw.mit.edu/terms.

Amplitude Response: Pole Diagram Applet


Open the applet and play with it. The main graph window is actually a 3D plot that you can rotate with your mouse. Exploring the Applet The applet shows the pole diagram and amplitude response of the system .. . x + bx + kx = k cos( t). It is designed to illustrate the connection between the pole diagram and the amplitude response of the system. To return the graph window to its original position click on the top button. The screen should look something like

The main graph window and the pole diagram in the lower right look the same. Notice that the real axis on both is green, the imaginary axis is yellow, and the poles are red. The amplitude response graph in the upper right is the same one weve seen in the Amplitude and Phase Second Order applets. The amplitude response graph is the same color as the imaginary axis because A = | p(ik )| , that is, because i is on the imaginary axis. The yellow

Amplitude Response: Pole Diagram Applet

OCW 18.03SC

dot in all the windows indicates the current value of . (Because cos( t) is the real part of ei and ei , there are dots at both i in the pole diagram and main graph.) Play with the b, k and sliders to see how the poles and amplitude change. Now set b = 0.75 and k = 2, then move to the position where the system has the maximum amplitude response. Now click on the side button and rotate the main graph so you can see all its features. It should look something like this:

The surface in the plot is the graph of the magnitude of the trans shown k fer function p(s) . The yellow curve on the surface above the imaginary axis is therefore the plot of p(ik ) . Notice that this is the same graph as the amplitude response graph in the upper right of the applet. (The main k graph also shows p(i ) for < 0. This is just the mirror image of the graph for > 0. We can now explain how the main graph illustrates why choosing i near a pole gives a big response: The yellow amplitude curve runs alongside the mountains that rise up near the poles. As i gets close to a pole, the amplitude curve moves up the side of the mountain.

Amplitude Response: Pole Diagram Applet

OCW 18.03SC

Questions 1. Why cant the amplitude response become innite by placing i directly on a pole? 2. Why are the poles in the left half-plane (Re(s) < 0) for all choices of b and k? 3. Move b to 1.5 and k to 0.4. What happens to the poles? At what frequency is the amplitude response maximized? 4. Leave k at 0.4 and move b to 1.1. The poles are now a complex conjugate pair. What frequency gives the maximum amplitude response? Answers 1. Because i must stay on the imaginary axis, and the poles are not on the imaginary axis. 2. Because both b and k are positive, we know that the system is stable (since all second order systems with positive coefcients are stable). Therefore, all the poles must have negative real part, which places them in the left half-plane 3. At these settings of b and k the poles are real. The peak amplitude response occurs at i = 0. 4. The peak amplitude is still at = 0. As we saw in the session on Frequency Response in unit 2, this system needs to be sufciently lightly damped, not just underdamped, in order to have a practical resonant frequency.

Linear Systems: Introduction


Suppose you have want to model the populations of cats and mice on an island, say c(t) and m(t). Cats feed on mice, and will starve without them. Therefore, any differential equation describing the rate-of-change of c should also involve m. Equally, the rate-of-change of m depends on the current number of cats. Up until now, we have studied differential equations with a single dependent variable. However, many real-life situations are modeled by collections of differential equations involving several dependent variables and their time-derivatives, the above scenario is one of them. These are called systems. In this nal unit, we will learn about some of the simpler ones: they will be of rst order, which means that only the rst time-derivatives of the dependents variables are involved. Also, there will usually only be two dependent variables. We will spend most of the unit studying linear systems, before devoting the nal few sessions to nonlinear ones. The present session introduces 2 2 linear systems; we will learn how to solve them by turning the problem into a second order ODE in one variable. Conversely, every second order ODE can be obtained from a system, and such a perspective can shed a new light on the ODE. Look out for the companion matrix and the applet Phase portraits: Matrix entry, which allows one to easily visualize the solutions to these sorts of systems. Perhaps most importantly, we will learn to present both our system and its solutions using matrices and vectors, paving the way to the later sessions in this unit.

First order Linear Systems 1. Models: Two Examples


Example 1. Farmer Jones and farmer McGregor each have a eld full of rabbits; farmer Jones eld contains x (t) rabbits, and farmer McGregors y(t) ones; t, time, is measured in months. These rabbits breed fast: they show a net growth rate of 0.5 rabbits per rabbit per month. The rabbits can also hop over the hedge between the elds; the grass is greener in Farmer McGregors eld, so Jones rabbits jump over a the rate of 0.2 per month, whereas McGregors jump at the rate of 0.1 per month. We can illustrate this with the following ow diagram:
.5 .5

.2 Jones .1 McGregor

Putting everything together, the equations governing x and y are: x = .5x .2x + .1y = .3x + .1y y = .5y .1y + .2x = .2x + .4y. This is an example of a rst order 2 2 linear system. Example 2. Consider the equations: R = J
1 4R 17 16 R

+ +

J
3 4J

This is another rst order 2 2 linear system. It is the result of an analysis by the MIT Humanities Department of the plot a famous Shakespeare play: R denotes Romeos love for Juliet, and J Juliets love for Romeo. What does this model mean? Let us try to work backwards. The change in Romeos feelings towards Juliet is mostly determined by how she feels about him: J is the most important term in the expression for R . His own feelings have a small reinforcing effect corresponding to the factor of 1/4 in front of the R.

First order Linear Systems

OCW 18.03SC

On the other hand, Juliet is more complex. She has a healthy selfawareness. If she loves Romeo, that very fact causes her to love him more: this is where the (3/4) J term comes from. On the other hand, if he seems S hence the to love her, she gets frightened and starts to love him less A (17/16) R term. We shall revisit this example in sections text companion matrix and applet companion matrix, and analyze mathematically the solutions to this tragic model.

2. Solving a Linear System by Elimination


Denition. A two-by-two rst order linear system of ODEs with constant coefcients is a collection of equations: x = ax + by y = cx + dy, where a, b, c, and d are constants. Well shorten this to rst order linear system or even linear system. Remark. Say you have dependent variables x1 , . . . , xn ; one can dene n n rst order linear systems (with constant coefcients) in exactly the same way: x1 = a11 x1 + . . . + a1n xn x n = an1 x1 + . . . + ann xn , where the aij are constants. In the next few sessions, we shall develop many tools for understanding 2 2 linear systems, both analytically and qualitatively. All these techniques generalize, in a fairly straightforward fashion, to n n systems. However, this goes beyond the scope of this course. The interested reader could for instance consult the textbook by Edwards and Penney. The naive way to solve a linear system of ODEs with constant coefcients is by eliminating variables, so as to change it into a single higher order equation, in one dependent variable. One then solves this equation using the techniques for constant-coefcient ODEs learned in unit 2. This is best illustrated with a worked example.

...

First order Linear Systems

OCW 18.03SC

Example. Let us consider the system of equations from example 1. x = 0.3x + 0.1y y = 0.2x + 0.4y. Step 1. Transform the equations to get a second order ODE for x. Use (1) to express y in terms of x: y = 10 x 3x Plug this into (2). (From equation (3) we get y = 10 x 3x). This gives 10x 3x = .2x + 4x 1.2x 10 x 7x + x = 0. Step 2. Solve the ODE for x. The characteristic equation for (4) is: 10s2 7s + 1 = 0

(1) (2)

(3)

..

..

..

(4)

s2 .7s + .1 = 0,

which has roots r1 = .5 and r2 = .2. Thus we get two basic solutions, x1 = e.5t and x2 = e.2t .

Step3. Solution for y. Each basic solution for x gives a corresponding solution for y, using equation (3) y1 = 2e.5t and y2 = e.2t . Step 4. Using superposition we get the general solution x (t) = c1 e0.5t + c2 e0.2t y(t) = 2c1 e0.5t c2 e0.2t Remarks. 1. It is important to understand that the constants c1 and c2 are the same for x and y; this follows from equation (3). 2. For certain ci , there will be negatively-valued solutions; these are clearly not biologically signicant: the model only holds for x,y 0. 3. We chose to eliminate y to have a second order equation in terms of x; we could just as well have chosen to eliminate x to get an equation in y. It might sometimes be computationally easier to go one way than the other; look out for this. 3

First order Linear Systems

OCW 18.03SC

4. We started by solving systems by elimination because it reduces to our previous methods. This will not be our preferred technique. In fact, in both theoretical and especially numerical work it is usually preferable to go the opposite way and convert a higher order ODE into a system of rst order equations and then use matrix methods.

Solving by Elimination
Exercise. Use the method of elimination to solve the following system. x = x + 3y y = x y. Answer. Step 1. Let us eliminate x by solving the second equation for x. We get x = y+y Replacing x everywhere by y + y in the rst equation gives

(1)

..

y 4y = 0.

(2)

Step 2. The characteristic equation for (2) is (r 2)(r + 2) = 0, so the general solution for y is y = c 1 e 2t + c 2 e 2t . Step 3. From the solution for y and equation (1), that was originally used to eliminate x, we get x = 3c1 e2t c2 e2t . Step 4. The solution to the system is thus x = 3 c 1 e 2t c 2 e 2t y = c 1 e 2t + c 2 e 2t .

Solving by Elimination
Exercise. Use the method of elimination to solve the following system. x = x + 3y y = x y. Try to solve these problems and then look at the solutions.

Review of Vectors and Matrices 1. Vectors


A vector (or n-vector) is an n-tuple of numbers; they are usually real numbers, but we will sometimes allow them to be complex numbers. All the rules and operations below apply just as well to n-tuples of complex numbers. (In the context of vectors, a single real or complex number, i.e., a constant, is called a scalar.) As we are dealing with 2 2 linear systems, we are primarily interested in scalars and 2-vectors: ordered pairs of numbers. The pair can be written horizontally as a row vector or vertically as a column vector. In these notes, it will almost always be a column. To save space, we will sometimes write the column vector as shown below; the small T stands for transpose, and means: change the row to a column. a = ( a, b ) row vector a = ( a, b ) T column vector

These notes use boldface for vectors hope; in handwriting, place an arrow a over the letter. Vector operations. Here are two standard operations on vectors: addition: ( a, b) + (c, d) = ( a + c, b + d). multiplication by a scalar: c ( a, b) = (ca, cb) scalar product: ( a, b)( c, d) = ac + bd

2. Matrices
An m n matrix A is a rectangular array of numbers (real or complex) having m rows and n columns. The element in the i-th row and j-th column is called the ij-th entry and written aij . The matrix itself is sometimes written ( aij ), i.e., by giving its generic entry, inside the matrix parentheses. We will be interested in matrices where m and n are at most 2. Note that a 1 2 matrix is a row vector; an 2 1 matrix is a column vector. Matrix operations. addition: if A and B are both m n matrices, they are added by adding the corresponding entries; i.e., if A = ( aij ) and B = (bij ), then A + B = ( aij + bij ).

Review of Vectors and Matrices

OCW 18.03SC

multiplication by a scalar: to get cA, multiply every entry of A by the scalar c; i.e., if A = ( aij ), then cA = (caij ). matrix multiplication: if A is an m n matrix and B is an n k matrix, their product AB is an m k matrix, dened by using the scalar product operation: ij-th entry of AB = (i-th row of A)( j-th column of B)T where the scalar product of two 1-vectors is just their normal product. The denition makes sense since both vectors on the right are vectors of the same length n. In what follows, the most important cases of matrix multiplication will be: (i) A and B are square 2 2 matrices. In this case, multiplication is always possible, and the product AB is again an 2 2 matrix. (ii) A is an 2 2 matrix and B = b, a column 2-vector. In this case, the matrix product Ab is again a column 2-vector. Laws satised by the matrix operations. For any matrices for which the products and sums below are dened, we have ( A B) C = A (B C) (associative law) (distributive laws) A ( B + C ) = A B + A C, ( A + B) C = A B + A C = B A (commutative law fails in general) AB The identity matrix I is the 2 2 matrix with 1s on the main diagonal (upper left and bottom right), and 0s elsewhere. If A is an arbitrary 2 2 matrix, it is easy to check from the denition of matrix multiplication that AI = A and I A = A.

The exercises later in this session should help you get familiar with all these concepts.

Vectors and Matrices


1. Complete the following vector operations. 1 a) (1, 2)T . Answer. (transpose). 2 a 3 a+3 b) + . Answer. . b 4 b+4 5 5c c) c . Answer. . 6 6c 2. Compute the following matrix products: 1 a) ( a, b) . Answer. a + 2b (1 2 times 2 1 = 1 1.) 2 a a 2a b) (1, 2). Answer. (2 1 times 1 2 = 2 2.) b b 2b a b 1 a + 2b c) . Answer. . c d 2 c + 2d a b d) (1, 2) . Answer. ( a + 2c, b + 2d). c d a b 1 2 a + 3b 2 a + 4b e) . Answer. . c d 3 4 c + 3d 2c + 4b

Vectors and Matrices


1. Complete the following vector operations. a) (1, 2)T . a 3 b) + . b 4 5 c) c . 6 2. Compute the following matrix products: 1 a) ( a, b) . 2 a b) (1, 2). b a b 1 c) . c d 2 a b d) (1, 2) . c d a b 1 2 e) . c d 3 4 Try to solve these problems and then look at the solutions.

Describing a First order System Using Matrix Notation 1. Description of the Equation
A general 2 2 linear system is given by: x = ax + by y = cx + dy The terms have been arranged in a suggestive manner. We can express this system using matrices and vectors: . x a b x . = . c d y y We can present this in the following even more compact form. a b x Let A = and write u for the column vector . c d y . . . x (t) . We have u(t) = and the system is simply u = Au. y(t) Example 1. Our favorite system, governing the rabbit populations in farmers Jones and McGregors elds, was x = 0.3x + 0.1y y = 0.2x + 0.4y, which has matrix form . x 0.3 0.1 x . = 0.2 0.4 y y

or

u = A u,

where A=

0.3 0.1 0.2 0.4

2. Description of the Solution


To describe the solution, we will use the column vector u(t) = x (t) y(t) .

Example 2. Earlier we used the method of elimination to solve the system in example 1. We found x (t) = c1 e0.5t + c2 e0.2t , y(t) = 2c1 e0.5t c2 e0.2t . Rewriting this in vector form we have c1 e0.5t + c2 e0.2t u(t) = . 2c1 e0.5t c2 e0.2t

Describing a First order System Using Matrix Notation

OCW 18.03SC

We can rewrite this as u ( t ) = c1 e


0.5t

1 2

+ c2 e

0.2t

1 1

which is a clearer way of presenting it. Let 1 1 .5t .2t u1 ( t ) = e and u2 (t) = e . 2 1 The column vectors u1 (t) and u2 (t) are both solutions. Since they both involve only one form of exponential, they are sometimes known as basic independent solutions, or normal modes. The general solution is a linear combination of them. We will learn much more about normal modes in the sessions on matrix methods and the phase portrait. Remark. As with linear second order ODEs in unit 2, the general solution to to a 2 linear system should always consist of linear combinations of two truly different solutions. It is not necessary , but usually our techniques will make these two solutions the normal modes.

3. Geometry of the Solutions


Suppose you want to plot a solution u(t). As time increases, it traces a curve in the xy-plane. Example. The solution u1 (t) traces a ray that passes through (1, 2) at t = 0 and move off towards innity in a straight line, with exponential speed. There is another ray through (1, 1), corresponding to the solution u2 (t). (This is tricky: the exponential in the formula for u1 might make you think the trajectory is curved. However, if you look carefully at the formula you will see u1 (t) is always a multiple of the vector (1, 2)T .) The applet Linear phase portrait: matrix entries will allow us to visualise this nicely, and get a feel for other sorts of trajectories. Later in this session you will look at this applet.

Matrix Notation
Exercise. The system (which we looked at earlier) x = x + 3y y = xy has general solution x = 3 c 1 e 2t c 2 e 2t y = c 1 e 2t + c 2 e 2t . Re-express this using matrix notation. What are two independent basic solutions? Answer. The matrix form for the system is . x 1 3 x . = . 1 1 y y and the solution can be expressed as x 3 c 1 e 2t c 2 e 2t 3 1 2t 2t = = c1 e + c2 e . c 1 e 2t + c 2 e 2t 1 1 y Two basic independent particular solutions are 3 1 2t 2t e and e . 1 1

Matrix Notation
Exercise. The system (which we looked at earlier) x = x + 3y y = xy has general solution x = 3 c 1 e 2t c 2 e 2t y = c 1 e 2t + c 2 e 2t . Re-express this using matrix notation. What are two independent basic solutions? Try to solve these problems and then look at the solutions.

Linear Phase Portraits: Matrix Entry


Open up the applet Linear phase portraits: Matrix Entry. Unselect companion matrix. This enables you to enter values for a, b, c, and d, between 4 and 4. These numbers are the entries of the matrix A of a rst order linear system, which you see on the left. You can also see some of the solution curves u(t) in the xy-plane. If you click on a point in the plane, the curve through that point will appear. Ignore all the other features of the applet for now; they will be used, and explained, later on. 0.3 0.1 1. Set the matrix to be . This corresponds to the system we 0.2 0.4 looked earlier in this session in the note called describing a rst order system using matrix notation (about farmers Jones and McGregor). Can you see the two rays that were described at the end of that section? What do the other curves look like? Do you notice anything about their behavior near zero? Examine the expression for the solution again, to see whether you can nd an explanation for this. 1 3 2. Set the matrix to be . This corresponds to the system that we 1 1 studied earlier in this session. What do the rays correspond to? What do the other curves look like? Examine the expression for the solution again. Why does one ray point towards zero, and one away from it? By the end of session phase portrait, you should have a thorough understanding of the geometry of these examples. In the mean time, this applet will be a very important tool, notably for illustrations.

The Companion Matrix 1. Introduction


We started the session by using elimination to convert a rst order linear system to an ordinary differential equation in one of the variables. In this note well see how to go the other way. That is, to convert a second order ODE to a 2 2 system of rst order equations. Looking at a second order system this way gives us an important way to visualize the second order equation. Since elimination takes us in the other direction we might call this process anti-elimination. This term is not entirely standard, but it will serve us nicely. We wont cover it in this course, but every method we saw for numerical solutions to rst order ODEs goes through without change to systems of rst order equations. Thus, anti-elimination allows us to use numerical techniques on second order ODEs. Indeed it can be used to convert any order ODE to a system of rst order equations.

2. Anti-elimination and the Companion Matrix


Suppose we have a second order homogeneous linear equation, say

..

x + bx + kx = 0.

(1)

We can derive a rst order linear system from this, by using the following trick. Introduce a second variable dened by y=x Substituting y = x and y = x into equation (1) we get y + by + kx = 0 We now have a rst order system x = y . y = kx by The corresponding coefcient matrix is 0 1 A= . k b

..

y = kx by.

The Companion Matrix

OCW 18.03SC

This is called the companion matrix of the equation (1). In this case, the x (t) . solution vector u(t) = . It records both the solution to (1) and its x (t) derivative. Example 1. Consider the equation x x + 5 4 x = 0. The companion matrix 0 1 is . 5/4 1 What do solutions of this system look like? The characteristic polynomial of the second order equation is p(s) = s2 s + 5/4 = (s (1/2))2 + 1. So, the roots are r = (1/2) i. From unit 2, the general solution in amplitudephase form is given by x (t) = Cet/2 cos(t ), where C and are constants. These oscillate under an exponentially growing envelope. The derivative does the same, but is off phase. This means . that the trajectory traced out by ( x, y) = ( x, x ) is an expanding spiral. Example 2. (Elimination followed by anti-elimination) Earlier in this session, we learned how to solve systems by elimination. What happens when we do elimination followed by anti-elimination? Let us re-visit the example about Romeo and Juliet. The second order system describing their mutual feelings was R = 1 R+J 4 17 3 J = R + J 16 4 (2) (3)

..

Let us rst eliminate J , to get a second order equation in R. From (2), J = R (1/4) R. Substituting this into (3) gives 1 17 3 3 R R = R + R R 4 16 4 16

5 R R R = 0. 4

(4)

(We saw this equation in example 1.) Applying anti-elimination gives R = Y =


5 4R

+ Y.

The Companion Matrix

OCW 18.03SC

This is a different system from the one we started with. The companion matrix of the ODE (4) is different from the original matrix associated to the system.

Companion Matrices
Open up the applet Linear phase portrait: Matrix entry again and uncheck companion matrix. Lets see what happens to Romeo and Juliet. Enter the matrix corresponding to their system: 0.25 1 A= . 1.125 0.75 What do the trajectories look like? How can they be interpreted? Remember that x corresponds to R, and y to J . Let us start at (.5,0): Romeo is fond of Juliet, but she is neutral towards him. However, she does notice that he is fond of her and this makes her somewhat hostile. As she becomes more distant, his affection wanes. Eventually, he is neutral and she really doesnt like him: the trajectory is (roughly) at (0,-1). This continues; presently he stays away from her, and this very fact makes her more interested. She warms to him; he notices, and R , while still negative, start to increase. Eventually she is neutral and the trajectory crosses the x-axis again, around (-2.4,0). He then starts to feel better towards her, but still stays away; now both his attitude and hers cause her to feel progressively more well disposed towards him. This causes him to continue to warm to her. Following this around, you wind up at J = 0 again, but now R has increased: it is already outside the applets screen. This is a cyclical evolution, but with each cycle the intensity of feelings increases. We all know the sad outcome. Now check companion matrix again. This corresponds to doing elimination followed by anti-elimination, as in example 2 in the note on the companion matrix. What do the trajectories look like? This should conrm the answer that we obtained analytically for example 1 in section text: companion matrix. How did the trajectories change when you checked companion matrix? Note that x is still R, but y is now R , not J . The fact that the pictures did not change dramatically is no accident. We will learn why in the phase portraits session. Play around with the applet a bit more, entering some other systems and then clicking companion matrix (thus performing elimination followed by anti-elimination). What sorts of things do you notice?

Elimination Followed by Anti-elimination


Quiz: Elimination converts the system x = 6 x + 5y y = to the ODE x 8x + 7x = 0. What is the coefcient matrix of the system that anti-elimination converts it back to? Choices: 1 2 a. 6 5 6 5 b. 1 2 0 c. 7

x + 2y

..

1 8

d. None of these. Answer: (c) 0 1 7 8 .

Note that neither a. nor b. has the form of a companion matrix.

Elimination Followed by Anti-elimination


Quiz: Elimination converts the system x = 6 x + 5y y = to the ODE x 8x + 7x = 0. What is the coefcient matrix of the system that anti-elimination converts it back to? Choices: 1 2 a. 6 5 6 5 b. 1 2 0 c. 7

x + 2y

..

1 8

d. None of these. Pick what you think is the correct choice and then look at the answer.

Elimination Followed by Anti-elimination


Quiz: Elimination converts the system x = 6 x + 5y y = to the ODE x 8x + 7x = 0. What is the coefcient matrix of the system that anti-elimination converts it back to? Think about your answer and then look at the choices.

x + 2y

..

Matrix Methods: Eigenvalues and Normal Modes


In the previous session, we learned how to solve rst order systems by elimination; we presented out results using vectors and matrices. This leads to a new approach, pursued in this session; it turns out the system can be largely understood by examining features of the coefcient matrix, notably its eigenvalues and the corresponding eigenvectors. These are terms belonging to the eld of linear algebra; the whole session is fairly algebra-heavy, so we start out by building up some background knowledge. Once this is in place, we develop our approach rst studying a long example, then the general case. It turns out that this splits into subcases classied by features of the eigenvalues of the coefcient matrix: these can be real and distinct; complex; real and repeated. Each case will be accompanied with a worked example. This session is a quite long, probably close to two normal sessions in length. This is because we cover all the possible cases: real, complex and repeated eigenvalues. Please dont rush through it. Take your time and learn this well. Eigenvalues and eigenvectors are not only the key idea in the rest of this unit they are also ubiquitous throughout math, science and engineering.

Vectors and Matrices: Homogeneous Systems


This is meant as a follow-up on the review of vectors and matrices in the previous session.

1. More on matrices
Associated with every square matrix A is a number, written | A| or |det( A)| called the determinant of A. For these notes, it will be enough if you can calculate the determinant of 2 2 matrices, which is as follows: a b det = ad bc c d The trace of a square matrix A is the sum of the elements on the main diagonal; it is denoted tr ( A): a b tr = a+d c d Remark. Theoretically, the determinant should not be confused with the matrix itself. The determinant is a number, the matrix is the square array. But, everyone puts vertical lines on either side of the matrix to indicate its determinant, and then uses phrases like the rst row of the determinant, meaning the rst row of the corresponding matrix. An important formula which everyone uses and no one can prove is det( A B) = det A det B. (1)

2. Homogeneous 2 2 systems
Matrices and determinants were originally invented to handle, in an efcient way, the solution of a system of simultaneous linear equations. This is still one of their most important uses. We give a brief account of what you need to know for now. We will restrict ourselves to square 2 2 homogeneous systems; they have two equations and two variables (or unknowns, as they are frequently called). Our notation will be: A = ( aij ), a square 2 2 matrix of constants,
T

x = ( x1 , x2 ) ,

a column vector of unknowns;

Vectors and Matrices: Homogeneous Systems

OCW 18.03SC

then the square system a11 x1 + a12 x2 = 0 a21 x1 + a21 x2 = 0

can be abbreviated by the matrix equation A x = 0. (2)

This always has the solution x = 0, which we call the trivial solution. The question is: when does it have a nontrivial solution? Theorem. Let A be a square matrix. The equation Ax = 0 has a nontrivial solution

det A = 0

(i.e., A is singular). (3)

We will use this, but not prove it in this course.

3. Linear independence of vectors


Conceptually, linear independence of vectors means each one provides something new to the mix. For two vectors this just means they are not zero and are not multiples of each of other. Example 1. (1, 2) and (3, 4) are linearly independent. Example 2. a = (1, 2) and b = (2, 4) are linearly dependent because b is a multiple of a. Notice that if we take linear combinations then b doesnt add anything to the set of vectors we can get from a alone. Example 3. a = (1, 2) and b = (0, 0)are linearly dependent because b is a multiple of a, i.e., b = 0 a. Determinantal criterion for linear independence Let a = ( a1 , a2 ) and b = (b1 , b2 ) be 2-vectors, and A the square matrix having these vectors for its rows (or columns). Then a, b are linearly independent

det A = 0.

(4)

Let us re-visit our previous examples. 1 2 Examples. 1. det = 4 6 = 2 = 0. Therefore, (1, 2) and (3, 4) 3 4 are linearly independent. 2

Vectors and Matrices: Homogeneous Systems

OCW 18.03SC

1 2 2. det = 1 4 2 2 = 0. Therefore, (1, 2) and (2, 4) are linearly 2 4 dependent. 1 2 3. det = 1 0 2 0 = 0. Therefore, (1, 2) and (0, 0) are linearly 0 0 dependent. Remark. The theorem on square homogeneous systems (3) follows from this criterion. We will prove neither. Two linearly independent 2-vectors v1 and v2 form a basis for the plane: every 2-vector w can be written as a linear combination of v1 and v2 . That is, there are scalars c1 and c2 such that c1 v1 + c1 v2 = w Remark. All of the notions and theorems mentioned in this section generalize to higher n (and a larger collection of vectors), though we will not need them.

Linear Algebra
1. Compute of the following matrices. determinants 1 2 a b 1 2 a) b) c) . 3 4 c d 2 4 Answer. a) -2 b) ad bc c) 0.

2. Find all solutions to 0 for Ax = 1 2 1 2 a) b) . 2 4 3 4 Answer. a) All multiples of (2, 1)T . b) 0 (zero-vector) only. 3. Which of the following pairs of vectors are linearly independent? a) (1, 0) and (1, 1) b) (2, 5) and (1, 3) c) (1, 3) and (2, 6)? Answer. a) and b), but not c): The pairs in (a) and (b) are not multiples of each other. In (c) (2, 6) = 2(1, 3).

Linear Algebra
1. Compute of the following matrices. determinants 1 2 a b 1 2 a) b) c) . 3 4 c d 2 4 2. Find all solutions to 0 for Ax = 1 2 1 2 a) b) . 2 4 3 4 3. Which of the following pairs of vectors are linearly independent? a) (1, 0) and (1, 1) b) (2, 5) and (1, 3) c) (1, 3) and (2, 6)?

Motivation and Derivation: Worked Example


Consider the system we studied in several examples in the opening session of unit 4: x 1 3 x = . y 1 1 y We are now going to show a new method of solving this system, which makes use of the matrix form for writing it. Recall that two modal solutions to the system are 3 1 2t 2t e and e . 1 1 Based on this, our new method is to look for solutions of the form x a1 t =e y a2

(1)

where a1 , a2 and are unknown constants. We substitute this into the system to determine what these unknown constants should be. This gives a1 1 3 a1 et = et (2) a2 1 1 a2 We can cancel the factor et from both sides, getting a1 1 3 a1 = a2 1 1 a2

(3)

This is a matrix equation for the three unknowns. It is not very clear how to solve it. When faced with equations in unfamiliar notation, a reasonable strategy is to rewrite them in more familiar notation. If we try this, we get the pair of equations a1 = a1 + 3 a2 a2 = a1 a2 . Technically speaking, these are a pair of nonlinear equations in three variables. The trick in solving them is to look at them as a pair of linear equations in the unknowns ai , with viewed as a parameter. If we think of them this way, it immediately suggests writing them in standard form

(1 ) a1 + 3 a2 a1 + ( 1 ) a2

0 = 0.

(4)

Motivation and Derivation: Worked Example

OCW 18.03SC

In this form, we recognize them as forming a square system of homogeneous linear equations. According to our theorem on square homogeneous systems they have a non-zero solution for the as if and only if the determinant of coefcients is zero: 1 3 = 0. 1 1 After calculation of the determinant this becomes the equation 2 4 = 0 . The roots of this equation are 2 and 2. What the argument shows is that the equations (4) (and therefore also (2)) have non-trivial solutions for the as exactly when = 2 or = 2. To complete the work, we see that for these values of the parameter , the system (4) becomes respectively

a1 + 3 a2 a1 3 a2 (for = 2)

= =

0 0

3 a1 + 3 a2 a1 + a2

= =

0 0

(5)

(for = 2)

Remark. It is of course no accident that in each case the two equations of the system become dependent, i.e., one is a constant multiple of the other. If this were not so, the two equations would have only the trivial solution (0, 0). All of our effort has been to locate the two values of for which this will not be so. The dependency of the two equations is thus a check on the correctness of the value of . To conclude, we solve the two systems in (5). This is best done by assigning the value 1 to one of the unknowns, and solving for the other. First try a1 = 1; if that does not work (in which case, the solution to (5) will have a1 = 0), try a2 = 1. We get a1 3 a1 1 = for = 2; = for = 2, a2 1 a2 1 which gives us, in view of (1), the two solutions: 3 1 2t 2t e and e , 1 1 which are essentially the two solutions we had found previously by the method of elimination. 2

Motivation and Derivation: Worked Example

OCW 18.03SC

Remarks. 1. With the elimination method, the basic normal solutions could be multiplied by an arbitrary non-zero constant without changing the validity of the general solution. Here, this corresponds to the fact that we get to select an arbitrary value of one of the as (the other value then being determined). 3. Is there some way of passing from (3) (the point at which we were temporarily stuck) to (4) by using matrices, without writing out the equations separately? The temptation in (3) is to try to combine the two column vectors a by subtraction, but this is impossible as the matrix equation stands. If we rewrite it however as 0 a1 1 3 a1 = , 0 a2 1 1 a2 it now makes sense to subtract the left side from the right. Using the distributive law for matrix multiplication, this becomes 1 3 a1 0 = , 1 1 a2 0 which is just the matrix form for (4). The trick therefore was in (3) to replace the scalar by the diagonal matrix I .

General Case: Eigenvalues and Eigenvectors


We are now ready to tackle the general case of a linear 2 2 system: x = ax + by y = cx + dy , where the a, b, c, d are constants. We will be following exactly the strategy that we laid out in the previous note. These are key concepts for the rest of the unit, and you should take the time to absorb them. We want to learn to write the system efciently in matrix form. So, throughout the derivation, we will give the expanded matrix form of our manipulations on the left, and the abridged form on the right. For example, our system is: . . x a b x . = x = Ax (1) c d y y Following the method in the previous note, we look for solutions to our system having the form x a1 a1 e t = et = x = e t a, y a2 a2 e t where a1 , a2 , and are unknown constants. We substitute this into the system (1) to determine these unknown constants. Since D ( aet ) = aet , we arrive at a1 a b a1 a1 t t t t e = e e a = e A . a2 c d a2 a2 We can cancel the factor et from both sides, getting a1 a b a1 a1 = a = A . a2 c d a2 a2 As it stands, we cannot combine the two sides by subtraction, since the scalar cannot be subtracted from the square matrix on the right. As in the previously worked example, the trick is to replace the scalar by the diagonal matrix I . This gives 0 a1 a b a1 a1 = Ia = A . 0 a2 c d a2 a2

General Case: Eigenvalues and Eigenvectors

OCW 18.03SC

If we now proceed as we did in the example, subtracting the left side from the right one and using the distributive law for matrix addition and multiplication, we get a 2 2 homogeneous linear system of equations: a b a1 0 = ( A I )a = 0/ c d a2 0 Written out without using matrices, the equations are

( a ) a1 + ba2 ca1 + (d ) a2

= 0 = 0.

(2)

According to the theorem on square homogeneous systems this system has a non-zero solution for the as if and only if the determinant of the coefcients is zero, i.e., a b = 0 | A I | = 0. c d Evaluating the determinant we get a quadratic equation in : 2 ( a + d) + ( ad bc) = 0 . Denition. This is called the characteristic equation of the matrix a b A = c d and if often denoted p A (). Its roots 1 and 2 are called the eigenvalues or characteristic values of the matrix A. Remark. In calculating the characteristic equation notice that ad bc = det A a + d = tr A.

Using this, the characteristic equation for a 2 2 matrix A can be written as 2 (tr A) + det A = 0 . In this form, the characteristic equation of A can be written down by inspection; you dont need the intermediate step of writing down | A I | = 0. Remark. Abridged vs. expanded notation In the manipulations above, the matrix notation on the right is compact 2

General Case: Eigenvalues and Eigenvectors

OCW 18.03SC

to write, which makes the derivation look simpler. On the other hand, its chief disadvantage for beginners is that it is very compressed. Practice writing the sequence of matrix equations so you get some skill in using the notation. Until you acquire some condence, keep referring to the writtenout form on the left, so you are sure you understand what the abridged form is actually saying. There are now various cases to consider, according to whether the eigenvalues of the matrix A are: 1. two distinct real numbers, 2. a single repeated real number, 3. a pair of conjugate complex numbers. We begin with the rst case: for the rest of this note, the eigenvalues are two distinct real numbers 1 and 2 .

1. Real distinct eigenvalues


To complete our work, we have to nd the solutions to the system (2) corresponding to the eigenvalues 1 and 2 . Formally, the systems become

( a 1 ) a1 + ba2 ca1 + (d 1 ) a2

= 0 = 0

( a 2 ) a1 + ba2 ca1 + (d 2 ) a2

= 0 = 0

(3)

The solutions to these two systems are column vectors, for which we will typically use v. Denition. The respective solutions a = v1 and a = v2 to the systems (3) are called eigenvectors (or characteristic vectors) corresponding to the eigenvalues 1 and 2 . Remarks. 1. If the work has been done correctly, in each of the two systems in (3), the two equations will be dependent, i.e., one will be a constant multiple of the other. Why? The two values of have been selected so that in each case the coefcient determinant A I will be zero, which means the equations will be dependent. 2. The solution v is determined only up to an arbitrary non-zero constant factor: if v is an eigenvector for , then so it cv, for any real constant c; because of this, the line through v is sometimes called an eigenline. A convenient way of nding the eigenvector v is to assign the value 1 to one of the ai , then use the equation to solve for the corresponding value of the other ai . (First try a1 = 1; if that does not work, then a2 = 1 will.) Once the eigenvalues and their corresponding eigenvectors have been 3

General Case: Eigenvalues and Eigenvectors

OCW 18.03SC

found, we have two independent solutions to the system (1). They are xi 1 t 2 t x1 ( t ) = e v1 , x2 (t) = 2e v2 , where xi ( t ) = . yi Denition. In science and engineering applications, these are usually called the normal modes. Using superposition, the general solution to the system (1) is x = c 1 x 1 + c 2 x 2 = c 1 e 1 t v 1 + c 2 2 e 2 t v 2 . Remarks. 1. The normal nodes often have physical interpretations; this means that it is sometimes possible to nd them just by inspection of the physical problem. 2. In the compact notation, the denitions and derivations are valid for square systems of any size. Thus, for instance, you know how to solve a 3 times3 system, if its eigenvalues turn out to be real and distinct. We wont consider any such systems in these notes, though. We will apply these techniques to a worked example in the next note in this session titled Worked Example: Distinct Real Roots.

Worked Example: Distinct Real Roots


Problem. Find the general solution to u = A u,

where A=

2 1 4 3

Find the solution with initial conditions u(0) = (1, 0)T . Throughout, comments are given in italics. Solution. Step 0. Write down A I Even if you nd the characteristic equation of A using its trace and determinant, you will need this later, for nding eigenvectors. Most students nd it useful to write it down clearly at the start of the question. 1 2 A I = . 4 3 Step 1. Find the characteristic equation of A. We use the method involving the trace and determinant of A. tr( A) = 2 + 3 = 1 det( A) = 2 3 1 (4) = 6 + 4 = 2 Thus p A () = det( A I ) = 2 2. Step 2. Find the eigenvalues of A. These are the roots of the characteristic equation. we complete the square. (We could also have used the quadratic formula.) p A () = ( 1/2)2 9/4. The roots are 1/2 3/2, so 1 = 1 and 2 = 2. Step 3. Find associated eigenvectors. 3a. Eigenvector for 1 . This is vector a = ( a1 , a2 )T that must satisfy 2 + 1 1 a1 0 ( A + I )a = 0 = 4 3+1 a2 0 1 1 a1 0 = 4 4 a2 0 a1 + a2 = 0 4 a1 + a2 = 0

Worked Example: Distinct Real Roots

OCW 18.03SC

Check: one equation is a multiple of the other, as should be the case. This is a good sign. Setting a1 = 1 gives a2 = 1; thus one eigenvector for 1 is (1, 1)T . 3b. Eigenvector for 2 . This is a vector ( a1 , a2 )T that must satisfy: 2 2 1 a1 0 4 a1 + a2 = 0 ( A 2 I )a = 0 = 4 32 a2 0 4 a1 + a2 = 0 Check: one equation is a (trivial) multiple of the other. Setting a1 = 1 gives a2 = 4. Thus, one eigenvector for 2 is (1, 4)T . Step 4. Normal modes and general solution 1 1 t 2 t The normal modes are e and e . 1 4 and the general solution is: 1 1 t 2t u ( t ) = c1 e + c2 e . 1 4 Step 5. Solution matching IC. We solve for c1 and c2 using our initial condition. From our expression for the general solution, u(0) = c1 (1, 1)T + c2 (1, 4)T = (c1 + c2 , c1 + 4c2 )T . Thus the initial condition u(0) = (1, 0)T gives: c1 + c2 = 1 c1 + 4c2 = 0

c2 = 1/3, c1 = 4/3

The solution we were asked for is: 4 1 1 1 u ( t ) = et e 2t . 1 4 3 3

Matrix/Vector Applet
Let us revisit the Matrix/Vector applet. 1 2 1. Set the matrix to . Can you nd a non-zero input vector that 2 4 produces zero output? Whats the determinant, and why does your nding make sense? Find other matrices where this is the case, and some where it is not. 2. In the previous session, we found some cases where the input vector lined up with the output, and noted that scaling the input did not change this. These are eigenvectors, and the line they span is called an eigenline. Note that if the eigenvalue is negative, the input and output vectors are lined up, but in point in opposite directions. Can you nd: a) a matrix with exactly two eigenlines; b) a matrix with exactly one eigenline; c) a matrix with no eigenlines; d) a matrix where all lines are eigenlines? We have already seen some examples, and will see more in the later notes: (a) corresponds to the case of a matrix with two distinct real eigenvalues; (b) to the case of a defective repeated eigenvalue; (c) to the case of complex eigenvalues, and (d) to the case of a complete repeated eigenvalue.

Complex Eigenvalues 1. Complex Eigenvalues


In the previous note, we obtained the solutions to a homogeneous linear system with constant coefcients x = Ax under the assumption that the roots of its characteristic equation

| A I | = 0,
i.e., the eigenvalues of A were real and distinct. In this section we consider what to do if there are complex eigenvalues. Since the characteristic equation has real coefcients, its complex roots must occur in conjugate pairs: = a + bi, = a bi .

Lets start with the eigenvalue a + bi. According to the solution method described in the note Eigenvectors and Eigenvalues, (from earlier in this session) the next step would be to nd the corresponding eigenvector v, by solving the equations

( a ) a1 + ba2 ca1 + (d ) a2

= 0 = 0

for its components a1 and a2 . Since is complex, the ai will also be complex, and therefore the eigenvector v corresponding to will have complex components. Putting together the eigenvalue and eigenvector gives us formally the complex solution x = e(a+bi)t v. (1)

Naturally, we want real solutions to the system, since it was real to start with. To get them, the following theorem tells us to just take the real and imaginary parts of (1). (This theorem is exactly analogous to what we did with ordinary differential equations.) Theorem. Given a system x = Ax, where A is a real matrix. If x = x1 + i x2 is a complex solution, then its real and imaginary parts x1 , x2 are also solutions to the system.

Complex Eigenvalues

OCW 18.03SC

Proof. Since x1 + i x2 is a solution, we have

( x1 + i x2 ) = A ( x1 + i x2 ) = A x1 + i A x2 .
Equating real and imaginary parts of this equation,
x1 = A x1 , x2 = A x2 ,

which shows exactly that the real vectors x1 and x2 are solutions to x = Ax . Example. Find the corresponding two real solutions to x = Ax if a complex eigenvalue and corresponding eigenvector are i = 1 + 2i , v = . 2 2i Solution. First write v in terms of its real and imaginary parts: 0 1 v = +i 2 2 The corresponding complex solution x = et v to the system can then be written 0 1 t x = e cos(2t) + i sin(2t) +i . 2 2 Now, using the theorem, the real and imaginary parts of x are 0 1 sin(2t) t t x1 = e cos(2t) i sin(2t) = e , 2 2 2 cos(2t) + 2 sin(2t) 1 0 cos(2t) t t x2 = e cos(2t) i sin(2t) = e . 2 2 2 cos(2t) + 2 sin(2t) These are two distinct real solutions to the system. In general, if the complex eigenvalue is a + bi, to get the real solutions to the system, we write the corresponding complex eigenvector v in terms of its real and imaginary part: v = v1 + i v2 , where v1 , v2 are real vectors;

(study carefully in the example above how this is done in practice). Then we substitute into (1) and calculate as in the example: x = e at (cos(bt) + i sin(bt)) (v1 + i v2 ), 2

Complex Eigenvalues

OCW 18.03SC

so that the real and imaginary parts of x give respectively the two real solutions x1 = e at (v1 cos(bt) v2 sin(bt)) , (2) x2 = e at (v1 sin(bt) + v2 cos(bt)) . These solutions are linearly independent: they are two truly different solutions. The general solution is given by their linear combinations c1 x1 + c2 x2 . Remarks 1. The complex conjugate eigenvalue a bi gives up to sign the same two solutions x1 and x2 . 2. The expression (2) was not written down for you to memorize, learn, or even use; the point was just for you to get some practice in seeing how a calculation like the one in our example looks when written out in general. To actually solve ODE systems having complex eigenvalues, imitate the procedure in the following example.

2. Worked Example
1 2 Problem. Solve u = Au, where A = . 2 1 Comments are given in italics; the steps initially follow those in section worked example: real distinct eigenvalues, then diverge. Solution. Step 0. Write down A I : A I = 1 2 2 1 .

Step 1. Find the characteristic equation of A. We use the method involving the trace and determinant of A. tr( A) = 1 + 1 = 2; det( A) = 1 1 2 (2) = 5. Thus p A () = det( A I ) = 2 tr( A) + det A = 2 2 + 5. Step 2. Find the eigenvalues of A. We complete the square. p A () = 2 2 + 5 = ( 1)2 + 4. The roots are 1 2i, so 1 = 1 + 2i and 2 = 1 2i. Step 3. Find the eigenvector associated to one eigenvalue The eigenvalues are complex, so well only need one eigenvector. We look for the eigenvector a1 for 1 = 1 + 2i. a2 3

Complex Eigenvalues

OCW 18.03SC

It must satisfy:

( A ( 1 + 2i ) I ) a = 0

2i 2 a1 0 = 2 2i a2 0 2ia1 + 2a2 = 0 2a1 2ia2 = 0.

You should check that these two equations are equivalent. This gives a1 + ia 2 = 0. Pick a1 = 1, this implies a2 = i. Thus an eigenvec1 tor for 1 is v1 = . i Step 4. Find the real and imaginary parts of solution associated to 1 The solution we associated to 1 is 1 1 1 t ( 1 + 2i ) t t e v1 = e = e (cos(2t) + i sin(2t)) i i This has real and imaginary parts: cos(2t) t x1 = e , sin(2t)

x2 = e

sin(2t) cos(2t)

If you are confused by steps 3 or 4, you should read over the note again. Step 5. General solution. cos(2t) sin(2t) t t c1 e + c2 e . sin(2t) cos(2t)

Repeated Eigenvalues 1. Repeated Eignevalues


Again, we start with the real 2 2 system x = A x.

(1)

We say an eigenvalue 1 of A is repeated if it is a multiple root of the characteristic equation of A; in our case, as this is a quadratic equation, the only possible case is when 1 is a double real root. We need to nd two linearly independent solutions to the system (1). We can get one solution in the usual way. Let v1 be an eigenvector corresponding to 1 . This is found by solving the system

( A 1 I ) a = 0.

(2)

This gives the solution x1 = e1 t v1 to the system (1). Our problem is to nd a second solution. To do this we have to distinguish two cases, called complete and defective. The rst one is easier, especially in the 2 2 case. A. The complete case. Still assuming 1 is a real double root of the characteristic equation of A, we say 1 is a complete eigenvalue if there are two linearly independent eigenvectors v1 and v2 corresponding to 1 ; i.e., if these two vectors are two linearly independent solutions to the system (2). In the 2 2 case, this only occurs when A is a scalar matrix that is, when A = 1 I . In this case, A 1 I = 0, and every vector is an eigenvector. It is easy to nd two independent solutions; the usual choices are 1 0 1 t 1 t e and e . 0 1 So the general solution is 1 0 c1 1 t 1 t 1 t c1 e + c2 e = e . 0 1 c2 Of course, we could choose any other pair of independent eigenvectors to generate the solutions, e.g., 5 1 e 1 t and e1 t . 1 1

Repeated Eigenvalues

OCW 18.03SC

Remark. For n = 3 and above the situation is more complicated. We will not discuss it here. The interested reader can consult, for instance, the textbook by Edwards and Penney. B. The defective case. If the eigenvalue is a double root of the characteristic equation, but the system (2) has only one non-zero solution v1 (up to constant multiples), then the eigenvalue is said to be incomplete or defective and x1 = e1 t v1 is the unique normal mode. However, a second order system needs two independent solutions. Our experience with repeated roots in second order ODEs suggests we try multiplying our normal solution by t. It turns out this doesnt quite work, but it can be xed as follows: a second independent solution is given by x 2 = e 1 t ( t v 1 + v 2 ). (3) where v2 is any vector satisfying

( A 1 I ) v2 = v1 .
(You can easily, if tediously, check by substitution that this does give a solution. You need to remember that Av1 = 1 v1 .) Fact. The equation for v2 is guaranteed to have a solution, provided that the eigenvalue 1 really is defective. When solving for v2 = (b1 , b2 )T , try setting b1 = 0, and solving for b2 . If that does not work, try setting b2 = 0 and solving for b1 . Remarks 1. Some people do not bother with (3). When they encounter the defective case (at least when n = 2), they give up on eigenvalues, and simply solve the original system (1) by elimination. 2. Although we will not go into it in this course, there is a well developed theory of defective matrices which gives insight into where this formula comes from. You will learn about all this when you study linear algebra. We will now do a worked example.

2. Worked example: Defective Repeated Eigenvalue


Problem. Solve u = Au, where A = Comments are given in italics. Solution. Step 0. Write down A I : A I = 2

2 1 1 0

2 1 1

Repeated Eigenvalues

OCW 18.03SC

Step 1. Find the characteristic equation of A: tr( A) = 2 + 0 = 2, det( A) = 2 0 1 (1) = 1. Thus, p A () = det( A I ) = 2 tr( A) + det( A) = 2 + 2 + 1 = 0. Step 2. Find the eigenvalues of A. The characteristic polynomial factors: p A () = ( + 1)2 . This has a repeated root, 1 = 1. As the matrix A is not the identity matrix, we must be in the defective repeated root case. Step 3. Find an eigenvector. This is vector v1 = ( a1 , a2 )T that must satisfy: 2 + 1 1 a1 0 ( A + I ) v1 = 0 = 1 1 a2 0 1 1 a1 0 = . 1 1 a2 0 Check: this gives two identical equations, which is good. Setting a1 = 1 gives a2 = 1. Thus, The equation is a1 + a2 = 0. one eigenvector for 1 is v1 = (1, 1)T . All other eigenvectors for 1 are multiples of this. Step 4. Find v2 : This vector v2 = (b1 , b2 )T must satisfy

( A 1 I ) v2 = v1

1 1 1 1

b1 b2

1 1

b1 + b2 = 1.

Setting b1 = 0 gives b2 = 1; so v2 = (0, 1)T is suitable. Step 5. General solution. The general solution is 1 1 0 1 1+t t t t t u ( t ) = c1 e + c2 (te +e = e c1 + c2 . 1 1 1 1 1 + 2t

Introduction
Up to now we have handled systems analytically, concentrating on a procedure for solving linear systems with constant coefcients. In this session, we consider methods for sketching graphs of the solutions. The emphasis is on the word sketching. Computers do the work of drawing reasonably accurate graphs. Here we want to see how to get quick qualitative information about the graph, without having to actually calculate points on it. These graphs of the solutions (also called the trajectories of the system) are called of phase portraits. In this session we consider 2 2 linear homogeneous systems x = Ax. In a later session we extend this program, known as the phase-plane analysis, to more general non-linear 2 2 DE systems. The analysis of the linear case will be the foundation for the more general program, so it is very important that we understand this case well. For that reason, this session is somewhat long, we wish to have the linear case well worked out in detail so that we can refer back to it later as needed. The Eigenvalues Rule There are a lot of details in this session. The one key fact tying them all together is the eignenvalues rule: We classify the linear phase portraits according to the eigenvalues of the matrix A.

The Phase Plane


The sort of system for which we will be trying to sketch solutions can be written in the form x = ax + by a b x = Ax, where A = , (1) y = cd + dy c d where a, b, c, d are constants. A solution of this system has the form (we write it two ways) x (t) x = x (t) x(t) = , y(t) y = y ( t ).

(2)

It is a vector function of t whose components satisfy the system (1) when they are substituted in for x and y. In general, you learned in 18.02 and physics that such a vector function describes a motion in the xy-plane; the equations in (2) tell how the point ( x, y) moves in the xy-plane as the time t varies. The moving point traces out a curve called the trajectory of the solution (2). The xy-plane itself is called the phase plane for the system (1). We show a sketch of a trajectory at right. Notice the arrow is used to indicate the direction of increasing time. We use the term phase portrait to mean the graphs of enough trajectories to give a good sense of all the solutions to the system (1)

1. Critical Points
Denition. A critical point is a point where the derivatives are 0. Therefore a point ( x0 , y0 ) is a critical point of the system (1) if x x0 0 =A = . y y0 0 The equations of the system (1) show this is equivalent to x = x0 , y = y0 is a (constant) solution to (1).

Critical points are the key to our qualitative view of systems. We classify the linear systems by their behavior near critical points. For the linear system constant coefcient system (1) there is always a critical point at (0, 0). If the matrix A is invertible then this is the only critical point.

The Phase Plane

OCW 18.03SC

2. Sketching Principle
When sketching integral curves for direction elds we saw that integral curves did not cross. For the system (1) we have a similar principle. Sketching Principle. Two trajectories of (1) cannot intersect.

Sketching the Basic Linear Systems


In this note we will only consider linear systems of the form x = Ax. Such a system always has a critical point at the origin. We start by sketching a few of the simple examples, so as to get an idea of the various possibilities for their trajectories. We will also introduce the terminology used to describe the resulting geometric pictures. Example 1. Lets consider the linear system on the left below. Its characteristic equation is 2 1 = 0, so the eigenvalues are 1. It is easy to see its general solution is the one on the right below: x = y 1 1 t ; x = c1 e + c2 et . (1) y = x 1 1 The only critical point of the system is (0, 0). Looking at the general solution in (1), we see that by giving one of the cs the value 0 and the other one the value 1 or 1, we get four easy solutions: 1 1 1 1 et , et , et , et . 1 1 1 1 These four solutions give four trajectories which are easy to plot. Consider the rst, for example. When t = 0, the point is at (1, 1). As t increases, the point moves outward along the line y = x. As t decreases through negative values, the point moves inwards along the line, toward (0, 0). Since t is always understood to be increasing on the trajectory, the whole trajectory consists of the ray y = x in the rst quadrant, excluding the origin (which is not reached in nite negative time), with an outward direction of motion. A similar analysis can be made for the other three solutions; see gure 1 below.

Sketching the Basic Linear Systems

OCW 18.03SC

Figure 1 As you can see, each of the four solutions has as its trajectory one of the four rays. The indicated direction of motion is outward or inward according to whether the exponential factor increases or decreases as t increases. There is even a fth trajectory: the origin itself, which is a stationary point, i.e., a solution all by itself. So the intersecting diagonal lines represent ve trajectories, no two of which intersect. For the other trajectories we can do a little algebra: We have x = c1 e t + c2 e t y = c1 e t c2 e t . This easily gives x2 y2 = 4c1 c2 = a constant, which is the equation of a hyperbola oriented with the axes we sketch in some of the hyperbolas. We know which direction to point the arrows indicating the direction of motion as t increases since they must be compatible with the motion along the rays for by continuity, nearby trajectories must have arrowheads pointing in similar directions. The only possibility therefore is the one shown in gure 1. A linear system whose trajectories show the general features of those in g. 1 is said to be an unstable saddle. It is called unstable because the trajectories go off to innity as t increases (there are three exceptions: what are they?). It is called a saddle because of its general resemblance to the level curves of a saddle-shaped surface in 3-space. Example 2. This time we consider the linear system below since it is decoupled, its general solution (on the right) can be obtained easily by in2

Sketching the Basic Linear Systems

OCW 18.03SC

spection:

x = x y = 2y

x = c1

1 0

+ c2

0 1

e 2t .

(2)

It is immediate that x = c1 et and y = c2 e2t implies y = cx2 . That is, the trajectories are a family of parabolas. Following the same plan as in Example 1, we single out the four solutions 1 1 0 0 t t 2t e , e , e , e 2t . (13) 0 0 1 1 Their trajectories are the four rays along the coordinate axes, the motion being always inward as t increases. Put compatible arrowheads on the parabolas and you get gure 2. A linear system whose trajectories have the general shape of those in g. 2 is called an asymptotically stable node or a sink node. The word node is used when the trajectories have a roughly parabolic shape (or exceptionally, they are rays); asymptotically stable or sink means that all the trajectories approach the critical point as t increases.

Figures 2 and 3. Trajectories from examples 2 and 3 Example 3. versed: This is the same as Example 2, except that the signs are rex = x y = 2y x = c1 1 0 0 1

e + c2

e 2t .

(3)

Sketching the Basic Linear Systems

OCW 18.03SC

The rst order differential equation remains the same, so we get the same parabolas. The only difference in the work is that the exponentials now have positive exponents. The picture remains exactly the same except that now the trajectories are all traversed in the opposite direction away from the origin as t increases. The resulting picture is g. 3, which we call an unstable node or source node. Example 4. A different type of simple system (eigenvalues i) and its solution is x = y sin t cos t ; x = c1 + c2 . (4) y = x cos t sin t For this example lets see a different way of nding the trajectories. Dividing y / x converts to a separable rst order ODE. dy/dt dy x = = , dx /dt dx y x 2 + y2 = c.

The trajectories are the family of circles centered at the origin. To determine the direction of motion, look at the solution in (4) for which c1 = 0, c2 = 1. Notice that it is the reection in the y-axis of the usual (counterclockwise) parametrization of the circle; hence the motion is clockwise around the circle. An even simpler procedure is to determine a single vector in the velocity eld thats enough to determine all of the directions. For example, the velocity vector at (1, 0) is < 0, 1 >= j, again showing the motion is clockwise. (The vector is drawn in on g. 4, which illustrates the trajectories.) This type of linear system is called a stable center . The word stable signies that any trajectory stays within a bounded region of the phase plane as t increases or decreases indenitely. (We cannot use asymptotically stable, since the trajectories do not approach the critical point (0, 0) as t increases.) The word center describes the geometric conguration: it would be used also if the curves were ellipses having the origin as center.

Figures 4, 5 and 6. Trajectories from examples 4 and 5 4

Sketching the Basic Linear Systems

OCW 18.03SC

Example 5. As a last example, a system having a complex eigenvalue = 1 + i is, with its general solution, x = x + y sin t cos t t t x = c1 e + c2 e . (5) y = x y cos t sin t The two fundamental solutions (using c1 = 0 and c1 = 1, and vice-versa) are typical. They are like the solutions in example 4, but multiplied by et . Their trajectories are therefore traced out by the tip of an origin vector that rotates clockwise at a constant rate, while its magnitude shrinks exponentially to 0. In other words, the trajectories spiral in toward the origin as t increases. We call this pattern an asymptotically stable spiral or a sink spiral; see g. 6. (An older terminology uses focus instead of spiral.) To determine the direction of motion, it is simplest to do what we did in the previous example: determine from the ODE system a single vector of the velocity eld. For instance, the system (5) has at (1, 0) the velocity vector i j, which shows that the motion is clockwise. x = x+y For the system , an eigenvalue is = 1 + i, and in (5) y = x + y et replaces et . The magnitude of the rotating vector increases as t increases, giving as pattern an unstable spiral, or source spiral, as in g. 6.

Sketching More General Linear Systems


In the preceding section we sketched trajectories for some particular linear systems. They were chosen to illustrate the different possible geometric pictures. Based on that experience, we can now describe how to sketch the general system x = A x, A = 2 2 constant matrix.

The geometric picture is largely determined by the eigenvalues and eigenvectors of A, so there are several cases. For the rst group of cases, we suppose the eigenvalues 1 and 2 are real and distinct. Case 1. The i have opposite signs: 1 > 0, 2 < 0 ; unstable saddle. Suppose the corresponding eigenvectors are 1 and 2 , respectively. Then four solutions to the system are x = 1 e 1 t , x = 2 e 2 t . (1)

How do the trajectories of these four solutions look? In gure 1 below, the four vectors 1 and 2 are drawn as origin vectors. In gure 2, the corresponding four trajectories are shown as solid lines, with the direction of motion as t increases shown by arrows on the lines. The reasoning behind this is the following. Look rst at x = 1 e1 t . We think of e1 t as a scalar factor changing the length of x; that is as t increases from to , this scalar factor increases from 0 to , since 1 > 0. The tip of this lengthening vector represents the 1 e1 t , which is therefore a ray going out from trajectory of the solution x = 1 . the origin in the direction of the vector Similarly, the trajectory of x = 1 e1 t is a ray going out from the origin in the opposite direction: that of the vector 1 . The trajectories of the other two solutions x = 2 e2 t will be similar, t except that since 2 < 0, the scalar factor e 2 decreases as t increases. Thus the solution vector will be shrinking as t increases. The trajectory traced 2 or 2 , but traversed out by its tip will be a ray having the direction of toward the origin as t increases, getting arbitrarily close but never reaching it in nite time.

Sketching More General Linear Systems

OCW 18.03SC

To complete the picture, we sketch some nearby trajectories. These will be smooth curves generally following the directions of the four rays described above. In example 1 in the previous note they were hyperbolas. In general they are not, but they look something like hyperbolas, and they do have the rays as asymptotes. They are the trajectories of the solutions x = c1 1 e1 t + c2 2 e 2 t , for different values of the constants c1 and c2 . (2)

Figures 1 and 2. Trajectories for case 1: saddle Case 2. 1 and 2 are distinct and negative: say 1 < 2 < 0; asymptotically stable (sink) node Formally, the solutions (1) are written the same way and we draw their trajectories just as before. The only difference is that now all four trajectories are represented by rays coming in towards the origin as t increases, since both of the i are negative. The four trajectories are represented as solid lines in gure 3, on the next page. The trajectories of the other solutions (2) will be smooth curves which generally follow the four rays. In the corresponding example 2 from the previous note, they were parabolas; here too they will be parabola-like, but this does not tell us how to draw them, so a little more thought is needed. The parabolic curves will certainly come in to the origin as t increases, but tangent to which of the rays? Briey, the answer is this: Node-sketching principle. Near the origin, the trajectories follow the ray attached to the i nearer to zero; far from the origin, they follow (i.e. are roughly parallel to) the ray attached to the i further from zero. You need not memorize the above. Instead learn the reasoning on which it is based, since this type of argument will be used over and over in science and engineering work having nothing to do with differential equations. Since we are assuming 1 < 2 < 0, it is 2 which is closer to 0. We want to know the behavior of the solutions near the origin and far from the 2

Sketching More General Linear Systems

OCW 18.03SC

origin. Since all solutions are approaching the origin, near the origin corresponds to large positive t (we write t 1); and far from the origin corresponds to large negative t (written t 1). As before, the general solution has the form x = c1 1 e1 t + c2 2 e 2 t , 1 < 2 < 0. (3)

If t 1, then x is near the origin, since both terms in (3) are small. However, the rst term is negligible compared with the second: for since 1 2 < 0, we have e 1 t = e(1 2 )t 0, e 2 t t1. (4)

Thus if 1 < 2 < 0 and t 1, we can neglect the rst term of (3), getting x c2 2 e 2 t for t 1

(x near the origin),

which shows that x(t) follows the ray corresponding to the the eigenvalue 2 closer to zero. Similarly, if t 1, then x is far from the origin since both terms in (3) are large. This time the ratio in (4) is large, so that it is the rst term in (3) that dominates the expression, which tells us that x c1 1 e 1 t for t 1

(x far from the origin).

This explains the reasoning behind the node-sketching principle in this case. Some of the trajectories of the solutions (3) are sketched in dashed lines in gure 3, using the node-sketching principle, and assuming 1 < 2 < 0.

Figures 3, 4 and 5. Trajectories for source and sink nodes Case 3. 1 and 2 are distinct and positive: say 1 > 2 > 0 (source) node 3 unstable

Sketching More General Linear Systems

OCW 18.03SC

The analysis is like the one we gave above. The direction of motion on the four rays coming from the origin is outwards, since the i > 0. The node-sketching principle is still valid and the reasoning for it is like the reasoning in case 2. The resulting sketch looks like the one in g. 5. Case 4. Eigenvalues are pure imaginary: = bi, center b > 0 stable

Here the solutions to the linear system have the form x = c1 cos bt + c2 sin bt, c1 , c2 constant vectors . (5)

(There is no exponential factor since the real part of is zero.) Since every solution (5) is periodic, with period 2 /b, the moving point representing it retraces its path at intervals of 2 /b. The trajectories therefore are closed curves; ellipses, in fact; see g. 7. Sketching the ellipse is a little troublesome, since the vectors ci do not have any simple relation to the major and minor axes of the ellipse. For this course, it will be enough if you determine whether the motion is clockwise or counterclockwise. As in example 4 in the previous note, this can be done by using the system x = Ax to calculate a single velocity vector x of the velocity eld. From this the sense of motion can be determined by inspection. The word stable means that each trajectory stays for all time within some circle centered at the critical point. Asymptotically stable is a stronger requirement: each trajectory must approach the critical point (here, the origin) as t . Case 5. The eigenvalues are complex, but not purely imaginary. There are two cases: a bi, a bi, a < 0, b > 0; a > 0, b > 0; asymptotically stable (sink) spiral; unstable (source) spiral.

Here the solutions to the linear system have the form x = e at (c1 cos bt + c2 sin bt), c1 , c2 constant vectors . (6)

They look like the solutions (5), except for a scalar factor e at which either decreases towards 0 as t increases towards as t

( a < 0), or ( a > 0) .

Sketching More General Linear Systems

OCW 18.03SC

Thus the point x travels in a trajectory which is like an ellipse, except that the distance from the origin is steadily shrinking or expanding. The result is a trajectory which does one of the following: spirals steadily towards the origin, spirals steadily away from the origin. (asymptotically stable spiral) : (unstable spiral); a > 0 a<0

The exact shape of the spiral is not obvious and perhaps best left to computers. You should determine the direction of motion by calculating from the linear system x = Ax a single velocity vector x near the origin. Typical spirals are pictured (gs. 7, 8).

Figures 6, 7 and 8. Trajectories centers and spirals Other cases. Repeated real eigenvalue = 0, defective: (incomplete: one independent eigenvector): defective node; unstable if > 0; asymptotically stable if < 0 (g. 9). Repeated real eigenvalue = 0, complete (two independent eigenvectors): star node; unstable if > 0; asymptotically stable if > 0. (g. 10). One eigenvalue = 0. (Picture left for exercises and problem sets.)

Figure 9. Stable and unstable nodes

Figures 10. Star nodes

Summary
In summary, the procedure of sketching trajectories of the 2 2 linear homogeneous system x = Ax, where A is a constant matrix, is the following. Begin by nding the eigenvalues of A. 1. If they are real, distinct, and non-zero: a) nd the corresponding eigenvectors; b) draw in the corresponding solutions whose trajectories are rays. Use the sign of the eigenvalue to determine the direction of motion as t increases; indicate it with an arrowhead on the ray; c) draw in some nearby smooth curves, with arrowheads indicating the direction of motion: (i) if the eigenvalues have opposite signs, this is easy; (ii) if the eigenvalues have the same sign, determine which is the dominant term in the solution for t 1 and t 1, and use this to determine which rays the trajectories are tangent to, near the origin, and which rays they are parallel to, away from the origin. (Or use the node-sketching principle.) 2. If the eigenvalues are complex, a bi, the trajectories will be a) ellipses if a = 0 b) spirals if a = 0; inward if a < 0, outward if a > 0.

In all cases, determine the direction of motion by using the system x = Ax to nd one velocity vector. 3. The details in the other cases (eigenvalues repeated, or zero) will be left as exercises using the reasoning in this note.

Trace-Determinant Diagram
Recall that the characteristic polynomial of a square matrix A is dened to be p() = det( A I ). a b a b For a 2 2 matrix A, A = , we have p() =| | c d c d = 2 ( a + d) + ( ad bc). If we now recall the denitions of trace and determinant for a 2 2 matrix A from the linear algebra and matrix review given at the end of the previous session on Matrix Methods, namely tr A = a + d and det A = ad bc, we see that we can write p() = 2 tr A + det A. Now if we use the abbreviations T = tr A and D = det A, we can write the characteristic polynomial as p() = 2 T + D. The eigenvalues are the roots of p(), so the quadratic formula immediately gives us that the eigenvalues will be real if and only if the discriminant T 2 4 D > 0 and complex if and only if T 2 4 D < 0. The separating curve D = T 2 /4 is shown on the trace-determinant graph below. Then looking at the full quadratic formula for p() = 0, = T 2T 4d , we can determine the conditions for the signs in the case of real eigenvalues and also the signs of the real part for the complex case. We leave this as an exercise (not difcult and highly recommended) for the reader. The results are as follows: 1. If D < 0, the eigenvalues are real and of opposite sign, and the phase portrait is a saddle (which is always unstable). 2. If 0 < D < T 2 /4, the eigenvalues are real, distinct, and of the same sign, and the phase portrait is a node, stable if T < 0, unstable if T > 0. 3. If 0 < T 2 /4 < D, the eigenvalues are neither real nor purely imaginary, and the phase portrait is a spiral, stable if T < 0, unstable if T > 0. Sketching this information in on the T D graph gives the trace-determinant diagram below. The boundary cases, where the either inequalities become equality and/or

Trace-Determinant Diagram

OCW 18.03SC

where T = 0 or D = 0, are called the borderline cases." We will discuss these further in a later session. The Mathlets Linear Phase Portraits: Cursor Entry and Linear Phase Portraits: Matrix Entry will allow you to explore the classication of the solution types provided by the T D diagram interactively, and are highly recommended. D = det A

center spiral source

spiral sink

defective or star node

nodal sink

nodal source T = tr A degenerate saddle

MIT OpenCourseWare http://ocw.mit.edu

Fall 2011

18.03SC Differential Equations

For information about citing these materials or our Terms of Use, visit: http://ocw.mit.edu/terms.

Appendix: A Computer-Generated Portrait Gallery


There are a number of public-domain computer programs which produce phase portraits for 2 2 autonomous systems. One has the option of displaying the trajectories with or without the little" arrows of the vector direction eld. We rst give an example where the direction eld is included, and then a portrait gallery with only the tractories themselves. In the pictures which illustrate these cases, only a few trajectories are shown, but these are sufcent to shown the qualitative behavior of the system. A much richer understanding of this gallery can be achieved using the Mathlets Linear Phase Portraits: Cursor Entry and Linear Phase Portraits: Matrix Entry.
3 2 1
G

Example. (direction eld included). 3 2 1 1 2 x y 1 = . y x 2 General solution: x = c1 cos t + c2 sin t, y = c1 sin t + c2 cos t 3 As we saw before, the trajectories are circles. We know the direction Vector field: x' = y, y' = x is clockwise by looking at a single tangent vector.
G

x
3

The portrait gallery. 1. A has real eigenvalues with two independent eigenvectors. Let 1 , 2 be the eigenvalues and v1 and v2 the corresponding eigenvectors. general solution to () is x = c1 e1 t v1 + c2 e2 t v2 . y

i) 1 > 2 > 0. Unstable nodal source. As t the term e1 t v1 dominates and x . As t the term e2 t v2 dominates and x 0.

Appendix: A Computer-Generated Portrait Gallery

OCW 18.03SC

ii) 1 < 2 < 0. Stable nodal sink (asymptotically stable). (Simply reverse the arrows on case (i).) As t 0 the term e2 t v2 dominates and x . As t the term e1 t v1 dominates and x .

iii) 1 > 0 > 2 . Saddle, unstable. As t the term e1 t v1 dominates and x . As t the term e2 t v2 dominates and x .

y iv) 1 = 0 > 2 . Line of critical points. The critical points are not isolated they lie on the line through 0 with direction v1 . x = c 1 v 1 + c 2 e 2 t v 2 As t x c1 v1 along a line parallel to v2 .

v) 1 = 0 < 2 . Line of critical points. (Simply reverse the arrows in case (iv).) The critical points are not isolated they lie on the line through 0 with direction v1 . x = c 1 v 1 + c 2 e 2 t v 2 As t x along a line parallel to v2 .

Appendix: A Computer-Generated Portrait Gallery

OCW 18.03SC y

vi) 1 = 2 > 0. Star nodal source (unstable). Let = 1 = 2 . Two independent eigenvectors A is a scalar matrix x = et c. That is all trajectories are straight rays. As t x along a line from 0. As t x 0

y vii) 1 = 2 < 0. Star nodal sink (asymptotically stable). (Simply reverse the arrows in case (vi).)

viii) 1 = 2 = 0. Non-isolated critical points. Every point is a critical point, every trajectory is a point. 2. Real defective case (repeated eigenvalue, only one eigenvector). Let be the eigenvalue and v1 the corresponding eigenvector. Let v2 be a generalized eigenvector associated with v1 .

general solution to () is x = et (c1 v1 + c2 (tv1 + v2 )).


i) > 0. Defective source node (unstable). As t the term tv1 dominates and x . As t the term tv1 dominates and x 0.

y ii) < 0. Defective sink node (asymptotically stable). (Simply reverse the arrows in case (i).)

Appendix: A Computer-Generated Portrait Gallery y iii) = 0. Line of critical points. x = c1 v1 + c2 ( t v1 + v2 ). Trajectories are parallel to v1 .

OCW 18.03SC y

3. Complext roots (pair of complex conjugate eigenvalues and vectors). Let an eigenvalue be = + i, with eigenvector v + i w. x = et (c1 (cos t v sin t w) + c2 (sin t v + cos t w)). The sines and cosines in the solution will cause the trajectory to spiral around the critical point. i) Re() > 0 (i.e. > 0). Spiral source (unstable). Trajectories can spiral clockwise or counterclockwise. As t , x . As t , x 0. ii) Re() < 0 (i.e. < 0). Spiral sink (asymptotically stable). (Reverse arrows from case (i).) Trajectories can spiral clockwise or counterclockwise. As t , x 0. As t , x . iii) Re() = 0 (i.e. = 0). Stable center. Trajectories can turn clockwise or counterclockwise. As t , x follows an ellipse. y

For the complex case you can nd the direction of rotation by checking the tangent vector at one point.

Appendix: A Computer-Generated Portrait Gallery Example. A = a spiral source. The tangent vector at the point x0 = 1 0 is A x0 = 2 3 2 3 3 2

OCW 18.03SC

has eigenvalues 2 3i. the critical point is .

spiral is clockwise.

Linear Phase Portraits: Cursor Entry


Open up the applet Linear phase portraits: Cursor Entry. This applet is similar to Linear phase portraits: Matrix Entry. In this exploration we want you to look at how the different regions in the trace-determinant diagram correspond to their phase portraits and eigenvalues. As usual, rst play with the applet a little to get familiar with it. Pay attention to the color coding. Without using the applet, for each different region in the diagram answer the following questions: 1. Are the eigenvalues real or complex? 2. What is the sign on the real part of the eigenvalues? 3. What will the phase portrait look like? Now check your answers by clicking in each region. Answer the same questions for each of the red boundary lines separating the different regions. A small movement in the trace-determinant plane should result in a small change in the eigenvalues and in the phase portrait. Verify this by dragging the cross-hairs around the diagram. As you drag the cross-hairs across a boundary line the phase portrait will switch types. If you do it slowly you can see how the portrait on the boundary is right between the types on either side. For example, if you drag it from the second to the third quadrant across the horizontal (trace) axis, the eigenvalues change from both negative to one positive and one negative. Right on the boundary one of them is zero. Likewise, the phase portraits change from turning in towards the origin to turning away from the origin. With the cursor right on the boundary they dont turn, they are just straight lines heading towards the line of critical points corresponding to the eigenvector with eigenvalue zero.

Introduction
In this session we learn general results about the solutions of any n n linear DE system (not necessarily constant coefcient). First we will learn the some general theory for linear systems. This will be familiar to you from our study of linear ODEs. After that we will discuss the fundamental matrix which is an efcient way to package all the solutions of a linear system of differential equations. It will also allow us to use matrix algebra when working with such systems. We will use the fundamental matrix to derive the variation of parameters formula, which is used to solve an inhomogeneous system with arbitrary output. Next we will dene the matrix exponential and express the fundamental matrix in terms of it. The matrix exponential has many nice theoretical properties. Finally we will recast what weve done in the language of decoupling which is commonly used by engineers. This session is long and covers many important topics. It is something of a sidelight in this course and will not be used in subsequent sessions.

General Linear ODE Systems and Independent Solutions


We have studied the homogeneous system of ODEs with constant coefcients, x = A x , (1) where A is an n n matrix of constants (n = 2, 3). We described how to calculate the eigenvalues and corresponding eigenvectors for the matrix A, and how to use them to nd n independent solutions to the system (1). With this concrete experience in solving low-order systems with constant coefcients, what can be said when the coefcients are functions of the independent variable t? We can still write the linear system in the matrix form (1), but now the matrix entries will be functions of t: x = a(t) x + b(t)y x a(t) b(t) x , = . , (2) y = c(t) x + d(t)y y c(t) d(t) y or in more abridged notation, valid for n n linear homogeneous systems, x = A ( t ) x . (3)

Note how the matrix becomes a function of t we call it a matrix-valued function of t, since to each value of t the function rule assigns a matrix: a ( t0 ) b ( t0 ) t0 A ( t0 ) = c ( t0 ) d ( t0 ) In the rest of this chapter we will often not write the variable t explicitly, but it is always understood that the matrix entries are functions of t. We will sometimes use n = 2 or 3 in the statements and examples in order to simplify the exposition, but the denitions, results, and the arguments which prove them are essentially the same for higher values of n. Denition 1 Solutions x1 (t), . . . , xn (t) to (3) are called linearly dependent if there are constants ci , not all of which are 0, such that c1 x1 (t) + . . . + cn xn (t) = 0, If there is no such relation, i.e., if c1 x1 (t) + . . . + cn xn (t) = 0 for all t for all t. (4)

all ci = 0,

(5)

the solutions are called linearly independent, or simply independent.

General Linear ODE Systems and Independent Solutions

OCW 18.03SC

The phrase for all t is often in practice omitted, as being understood. This can lead to ambiguity. To avoid it, we will use the symbol 0 for identically 0, meaning zero for all t; the symbol 0 means not identically 0, i.e., there is some t-value for which it is not zero. For example, (4) would be written c1 x1 (t) + . . . + cn xn (t) 0 . Theorem 1 If x1 , . . . , xn is a linearly independent set of solutions to the n n system x = A(t)x, then the general solution to the system is x = c1 x1 + . . . + cn xn . (6)

Such a linearly independent set is called a fundamental set of solutions. This theorem is the reason for expending so much effort to nd two independent solutions, when n = 2 and A is a constant matrix. In this chapter, the matrix A is not constant; nevertheless, (6) is still true. Proof. There are two things to prove: (a) All vector functions of the form (6) really are solutions to x = A x. This is the superposition principle for solutions of the system; its true because the system is linear. The matrix notation makes it really easy to prove. We have

( c 1 x1 + . . . + c n x n )

+ . . . + c x = c1 x1 n n = c 1 A x1 + . . . + c n A x n , = A ( c 1 x1 + . . . + c n x n ) ,

since xi = A xi ; by the distributive law.

(b) All solutions to the system are of the form (6). This is harder to prove and will be the main result of the next note.

The Existence and Uniqueness Theorem for Linear Systems


For simplicity, we stick with n = 2, but the results here are true for all n. There are two questions about the following general linear system that we need to consider. x = a(t) x + b(t)y x a(t) b(t) x ; in matrix form, = y = c(t) x + d(t)y y c(t) d(t) y (1) The rst is from the previous section: to show that all solutions are of the form x = c1 x1 + x2 x2 , where the xi form a fundamental set, that is, no xi is a constant multiple of the other). (The fact that we can write down all solutions to a linear system in this way is one of the main reasons why such systems are so important.) An even more basic question for the system (1) is: how do we know that it has two linearly independent solutions? For systems with a constant coefcient matrix A, we showed in the previous chapters how to solve them explicitly to get two independent solutions. But the general non-constant linear system (1) does not have solutions given by explicit formulas or procedures. The answers to these questions are based on following theorem. Theorem 2 Existence and uniqueness theorem for linear systems. If the entries of the square matrix A(t) are continuous on an open interval I containing t0 , then the initial value problem x = A(t) x, x ( t 0 ) = x0 (2)

has one and only one solution x(t) on the interval I . The proof is difcult and we shall not attempt it. More important is to see how it is used. The following three theorems answer the questions posed for the 2 2 system (1). They are true for n > 2 as well, and the proofs are analogous. In the following theorems, we assume the entries of A(t) are continuous on an open interval I . Here the conclusions are valid on the interval I , fFor example, I could be the whole t-axis. Theorem 2A Linear independence theorem.

The Existence and Uniqueness Theorem for Linear Systems OCW 18.03SC

Let x1 (t) and x2 (t) be two solutions to (1) on the interval I, such that at some point t0 in I , the vectors x1 (t0 ) and x2 (t0 ) are linearly independent. Then a) the solutions x1 (t) and x2 (t) are linearly independent on I, and b) the vectors x1 (t1 ) and x2 (t1 ) are linearly independent at every point t1 of I . Proof. a) By contradiction. If they were dependent on I, one would be a constant multiple of the other, say x2 (t) = c1 x1 (t). Then x2 (t0 ) = c1 x1 (t0 ), showing them dependent at t0 . b) By contradiction. If there were a point t1 on I where they were dependent, say x2 (t1 ) = c1 x1 (t1 ), then x2 (t) and c1 x1 (t) would be solutions to (1) which agreed at t1 . Hence, by the uniqueness statement in Theorem 2, x2 (t) = c1 x1 (t) on all of I , showing them linearly dependent on I . Theorem 2B General solution theorem. a) The system (1) has two linearly independent solutions. b) If x1 (t) and x2 (t) are any two linearly independent solutions, then every solution x can be written in the form (3), for some choice of c1 and c2 : (3) x = c 1 x1 + c 2 x2 . Proof. Choose a point t = t0 in the interval I . a) According to Theorem 2, there are two solutions x1 , x2 to (1), satisfying respectively the initial conditions x1 (t0 ) = i, x2 (t0 ) = j , (4) where i and j are the usual unit vectors in the xy-plane. Since the two solutions are linearly independent when t = t0 , they are linearly independent on I , by Theorem 5.2A. b) Let u(t) be a solution to (1) on I . Since x1 and x2 are independent at t0 by Theorem 2, using the parallelogram law of addition we can nd and c such that constants c1 2
u( t0 ) = c1 x1 (t0 ) + c2 x2 (t0 ).

(5)

x (t) + c x (t) The vector equation (5) shows that the solutions u(t) and c1 1 2 2 agree at t0 . Therefore by the uniqueness statement in Theorem 2, they are equal on all of I ; that is, u( t ) = c1 x1 ( t ) + c 2 x2 ( t )

on I .

The Wronskian
We know that a standard way of testing whether a set of n n-vectors are linearly independent is to see if the n n determinant having them as its rows or columns is non-zero. This is also an important method when the nvectors are solutions to a system; the determinant is given a special name. (Again, we will assume n = 2, but the denitions and results generalize to any n.) Denition 3 Let x1 (t) and x2 (t) be two 2-vector functions. We dene their Wronskian to be the determinant x1 ( t ) x2 ( t ) W (x1 , x2 )(t) = (1) y1 ( t ) y2 ( t ) whose columns are the two vector functions. The independence of the two vector functions should be connected with their Wronskian not being zero. For points, the relationship is clear. Using the result mentioned above, we can say x1 ( t0 ) x2 ( t0 ) = 0 x1 (t0 ) and x2 (t0 ) are dependent. W (x1 , x2 )(t0 ) = y1 ( t0 ) y2 ( t0 ) (2) However for vector functions, the relationship is clear-cut only when x1 and x2 are solutions to a well-behaved ODE system (3). The theorem is: We are still considering the system x y

= a(t) x + b(t)y = c(t) x + d(t)y

x y

a(t) b(t) c(t) d(t)

x y

, (3)

Theorem 3 Wronskian vanishing theorem. On an interval I where the entries of A(t) are continuous, let x1 and x2 be two solutions to (3) and W (t) their Wronskian (1). Then either a) W (t) 0 on I , and x1 and x2 are linearly dependent on I , or b) W (t) is never 0 on I , and x1 and x2 are linearly independent on I . Proof. Using (2), there are just two possibilities. a) x1 and x2 are linearly dependent on I ; say x2 = c1 x1 . In this case they are dependent at each point of I , and W (t) 0 on I , by (2).

The Wronskian

OCW 18.03SC

b) x1 and x2 are linearly independent on I , in which case by Theorem 2A they are linearly independent at each point of I , and so W (t) is never zero on I , by (2).

Existence and Uniqueness and Superposition in the General Case


We can extend the results above to the inhomogeneous case. x = A(t)x (homogeneous) x = A(t)x + F(t) (inhomogeneous), where F (t) is the input to the system. Linearity/superposition: 1. x1 and x2 solutions to (H) x = c1 x1 + c2 x2 proof: x = c1 x1 + c2 x2 = c1 Ax1 + c2 Ax2 = A(c1 x1 + c2 x2 ) = x. 2. xh solution to (H) and xp solution to (I) x = xh + xp is a solution to (I). proof: x = xh + xp = Axh + Axp + F = A(xh + xp ) + F = Ax + F. 3. x1 = Ax1 + F1 , x2 = Ax2 + F2 x1 + x2 satises x = Ax + F1 + F2 i.e., superposition of inputs leads to superposition of outputs. (H) (I)

Existence and uniqueness: We start with an initial time t0 and the initial value problem: x = A ( t ) x + F ( t ), x ( t0 ) = x 0 . (IVP)

Theorem: If A(t) and F(t) are continuous then there exists a unique solution to (IVP).

Fundamental Matrices
In the literature, solutions to linear systems often are expressed using square matrices rather than vectors. This is an elegant bookkeeping technique and a very compact, efcient way to express these formulas. As before, we state the denitions and results for a 2 2 system, but they generalize immediately to n n systems. We return to the system x = A ( t ) x , with the general solution x = c 1 x1 ( t ) + c 2 x2 ( t ) , (2) (1)

where x1 and x2 are two independent solutions to (1), and c1 and c2 are arbitrary constants. We form the matrix whose columns are the solutions x1 and x2 : x1 x1 x2 (t) = = . x2 y1 y2

(3)

Since the solutions are linearly independent, we called them a fundamental set of solutions, and therefore we call the matrix in (3) a fundamental matrix for the system (1). Writing the general solution using (t). As a rst application of (t), we can use it to write the general solution (2) efciently. For according to (2), it is x1 x2 x1 x2 c1 x = c1 + c2 = , y1 y2 y1 y2 c2 which becomes using the fundamental matrix c1 x = (t) c where c = , (general solution to (1)). c2

(4)

Note that the vector c must be written on the right, even though the cs are usually written on the left when they are the coefcients of the solutions xi .

Fundamental Matrices

OCW 18.03SC

Solving the IVP using (t). We can now write down the solution to the IVP x = A ( t ) x , x ( t 0 ) = x0 . (5) Starting from the general solution (4), we have to choose the c so that the initial condition in (6) is satised. Substituting t0 into (5) gives us the matrix equation for c : ( t 0 ) c = x0 . Since the determinant |(t0 )| is the value at t0 of the Wronskian of x1 and x2 , it is non-zero since the two solutions are linearly independent (Theorem 3 in the note on the Wronskian). Therefore the inverse matrix exists and the matrix equation above can be solved for c: c = ( t 0 ) 1 x0 . Using the above value of c in (4), the solution to the IVP (1) can now be written x = ( t ) ( t 0 ) 1 x0 . (6) Note that when the solution is written in this form, its obvious that x(t0 ) = x0 , i.e., that the initial condition in (5) is satised. An equation for fundamental matrices We have been saying a rather than the fundamental matrix since the system (1) doesnt have a unique fundamental matrix: there are many ways to pick two independent solutions of x = A x to form the columns of . It is therefore useful to have a way of recognizing a fundamental matrix when you see one. The following theorem is good for this; well need it shortly. Theorem 1 (t) is a fundamental matrix for the system (1) if its determinant |(t)| is non-zero and it satises the matrix equation = A , where means that each entry of has been differentiated. Proof. Since || 0, its columns x1 and x2 are linearly independent, as we x1 saw in the previous note. Let = . According to the rules for x2 matrix multiplication (7) becomes x1 x1 Ax1 = A = . x2 x2 Ax2 2 (7)

Fundamental Matrices

OCW 18.03SC

which shows that


= A x1 x1

and

x2 = A x2 ;

this last line says that x1 and x2 are solutions to the system (1).

The Normalized Fundamental Matrix


In the previous note we saw to main facts about the a funcdamemtal matrix: x1 x1 x2 (t) = = . (1) x2 y1 y2 and x = ( t ) ( t 0 ) 1 x0 . (2)

Is there a best choice for fundamental matrix? There are two common choices, each with its advantages. If the ODE system has constant coefcients, and its eigenvalues are real and distinct, then a natural choice for the fundamental matrix would be the one whose columns are the normal modes the solutions of the form xi = i e i t , i = 1, 2.

There is another choice however which is suggested by (2) and which is particularly useful in showing how the solution depends on the initial conditions. Suppose we pick (t) so that 1 0 ( t0 ) = I = . (3) 0 1 Referring to the denition (1), this means the solutions x1 and x2 are picked so 1 0 x1 ( t 0 ) = , x2 (t0 ) = . (3 ) 0 1 Since the xi (t) are uniquely determined by these initial conditions, the fundamental matrix (t) satisfying (3) is also unique; we give it a name. t0 (t) satisfying Denition 2 The unique matrix t0 = A t0 , t0 ( t 0 ) = I (4)

is called the normalized fundamental matrix at t0 for A. For convenience in use, the denition uses Theorem 1 to t0 will actually be a fundamental matrix. The guarantee t0 (t)| = 0 in Theorem 1 is satised, since the condition | t0 (t0 )| = 1. denition implies |

The Normalized Fundamental Matrix

OCW 18.03SC

To keep the notation simple, we will assume in the rest of this section 0 is the normalized fundamental that t0 = 0, as it almost always is; then 0 (0) = I , we get from (2) the matrix form for the solution to matrix. Since the IVP: x = A(t) x, x(0) = x0 is 0 (t)x0 . x( t ) = (5)

0 . One way is to nd the two solutions in (3 ) and use Calculating 0 . This is ne if the two solutions can be deterthem as the columns of mined by inspection. If not, a simpler method is this: nd any fundamental matrix (t); then 0 ( t ) = ( t ) (0 ) 1 . (6)

To verify this, we have to see that the matrix on the right of (6) satises the two conditions in Denition 2. The second is trivial. The rst is easy using the rule for matrix differentiation: If M = M(t) and B, C are constant matrices, then

( BM) = BM , ( MC ) = M C,

from which we see that since is a fundamental matrix,

( ( t ) (0 ) 1 ) = ( t ) (0 ) 1 = A ( t ) (0 ) 1 = A ( ( t ) (0 ) 1 ),
showing that (t)(0)1 also satises the rst condition in Denition 2. 0 1 Example 2A. Find the solution to the IVP: x = x , x(0) = 1 0 x0 . Solution. Since the system is x = y, y = x, we can nd by inspection the fundamental set of solutions satisfying (3 ) : x = cos t y = sin t and x = sin t y = cos t .

Thus by (5) the normalized fundamental matrix at 0 and solution to the IVP is cos t sin t x0 cos t sin t x = x0 = = x0 + y0 . sin t cos t y0 sin t cos t
Example 2B. Give the normalized fundamental matrix at 0 for x 1 3 x. 1 1

The Normalized Fundamental Matrix

OCW 18.03SC

Solution. This time the solutions (3 ) cannot be obtained by inspection, so we use the second method. You can easily nd the eigenvalues and eigenvectors for this system. Doing so produces the normal modes. Using them as the columns of a fundamental matrix gives us 2t 3e e 2t (t) = . e 2t e 2t Using (6) and the formula for calculating the inverse matrix we get 1 3 1 1 1 1 (0) = , (0) = , 1 1 1 3 4 so that (t) = 1 4 3 e 2t e 2t e 2t e 2t 1 1 1 3 1 = 4 3 e 2t + e 2t 3 e 2t 3 e 2t e 2t e 2t e 2t + 3 e 2t .

The Exponential Matrix


The work in the preceding note with fundamental matrices was valid for any linear homogeneous square system of ODEs, x = A ( t ) x . However, if the system has constant coefcients, i.e., the matrix A is a constant matrix, the results are usually expressed by using the exponential matrix, which we now dene. Recall that if x is any real number, then ex = 1 + x + x2 xn +...+ +... . 2! n! (1)

Denition 3 Given an n n constant matrix A, the exponential matrix e A is the n n matrix dened by eA = I + A + A2 An +...+ +... . 2! n! (2)

Each term on the right side of (2) is an n n matrix adding up the ij-th entry of each of these matrices gives you an innite series whose sum is the ij-th entry of e A . (The series always converges.) In the applications, an independent variable t is usually included: e At = I + A t + A2 t2 tn + . . . + An +... . 2! n! (3)

This is not a new denition, its just (2) above applied to the matrix A t in which every element of A has been multiplied by t, since for example

( At)2 = At At = A A t2 = A2 t2 .
Try out (2) and (3) on these two examples (the second is very easy, since it is not an innite series). a e 0 a 0 A Example 3A. Let A = . Show: e = ; and 0 b 0 eb at e 0 e At = 0 ebt

The Exponential Matrix Example 3B. Let A = 1 t At e = . 0 1 0 1 0 0 , show:

OCW 18.03SC 1 1 0 1 and

eA

Whats the point of the exponential matrix? The answer is given by the theorem below, which says that the exponential matrix provides a royal road to the solution of a square system with constant coefcients: no eigenvectors, no eigenvalues, you just write down the answer! Theorem 3 (1) (a) e At Let A be a square constant matrix. Then 0 (t), the normalized fundamental matrix at 0; = x ( 0 ) = x0 is x=

(2) (b) the unique solution to the IVP x = Ax, At e x0 .

0 (t) is the normalProof. Recall that in the previous note we saw that if ized fundamental matrix then 0 ( t ) x0 . x( t ) = (4) Statement (2) follows immediately from (1), in view of (4). The solution to the IVP : is We prove (1) is true by using the fact that if t0 = 0 then the normalized fundamental matrix has (0) = I . Letting = e At , we must show = A and (0) = I . The second of these follows from substituting t = 0 into the innite series denition (3) for e At . To show = A, we assume that we can differentiate the series (3) term-by-term; then we have for the individual terms d n tn t n 1 A = An , ( n 1) ! dt n! since An is a constant matrix. Differentiating (3) term-by-term then gives
d dt

x = A(t) x, x(0) = x0

d dt

e At

t = A + A2 t + . . . + A n ( n +... 1) ! At = Ae = A .

n 1

(5)

Calculation of e At . The main use of the exponential matrix is in Theorem 3 writing down explicitly the solution to an IVP. If e At has to be calculated for a specic system, several techniques are available. 2

The Exponential Matrix

OCW 18.03SC

a) In simple cases, it can be calculated directly as an innite series of matrices. b) It can always be calculated, according to Theorem 3, as the normal 0 (t), using (11): 0 ( t ) = ( t ) (0 ) 1 . ized fundamental matrix c) A third technique uses the exponential law e( B+C)t = e Bt eCt , valid if BC = CB. (6)

To use it, one looks for constant matrices B and C such that A = B + C, then e At = e B t eC t . (8) 2 1 1 Let A = . Solve x = A x, x(0) = , 0 2 2 Solution. We set satised, and e
At

BC = CB,

e Bt and eCt are computable;

(7)

Example 3C. using e At .

B =

2 0 0 2

and C =

0 1 0 0 1 t 0 1

; then (7) is

e 2t 0 0 e2t

1 t 0 1

= e

2t

by (8) and Examples 3A and 3B. Therefore, by Theorem 3 (2), we get 1 t 1 1 + 2t At 2t 2t x = e x0 = e = e . 0 1 2 2

Inhomogeneous Case: Variation of Parameters Formula


The fundamental matrix (t) also provides a very compact and efcient integral formula for a particular solution to the inhomogeneous equation x = A(t)x + F(t). (presupposing of course that one can solve the homogeneous equation x = A(t)x rst to get .) In this short note we give the formula (with proof!) and one example. Variation of parameters: (solving inhomegeneous systems) (H) x = A(t)x (t) = fundamental matrix (I) x = A(t)x + F(t) Variation of parameters formula for solution to (I) (just like order 1 DEs): x = 1 F dt + C . proof (remember this) General homogeneous solution: x = c for a constant vector c. Make c variable trial solution x = v(t). Plug this into (I): x = Ax + F v + v = A v + F. Now substitute for = A: A v + v = A v + F. v = F v = 1 F

1 F + C . 1 x = v = F dt + C . QED. v=

Denite integral version of variation of parameters t 1 x(t) = (t) (u) F(u) du + C , where C = 1 (t0 ) x(t0 ).
t0

t 6 5 e Example. Solve = x+ 1 2 e 5t 6 5 1 Notation: A = , F= . 1 2 t x Fundamental matrix (earlier example): = e t 5 e 7t e t e 7t 1 =


e 8t 6

e 7t 5 e 7t et et

Inhomogeneous Case: Variation of Parameters Formula

OCW 18.03SC

Variation of parameters: x = 1 F dt t 8t 7t e 1 e 5 e7t e 1 5 e4t = = dt et et e 5t e 6t + e 2t 6 6 1 1 t 5 e 4t + c 1 e 5t 5 et 5 e 5t + c 1 e t + 5 c2 e 7t tet 5 4 4 6 2 = = 1 t 1 5t 6t 1 e 2t + c 5t t 7t tet + 5 1 6 6 2 6e 2 4 e 6 e 2 e c1 e + c2 e 1 1 15/4 5/6 1 5 t 5t t t 7t te +e +e + c1 e + c2 e . = 1 3/4 1/6 1 1 6
(Notice the homogeneous solution appearing with the constants of integration).

Introduction
In this session we introduce and develop the basic properties of autonomous 2 2 systems. In the next session we will see how to get key information about the solutions to such a system directly from the DE itself, without having to actually solve it. This is an example of what is called the qualitative theory of differential equations.

The Phase Plane 1. General First Order Autonomous Systems


The sort of system for which we will be trying to sketch solutions can be written in the form x = f ( x, y) (1) y = g( x, y) This is called an autonomous system. The word autonomous means selfregulating. These systems are self-regulating in the sense that their rate of change (i.e. derivatives) depends only on the state of the system (values of x and y) and not on the time t. You can easily spot an autonomous system because the independent variable (which we understand to be t) does not appear explicitly on the right, though of course it lurks in the derivatives on the left. The system (1) is a rst-order autonomous system; it is in standard form the derivatives on the left, the functions on the right. Just as for linear constant coefcient systems autonomous systems have trajectories in the phase plane. We will repeat the denitions of these objects in this more general setting. A solution of this system has the form (we write it two ways) x (t) x = x (t) x(t) = , y(t) y = y ( t ).

(2)

It is a vector function of t, whose components satisfy the system (1) when they are substituted in for x and y. In general, you learned in 18.02 and physics that such a vector function describes a motion in the xy-plane; the equations in (2) tell how the point ( x, y) moves in the xy-plane as the time t varies. The moving point traces out a curve called the trajectory of the solution (2) The xy-plane itself is called the phase plane for the system (1). We show a sketch of a trajectory at right. Notice the arrow is used to indicate the direction of in increasing time. We use the term phase portrait to mean the graphs of enough trajectories to give a good sense of all the solutions to the system (1) We have seen how we can picture the solutions (2) to the system. But how can we picture the system (1) itself? We can think of the derivative of

The Phase Plane

OCW 18.03SC

a solution x (t) =

x (t) y (t)

(3)

as representing the velocity vector of the point ( x, y) as it moves according to (2). From this viewpoint, we can interpret geometrically the system (1) as prescribing for each point ( x0 , y0 ) in the xy-plane a velocity vector having its tail at ( x0 , y0 ): f ( x0 , y0 ) = f ( x0 , y0 ) i + g ( x0 , y0 ) j. (4) x = g ( x0 , y0 ) The system (1) is thus represented geometrically as a vector eld, the velocity eld. A solution (2) of the system is a point moving in the xyplane so that at each point of its trajectory, it has the velocity prescribed by the eld. The trajectory itself will be a curve which at each point has the direction of the velocity vector at that point.

2. Critical Points
Denition. A point ( x0 , y0 ) is a critical point of the system (1) if f ( x0 , y0 ) = 0 and g( x0 , y0 ) = 0.

In considering how to sketch trajectories of the system (1), the rst thing to consider are the critical points (they are sometimes called stationary points). If we adopt the geometric viewpoint, thinking of the system as represented by a velocity vector eld, then a critical point is one where the velocity vector is zero. That is ( x0 , y0 ) is a critical point is equativalent to x = x0 , y = y0 is a (constant) solution to (1).

Such a point is a trajectory all by itself, since by not moving it satises the equations (1) of the system (and hence the alternative designation stationary point). The critical points represent the simplest possible solutions to (1), so you begin by nding them; this is done by solving the pair of simultaneous equations f ( x, y) = 0 g( x, y) = 0

The Phase Plane

OCW 18.03SC

Next, you can try the strategy indicated in the following note of passing to the associated rst-order ODE and trying to solve that and sketch the solutions; or you can try to locate some sketchable solutions to (??) and draw them in, as we did for linear constant coefcient systems in the session on Phase Potraits.

3. Sketching Principle
When sketching integral curves for direction elds we saw that integral curves did not cross. For the system (1) we have a similar principle. Sketching Principle. Assuming the the functions f ( x, y) and g( x, y) are smooth, (i.e. have continuous partial derivatives) then two trajectories of (1) cannot intersect.

First Order Autonomous ODE Systems and First Order ODEs


We are still considering the rst order autonomous system x = f ( x, y) y = g( x, y) (1)

We can eliminate t from the system by dividing one equation by the other. Since by the chain rule y dy/dt dy = = . x dx /dt dx we get after the division a single rst-order ODE in x and y : x y

= f ( x, y) = g( x, y)

dy g( x, y) = . dx f ( x, y)

(2)

If the rst order equation on the right is solvable, this is an important way of getting information about the solutions to the system on the left. Indeed, in the older literature, little distinction was made between the system and the single equation solving meant to solve either one. There is however a difference between them: the system involves time, whereas the single ODE does not. Consider how their respective solutions are related: x = x (t) F ( x, y) = 0 , (3) y = y(t) where the equation on the right is the result of eliminating t from the pair of equations on the left. Geometrically, F ( x, y) = 0 is the equation for the trajectory of the solution x(t) on the left. in other words is The trajectory the path traced out by the moving point x (t), y(t) ; it doesnt contain any record of how fast the point was moving; it is only the track (or trace, as one sometimes says) of its motion. In the same way, we have the difference between the velocity eld, which represents the left side of (2), and the direction eld, which represents the right side. The velocity vectors have magnitude and sense, whereas the line segments that make up the direction eld only have slope. The passage from the left side of (2) to the right side is represented geometrically by changing each of the velocity vectors to a line segment of standard length.

First Order Autonomous ODE Systems and First Order ODEs OCW 18.03SC

Even the arrowhead is dropped, since it represents the direction of increasing time, and time has been eliminated; only the slope of the vector is retained.

Introduction
In this session, we continue to develop the methods for sketching graphs of the solutions to DE systems which we carried out for linear systems in the session on Phase Portraits. The goal is to see how to get quick qualitative information about the graphs of the solutions, without having to actually calculate points on them.

Sketching Non-linear Systems


In session on Phase Portraits, we described how to sketch the trajectories of a linear system x y

= ax + by = cx + dy

a, b, c, d constants.

We now return to the general (i.e., non-linear) 2 2 autonomous system discussed at the beginning of this chapter, in sections 1 and 2: x y

= f ( x, y) = g( x, y)

(1)

it is represented geometrically as a vector eld, and its trajectories the solution curves are the curves which at each point have the direction prescribed by the vector eld. Our goal is to see how one can get information about the trajectories of (1), without determining them analytically or using a computer to plot them numerically. Linearizing at the origin. To illustrate the general idea, lets suppose that (0, 0) is a critical point of the system (1), i.e., f (0, 0) = 0, g(0, 0) = 0, (2)

Then if f and g are sufciently differentiable, we can approximate them near (0, 0) (the approximation will have no constant term by (2)): f ( x, y) = a1 x + b1 y + higher order terms in x and y g( x, y) = a2 x + b2 y + higher order terms in x and y. If ( x, y) is close to (0, 0), then x and y will be small and we can neglect the higher order terms. Then the non-linear system (2) is approximated near (0, 0) by a linear system, the linearization of (2) at (0,0): x = a1 x + b1 y , y = a2 x + b2 y (3)

and near (0,0), the solutions of (1) about which we know nothing will be like the solutions to (4), about which we know a great deal from our work in the previous sessions. x = y cos x Example 1. Linearize the system at the critical y = x (1 + y )2 point (0, 0).

Sketching Non-linear Systems Solution. We have x = y . y = x x y


2 y (1 1 2x ) = x (1 + 2y + y2 )

OCW 18.03SC

so the linearization is

Linearising at a general point More generally, suppose now the critical point of (1) is ( x0 , y0 ), so that f ( x0 , y0 ) = 0, g( x0 , y0 ) = 0.

One way this can be handled is to make the change of variable x1 = x x0 , y1 = y y0 ; (4)

in the x1 y1 -coordinate system, the critical point is (0, 0), and we can proceed as before. x = x x2 2xy Example 2. Linearize at its critical points on y = y y2 3 2 xy the x-axis. Solution. When y = 0, the functions on the right are zero when x = 0 and x = 1, so the critical points on the x-axis are (0, 0) and (1, 0). The linearization at (0, 0) is x = x, y = y. To nd the linearization at (1, 0) we change of variable as in (4): x1 = x 1, y1 = y ; substituting for x and y in the system and keeping just the linear terms on the right gives us as the linearization:
x1 y1

= ( x1 + 1) ( x1 + 1)2 2( x1 + 1) y1 3 = y1 y2 1 2 ( x1 + 1) y1

x1 2y1 1 2 y1 .

Linearization using the Jacobian matrix Though the above techniques are usable if the right sides are very simple, it is generally faster to nd the linearization by using the Jacobian matrix, especially if there are several critical points, or the functions on the right are not simple polynomials. We derive the procedure. We need to approximate f and g near ( x0 , y0 ). While this can sometimes be done by changing variable, a more basic method is to use the main approximation theorem of multivariable calculus. For this we use the notation x = x x0 , y = y y0 , f = f ( x , y ) f ( x0 , y0 ) (5)

Sketching Non-linear Systems

OCW 18.03SC

and we have then the basic approximation formula f f f x + y, y 0 x 0 f f f ( x, y) x + y , x 0 y 0 x1 = x x0 = x ,

or (6)

since by hypothesis f ( x0 , y0 ) = 0. We now make the change of variables (4) y1 = y y0 = y,

and use (6) to approximate f and g by their linearizations at ( x0 , y0 ). The result is that in the neighborhood of the critical point ( x0 , y0 ), the linearization of the system (1) is f f x1 = x1 + y1 , x y 0 0 (7) g g y1 = x1 + y1 . x 0 y 0 In matrix notation, the linearization is therefore x1 fx x1 = A x1 , where x1 = and A = y1 gx fy gy ;
( x0 ,y0 )

(8) the matrix A is the Jacobian matrix, evaluated at the critical point ( x0 , y0 ). General procedure for sketching the trajectories of non-linear systems. We can now outline how to sketch in a qualitative way the solution curves of a 2 2 non-linear autonomous system, x y

= f ( x, y) = g ( x , y ).

(9)

1. Find all the critical points (i.e., the constant solutions), by solving the system of simultaneous equations f ( x, y) = 0 g( x, y) = 0 . 2. For each critical point ( x0 , y0 ), nd the matrix A of the linearized system at that point, by evaluating the Jacobian matrix at ( x0 , y0 ): fx fy . g x g y ( x ,y )
0 0

Sketching Non-linear Systems

OCW 18.03SC

(Alternatively, make the change of variables x1 = x x0 , y1 = y y0 , and drop all terms having order higher than one; then A is the matrix of coefcients for the linear terms.) 3. Find the geometric type and stability of the linearized system at the critical point point ( x0 , y0 ), by carrying out the analysis in sections 4 and 5. sl The subsequent steps require that the eigenvalues be nonzero, real, and distinct, or complex, with a non-zero real part. The remaining cases: eigenvalues which are zero, repeated, or pure imaginary are classied as borderline, and the subsequent steps dont apply, or have limited application. See the next section. 4. According to the above, the acceptable geometric types are a saddle, node (not a star or a defective node, however), and a spiral. Assuming that this is what you have, for each critical point determine enough additional information (eigenvectors, direction of motion) to allow a sketch of the trajectories near the critical point. 5. In the xy-plane, mark the critical points. Around each, sketch the trajectories in its immediate neighborhood, as determined in the previous step, including the direction of motion. 6. Finally, sketch in some other trajectories to ll out the picture, making them compatible with the behavior of the trajectories you have already sketched near the critical points. Mark with an arrowhead the direction of motion on each trajectory. If you have made a mistake in analyzing any of the critical points, it will often show up here it will turn out to be impossible to draw in any plausible trajectories that complete the picture. Remarks about the steps. 1. In the homework problems, the simultaneous equations whose solutions are the critical points will be reasonably easy to solve. In the real world, they may not be; a simultaneous-equation solver will have to be used (the standard programs MatLab, Maple, Mathematica, Macsyma all have them, but they are not always effective.) 2. If there are several critical points, one almost always uses the Jacobian matrix; if there is only one, use your judgment. 3. This method of analyzing non-linear systems rests on the assumption

Sketching Non-linear Systems

OCW 18.03SC

that in the neighborhood of a critical point, the non-linear system will look like its linearization at that point. For the borderline cases this may not be so that is why they are rejected. The next two notes explain this more fully. If one or more of the critical points turn out to be borderline cases, one usually resorts to numerical computation on the non-linear system. Occasionally one can use the reduction to a rst order equation: dy g( x, y) = dx f ( x, y) to get information about the system. Example 3. Sketch some trajectories of the system x = x + xy . y = 2y + xy Solution. We rst nd the critical points, by solving

x + xy = x (1 + y) = 0 . 2y + xy = y(2 + x ) = 0
From the rst equation, either x = 0 or y = 1. From the second equation, x = 0 y = 0; y = 1 x = 2; critical points : (0, 0), (2, 1).

To linearize at the critical points, we compute the Jacobian matrices 1 + y x 1 0 0 2 J= ; J(0,0) = J(2,1) = . y 2 + x 0 2 1 0 Next we analyze the geometric type and stability of each critical point: (0, 0): 1, 2 = 2 sink node eigenvalues: 1 = 1 0 eigenvectors: 1 = ; 2 = 0 1 1 near the origin, By the node-sketching principle, trajectories follow 2 away from the origin. are parallel to

2, 1 = 2 eigenvectors: 1 = ; 1

(2, 1): eigenvalues:

2 = unstable saddle 2 2 2 = 1
5

Sketching Non-linear Systems

OCW 18.03SC

Draw in these eigenvectors at the respective points (0, 0) and (2, 1), with arrowhead indicating direction of motion (into the critical point if < 0, away from critical point if > 0.) Draw in some nearby trajectories. Then guess at some other trajectories compatible with these. See the gure for one attempt at this. Further information could be gotten by considering the associated rst-order ODE in x and y.

Example. Sketch the phase portrait of the following system. 1 x = 14 x x2 xy 2 1 2 y = 16y y xy 2 Critical points: 1 1 x 14 x y = 0 x = 0 or 14 x y = 0 2 2 1 1 y 16 y x = 0 y = 0 or 16 y x = 0. 2 2 x = 0 y = 0 or y = 32. y = 0 x = 0 or x = 28. x = 0, y = 0 x = 12, y = 8. all critical points: (0, 0), (0, 32), (28, 0), (12, 8).

Sketching Non-linear Systems

OCW 18.03SC

14 x y x J ( x, y) = y 16 y x Looking at each of the critical points in turn: 14 0 1 0 J (0, 0) = : eigenvalues 14, 16; eigenvectors , 0 16 0 1 source node (see Source node picture below). 0 1 0 18 J (0, 32) = : eigenvalues -18, -16; eigenvectors , 32 16 16 1 sink node (see Sink node 1 picture below). 14 28 1 14 J (28, 0) = : eigenvalues -14, -12; eigenvectors , 0 12 0 1 sink node (see Sink node 2 picture below). 6 12 J (12, 8) = : 8 4 eigenvalues 5 97 15, 5; 11 9 1 + 97 1 97 eigenvectors , , 8 8 8 8 saddle (see Saddle picture below). Rough sketch of system: First we sketch each of the critical points. v v

Source node v

Sink node 1

Sink node 2

Saddle 7

Sketching Non-linear Systems

OCW 18.03SC

Hand sketch phase plane portrait.

Computer plot phase plane portrait.

Structural Stability
In the previous Note, we described how to get a rough picture of the trajectories of a non-linear system by linearizing at each of its critical points. The basic assumption of the method is that the linearized system will be a good approximation to the original non-linear system if you stay near the critical point. The method only works however if the linearized system turns out to be a node, saddle, or spiral. What is it about these geometric types that allows the method to work, and why wont it work if the linearized system turns out to be one of the other possibilities (dismissed as borderline types in the previous section)? Briey, the answer is that nodes, saddles, and spirals are structurally stable, while the other possibilities are not. We call a system x = f ( x, y) y = g( x, y) (1)

Structurally Stability: We say a system is structural if small changes in the system parameters (i.e., the constants that enter into the functions on the right hand side) do not change the geometric type or stability of its critical points (or its limit cycles, which will be dened in a later session -dont worry about them for now). Theorem. The 2 2 autonomous linear system x = ax + by y = cx + dy (2)

is structurally stable if it is a spiral, saddle, or node (but not a degenerate or star node). Proof. The characteristic equation is 2 ( a + d) + ( ad bc) = 0, and its roots (the eigenvalues) are 1 , 2 =

( a + d)

( a + d)2 4( ad bc) . 2

(3)

Lets look at the cases one-by-one; assume rst that the roots 1 and 2 are real and distinct. The possibilities in the theorem are given by the

Structural Stability

OCW 18.03SC

following (note that since the roots are distinct, the node will not be degenerate or a star node): 1 > 0, 2 > 0 1 < 0, 2 < 0 1 > 0, 2 < 0 unstable node asymptotically stable node unstable saddle.

The quadratic formula (3) shows that the roots depend continuously on the coefcients a, b, c, d. Thus if the coefcients are changed a little, the and respectively; the new roots 1 and 2 will also be changed a little to 1 2 roots will still be real, and will have the same sign if the change is small enough. Thus the changed system will still have the same geometric type and stability. If the roots of the characteristic equation are complex, the reasoning is similar. Let us denote the complex roots by r si; we use the root = r + si, s > 0; then the possibilities to be considered for structural stability are r > 0, s > 0 unstable spiral asymptotically stable spiral. r < 0, s > 0 If a, b, c, d are changed a little, the root is changed to = r + s i, where r and s are close to r and s respectively, since the quadratic formula (3) shows r and s depend continuously on the coefcients. If the change is small enough, r will have the same sign as r and s will still be positive, so the geometric type of the changed system will still be a spiral, with the same stability type. Structural Stability of a non-linear system Theorem: For an autonomous non-linear system, the linearized system correctly classies the crititcal point if the linear system is a spiral node, a nodal source or sink or a saddle. It may not however correctly classify a center, defective node, star node or non-isolated critical point. That is, it is correct in open regions of the tracedeterminant diagram and untrustworthy on the boundary lines.

Structural Stability

OCW 18.03SC

det A

spiral sink nodal sink

center spiral source

defective or star node

saddle Trace-determinant diagram

nodal source tr A degenerate

Idea: small changes in the eigenvalues dont move far in tracedeterminant diagram.

The Borderline Geometric Types


All the other possibilities for the linear system (??) we call borderline types. We will show now that none of them is structurally stable; we begin with the center. Eigenvalues pure imaginary. Once again we use the eigenvalue with the positive imaginary part: = 0 + si, s > 0. It corresponds to a center: the trajectories are a family of concentric ellipses, centered at the origin. If the coefcients a, b, c, d are changed a little, the eigenvalue 0 + si changes a little to r + s i, where r 0, s s, and there are three possibilities for the new eigenvalue: 0 + si r + s i : s>0 center r > 0 s > 0 source spiral r < 0 s > 0 sink spiral r = 0 s > 0 center

Correspondingly, there are three possibilities for how the geometric picture of the trajectories can change:

Eigenvalues real; one eigenvalue zero. Here 1 = 0, and 2 > 0 or 2 < 0. The general solution to the system has the form (1 , 2 are the eigenvectors) x = c1 1 + c2 2 e2 t . If 2 < 0, the geometric picture of its trajectories shows a line of critical points (constant solutions, corresponding to c2 = 0), with all other trajectories being parallel lines ending up (for t = ) at one of the critical points, as shown below.

We continue to assume 2 < 0. As the coefcients of the system change a little, the two eigenvalues change a little also; there are three possibilities,

The Borderline Geometric Types

OCW 18.03SC

since the eigenvalue = 0 can become positive, negative, or stay zero:


: 1 = 0 1 : 2 < 0 2 critical line >0 1 <0 2 unstable saddle =0 1 <0 2 critical line

1 < 0 <0 2 sink node

Here are the corresponding pictures. (The pictures would look the same if we assumed 2 > 0, but the arrows on the trajectories would be reversed.)

One repeated real eigenvalue. Finally, we consider the case where 1 = 2 . Here there are a number of possibilities, depending on whether 1 is positive or negative, and whether the repeated eigenvalue is complete (i.e., has two independent eigenvectors), or defective (i.e., incomplete: only one eigenvector). Let us assume that 1 < 0. We vary the coefcients of the system a little. By the same reasoning as before, the eigenvalues change a little, and by the same reasoning as before, we get as the main possibilities (omitting this time the one where the changed eigenvalue is still repeated): 1 < 0 2 < 0 1 = 2 sink node

<0 1 <0 2 = 1 2 sink node

r + si r si r 1 , s 0, sink spiral

Typical corresponding pictures for the complete case and the defective (incomplete) case are (the last one is left for you to experiment with on the computer screen)

complete: star node 2

incomplete: defective node

The Borderline Geometric Types

OCW 18.03SC

Remarks. Each of these three casesone eigenvalue zero, pure imaginary eigenvalues, repeated real eigenvaluehas to be looked on as a borderline linear system: altering the coefcients slightly can give it an entirely different geometric type, and in the rst two cases, possibly alter its stability as well.

Structural Stability for Non-linear Systems


In the preceding note we discussed the structural stability of a linear system. How does it apply to non-linear systems? Suppose our non-linear system has a critical point at P, and we want to study its trajectories near P by linearizing the system at P. This linearization is only an approximation to the original system, so if it turns out to be a borderline case, i.e., one sensitive to the exact value of the coefcients, the trajectories near P of the original system can look like any of the types obtainable by slightly changing the coefcients of the linearization. It could also look like a combination of types. For instance, if the linearized system had a critical line (i.e., one eigenvalue zero), the original system could have a sink node on one half of the critical line, and an unstable saddle on the other half. (This actually occurs.) In other words, the method of linearization to analyze a non-linear system near a critical point doesnt fail entirely, but we dont end up with a denite picture of the non-linear system near P; we only get a list of possibilities. In general one has to rely on computation or more powerful analytic tools to get a clearer answer. The rst thing to try is a computer picture of the non-linear system, which often will give the answer. x = y x 2 , y = x + y2 2 x 1 Jacobian: J ( x, y) = 1 2y Example. Crititcal points: y x2 = 0 y = x2 x + y2 = 0 x + x4 = 0 x = 0, 1. (0, 0) and (1, 1) are the critical points. 2 1 J (1, 1) = : 1 2 characteristic equation: 2 3 = 0 = 3 linearized system has a saddle. This is structurally stable the nonlinear system has a saddle at (1, 1). 0 1 J (0, 0) = : eigenvalues = i a linearized center. 1 0 This is not structurally stable. The nonlinear system could be any one of a

Structural Stability for Non-linear Systems

OCW 18.03SC

center, spiral out or spiral in. Using a computer program it appears that (0,0) is in fact a center. (This can be proven using more advanced methods.) We can show the trajectories near (0,0) are not spirals by exploiting the symmetry of the picture. First note, if ( x (t), y(t) is a solution then so is (y(t), x (t). That is, the trajectory is symmetric in the line x = y. This implies it cant be a spiral. Since the only other choice choice is that the critical point (0,0) is a center, the trajectories must be closed. The following two examples show that a linearized center might also be a spiral in or a spiral out in the nonlinear system. Example a. x = y, y = x y3 . Example b. x = y, y = x + y3 . In both examples the only critical point is (0, 0). 0 1 J (0, 0) = linearized center. This is not structurally stable. 1 0 In example a the critical point turns out to be a spiral sink. In example b it is a spiral source. Below are computer-generated pictures. Because the y3 term causes the spiral to have a lot of turns we improved the pictures by using the power 1.1 instead.

Spiral in

Spiral out

Introduction
In this nal section we look at two important and interesting extensions of the ideas from qualitative DE theory we have been exploring inthis unit, namely limit cycles and chaos.

Limit Cycles
In analyzing non-linear systems in the xy-plane, we have so far concentrated on nding the critical points and analysing how the trajectories of the system look in the neighborhood of each critical point. This gives some feeling for how the other trajectories can behave, at least those which pass near anough to critical points. Another important possibility which can inuence how the trajectories look is if one of the trajectories traces out a closed curve C. If this happens, the associated solution x(t) will be geometrically realized by a point which goes round and round the curve C with a certain period T . That is, the solution vector x(t) = ( x (t), y(t)) will be a pair of periodic functions with period T : x ( t + T ) = x ( t ), y(t + T ) = y(t) for all t.

If there is such a closed curve, the nearby trajectories must behave something like C. The possibilities are illustrated below. The nearby trajectories can either spiral in toward C, they can spiral away from C, or they can themselves be closed curves. If the latter case does not hold in other words, if C is an isolated closed curve then C is called a limit cycle: stable, unstable, or semi-stable according to whether the nearby curves spiral towards C, away from C, or both.

The most important kind of limit cycle is the stable limit cycle, where nearby curves spiral towards C on both sides. Periodic processes in nature can often be represented as stable limit cycles, so that great interest is attached to nding such trajectories if they exist. Unfortunately, surprisingly little is known about how to do this, or how to show that a system has no

Limit Cycles

OCW 18.03SC

limit cycles. There is active research in this subject today. We will present a few of the things that are known.

Showing Limit Cycles Exist


The main tool which historically has been used to show that the system x = f ( x, y) y = g( x, y) has a stable limit cycle is the Poincare-Bendixson Theorem Suppose R is the nite region of the plane lying between two simple closed curves D1 and D2 , and Fis the velocity vector eld for the system (1). If (i) at each point of D1 and D2 , the eld Fpoints toward the interior of R, and (ii) R contains no critical points, then the system (1) has a closed trajectory lying inside R. The hypotheses of the theorem are illustrated by g. 1. We will not give the proof of the theorem, which requires a background in Mathematical Analysis. Fortunately, the theorem strongly appeals to intuition. If we start on one of the boundary curves, the solution will enter R, since the velocity vector points into the interior of R. As time goes on, the solution can never leave R, since as it approaches a boundary curve, trying to escape from R, the velocity vectors are always pointing inwards, forcing it to stay inside R. Since the solution can never leave R, the only thing it can do as t is either approach a critical point but there are none, by hypothesis or spiral in towards a closed trajectory. Thus there is a closed trajectory inside R. (It cannot be an unstable limit cycleit must be one of the other three cases shown above.) (1)

To use the Poincare-Bendixson theorem, one has to search the vector eld for closed curves D along which the velocity vectors all point towards

Showing Limit Cycles Exist

OCW 18.03SC

the same side. Here is an example where they can be found. Example 1. Consider the system x = y + x (1 x 2 y2 ) y = x + y (1 x 2 y2 ) (2)

Figure 2 shows how the associated velocity vector eld looks on two circles. On a circle of radius 2 centered at the origin, the vector eld points inwards, while on a circle of radius 1/2, the vector eld points outwards. To prove this, we write the vector eld along a circle of radius r as x = (yi + xj) + (1 r2 )( xi + yj) . (3)

The rst vector on the right side of (3) is tangent to the circle; the second vector points radially in for the big circle (r = 2), and radially out for the small circle (r = 1/2). Thus the sum of the two vectors given in (3) points inwards along the big circle and outwards along the small one. We would like to conclude that the Poincare-Bendixson theorem applies to the ring-shaped region between the two circles. However, for this we must verify that R contains no critical points of the system. We leave you to show as an exercise that (0, 0) is the only critical point of the system; this shows that the ring-shaped region contains no critical points. The above argument shows that the Poincare-Bendixson theorem can be applied to R, and we conclude that R contains a closed trajectory. In fact, it is easy to verify that x = cos t, y = sin t solves the system, so the unit circle is the locus of a closed trajectory. We leave as another exercise to show that it is actually a stable limit cycle for the system, and the only closed trajectory.

Non-Existence of Limit Cycles


We turn our attention now to the negative side of the problem of showing limit cycles exist. Here are two theorems which can sometimes be used to show that a limit cycle does not exist.

1. Bendixsons Criterion
If f x and gy are continuous in a region R which is simply-connected (i.e., without holes), and f g + = 0 at any point of R, x y then the system x = f ( x, y) y = g( x, y) has no closed trajectories inside R. Proof. Assume there is a closed trajectory C inside R. We shall derive a contradiction, by applying Greens theorem, in its normal (or ux) form. This theorem says

(1)

( f i + gj) n ds

f dy g dx

f g + ) dx dy . (2) x y

where D is the region inside the simple closed curve C. This however is a contradiction. Namely, by hypothesis, the integrand on the right-hand side is continuous and never 0 in R; thus it is either always positive or always negative, and the right-hand side of (2) is therefore either positive or negative. On the other hand, the left-hand side must be zero. For since C is a closed trajectory, C is always tangent to the velocity eld f i + gj dened by the system. This means the normal vector n to C is always perpendicular to the velocity eld f i + gj, so that the integrand ( f i + gj) n on the left is identically zero. This contradiction means that our assumption that R contained a closed trajectory of (1) was false, and Bendixsons Criterion is proved. Critical-point Criterion terior. A closed trajectory has a critical point in its in-

Non-Existence of Limit Cycles

OCW 18.03SC

If we turn this statement around, we see that it is really a criterion for non-existence: it says that if a region R is simply-connected (i.e., without holes) and has no critical points, then it cannot contain any limit cycles. For if it did, the Critical-point Criterion says there would be a critical point inside the limit cycle, and this point would also lie in R since R has no holes. (Note carefully the distinction between this theorem, which says that limit cycles enclose regions which do contain critical points, and the PoincareBendixson theorem, which seems to imply that limit cycles tend to lie in regions which dont contain critical points. The difference is that these latter regions always contain a hole; the critical points are in the hole. Example 1 illustrated this. x = ax + by Example 2. For what a and d does have closed trajecy = cx + dy tories? Solution. By Bendixsons criterion, ries. a + d = 0

no closed trajecto-

What if a + d = 0? Bendixsons criterion says nothing. We go back to our analysis of the linear system. The characteristic equation of the system is 2 ( a + d) + ( ad bc) = 0 . Assume a + d = 0. Then the characteristic roots have opposite sign if ad bc < 0 and the system is a saddle; the roots are pure imaginary if ad bc > 0 and the system is a center, which has closed trajectories. Thus the system has closed trajectories

a + d = 0,

ad bc > 0.

The Van der Pol Equation


An important kind of second-order non-linear autonomous equation has the form x + u( x ) x + v( x ) = 0 (Linard equation) . (1)

One might think of this as a model for a spring-mass system where the damping force u( x ) depends on position (for example, the mass might be moving through a viscous medium of varying density), and the spring constant v( x ) depends on how much the spring is stretchedthis last is true of all springs, to some extent. We also allow for the possibility that u( x ) < 0 (i.e., that there is "negative damping"). The system equivalent to (1) is x = y y = v( x ) u( x ) y (2)

Under certain conditions, the system (2) has a unique stable limit cycle, or what is the same thing, the equation (1) has a unique periodic solution; and all nearby solutions tend towards this periodic solution as t . The conditions which guarantee this were given by Linard, and generalized in the following theorem. Levinson-Smith Theorem Suppose the following conditions are satised. (a) u( x ) is even and continuous, (b) v( x ) is odd, v( x ) > 0 i f x > 0, and v( x ) is continuous for all x, x (c) V ( x ) as x , where V ( x ) = 0 v(t) dt , (d) for some k > 0, we have U ( x ) < 0, U ( x ) > 0 and increasing, U ( x ) , Then, the system (2) has i) a unique critical point at the origin; ii) a unique non-zero closed trajectory C, which is a stable limit cycle around the origin; iii) all other non-zero trajectories spiralling towards C as t . for 0 < x < k, for x > k, as x , where U ( x ) =
x
0

u(t) dt.

The Van der Pol Equation

OCW 18.03SC

We omit the proof, as too difcult. A classic application is to the equation x a(1 x2 ) x + x = 0 (van der Pol equation) (3) which describes the current x (t) in a certain type of vacuum tube. (The constant a is a positive parameter depending on the tube constants.) The equation has a unique non-zero periodic solution. Intuitively, think of it as modeling a non-linear spring-mass system. When | x | is large, the restoring and damping forces are large, so that | x | should decrease with time. But when | x | gets small, the damping becomes negative, which should make | x | tend to increase with time. Thus it is plausible that the solutions should oscillate; that it has exactly one periodic solution is a more subtle fact. There is a lot of interest in limit cycles, because of their appearance in systems which model processes exhibiting periodicity. But not a great deal is known about them this is still an area of active research.

Chaos
We give a very brief introduction to this subject using DEs as the starting point. The interested reader who wishes to explore this subject further will nd many good sources on the web.

1. Discrete Logistic Equation


The difference equation xn+1 = rxn (1 xn ) (r a constant) is the discrete logistic equation. One way it arises is as follows. dP = aP bP2 = model of logistic population growth. dt Eulers numerical method makes this a discrete system:
2 Pn+1 = Pn + ( aPn bPn ) h. 2 Rewrite this as Pn+1 = rPn sPn . r Let Pn = xn xn+1 = rxn (1 xn ). s Since r = 1 + ah we will only consider r > 1 Given r and x0 iteration gives the sequence

x0 , x1 , x2 , . . . , x n . . .

Figure 1.

This is easy to implement on a computer: Figure 1 shows an x vs. r diagram. To make it we used the following recipe. 1. We choose a value of r and a starting point x0 = .5. 2. We iterate out to x500 in order to eliminate any transient behavior. 3. We then plot 1000 points (r, xn ) for n = 501 to 1500. The darker the plotted point the more times that we got that value of x. Look for, instance at the value r = 1.5. The only x value plotted is the one at x = .333. This says that the iterated sequence x0 , x1 , . . . goes to a limit of .333. The values r = 2 and r = 2.5 behave similarly. At around r = 3.1 the diagram bifurcates. That is, it splits into two branches. What this means is that the value of xn is cycles back and forth between two values. In the case r = 3.1 we get x1001 = .5580, x1002 = .7646, x1003 = .5580, . . .

Chaos

OCW 18.03SC

We call this a period 2 cycle. As r increases from 3.1 we continue to get period 2 cycles until around r = 3.5. At this point both branches of the diagram bifurcate and we see four values plotted. This means the values of xn are cycling between four values. This is called a cycle with period 4 (or a 4-cycle for short). This continues as r increases until the next bifurcation point where we get cycles of period 8. As r increases further, this period doubling continues to cycles of period 16, 32, etc. Then around r = 3.57 something new happens: the periodic behavior disappears and seemingly random behavior occurs. This is called chaos. At around r = 3.83 periodic behavior returns with cycles of period 3. As r increases we again see period doubling with cycles of period 6, then 12, then 24 etc. until this leads to chaos again. After the chaotic region there is a value of r where we see period 5cycles. This is followed by period doubling, leading to chaos again. Then 7-cycles followed by period doubling to chaos, etc.

Figures 2-4. The pitchfork at various resolutions. Remarks: 1. This period doubling to chaos is a phenomenon seen in many systems. 2. For any value of r there are xed points and, often points with other periods. The computer doesnt nd them because they are not stable. In fact, there is a theorem that says if there is a point of period 3 then there are points of all orders.

2. Feigenbaum constant
If r1 = rst bifurcation point, r2 = second etc. then

Chaos r k r k 1 = 4.6692 . . . = the Feigenbaum constant. rk The same value occurs in many period doubling systems.
k r k +1

OCW 18.03SC

lim

3. The forced Dufng Equation


Period doubling also happens in mechanical systems. If we apply a periodic force to the damped nonlinear spring we get the equation mx + cx + kx + x3 = F0 cos t. A mass atop a thin metal wire is modeled by this equation with k < 0. We look at the forced Dufng equation x + x x + x3 = F0 cos t and the equivalent nonlinear system x = y y = x x3 y + F0 cos t.

If F0 = 0 (unforced) then there are 3 equilibrium points: (0, 0) unstable (saddle); (1, 0) stable (spiral sinks). These are shown in the pictures at right.

stable unstable stable In a linear spring system the single critical point at the origin is stable and the frequency of the periodic response would equal (which in this case is 1) and doubling the amplitude of the input would simply double the amplitude of the output. In the Dufng system, the behavior is very different. The plots below were made by taking x (0) = 1, x (0) = 0, running the ode solver for t = 0 to 200, and plotting for t = 100 to 200. (We throw away t = 0 to 100 as transient.) Just like the discrete logistic equation, we see period doubling to chaos.

Chaos

OCW 18.03SC

Chaos

OCW 18.03SC

4. Lorenz Strange Attractor


This example is the Lorenz System. It is a 3 dimensional system x = sx + sy y z where s, r, b are constants. The following picture shows the famous buttery. (The plot should be three dimensional, showing x, y and z. In this case we just plotted z vs. x.) Like limit cycles or the periodic points in the pitchfork example this trajectory has a limiting set. That is. a set of points that are arbitrarily close to the trajectory for arbitrarily large t. In this case, it is called a strange attractor because it is such a complicated set.

= xz + rx y = xy bz

You might also like